*NURSING > EXAM > PATHOPHYSIOLOGY 8TH EDITION MCCANCE TEST BANK Chapter 1 to 50,100% CORRECT (All)

PATHOPHYSIOLOGY 8TH EDITION MCCANCE TEST BANK Chapter 1 to 50,100% CORRECT

Document Content and Description Below

PATHOPHYSIOLOGY 8TH EDITION MCCANCE TEST BANK Chapter 1 to 50 Chapter 1: Cellular Biology MULTIPLE CHOICE 1. Which statement best describes the cellular function of metabolic absorption? a.... Cells can produce proteins. c. Cells can take in and use nutrients. b. Cells can secrete digestive enzymes. d. Cells can synthesize fats. ANS: C In metabolic absorption, all cells take in and use nutrients and other substances from their surroundings. The remaining options are not inclusive in their descriptions of cellular metabolic absorption. PTS: 1 REF: Page 2 2. Most of a cell’s genetic information, including RNA and DNA, is contained in the: a. Mitochondria c. Nucleolus b. Ribosome d. Lysosome ANS: C The nucleus contains the nucleolus, a small dense structure composed largely of RNA, most of the cellular DNA, and the DNA-binding proteins, such as the histones, which regulate its activity. The other options do not contain most of a cell’s genetic information. PTS: 1 REF: Page 2 3. Which component of the cell prodNuUceRsSIhNyGdTroBg.CenOMperoxide (H2O2) by using oxygen to remove hydrogen atoms from specific substrates in an oxidative reaction? a. Lysosomes c. Ribosomes b. Peroxisomes d. Oxyhydrosomes ANS: B Peroxisomes are so named because they usually contain enzymes that use oxygen to remove hydrogen atoms from specific substrates in an oxidative reaction that produces H2O2, which is a powerful oxidant and potentially destructive if it accumulates or escapes from peroxisomes. Ribosomes are RNA-protein complexes (nucleoproteins) that are synthesized in the nucleolus and secreted into the cytoplasm through pores in the nuclear envelope called nuclear pore complexes. Lysosomes are saclike structures that originate from the Golgi complex and contain more than 40 digestive enzymes called hydrolases, which catalyze bonds in proteins, lipids, nucleic acids, and carbohydrates. Oxyhydrosomes are involved in enzyme production. PTS: 1 REF: Page 8 4. Which cell component is capable of cellular autodigestion when it is released during cell injury? a. Ribosome c. Smooth endoplasmic reticulum b. Golgi complex d. Lysosomes ANS: D The lysosomal membrane acts as a protective shield between the powerful digestive enzymes within the lysosome and the cytoplasm, preventing their leakage into the cytoplasmic matrix. Disruption of the membrane by various treatments or cellular injury leads to a release of the lysosomal enzymes, which can then react with their specific substrates, causing cellular self-digestion. The other options do not correctly describe this process. PTS: 1 REF: Pages 7-8 5. What is the sequence of steps in the development of a digestive enzyme by the pancreas cells from the initial transcription to the release from the cell? a. The enzyme is transcribed from DNA by RNA in the nucleus, proceeds to the ribosome for synthesis, and is transported in a secretory vesicle to the cell membrane. b. The enzyme is transcribed from RNA by DNA in the nucleus, proceeds to the lysosome for synthesis, and is transported in an encapsulated membrane to the cell membrane. c. The enzyme is transcribed by the mitochondria in the nucleus, proceeds to the ribosome for synthesis, and is transported in a cytoskeleton to the cell membrane. d. The enzyme is transcribed from DNA by RNA in the nucleus, proceeds to the Golgi complex for synthesis, and is transported in a cytosol to the cell membrane. ANS: A The enzyme is transcribed from DNA by RNA in the nucleus, proceeds to the ribosome for synthesis, and is transported in a secretory vesicle to the cell membrane. The other options do not correctly describe this process. NURSINGTB.COM PTS: 1 REF: Page 7 | Figure 1-5 6. During which phase of the cell cycle is DNA synthesized? a. G1 c. G2 b. S d. M ANS: B The four designated phases of the cell cycle are: (1) the G1 phase (G = gap), which is the period between the M phase (M = mitosis) and the start of DNA synthesis; (2) the S phase (S = synthesis), during which DNA is synthesized in the cell nucleus; (3) the G2 phase, during which RNA and protein synthesis occurs, the period between the completion of DNA synthesis and the next phase (M); and (4) the M phase, which includes nuclear and cytoplasmic division. PTS: 1 REF: Page 37 7. What organic compound facilitates transportation across cell membranes by acting as receptors, transport channels for electrolytes, and enzymes to drive active pumps? a. Lipids c. Proteins b. Proteases d. Carbohydrates ANS: C Proteins act as (1) recognition and binding units (receptors) for substances moving in and out of the cell; (2) pores or transport channels for various electrically charged particles called ions or electrolytes and specific carriers for amino acids and monosaccharides; and (3) specific enzymes that drive active pumps that promote the concentration of certain ions, particularly potassium (K+), within the cell while keeping concentrations of other ions, for example, sodium (Na+), below the concentrations found in the extracellular environment. The other options do not correctly describe this process. PTS: 1 REF: Page 13 | Page 15 8. Understanding the various steps of proteolytic cascades, such as caspase-mediated apoptosis and complement cascades, may be useful in designing drug therapy for which human diseases? a. Cardiac and vascular disorders b. Autoimmune and malignant disorders c. Gastrointestinal and renal disorders d. Endocrine and gastrointestinal disorders ANS: B Understanding the various steps involved in this process is crucial for designing drug interventions. Dysregulation of proteases features prominently in many human diseases, including cancer, autoimmunity, and neurodegenerative disorders. The other options do not correctly describe this process. PTS: 1 REF: Page 15 9. Which structure prevents water-soluble molecules from entering cells across the plasma membrane? NURSINGTB.COM a. Carbohydrate chains c. Membrane channel proteins b. Glycoprotein channels d. Lipid bilayer ANS: D The bilayer’s structure accounts for one of the essential functions of the plasma membrane. It is impermeable to most water-soluble molecules (molecules that dissolve in water) because the water-soluble molecules are insoluble in the oily core region. The bilayer serves as a barrier to the diffusion of water and hydrophilic substances while allowing lipid-soluble molecules, such as oxygen (O2) and carbon dioxide (CO2), to diffuse through it readily. The other options do not correctly describe this process. PTS: 1 REF: Pages 12-13 10. The fluid mosaic model explains: a. How a cell membrane functions b. Why our bodies appear to be solid c. How tissue is differentiated d. How fluid moves between the intracellular and extracellular compartments ANS: A The fluid mosaic model accounts for the flexibility of cellular membranes, their self-sealing properties, and their impermeability to many substances. The remaining options do not explain the mosaic model. PTS: 1 REF: Page 12 | What's New box 11. Which form of cell communication is used to communicate within the cell itself and with other cells in direct physical contact? a. Protein channel (gap junction) b. Plasma membrane–bound signaling molecules (involving receptors) c. Hormone secretion such as neurotransmitters d. Extracellular chemical messengers such as ligands ANS: A Cells communicate by using hundreds of kinds of signal molecules, for example, insulin. Cells communicate in three main ways; they display plasma membrane–bound signaling molecules (receptors) that affect the cell itself and other cells in direct physical contact. The other options do not correctly describe this process. PTS: 1 REF: Page 20 12. Which mode of chemical signaling uses blood to transport communication to cells some distance away? a. Paracrine c. Neurotransmitter b. Autocrine d. Hormonal ANS: D Chemical signaling can be classified into three categories: (1) local-chemical mediator, (2) hormone, and (3) neurotransmitter. In the local-chemical mediator model, the secreted chemical acts on the cells in the immediate environment. Hormones are used for communication with distant targetNcUeRllSsI.NFGoTrBe.xCaOmMple, cells can secrete a chemical and rely on the blood system to deliver the signal to a distant cell. Finally, neurotransmitters are secreted by neurons to stimulate an adjoining cell. For example, a neuron might secrete acetylcholine to stimulate the movement of a muscle cell. PTS: 1 REF: Page 20 13. Which mode of chemical signaling uses local chemical mediators that are quickly taken up, destroyed, or immobilized? a. Paracrine c. Neurotransmitter b. Autocrine d. Hormone ANS: A In paracrine signaling, cells secrete local chemical mediators that are quickly taken up, destroyed, or immobilized. The other options do not correctly describe this process. PTS: 1 REF: Page 20 14. Neurotransmitters affect the postsynaptic membrane by binding to: a. Lipids c. Amphipathic lipids b. Ribosomes d. Receptors ANS: D In each type of chemical signaling, the target cell receives the signal by first attaching to its receptors. The other options do not correctly describe this process. PTS: 1 REF: Page 17 15. How do cells receive communication from the extracellular fluid surrounding them? a. Protein channel (gap junction) b. Plasma membrane–bound signaling molecules (involving receptors) c. Hormone secretion such as neurotransmitters d. Chemical messengers such as ligands ANS: D Cellular communication can occur by the binding of a chemical massager (a ligand) to a specific membrane receptor that is closely associated with the channel (e.g., G proteins). The other options do not correctly describe how cells communicate. PTS: 1 REF: Pages 21-22 16. When a second message is necessary for extracellular communication to be activated, it is provided by which one? a. Guanosine triphosphate (GTP) c. Adenosine triphosphate (ATP) b. Adenosine monophosphate (AMP) d. Guanosine diphosphate (GDP) ANS: B The two major second messenger pathways are cyclic AMP (cAMP) and calcium (Ca++). PTS: 1 REF: Pages 22-23 17. Under anaerobic conditions, what process provides energy for the cell? a. Oxidative phosphorylation NURSINGTcB..COLMactolysis b. Glycolysis d. Passive transport ANS: B Glycolysis produces a net of two molecules of ATP per glucose molecule through the process of oxidation or the removal and transfer of a pair of electrons. The other options do not correctly identify an anaerobic process that provides energy to the cell. PTS: 1 REF: Page 28 18. What is the mechanism by which the energy produced from carbohydrates, proteins, and lipids is transferred to adenosine triphosphate (ATP)? a. Anaerobic glycolysis c. Oxidative phosphorylation b. Oxidative cellular metabolism d. Tricarboxylic acid phosphorylation ANS: C Oxidative phosphorylation occurs in the mitochondria and is the mechanism by which the energy produced from carbohydrates, fats, and proteins is transferred to ATP. The other options do not correctly identify the mechanism described in the question. PTS: 1 REF: Pages 27-28 19. Passive transport is best described with which statement? a. Being driven by osmosis, hydrostatic pressure, and diffusion b. Involving receptors that can bind with substances being transported c. Being capable of transporting macromolecules d. Requiring energy generated by the cell ANS: A Water and small electrically uncharged molecules move easily through pores in the plasma membrane’s lipid bilayer. This process, called passive transport, naturally occurs through any semipermeable barrier. It is driven by osmosis, hydrostatic pressure, and diffusion, all of which depend on the laws of physics and do not require life. The other options do not correctly describe passive transport. PTS: 1 REF: Page 28 20. Active transport occurs across which type of membranes? a. Membranes that have a higher concentration of the solute on the outside of the cell b. Membranes that are semipermeable to water and small electrically uncharged molecules c. Membranes that have receptors that are capable of binding with the substances to be transported d. Membranes that have a cell membrane that is hydrophobic rather than hydrophilic ANS: C Some molecules are moved into the cell by mechanisms of active transport, which require receptors that are capable of recognizing and binding with the substance to be transported. Diffusion is the movement of a solute molecule from an area of greater solute concentration to an area of lesser solute concentration. Hydrostatic pressure is the mechanical force of water pushing against cellular membranes. Osmosis is the movement of water down a concentration gradient; that is, across a semipermeable membrane from a region of higher water concentratiNoUnRtoSIaNGloTwBe.Cr OwMater concentration PTS: 1 REF: Page 28 21. Which method of transport uses transmembrane proteins with receptors with a high degree of specificity for the substance being transported? a. Active c. Transmembranous b. Mediated d. Passive ANS: B Mediated transport (passive and active) involves integral or transmembrane proteins with receptors having a high degree of specificity for the substance being transported. Inorganic anions and cations (e.g., Na+, K+, Ca++, chloride [Cl–], bicarbonate [HCO3–]) and charged and uncharged organic compounds (e.g., amino acids, sugars) require specific transport systems to facilitate movement through different cellular membranes. The remaining options do not correctly identify the process described. PTS: 1 REF: Page 31 22. The movement of fluid across the arterial end of capillary membranes into the interstitial fluid surrounding the capillary is an example of which fluid movement process? a. Hydrostatic pressure c. Diffusion b. Osmosis d. Active transport ANS: A Hydrostatic pressure is the mechanical force of water pushing against cellular membranes. In the vascular system, hydrostatic pressure is the blood pressure generated in vessels by the contraction of the heart. Blood reaching the capillary bed has a hydrostatic pressure of 25 to 30 mm Hg, which is sufficient force to push water across the thin capillary membranes into the interstitial space. The remaining options do not correctly identify the process described. PTS: 1 REF: Pages 29-30 23. Why is osmolality preferred over osmolarity as the measurement of osmotic activity in the clinical assessment of individuals? a. Plasma contains sodium and chloride, which influence the volume of solution. b. Volume affects perfusion more than the weight of solutes. c. More of the weight of plasma is influenced by solutes, such as protein and glucose, rather than by water. d. Osmotic activity depends on the concentration of solutes present in plasma, such as proteins and glucose. ANS: C In plasma, less of the plasma weight is water; therefore the overall concentration of particles is greater. The osmolality will be greater than the osmolarity because of the smaller proportion of water. Osmolality is thus the preferred measure of osmotic activity in clinical assessment of individuals. PTS: 1 REF: Page 30 24. A patient who has diarrhea receives a 3% saline solution intravenously to replace the sodium and chloride lost in the stoNoUl.RWSINhaGtTeBf.fCeOctMwill this fluid replacement have on cells? a. Become hydrated c. Shrink b. Swell or burst d. Divide ANS: C A hypertonic solution has a concentration of greater than 285 to 294 mOsm/kg. An example of a hypertonic solution is 3% saline solution. Water can be pulled out of the cells by a hypertonic solution; therefore the cells shrink. The remaining options do not correctly describe the effect identified in the stem. PTS: 1 REF: Page 31 25. The transport of glucose from the blood to the cell is accomplished by which process? a. Active-mediated transport (active transport) b. Active diffusion c. Passive osmosis d. Passive-mediated transport (facilitated diffusion) ANS: D Facilitated diffusion is the means by which glucose is transported from the blood to the cells. The remaining options do not correctly identify this process. PTS: 1 REF: Pages 31-32 26. Potassium and sodium are transported across plasma membranes by: a. Passive electrolyte channels b. Coupled channels c. Adenosine triphosphatase (ATPase) enzyme d. Diffusion ANS: C The exact mechanism for the transport of Na+ and K+ across the membrane is uncertain. One proposal is that ATPase enzyme induces the transporter protein to undergo several conformational changes, causing Na+ and K+ to move short distances (see Figure 1-29). The remaining options do not correctly describe the means by which K+ and Na+ are transported. PTS: 1 REF: Pages 32-33 27. What occurs during exocytosis? a. Macromolecules can be secreted across eukaryotic cell membranes. b. All substances are secreted into the cellular matrix. c. No repairs in the plasma membrane can take place. d. Solute molecules flow freely into and out of the cell. ANS: A In eukaryotic cells, secretion of macromolecules almost always occurs by exocytosis. The remaining options do not correctly describe exocytosis. PTS: 1 REF: Pages 35-36 28. Why is it possible for potassium to diffuse easily into and out of cells? a. Potassium has a greater conceNntUraRtSioINnGinTBth.CeOiMntracellular fluid (ICF). b. Sodium has a greater concentration in the extracellular fluid (ECF). c. The resting plasma membrane is more permeable to potassium. d. An excess of anions are inside the cell. ANS: C Because the resting plasma membrane is more permeable to K+ than to Na+, K+ can easily diffuse from its area of higher concentration in the ICF to its area of lower concentration in the ECF. Because Na+ and K+ are both cations, the net result is an excess of anions inside the cell, resulting in the resting membrane potential. The remaining options do not correctly identify the process that most easily diffuses K+. PTS: 1 REF: Page 36 29. The cellular uptake of the nutrient cholesterol depends on which process? a. Receptor-mediated exocytosis c. Receptor-mediated endocytosis b. Antiport system d. Passive transport ANS: C The cellular uptake of nutrients, such as cholesterol, for example, depends on receptor-mediated endocytosis. Nutrients are not transported via the other options. PTS: 1 REF: Page 33 30. What causes the rapid change in the resting membrane potential to initiate an action potential? a. Potassium gates open, and potassium rushes into the cell, changing the membrane potential from negative to positive. b. Sodium gates open, and sodium rushes into the cell, changing the membrane potential from negative to positive. c. Sodium gates close, allowing potassium into the cell to change the membrane potential from positive to negative. d. Potassium gates close, allowing sodium into the cell to change the membrane potential from positive to negative. ANS: B When a resting cell is stimulated through voltage-regulated channels, the cell membranes become more permeable to Na+. A net Na+ moves into the cell, and the membrane potential decreases, or moves forward, from a negative value (in millivolts) to zero. The Na+ gates open, and Na+ rushes into the cell, causing the membrane potential to reduce to zero and then become positive (depolarization). The remaining options do not correctly describe the change that initiates an action potential. PTS: 1 REF: Page 36 31. The action of platelet-derived growth factor is to stimulate the production of which cells? a. Platelets c. Connective tissue cells b. Epidermal cells d. Fibroblast cells ANS: C Different types of cells require different factors; for example, platelet-derived growth factor stimulates the production ofNcUoRnSnINecGtiTvBe.CtiOssMue cells. The remaining options do not correctly describe the action of platelet-derived growth factor. PTS: 1 REF: Page 39 32. The role of cytokines in cell reproduction is that they: a. Provide growth factor for tissue growth and development. b. Block progress of cell reproduction through the cell cycle. c. Restrain cell growth and development. d. Provide nutrients for cell growth and development. ANS: A Growth factors, also called cytokines, are peptides that transmit signals within and among cells. They have a major role in the regulation of tissue growth and development (see Table 1-6). The remaining options do not correctly describe the role of cytokines in cell reproduction. PTS: 1 REF: Pages 38-39 33. What is the process of cellular reproduction? a. The process often takes months or years to complete. b. Cellular reproduction typically has a short interphase. c. Two diploid cells, called daughter cells, have been formed. d. The process involves the interaction of male and female cells. ANS: C During telophase, the final stage, a new nuclear membrane is formed around each group of 46 chromosomes, the spindle fibers disappear, and the chromosomes begin to uncoil. Cytokinesis causes the cytoplasm to divide into roughly equal parts during this phase. At the end of telophase, two identical diploid cells, called daughter cells, have been formed from the original cell. PTS: 1 REF: Page 37 34. Which statement is true about eukaryotic cells? a. They lack distinct nucleus. b. They contain compartments called organelles. c. They lack an encasing nuclear membrane. d. They are smaller than the typical prokaryote cell. ANS: B Eukaryotic cells have a characteristic set of membrane-bound intracellular compartments called organelles that include a well-defined nucleus and are larger than prokaryotes. The remaining statements are not true regarding eukaryotic cells. PTS: 1 REF: Page 2 35. Which statement is true about phagocytosis? a. Phagocytosis is an example of exocytosis. b. Phagocytosis is dependent on small vesicles. c. Phagocytosis involves the ingestion of bacteria. d. Phagocytosis focuses on solute molecules. NURSINGTB.COM ANS: C In phagocytosis, the large molecular substances are engulfed by the plasma membrane and enter the cell so that they can be isolated and destroyed by lysosomal enzymes. Two types of endocytosis are designated, based on the size of the vesicle formed. Pinocytosis (cell drinking) involves the ingestion of fluids and solute molecules through the formation of small vesicles, and phagocytosis (cell eating) involves the ingestion of large particles, such as bacteria, through formation of large vesicles (also called vacuoles). Phagocytosis in an example of endocytosis, not exocytosis. PTS: 1 REF: Pages 33-34 36. A muscle cell possesses which specialized function? a. Movement c. Secretion b. Conductivity d. Respiration ANS: A A cell has the potential to differentiation and to gain the ability to perform one of eight specialized functions. Muscle cells can generate forces that produce motion. Nerves cells are capable of conductivity. Cells of the adrenal gland, testis, and ovary can secrete. Respiration is a function that all cells possess. PTS: 1 REF: Page 2 37. When a mucous gland cell creates a new substance from previously absorbed material, this process is known as which specialized cellular function? a. Excretion c. Reproduction b. Metabolic absorption d. Secretion ANS: D Certain cells, such as mucous gland cells, can synthesize new substances from substances they absorb and then secrete the new substances to serve elsewhere as needed. The other options are not used to describe the function described in the stem. PTS: 1 REF: Page 2 38. All cells are capable of what process? a. Excretion c. Metabolic absorption b. Movement d. Continuous division ANS: A All cells have the capacity to excrete, thus allowing them to rid themselves of waste products resulting from the metabolic breakdown of nutrients. The remaining options are not functions possessed by all cells. PTS: 1 REF: Page 2 MULTIPLE RESPONSE 39. What are the major chemical components of the cell membranes? (Select all that apply.) a. Lipids b. Sodium ions c. Carbohydrates d. DNA e. Proteins ANS: A, E NURSINGTB.COM The major chemical components of all cell membranes are lipids and proteins, but the percentage of each varies among different membranes. PTS: 1 REF: Page 12 40. Which cells lose their ability to replicate and divide? (Select all that apply.) a. Intestines b. Nerves c. Skin d. Lens of the eye e. Skeletal muscle ANS: B, D, E All types of cells undergo mitosis during the formation of the embryo, but many adult cells, such as nerve cells, lens cells of the eye, and muscle cells, lose their ability to replicate and divide. Intestines and skin cells retain their ability to replicate and divide. PTS: 1 REF: Page 37 41. Which statements are true concerning the process of facilitated diffusion? (Select all that apply). a. Facilitated diffusion is also referred to as passive mediated transport. b. This process expends no metabolic energy. c. Moving solute molecules through cellular membranes are involved in this process. d. Movement up a concentration gradient is necessary. e. Facilitated diffusion is the primary means for water transport. ANS: A, B, C In passive mediated transport, also called facilitated diffusion, the protein transporter moves solute molecules through cellular membranes without expending metabolic energy. Downward movement along a concentration gradient is necessary. Osmosis is the movement of water down a concentration gradient. PTS: 1 REF: Pages 31-32 42. Passive transport is dependent on: (Select all that apply.) a. Semipermeable barrier membrane b. The process of osmosis c. Diffusion as a driving force d. A living host e. Hydrostatic pressure ANS: A, B, C, E Passive transport naturally occurs through any semipermeable barrier. It is driven by osmosis, hydrostatic pressure, and diffusion, all of which depend on the laws of physics and do not require life. NURSINGTB.COM PTS: 1 REF: Page 28 43. What is the primary function of proteins? (Select all that apply.) a. Proteins are binding units. b. Proteins are transport channels. c. Proteins are ribonucleoproteins. d. Proteins provide cell surface markers. e. Proteins are chemical reaction catalysts. ANS: A, B, D, E Protein functions include (a) recognition and binding units (receptors) for substances moving in and out of the cell; (b) pores or transport channels; (c) enzymes that drive active pumps; (d) cell surface markers, such as glycoproteins; (e) cell adhesion molecules; and (f) catalysts of chemical reactions. PTS: 1 REF: Page 15 MATCHING Match the structure with its function. Answers may be used more than once. A. Endoplasmic reticulum B. Ribosome C. Secretory vesicle D. Lysosomes 44. Packages and transports proteins. 45. Fuses with the plasma membrane to release contents from the cell. 46. Synthesizes and transports lipids. 47. Provides energy to digest proteins into amino acids. 44. ANS: A PTS: 1 REF: Pages 5-6 MSC: The endoplasmic reticulum (endo = within; plasma = cytoplasm; reticulum = network) is a membrane factory that specializes in the synthesis and transport of the protein and lipid components of most of the cell's organelles. 45. ANS: C PTS: 1 REF: Pages 6-7 MSC: Proteins from the endoplasmic reticulum are processed and packaged into small membrane-bound sacs or vesicles called secretory vesicles that collect at the end of the membranous folds of the Golgi bodies. The secretory vesicles then break off from the Golgi complex and migrate to a variety of intracellular and extracellular destinations, including the plasma membrane. The vesicles fuse with the plasma membrane, and their contents are released from the cell. 46. ANS: A PTS: 1 REF: Pages 5-6 MSC: The endoplasmic reticulum (endo = within; plasma = cytoplasm; reticulum = network) is a membrane factory that specializes in the synthesis and transport of the protein and lipid components of most of the cell's organelles. 47. ANS: D PTS: 1 REF: Page 7 MSC: Lysosomes function as the intracellular digestive system (see Figure 1-6). Lysosomal enzymes are capable of digesting most cellular constituents down to their basic forms, such as amino acids, fatty acids, and sugars. Match the structure with its functiNoUn.RASInNsGwTeBrs.CmOaMy be used more than once. A. Passive-mediated transport B. Active-mediated transport C. Osmosis 48. Movement of water 49. Protein carrier 50. Facilitated diffusion 48. ANS: C PTS: 1 REF: Page 30 MSC: Osmosis is the movement of water down a concentration gradient; that is, across a semipermeable membrane from a region of higher water concentration to a lower water concentration. 49. ANS: B PTS: 1 REF: Page 32 MSC: In active-mediated transport, also called active transport, the protein transporter moves molecules against, or up, the concentration gradient. Unlike passive-mediated transport, active-mediated transport requires the expenditure of energy. 50. ANS: A PTS: 1 REF: Pages 31-32 MSC: In passive-mediated transport, also called facilitated diffusion, the protein transporter moves solute molecules through cellular membranes without expending metabolic energy. Chapter 2: Altered Cellular and Tissue Biology MULTIPLE CHOICE 1. Which type of cell adaptation occurs when normal columnar ciliated epithelial cells of the bronchial lining have been replaced by stratified squamous epithelial cells? a. Hyperplasia c. Dysplasia b. Metaplasia d. Anaplasia ANS: B Metaplasia is the reversible replacement of one mature cell by another, sometimes a less differentiated cell type. The best example of metaplasia is the replacement of normal columnar ciliated epithelial cells of the bronchial (airway) lining by stratified squamous epithelial cells. The other options do not accurately describe the event in the question. PTS: 1 REF: Page 54 2. The loss of the adenosine triphosphate (ATP) during ischemia causes cells to: a. Shrink because of the influx of calcium (Ca). b. Shrink because of the influx of potassium chloride (KCl). c. Swell because of the influx of sodium chloride (NaCl). d. Swell because of the influx of nitric oxide (NO). ANS: C A reduction in ATP levels causes the plasma membrane’s sodium-potassium (Na+–K+) pump and sodium-calcium exchange to fail, which leads to an intracellular accumulation of sodium and calcium and diffusiNoUnRoSfINpoGtTaBss.CiuOmMout of the cell. (The Na+–K+ pump is discussed in Chapter 1.) Sodium and water can then freely enter the cell, and cellular swelling results. The other options do not accurately describe the result of ATP at the cellular level. PTS: 1 REF: Page 57 3. The mammary glands enlarge during pregnancy primarily as a consequence of hormonal: a. Atrophy c. Anaplasia b. Hyperplasia d. Dysplasia ANS: B Hormonal hyperplasia occurs chiefly in estrogen-dependent organs, such as the uterus and breast. The remaining options do not adequately describe the consequence of hormones on breast tissue during pregnancy. PTS: 1 REF: Page 53 4. Free radicals play a major role in the initiation and progression of which diseases? a. Cardiovascular diseases such as hypertension and ischemic heart disease b. Renal diseases such as acute tubular necrosis and glomerulonephritis c. Gastrointestinal diseases such as peptic ulcer disease and Crohn disease d. Muscular disease such as muscular dystrophy and fibromyalgia Emerging data indicate that reactive oxygen species play major roles in the initiation and progression of cardiovascular alterations associated with hyperlipidemia, diabetes mellitus, hypertension, ischemic heart disease, and chronic heart failure. No current research connects the disorders mentioned in the other options to the effects of free radicals. PTS: 1 REF: Pages 59-60 5. Free radicals cause cell damage by: a. Stealing the cell’s oxygen to stabilize the electron, thus causing hypoxia b. Stimulating the release of lysosomal enzymes that digest the cell membranes c. Transferring one of its charged, stabilized atoms to the cell membrane, which causes lysis d. Giving up an electron, which causes injury to the chemical bonds of the cell membrane ANS: D A free radical is an electrically uncharged atom or group of atoms having an unpaired electron. Having one unpaired electron makes the molecule unstable; thus to stabilize, the molecule gives up an electron to another molecule or steals one. Therefore it is capable of forming injurious chemical bonds with proteins, lipids, or carbohydrates—key molecules in membranes and nucleic acids. The remaining options do not accurately describe the role played by free radicals in cell damage. PTS: 1 REF: Page 60 6. What is a consequence of plasma membrane damage to the mitochondria? a. Enzymatic digestion halts DNA synthesis. b. Influx of calcium ions halts ANTUPRpSrIoNdGuTctBio.CnO. M c. Edema from an influx in sodium causes a reduction in ATP production. d. Potassium shifts out of the mitochondria, which destroys the infrastructure. ANS: B The most serious consequence of plasma membrane damage is, as in hypoxic injury, to the mitochondria. An influx of calcium ions from the extracellular compartment activates multiple enzyme systems, resulting in cytoskeleton disruption, membrane damage, activation of inflammation, and eventually DNA degradation. Calcium ion accumulation in the mitochondria causes the mitochondria to swell, which is an occurrence that is associated with irreversible cellular injury. The injured mitochondria can no longer generate ATP, but they do continue to accumulate calcium ions. The remaining options do not accurately describe the consequence of plasma membrane damage to the mitochondria. PTS: 1 REF: Page 63 7. What is a consequence of leakage of lysosomal enzymes during chemical injury? a. Enzymatic digestion of the nucleus and nucleolus occurs, halting DNA synthesis. b. Influx of potassium ions into the mitochondria occurs, halting the ATP production. c. Edema of the Golgi body occurs, preventing the transport of proteins out of the cell. d. Shift of calcium out of the plasma membrane occurs, destroying the cytoskeleton. Enzymatic digestion of cellular organelles, including the nucleus and nucleolus, ensues, halting the synthesis of DNA and ribonucleic acid (RNA). The remaining options do not accurately describe the consequence of lysosomal enzyme leakage during chemical injury. PTS: 1 REF: Page 63 8. Lead causes damage within the cell by interfering with the action of: a. Sodium and chloride c. Calcium b. Potassium d. ATP ANS: C Lead affects many different biologic activities at the cellular and molecular levels, many of which may be related to its ability to interfere with the functions of calcium. Lead does not appear to cause damage by interfering with the action of the other options. PTS: 1 REF: Page 66 9. Which statement is a description of the characteristics of apoptosis? a. Programmed cell death of scattered, single cells b. Characterized by swelling of the nucleus and cytoplasm c. Unpredictable patterns of cell death d. Results in benign malignancies ANS: A Apoptosis is an active process of cellular self-destruction, also known as programmed cell death, which is implicated in normal and pathologic tissue changes. The remaining options do not accurately describe the characteristics of apoptosis. NURSINGTB.COM PTS: 1 REF: Page 91 10. Lead poisoning affects the nervous system by: a. Interfering with the function of neurotransmitters b. Inhibiting the production of myelin around nerves c. Increasing the resting membrane potential d. Altering the transport of potassium into the nerves ANS: A Alterations in calcium may play a crucial role in the interference with neurotransmitters, which may cause hyperactive behavior and the proliferation of capillaries of the white matter and intercerebral arteries. The remaining options do not accurately describe the effects of lead poisoning of the nervous system. PTS: 1 REF: Page 66 11. Carbon monoxide causes tissue damage by: a. Competing with carbon dioxide so that it cannot be excreted b. Binding to hemoglobin so that it cannot carry oxygen c. Destroying the chemical bonds of hemoglobin so it cannot carry oxygen d. Removing iron from hemoglobin so it cannot carry oxygen ANS: B Because carbon monoxide’s affinity for hemoglobin is 200 times greater than that of oxygen, it quickly binds with the hemoglobin, preventing oxygen molecules from doing so. The remaining options do not accurately describe the means by which carbon monoxide damages tissue. PTS: 1 REF: Page 67 12. Acute alcoholism mainly affects which body system? a. Hepatic c. Renal b. Gastrointestinal d. Central nervous ANS: D Acute alcoholism mainly affects the central nervous system but may induce reversible hepatic and gastric changes. Other systems may evidentially be affected by chronic alcoholism. PTS: 1 REF: Page 68 13. During cell injury caused by hypoxia, an increase in the osmotic pressure occurs within the cell because: a. Plasma proteins enter the cell. b. The adenosine triphosphatase (ATPase)–driven pump is stronger during hypoxia. c. Sodium chloride enters the cell. d. An influx of glucose occurs through the injured cell membranes. ANS: C In hypoxic injury, movement of fluid and ions into the cell is associated with acute failure of metabolism and a loss of ATP pNrUoRdSuIcNtiGoTnB. .NCoOrMmally, the pump that transports sodium ions out of the cell is maintained by the presence of ATP and ATPase, the active-transport enzyme. In metabolic failure caused by hypoxia, reduced ATP and ATPase levels permit sodium to accumulate in the cell, whereas potassium diffuses outward. The increase of intracellular sodium increases osmotic pressure, which draws more water into the cell. (Transport mechanisms are described in Chapter 1.) The remaining options do not accurately describe the cell injury that results in increased osmotic pressure caused by hypoxia. PTS: 1 REF: Page 84 14. Which statement is true regarding the difference between subdural hematoma and epidural hematoma? a. No difference exists, and these terms may be correctly used interchangeably. b. A subdural hematoma occurs above the dura, whereas an epidural hematoma occurs under the dura. c. A subdural hematoma is often the result of shaken baby syndrome, whereas an epidural hematoma rapidly forms as a result of a skull fracture. d. A subdural hematoma usually forms from bleeding within the skull, such as an aneurysm eruption, whereas an epidural hematoma occurs from trauma outside the skull, such as a blunt force trauma. ANS: C A subdural hematoma is a collection of blood between the inner surface of the dura mater and the surface of the brain, resulting from the shearing of small veins that bridge the subdural space. Subdural hematomas can be the result of blows, falls, or sudden acceleration-deceleration of the head, which occurs in the shaken baby syndrome. An epidural hematoma is a collection of blood between the inner surface of the skull and the dura and is almost always associated with a skull fracture. The other options do not accurately describe the differences between the two hematomas. PTS: 1 REF: Page 72 | Table 2-6 15. What physiologic change occurs during heat exhaustion? a. Hemoconcentration occurs because of the loss of salt and water. b. Cramping of voluntary muscles occurs as a result of salt loss. c. Thermoregulation fails because of high core temperatures. d. Subcutaneous layers are damaged because of high core temperatures. ANS: A Heat exhaustion occurs when sufficient salt and water loss results in hemoconcentration. The other options do not accurately describe the physiologic changes that occur during heat exhaustion. PTS: 1 REF: Page 77 16. In hypoxic injury, sodium enters the cell and causes swelling because: a. The cell membrane permeability increases for sodium during periods of hypoxia. b. ATP is insufficient to maintain the pump that keeps sodium out of the cell. c. The lactic acid produced by the hypoxia binds with sodium in the cell. d. Sodium cannot be transportedNtoURthSeINcGelTlBm.CeOmMbrane during hypoxia. ANS: B In hypoxic injury, movement of fluid and ions into the cell is associated with acute failure of metabolism and a loss of ATP production. Normally, the presence of ATP and ATPase, the active-transport enzyme, maintains the pump that transports sodium ions out of the cell. In metabolic failure caused by hypoxia, reduced ATP and ATPase levels permit sodium to accumulate in the cell, whereas potassium diffuses outward. The other options do not accurately describe the cause of the swelling caused by hypoxia. PTS: 1 REF: Page 84 17. What is the most common site of lipid accumulation? a. Coronary arteries c. Liver b. Kidneys d. Subcutaneous tissue ANS: C Although lipids sometimes accumulate in heart and kidney cells, the most common site of intracellular lipid accumulation, or fatty change, is liver cells. Subcutaneous tissue is not a common site of lipid accumulation. PTS: 1 REF: Pages 84-85 18. What mechanisms occur in the liver cells as a result of lipid accumulation? a. Accumulation of lipids that obstruct the common bile duct, preventing flow of bile from the liver to the gallbladder b. Increased synthesis of triglycerides from fatty acids and decreased synthesis of apoproteins c. Increased binding of lipids with apoproteins to form lipoproteins d. Increased conversion of fatty acids to phospholipids ANS: B Lipid accumulation in liver cells occurs after cellular injury sets the following mechanisms in motion: increased synthesis of triglycerides from fatty acids (increases in the enzyme, -glycerophosphatase, which can accelerate triglyceride synthesis) and decreased synthesis of apoproteins (lipid-acceptor proteins). The other options do not accurately describe this event. PTS: 1 REF: Pages 84-85 19. Hemoprotein accumulations are a result of the excessive storage of: a. Iron, which is transferred from the cells to the bloodstream b. Hemoglobin, which is transferred from the bloodstream to the cells c. Albumin, which is transferred from the cells to the bloodstream d. Amino acids, which are transferred from the cells to the bloodstream ANS: A Excessive storage of iron, which is transferred to the cells from the bloodstream, causes hemoprotein accumulations in cells. Hemoglobin, albumin, or amino acids will not cause hemoprotein accumulations. PTS: 1 REF: Page 86 NURSINGTB.COM 20. Hemosiderosis is a condition that results in the excess of what substance being stored as hemosiderin in cells of many organs and tissues? a. Hemoglobin c. Iron b. Ferritin d. Transferrin ANS: C Hemosiderosis is a condition that occurs only when excess iron is stored as hemosiderin in the cells of many organs and tissues. PTS: 1 REF: Page 86 21. What is the cause of free calcium in the cytosol that damages cell membranes by uncontrolled enzyme activation? a. Activation of endonuclease interferes with the binding of calcium to protein. b. Activation of phospholipases, to which calcium normally binds, degrades the proteins. c. An influx of phosphate ions competes with calcium for binding to proteins. d. Depletion of ATP normally pumps calcium from the cell. ANS: D If abnormal direct damage occurs to membranes or ATP is depleted, then calcium increases in the cytosol. The other options do not accurately describe the cause of free calcium in cytosol to damage cell membranes. PTS: 1 REF: Pages 57-58 | Page 87 | Figure 2-24 22. What two types of hearing loss are associated with noise? a. Acoustic trauma and noise-induced c. High frequency and acoustic trauma b. High frequency and low frequency d. Noise-induced and low frequency ANS: A Two types of hearing loss are associated with noise: (1) acoustic trauma or instantaneous damage caused by a single sharply rising wave of sound (e.g., gunfire), and (2) noise-induced hearing loss, the more common type, which is the result of prolonged exposure to intense sound (e.g., noise associated with the workplace and leisure-time activities). The remaining options are not related to noise but rather to the amplitude of the sound. PTS: 1 REF: Page 83 23. What type of necrosis results from ischemia of neurons and glial cells? a. Coagulative c. Caseous b. Liquefactive d. Gangrene ANS: B Liquefactive necrosis commonly results from ischemic injury to neurons and glial cells in the brain. The other types of necrosis are not related to ischemic injuries in the brain. PTS: 1 REF: Page 90 24. What type of necrosis is often associated with pulmonary tuberculosis? a. Bacteriologic NURSINGTcB..COLMiquefactive b. Caseous d. Gangrenous ANS: B Caseous necrosis, which commonly results from tuberculous pulmonary infection, particularly Mycobacterium tuberculosis, is a combination of coagulative and liquefactive necrosis. The other types of necrosis are not observed in pulmonary tuberculosis. PTS: 1 REF: Page 90 25. What type of necrosis is associated with wet gangrene? a. Coagulative c. Caseous b. Liquefactive d. Gangrene ANS: B Wet gangrene develops only when neutrophils invade the site, causing liquefactive necrosis. PTS: 1 REF: Page 91 26. Current research supports the believe that, after heart muscle injury, the damage: a. Remains indefinitely because cardiac cells do not reproduce. b. Is repaired by newly matured cardiomyocytes. c. Gradually decreases in size as mitotic cell division occurs. d. Is replaced by hypertrophy of remaining cells. ANS: B The recent discovery that cardiac stem cells exist in the heart and differentiate into various cardiac cell lineages has profoundly changed the understanding of myocardial biology; it is now believed that bone marrow–derived cardiac stem cells or progenitor cells that have the ability to mature into cardiomyocytes may populate the heart after injury. The other options do not accurately describe the process that is believed to occur to address cardiac muscle damage. PTS: 1 REF: Page 52 | What's New box 27. After ovulation, the uterine endometrial cells divide under the influence of estrogen. This process is an example of hormonal: a. Hyperplasia c. Hypertrophy b. Dysplasia d. Anaplasia ANS: A Hormonal hyperplasia chiefly occurs in estrogen-dependent organs, such as the uterus and breast. After ovulation, for example, estrogen stimulates the endometrium to grow and thicken for reception of the fertilized ovum. The other options do not accurately describe the process identified in the question. PTS: 1 REF: Pages 51-53 28. The abnormal proliferation of cells in response to excessive hormonal stimulation is called: a. Dysplasia c. Hyperplasia b. Pathologic dysplasia d. Pathologic hyperplasia NURSINGTB.COM ANS: D Pathologic hyperplasia is the abnormal proliferation of normal cells and can occur as a response to excessive hormonal stimulation or the effects of growth factors on target cells (see Figure 2-4). The other options do not accurately identify the term for the results of excessive hormonal stimulation on cells. PTS: 1 REF: Page 53 29. Removal of part of the liver leads to the remaining liver cells undergoing compensatory: a. Atrophy c. Hyperplasia b. Metaplasia d. Dysplasia ANS: C Compensatory hyperplasia is an adaptive mechanism that enables certain organs to regenerate. For example, the removal of part of the liver leads to hyperplasia of the remaining liver cells (hepatocytes) to compensate for the loss. The other options do not accurately identify the compensatory process described in the question. PTS: 1 REF: Pages 52-53 30. What is the single most common cause of cellular injury? a. Hypoxic injury c. Infectious injury b. Chemical injury d. Genetic injury ANS: A Hypoxia, or lack of sufficient oxygen, is the single most common cause of cellular injury (see Figure 2-8). The other options are not a commonly observed as is the correct option. PTS: 1 REF: Page 56 31. During cell injury caused by hypoxia, sodium and water move into the cell because: a. Potassium moves out of the cell, and potassium and sodium are inversely related. b. The pump that transports sodium out of the cell cannot function because of a decrease in ATP levels. c. The osmotic pressure is increased, which pulls additional sodium across the cell membrane. d. Oxygen is not available to bind with sodium to maintain it outside of the cell. ANS: B A reduction in ATP levels causes the plasma membrane’s sodium-potassium (Na+–K+) pump and sodium-calcium exchange to fail, which leads to an intracellular accumulation of sodium and calcium and diffusion of potassium out of the cell. (The Na+–K+ pump is discussed in Chapter 1.) PTS: 1 REF: Page 57 32. In decompression sickness, emboli are formed by bubbles of: a. Oxygen c. Carbon monoxide b. Nitrogen d. Hydrogen ANS: B If water pressure is too rapidly redNuUcRedS,INthGeTgBa.CseOsMdissolved in blood bubble out of the solution, forming emboli. Oxygen is quickly redissolved, but nitrogen bubbles may persist and obstruct blood vessels. Ischemia, resulting from gas emboli, causes cellular hypoxia, particularly in the muscles, joints, and tendons, which are especially susceptible to changes in oxygen supply. The remaining options are not involved in the formation of decompression sickness emboli. PTS: 1 REF: Page 77 33. Which is an effect of ionizing radiation exposure? a. Respiratory distress c. DNA aberrations b. Sun intolerance d. Death ANS: C The effects of ionizing radiation may be acute or delayed. Acute effects of high doses, such as skin redness, skin damage, or chromosomal aberrations, occur within hours, days, or months. The delayed effects of low doses may not be evident for years. The other options are not commonly considered effects of radiation exposure. PTS: 1 REF: Pages 78-79 34. What is an example of compensatory hyperplasia? a. Hepatic cells increase cell division after part of the liver is excised. b. Skeletal muscle cells atrophy as a result of paralysis. c. The heart muscle enlarges as a result of hypertension. d. The size of the uterus increases during pregnancy. ANS: A Compensatory hyperplasia is an adaptive mechanism that enables certain organs to regenerate. For example, the removal of part of the liver leads to hyperplasia of the remaining liver cells (hepatocytes) to compensate for the loss. The other options do not accurately describe the term compensatory hyperplasia. PTS: 1 REF: Pages 52-53 35. It is true that nondividing cells are: a. Found in gastrointestinal lining c. Incapable of synthesizing DNA b. Affected by hyperplasia d. Affected by only hypertrophy ANS: A Gastrointestinal lining is made up of rapidly dividing cells. Hyperplasia and hypertrophy take place if the cells are capable of synthesizing DNA; however, only hypertrophy occurs in nondividing cells. PTS: 1 REF: Pages 51-53 36. Dysplasia refers to a(n): a. Abnormal increase in the number of a specific cell type b. True adaptive process at the cellular level c. Modification in the shape of a specific cell type d. Lack of oxygen at the cellular level ANS: C NURSINGTB.COM Dysplasia refers only to abnormal changes in the size, shape, and organization of mature cells. PTS: 1 REF: Pages 53-54 37. Current research has determined that chemical-induced cellular injury: a. Affects the permeability of the plasma membrane. b. Is often the result of the damage caused by reactive free radicals. c. Is rarely influenced by lipid peroxidation. d. Seldom involves the cell’s organelles. ANS: B Not all the mechanisms causing chemical-induced membrane destruction are known; however, the only two general mechanisms currently accepted include: (1) direct toxicity by combining with a molecular component of the cell membrane or organelles, and (2) reactive free radicals and lipid peroxidation. PTS: 1 REF: Pages 62-63 MULTIPLE RESPONSE 38. Which organs are affected by lead consumption? (Select all that apply.) a. Bones b. Muscles c. Pancreas d. Nerves e. Eyes ANS: A, D The only organ systems provided as options that are primarily affected by lead include the nervous system, bones, kidneys, teeth, cardiovascular, and reproductive and immune systems. PTS: 1 REF: Page 66 39. What effect does fetal alcohol syndrome have on newborns? (Select all that apply.) a. Failure of alveoli to open b. Cognitive impairment c. Incompetent semilunar values d. Esophageal stricture e. Facial anomalies ANS: B, E Fetal alcohol syndrome (FAS) can lead to growth restriction, cognitive impairment, facial anomalies, and ocular disturbances. The other options do not accurately describe the effects of FAS. NURSINGTB.COM PTS: 1 REF: Page 69 40. What organs are affected by the type of necrosis that results from either severe ischemia or chemical injury? (Select all that apply.) a. Lungs b. Brain c. Kidneys d. Muscles e. Heart ANS: C, E Coagulative necrosis, which occurs primarily in the kidneys, heart, and adrenal glands, is a common result of hypoxia from severe ischemia or hypoxia caused by chemical injury, especially the ingestion of mercuric chloride. The other options do not accurately identify organs affected by necrosis resulting from ischemia or chemical injury. PTS: 1 REF: Page 90 41. It is true that melanin is: (Select all that apply.) a. Rarely found in epithelial cells b. Found in cells called keratinocytes, which are present in the retina c. A factor in the prevention of certain types of cancer d. Most influential in managing the effects of short-term sunlight exposure e. Accumulated in specific cells found in the skin ANS: B, C, E Melanin accumulates in epithelial cells (keratinocytes) of the skin and retina and is an extremely important pigment because it protects the skin against long exposure to sunlight and is considered an essential factor in the prevention of skin cancer. PTS: 1 REF: Pages 85-86 42. Examples of adaptive cellular responses include: (Select all that apply.) a. Atrophy b. Dysplasia c. Hypertrophy d. Hyperplasia e. Metaplasia ANS: A, C, D, E Atrophy, hypertrophy, hyperplasia, and metaplasia are considered to be adaptive cellular responses. PTS: 1 REF: Page 50 43. Blunt force injuries would include a: (Select all that apply.) a. Bruise to the upper arm, resulting from a fall b. Simple tibia fracture sustained in a skiing accident c. Cut on the finger while slicing vegetables for a salad d. Spleen laceration caused by a punch during a physical fight e. Small caliber gunshot wound tNoUtRhSeIfNoGoTt Bw.ChOileMtarget shooting ANS: A, B, D Blunt force injuries are the result of tearing, shearing, or crushing types of injuries, resulting in bruises, fractures, and lacerations caused by blows or impacts. Sharp force injuries include cuts. Gunshot wounds require the penetration of the skin and muscle by a bullet. PTS: 1 REF: Page 72 | Table 2-6 44. Which statements are true regarding the effects of marijuana use? (Select all that apply.) a. Smoking the drug results in greater absorption that eating it. b. Heavy use can result in psychomotor impairments. c. Smoking four “joints” a day equals smoking approximately 20 cigarettes. d. Research does not support marijuana use as a factor in developing lung cancer. e. Fetal development appears to be unharmed by marijuana use. ANS: A, B, C With marijuana smoking, approximately 50% of the potent agents are absorbed through the lungs; when marijuana is ingested, however, only 10% is absorbed. With heavy marijuana use, the following adverse effects have been reported: (1) alterations of sensory perceptions, cognitive and psychomotor impairment (e.g., inability to judge time, speed, distance); (2) smoking three or four joints per day is similar to smoking 20 cigarettes per day, in relation to the frequency of chronic bronchitis and may contribute to lung cancer; (3) data from animal studies only, indicate reproductive changes that include reduced fertility, decreased sperm motility, and decreased circulatory testosterone; (4) fetal abnormalities including low birth weight and increased frequency of childhood leukemia; (5) increased frequency of infectious illness, which is thought to be the result of depressed cell-mediated and humoral immunity. PTS: 1 REF: Page 70 | Table 2-5 NURSINGTB.COM Chapter 3: The Cellular Environment: Fluids and Electrolytes, Acids and Bases MULTIPLE CHOICE 1. Infants are most susceptible to significant losses in total body water because of an infant’s: a. High body surface–to–body size ratio b. Slow metabolic rate c. Kidneys are not mature enough to counter fluid losses d. Inability to communicate adequately when he or she is thirsty ANS: C Renal mechanisms that regulate fluid and electrolyte conservation are often not mature enough to counter the losses; consequently, dehydration may rapidly develop. Infants can be susceptible to changes in total body water because of their high metabolic rate and the turnover of body fluids caused by their greater body surface area in proportion to their total body size. The inability to communicate their thirst is a problem only when they are poorly cared for. PTS: 1 REF: Page 104 2. Obesity creates a greater risk for dehydration in people because: a. Adipose cells contain little water because fat is water repelling. b. The metabolic rate of obese adults is slower than the rate of lean adults. c. The rate of urine output of obese adults is higher than the rate of output of lean adults. d. The thirst receptors of the hypNoUthRaSlaINmGuTsBd.CoOnMot function effectively. ANS: A The percentage of total body water (TBW) varies with the amount of body fat and age. Because fat is water repelling (hydrophobic), very little water is contained in adipose cells. Individuals with more body fat have proportionately less TBW and tend to be more susceptible to fluid imbalances that cause dehydration. PTS: 1 REF: Page 104 3. A patient’s blood gases reveal the following findings: pH, 7.3; bicarbonate (HCO3) 27 mEq/L; carbon dioxide (CO2), 58 mm Hg. What is the interpretation of these gases? a. Respiratory alkalosis c. Respiratory acidosis b. Metabolic acidosis d. Metabolic alkalosis ANS: C The values provided in this question characterize only acute uncompensated respiratory acidosis. PTS: 1 REF: Pages 129-130 4. Water movement between the intracellular fluid (ICF) compartment and the extracellular fluid (ECF) compartment is primarily a function of: a. Osmotic forces c. Antidiuretic hormone b. Plasma oncotic pressure d. Hydrostatic forces ANS: A The movement of water between the ICF and ECF compartments is primarily a function of osmotic forces. (Osmosis and other mechanisms of passive transport are discussed in Chapter 1.) PTS: 1 REF: Page 105 5. In addition to osmosis, what force is involved in the movement of water between the plasma and interstitial fluid spaces? a. Oncotic pressure c. Net filtration b. Buffering d. Hydrostatic pressure ANS: D Water moves between the plasma and interstitial fluid through the forces of only osmosis and hydrostatic pressure, which occur across the capillary membrane. Buffers are substances that can absorb excessive acid or base to minimize pH fluctuations. Net filtration is a term used to identify fluid movement in relationship to the Starling hypothesis. Oncotic pressure encourages water to cross the barrier of capillaries to enter the circulatory system. PTS: 1 REF: Page 105 6. Venous obstruction is a cause of edema because of an increase in which pressure? a. Capillary hydrostatic c. Capillary oncotic b. Interstitial hydrostatic d. Interstitial oncotic ANS: A Venous obstruction can increase thNeUhRySdINroGsTtaBt.iCcOpMressure of fluid in the capillaries enough to cause fluid to escape into the interstitial spaces. The remaining options are not causes of edema resulting from venous obstruction. PTS: 1 REF: Page 106 7. At the arterial end of capillaries, fluid moves from the intravascular space into the interstitial space because the: a. Interstitial hydrostatic pressure is higher than the capillary hydrostatic pressure. b. Capillary hydrostatic pressure is higher than the capillary oncotic pressure. c. Interstitial oncotic pressure is higher than the interstitial hydrostatic pressure. d. Capillary oncotic pressure is lower than the interstitial hydrostatic pressure. ANS: B At the arterial end of capillaries, fluid moves from the intravascular space into the interstitial because capillary hydrostatic pressure is higher than the capillary oncotic pressure. PTS: 1 REF: Page 105 8. Low plasma albumin causes edema as a result of a reduction in which pressure? a. Capillary hydrostatic c. Plasma oncotic b. Interstitial hydrostatic d. Interstitial oncotic ANS: C Losses or diminished production of plasma albumin is the only option that contributes to a decrease in plasma oncotic pressure. PTS: 1 REF: Pages 106-107 9. Secretion of antidiuretic hormone (ADH) and the perception of thirst are stimulated by a(n): a. Decrease in serum sodium c. Increase in glomerular filtration rate b. Increase in plasma osmolality d. Decrease in osmoreceptor stimulation ANS: B Secretion of ADH and the perception of thirst are primary factors in the regulation of water balance. Thirst is a sensation that stimulates water-drinking behavior. Thirst is experienced when water loss equals 2% of an individual’s body weight or when osmotic pressure increases. The other options do not accurately describe how ADH and the perception of thirst are related. PTS: 1 REF: Page 109 10. Thirst activates osmoreceptors by an increase in which blood plasma? a. Antidiuretic hormone c. Hydrostatic pressure b. Aldosterone d. Osmotic pressure ANS: D Thirst is experienced when water loss equals 2% of an individual’s body weight or when osmotic pressure increases. Dry mouth, hyperosmolality, and plasma volume depletion activate osmoreceptors (neurons located in the hypothalamus that are stimulated by increased osmotic pressure). The oNtUhRerSIoNpGtiToBn.sCdOoMnot accurately identify what increases to activate osmoreceptors. PTS: 1 REF: Page 109 11. It is true that natriuretic peptides: a. Decrease blood pressure and increase sodium and water excretion. b. Increase blood pressure and decrease sodium and water excretion. c. Increase heart rate and decrease potassium excretion. d. Decrease heart rate and increase potassium excretion. ANS: A Natriuretic peptides are hormones that include atrial natriuretic peptide (ANP) produced by the myocardial atria, brain natriuretic peptide (BNP) produced by the myocardial ventricles, and urodilatin within the kidney. Natriuretic peptides decrease blood pressure and increase sodium and water excretion. PTS: 1 REF: Page 109 12. When changes in total body water are accompanied by proportional changes in electrolytes, what type of alteration occurs? a. Isotonic c. Hypotonic b. Hypertonic d. Normotonic ANS: A Only isotonic alterations occur when proportional changes in electrolytes and water accompany changes in total body water . PTS: 1 REF: Pages 109-110 13. Which enzyme is secreted by the juxtaglomerular cells of the kidney when circulating blood volume is reduced? a. Angiotensin I c. Aldosterone b. Angiotensin II d. Renin ANS: D When circulating blood volume or blood pressure is reduced, renin, an enzyme secreted by the juxtaglomerular cells of the kidney, is released in response to sympathetic nerve stimulation and decreased perfusion of the renal vasculature. The other options are not released by the situation described in the question. PTS: 1 REF: Pages 108-109 14. What mechanism can cause hypernatremia? a. Syndrome of inappropriate antidiuretic hormone b. Hypersecretion of aldosterone c. Brief bouts of vomiting or diarrhea d. Excessive diuretic therapy ANS: B Hypernatremia occurs because of (1) inadequate free water intake, (2) inappropriate administration of hypertonic saline solution (e.g., sodium bicarbonate for treatment of acidosis during cardiac arrest), (3)NhUiRghSINsoGdTiuBm.COleMvels as a result of oversecretion of aldosterone (as in primary hyperaldosteronism), or (4) Cushing syndrome (caused by the excess secretion of adrenocorticotropic hormone [ACTH], which also causes increased secretion of aldosterone). The other options do not result in hypernatremia. PTS: 1 REF: Page 111 15. What causes the clinical manifestations of confusion, convulsions, cerebral hemorrhage, and coma in hypernatremia? a. High sodium in the blood vessels pulls water out of the brain cells into the blood vessels, causing brain cells to shrink. b. High sodium in the brain cells pulls water out of the blood vessels into the brain cells, causing them to swell. c. High sodium in the blood vessels pulls potassium out of the brain cells, which slows the synapses in the brain. d. High sodium in the blood vessels draws chloride into the brain cells followed by water, causing the brain cells to swell. ANS: A Hypertonic (hyperosmolar) imbalances result in an extracellular fluid concentration greater than 0.9% salt solution (e.g., water loss or solute gain); cells shrink in a hypertonic fluid (see Table 3-7). This shrinking of cells results in the symptoms described in the question. The other options do not accurately describe the cause of these symptoms as they relate to hypernatremia. PTS: 1 REF: Page 111 16. Vomiting-induced metabolic alkalosis, resulting in the loss of chloride, causes: a. Retained sodium to bind with the chloride b. Hydrogen to move into the cell and exchange with potassium to maintain cation balance c. Retention of bicarbonate to maintain the anion balance d. Hypoventilation to compensate for the metabolic alkalosis ANS: C When vomiting with the depletion of ECF and chloride (hypochloremic metabolic alkalosis) causes acid loss, renal compensation is not effective; the volume depletion and loss of electrolytes (sodium [Na+], potassium [K+], hydrogen [H+], chlorine [Cl–]) stimulate a paradoxic response by the kidneys. The kidneys increase sodium and bicarbonate reabsorption with the excretion of hydrogen. Bicarbonate is reabsorbed to maintain an anionic balance because the ECF chloride concentration is decreased. The other options do not accurately describe the mechanism that results from vomiting-induced metabolic alkalosis. PTS: 1 REF: Page 128 17. The pathophysiologic process of edema is related to which mechanism? a. Sodium depletion b. Decreased capillary hydrostatic pressure c. Increased plasma oncotic pressure d. Lymphatic obstruction ANS: D NURSINGTB.COM The pathophysiologic process of edema is related to an increase in the forces favoring fluid filtration from the capillaries or lymphatic channels into the tissues. The most common mechanisms are increased capillary hydrostatic pressure, decreased plasma oncotic pressure, increased capillary membrane permeability and lymphatic obstruction, and sodium retention. PTS: 1 REF: Page 105 18. Insulin is used to treat hyperkalemia because it: a. Stimulates sodium to be removed from the cell in exchange for potassium. b. Binds to potassium to remove it through the kidneys. c. Transports potassium from the blood to the cell along with glucose. d. Breaks down the chemical components of potassium, causing it to be no longer effective. ANS: C Insulin contributes to the regulation of plasma potassium levels by stimulating the Na+, potassium–adenosine triphosphatase (K+–ATPase) pump, thereby promoting the movement of potassium simultaneously into the liver and muscle cells with glucose transport after eating. The intracellular movement of potassium prevents an acute hyperkalemia related to food intake. The other options do not accurately describe how insulin is used to treat hyperkalemia. PTS: 1 REF: Page 114 19. A major determinant of the resting membrane potential necessary for the transmission of nerve impulses is the ratio between: a. Intracellular and extracellular Na+ c. Intracellular Na+ and extracellular K+ b. Intracellular and extracellular K+ d. Intracellular K+ and extracellular Na+ ANS: B The ratio of K+ in the ICF to K+ in the ECF is the major determinant of the resting membrane potential, which is necessary for the transmission and conduction of nerve impulses, for the maintenance of normal cardiac rhythms, and for the skeletal and smooth muscle contraction. This is not true of the other options. PTS: 1 REF: Page 114 20. During acidosis, the body compensates for the increase in serum hydrogen ions by shifting hydrogen ions into the cell in exchange for which electrolyte? a. Oxygen c. Potassium b. Sodium d. Magnesium ANS: C In states of acidosis, hydrogen ions shift into the cells in exchange for intracellular fluid potassium; hyperkalemia and acidosis therefore often occur together. This is not true of the other options. PTS: 1 REF: Page 117 | Pages 126-127 NURSINGTB.COM 21. Causes of hyperkalemia include: a. Hyperparathyroidism and malnutrition b. Vomiting and diarrhea c. Renal failure and Addison disease d. Hyperaldosteronism and Cushing disease ANS: C Hyperkalemia should be investigated when a history of renal disease, massive trauma, insulin deficiency, Addison disease, use of potassium salt substitutes, or metabolic acidosis exists. The other options are not known to be causes of hyperkalemia. PTS: 1 REF: Page 119 22. In hyperkalemia, what change occurs to the cells’ resting membrane potential? a. Hypopolarization c. Depolarization b. Hyperexcitability d. Repolarization ANS: A If extracellular potassium concentration increases without a significant change in intracellular potassium, then the resting membrane potential becomes more positive (i.e., changes from –90 to –80 mV) and the cell membrane is hypopolarized (i.e., the inside of the cell becomes less negative or partially depolarized [increase excitability]). PTS: 1 REF: Pages 117-118 23. The calcium and phosphate balance is influenced by which three substances? a. Parathyroid hormone, vasopressin, and vitamin D b. Parathyroid hormone, calcitonin, and vitamin D c. Thyroid hormone, vasopressin, and vitamin A d. Thyroid hormone, calcitonin, and vitamin A ANS: B Three hormones regulate calcium and phosphate balance: parathyroid hormone (PTH), vitamin D, and calcitonin. Vasopressin, thyroid hormone, and vitamin A do not influence calcium and phosphate balance. PTS: 1 REF: Page 119 24. It is true that Kussmaul respirations indicate: a. Anxiety is a cause of respiratory acidosis. b. A compensatory measure is needed to correct metabolic acidosis. c. Diabetic ketoacidosis is the cause of the metabolic acidosis. d. More oxygen is necessary to compensate for respiratory acidosis. ANS: B Deep, rapid respirations (Kussmaul respirations) are indicative of respiratory compensation for metabolic acidosis. The other options are not true. PTS: 1 REF: Page 128 25. Chvostek and Trousseau signs indicate which electrolyte imbalance? a. Hypokalemia NURSINGTcB..COHMypocalcemia b. Hyperkalemia d. Hypercalcemia ANS: C Two clinical signs of hypocalcemia are the Chvostek sign and Trousseau sign. These clinical signs are not indicative of any of the other options. PTS: 1 REF: Page 120 26. An excessive use of magnesium-containing antacids and aluminum-containing antacids can result in: a. Hypomagnesemia c. Hyponatremia b. Hypophosphatemia d. Hypokalemia ANS: B The most common causes of hypophosphatemia are intestinal malabsorption and increased renal excretion of phosphate. Inadequate absorption is associated with vitamin D deficiency, the use of magnesium and aluminum-containing antacids (which bind with phosphorus), long-term alcohol abuse, and malabsorption syndromes. The excessive use of such antacids will not result in the other options. PTS: 1 REF: Page 121 27. The most common cause of hypermagnesemia is: a. Hepatitis c. Trauma to the hypothalamus b. Renal failure d. Pancreatitis ANS: B Renal failure usually causes hypermagnesemia, in which magnesium concentration is greater than 2.5 mEq/L. Hypermagnesemia is not a result of the other options. PTS: 1 REF: Page 122 28. Physiologic pH is maintained at approximately 7.4 because bicarbonate (HCO3) and carbonic acid (H2CO3) exist in a ratio of: a. 20:1 c. 10:2 b. 1:20 d. 10:5 ANS: A The relationship between HCO3 and H2CO3 is usually expressed as a ratio. When the pH is 7.4, this ratio is 20:1 (HCO3:H2CO3). The other options do not accurately identify physiologic pH by the correct ratio of HCO3 and H2CO3. PTS: 1 REF: Page 124 29. Which arterial pH will initiate the formation of ammonium (NH4) from ammonia (NH3), referred to as academia, in the tubular lumen of the kidney? a. 7.25 c. 7.55 b. 7.35 d. 7.65 ANS: A Pathophysiologic changes in the concentration of hydrogen ion or base in the blood lead to acid-base imbalances. Acidemia is a state in which the pH of arterial blood is less than NURSINGTB.COM7.35. NH3 is produced from gl lial cell and diffuses to the tubular lumen, where it combines with H+ to form NH4. PTS: 1 REF: Page 126 30. Two thirds of the body’s water is found in its: a. Interstitial fluid spaces c. Intracellular fluid compartments b. Vascular system d. Intraocular fluids ANS: C Two thirds of the body’s water is in the intracellular fluid (ICF) compartment, and one third is in the extracellular fluid (ECF) compartment. The two main ECF compartments are the interstitial fluid and the intravascular fluid, which is the blood plasma. Other ECF compartments include the lymph and the transcellular fluids, such as the synovial, intestinal, biliary, hepatic, pancreatic, and cerebrospinal fluids; sweat; urine; and pleural, synovial, peritoneal, pericardial, and intraocular fluids. PTS: 1 REF: Pages 103-104 31. It is true that when insulin is administered: a. The Na+, K+–ATPase pump is turned off. b. Potassium is moved out of muscle cells. c. The liver increases its potassium levels. d. Glucose transport is impaired. ANS: C Insulin contributes to the regulation of plasma potassium levels by stimulating the Na+, K+–ATPase pump, thereby promoting the movement of potassium simultaneously into the liver and muscle cells with glucose transport after eating. The other options do not accurately describe the effect of insulin administration. PTS: 1 REF: Page 114 32. Increased capillary hydrostatic pressure results in edema because of: a. Losses or diminished production of plasma albumin b. Inflammation resulting from an immune response c. Blockage within the lymphatic channel system d. Sodium and water retention ANS: D Increased capillary hydrostatic pressure can result from venous obstruction or sodium and water retention. The other options do not accurately describe the cause of edema related to increased capillary hydrostatic pressure. PTS: 1 REF: Page 106 33. The existence of hyperkalemia is likely to result in which changes to a person’s electrocardiogram (ECG)? a. Flattened U waves c. Depressed ST segments b. Peaked T waves d. Peaked P waves ANS: B Observed ECG changes include pNeaUkReSdINTGwTaBv.CeOs,Mprolonged PR interval, and absent P wave with a widened QRS complex. The other options are not related to hyperkalemia. PTS: 1 REF: Page 118 MULTIPLE RESPONSE 34. Which groups are at risk for fluid imbalance? (Select all that apply.) a. Women b. Infants c. Men d. Obese persons e. Older adults ANS: B, D, E Kidney function, surface area, total body water, and the hydrophobic nature of fat cells all contribute to the increased risk for fluid imbalance among obese individuals, infants, and older adults. Gender alone is not a risk factor for fluid imbalance. PTS: 1 REF: Pages 104-105 35. Dehydration can cause which result? (Select all that apply.) a. Moist mucous membranes b. Weak pulses c. Tachycardia d. Polyuria e. Weight loss ANS: B, C, E Significant water deficit is demonstrated by symptoms of dehydration that include headache, thirst, dry skin and mucous membranes, elevated temperature, weight loss, and decreased or concentrated urine (with the exception of diabetes insipidus). Skin turgor may be normal or decreased. Symptoms of hypovolemia include tachycardia, weak pulses, and postural hypotension. PTS: 1 REF: Page 112 36. Causes of hypocalcemia include: (Select all that apply.) a. Repeated blood administration b. Pancreatitis c. Decreased reabsorption of calcium d. Hyperparathyroidism e. Kidney stones ANS: A, B Blood transfusions are a common cause of hypocalcemia because the citrate solution used in storing whole blood binds with calcium. Pancreatitis causes a release of lipases into soft-tissue spaces; consequently, the free fatty acids that are formed bind calcium, causing a decrease in ionized calcium. The other options are not recognized causes of hypocalcemia. PTS: 1 REF: Page 12N0URSINGTB.COM 37. The electrolyte imbalance called hyponatremia exhibits which clinical manifestations? (Select all that apply.) a. Headache b. Seizures c. Paranoia d. Confusion e. Lethargy ANS: A, B, D, E Behavioral and neurologic changes characteristic of hyponatremia include lethargy, headache, confusion, apprehension, seizures, and coma. Paranoia is not associated with hyponatremia. PTS: 1 REF: Page 113 38. The electrolyte imbalance hypercalcemia exhibits which clinical manifestations? (Select all that apply.) a. Diarrhea b. Calcium based kidney stones c. ECG showing narrow T waves d. Lethargy e. Bradycardia ANS: B, D, E Fatigue, weakness, lethargy, anorexia, nausea, and constipation are common. Behavioral changes may occur. Impaired renal function frequently develops, and kidney stones form as precipitates of calcium salts. A shortened QT segment and depressed widened T waves also may be observed on the ECG, with bradycardia and varying degrees of heart block. PTS: 1 REF: Pages 120-121 39. The electrolyte imbalance hypokalemia exhibits which clinical manifestations? (Select all that apply.) a. Paralytic ileus b. Sinus bradycardia c. Atrioventricular block d. Dry mucous membranes e. Tetany ANS: A, B, C A variety of dysrhythmias may occur, including sinus bradycardia, atrioventricular block, paroxysmal atrial tachycardia, and paralytic ileus. The other options are not related to hypokalemia. PTS: 1 REF: Pages 116-117 40. A third of the body’s fluid is contained in the extracellular interstitial fluid spaces that include: (Select all that apply.) a. Urine b. Intraocular fluids c. Lymph d. Blood plasma e. Sweat ANS: A, B, C, E NURSINGTB.COM Two thirds of the body’s water is in the intracelluarl fluid (ICF) compartment, and one third is in the extracellular fluid (ECF) compartments. The two main ECF compartments are the interstitial fluid and the intravascular fluid, such as the blood plasma. Interstitial ECF compartments include the lymph and the transcellular fluids, such as the synovial, intestinal, biliary, hepatic, pancreatic, and cerebrospinal fluids; sweat; urine; and pleural, synovial, peritoneal, pericardial, and intraocular fluids. PTS: 1 REF: Pages 103-104 41. An imbalance of potassium can produce which dysfunctions? (Select all that apply.) a. Weakness skeletal muscles b. Cardiac dysrhythmias c. Smooth muscle atony d. Visual impairment e. Hearing loss ANS: A, B, C Symptoms of hyperkalemia vary, but common characteristics are muscle weakness or paralysis and dysrhythmias with changes in the ECG. A wide range of metabolic dysfunctions may result from hypokalemia. Neuromuscular excitability is decreased, causing skeletal muscle weakness, smooth muscle atony, and cardiac dysrhythmias. PTS: 1 REF: Pages 116-118 42. Which statements regarding total body water (TBW) are true? (Select all that apply.) a. During childhood, TBW slowly decreases in relationship to body weight. b. Gender has no influence on TBW until old age. c. Men tend to have greater TBW as a result of their muscle mass. d. Estrogen plays a role in female TBW. e. Older adults experience a decrease in TBW as a result of decreased muscle mass. ANS: A, C, D, E During childhood, TBW slowly decreases to 60% to 65% of body weight. At adolescence, the percentage of TBW approaches adult proportions, and gender differences begin to appear. Men eventually have a greater percentage of body water as a function of increasing muscle mass. Women have more body fat and less muscle as a function of estrogens and therefore have less body water. With increasing age, the percentage of TBW declines further still. The decrease is caused, in part, by an increased amount of fat and a decreased amount of muscle, as well as by a reduced ability to regulate sodium and water balance. PTS: 1 REF: Pages 104-105 MATCHING NURSINGTB.COM Match the electrolytes with the corresponding descriptions. Terms may be used more than once. A. Sodium B. Chloride C. Potassium D. Magnesium E. Phosphate 43. Regulates osmolality in the extracellular fluid (ECF) space. 44. Is inversely related to HCO3 concentration. 45. Is a major determinant of resting membrane potential. 46. An intracellular metabolic form is adenosine triphosphate (ATP). 47. Changes in hydrogen ion concentration affect this electrolyte. 43. ANS: A PTS: 1 REF: Page 105 MSC: Sodium is the most abundant ECF ion and is responsible for the osmotic balance of the ECF space. Potassium maintains the osmotic balance of the ICF space. 44. ANS: B PTS: 1 REF: Page 109 MSC: Chloride levels are inversely related to HCO3 concentration. 45. ANS: C PTS: 1 REF: Page 114 MSC: The ratio of K+ in the ICF to K+ in the ECF is the major determinant of the resting membrane potential, which is necessary for the transmission and conduction of nerve impulses, for the maintenance of normal cardiac rhythms, and for skeletal and smooth muscle contraction. (Membrane transport and membrane potentials are discussed in Chapter 1.) 46. ANS: E PTS: 1 REF: Page 119 MSC: Phosphate acts as an intracellular and extracellular anion buffer in the regulation of acid-base balance; it provides energy for muscle contraction in the form of ATP. 47. ANS: C PTS: 1 REF: Page 117 MSC: In states of acidosis, hydrogen ions shift into the cells in exchange for ICF potassium; hyperkalemia and acidosis therefore often occur together. NURSINGTB.COM Chapter 4: Genes and Genetic Diseases MULTIPLE CHOICE 1. Inserting bone marrow cells into an individual who produces abnormal erythrocytes is an example of what type of therapy? a. Somatic cell c. Genetic engineering b. Germ cell d. Recombinant DNA ANS: A Gene therapy can be applied in two ways. The less controversial approach is somatic cell therapy, which consists of inserting normal genes into the cells of an individual who has a genetic disease. In this approach, a particular tissue, such as bone marrow cells that produce abnormal erythrocytes, is treated. The correct option is the only one that accurately identifies the therapy described in the question. PTS: 1 REF: Page 137 | Box 4-2 2. DNA replication requires the enzyme DNA polymerase to: a. Travel along the single DNA strand, adding the correct nucleotide to the new strand b. Move along the double strand of DNA to unwind the nucleotides of the double helix c. Hold the double strand apart while the correct nucleotides are added to the strand d. Transport the double strand of DNA from the nucleus to the cytoplasm for protein formation ANS: A NURSINGTB.COM The DNA polymerase enzyme travels along the single DNA strand, adding the correct nucleotides to the free end of the new strand (see Figure 4-2, B). The correct option is the only one that accurately describes the process involved in DNA replication using DNA polymerase. PTS: 1 REF: Page 137 3. Transcription is best defined as a process by which: a. DNA polymerase binds to the promoter site on ribonucleic acid (RNA). b. RNA directs the synthesis of polypeptides for protein synthesis. c. RNA is synthesized from a DNA template. d. A base pair substitution results in a mutation of the amino acid sequence. ANS: C Transcription is the process by which RNA is synthesized from a DNA template. The correct option is the only one that accurately defines the term transcription. PTS: 1 REF: Page 141 4. The purpose of a staining technique of chromosomes such as Giemsa is to: a. Permit the mitotic process to be followed and monitored for variations. b. Allow for the numbering of chromosomes and the identification of variations. c. Identify new somatic cells formed through mitosis and cytokinesis. d. Distinguish the sex chromosomes from the homologous chromosomes. ANS: B One of the most commonly used stains is Giemsa stain. By using banding techniques, chromosomes can be unambiguously numbered, and individual variation in chromosome composition can be studied. Missing or duplicated portions of chromosomes, which often result in serious diseases, also can be readily identified. The correct option is the only one that accurately describes the purpose of the Giemsa staining technique. PTS: 1 REF: Pages 142-143 5. An amniocentesis indicates a neural tube defect when an increase in which protein is evident? a. Chorionic c. Amniotic b. Alpha fetoprotein d. Embryonic ANS: B Other disorders can be detected with this procedure. These include most neural tube defects, which cause an elevation of alpha fetoprotein in the amniotic fluid, and hundreds of diseases caused by mutations of single genes. The correct option is the only one that accurately identifies the protein responsible for a neural tube defect. PTS: 1 REF: Page 136 | Box 4-1 6. An amniocentesis is recommended for pregnant women who: a. Have a history of chronic illness b. Have a family history of genetNicUdRiSsIoNrGdeTrBs.COM c. Have experienced in vitro fertilization d. Had a late menarche ANS: B Amniocentesis is recommended only for pregnancies known to have an elevated risk for a genetic disease or in women older than 30 to 35 years of age. The correct option is the only one that accurately describes a criterion for ordering an amniocentesis. PTS: 1 REF: Page 136 | Box 4-1 7. The most clinically useful technique for prenatal diagnosis of chromosomal abnormalities at 3 months’ (12 weeks’) gestation is: a. Gene mapping c. Amniocentesis b. Linkage analysis d. Chorionic villus sampling ANS: D Chorionic villus sampling consists of extracting a small amount of villous tissue directly from the chorion. This procedure can be performed at 10 weeks’ gestation and does not require in vitro culturing of cells; sufficient numbers are directly available in the extracted tissue. Thus the procedure allows prenatal diagnosis at approximately 3 months’ gestation rather than at nearly 5 months’ gestation. The correct option is the only one that accurately describes the most useful technique for prenatal diagnosis of chromosomal abnormalities. PTS: 1 REF: Page 136 | Box 4-1 8. The term for an error in which homologous chromosomes fail to separate during meiosis or mitosis is: a. Aneuploidy c. Polyploidy b. Nondisjunction d. Translocation ANS: B Aneuploidy is usually the result of nondisjunction, an error in which homologous chromosomes or sister chromatids fail to separate normally during meiosis or mitosis. The correct option is the only one that is used to describe an error in chromosomal separation during reproduction. PTS: 1 REF: Page 145 9. Which clinical manifestations would be expected for a child who has complete trisomy of the twenty-first chromosome? a. Widely spaced nipples, reduced carrying angle at the elbow, and sparse body hair b. An IQ of 25 to 70, low nasal bridge, protruding tongue, and flat, low-set ears c. High-pitched voice, tall stature, gynecomastia, and an IQ of 60 to 90 d. Circumoral cyanosis, edema of the feet, short stature, and mental slowness ANS: B Individuals with this disease are mentally retarded, with IQs usually ranging from 25 to 70. The facial appearance is distinctive and exhibits a low nasal bridge, epicanthal folds (which produce a superficially Asian appearance), protruding tongue, and flat, low-set ears. The correct option is the only one that accurately describes the clinical manifestations of the complete trisomy of the twenty-first chromosome. NURSINGTB.COM PTS: 1 REF: Pages 146-147 10. What is the most common cause of Down syndrome? a. Paternal nondisjunction c. Maternal nondisjunction b. Maternal translocations d. Paternal translocation ANS: C Nondisjunction during the formation of one of the parent’s gametes or during early embryonic development occurs in approximately 97% of infants born with Down syndrome. In approximately 90% to 95% of infants, the nondisjunction occurs in the formation of the mother’s egg cell. The correct option is the only one that accurately describes the most common cause of Down syndrome. PTS: 1 REF: Page 147 11. What syndrome, characterized by an absent homologous X chromosome with only a single X chromosome, exhibits features that include a short stature, widely spaced nipples, and webbed neck? a. Down c. Turner b. Cri du chat d. Klinefelter In Turner syndrome, a sex chromosome is missing, and the person’s total chromosome count is 45. Characteristic signs include short stature, female genitalia, webbed neck, shieldlike chest with underdeveloped breasts and widely spaced nipples, and imperfectly developed ovaries. The correct option is the only one that accurately describes the clinical manifestations described in the question. PTS: 1 REF: Page 147 12. A person with 47, XXY karyotype has the genetic disorder resulting in which syndrome? a. Turner c. Down b. Klinefelter d. Fragile X ANS: B A disorder in the chromosome (47, XXY karyotype) results in a disorder known as Klinefelter syndrome. The correct option is the only one that accurately describes a genetic disorder that exhibits the described genetic configuration. PTS: 1 REF: Page 148 13. What is the chromosomal variation that causes Klinefelter syndrome? a. Nondisjunction of the X chromosome in the father b. Translocation of the X chromosome in the mother c. Nondisjunction of X chromosome in the mother d. Translocation of the Y chromosome in the father ANS: C Nondisjunction of the X chromosomes in the mother causes Klinefelter syndrome in the majority of infants, and the frequeNnUcyRSoIfNtGhTeBd.iCsOorMder rises with maternal age. The correct option is the only one that accurately describes the chromosomal variation characteristic of Klinefelter syndrome. PTS: 1 REF: Page 148 14. What is the second most commonly recognized genetic cause of mental retardation? a. Down syndrome c. Klinefelter syndrome b. Fragile X syndrome d. Turner syndrome ANS: B The fragile X syndrome is the second most common genetic cause of mental retardation (after Down syndrome). The correct option is not observed with enough frequency to be recognized as the second most common cause of mental retardation. PTS: 1 REF: Page 151 15. What is the blood type of a person who is heterozygous, having A and B alleles as codominant? a. A c. O b. B d. AB ANS: D When the heterozygote is distinguishable from both homozygotes, the locus is said to exhibit codominance. An example is the ABO blood group, in which heterozygotes having the A and B alleles express both of them as A and B antigens on their red cells (forming blood group AB). The correct option is the only one that accurately describes codominance. PTS: 1 REF: Page 151 16. A couple has two children diagnosed with an autosomal dominant genetic disease. What is the probability that the next child will have the same genetic disease? a. One sixth c. One third b. One fourth d. One half ANS: D Affected heterozygous individuals transmit the trait to approximately one half of their children; however, because gamete transmission is subject to chance fluctuations, it is possible that all or none of the children of an affected parent may have the trait. Nevertheless, when large numbers of matings of this type are studied, the proportion of affected children closely approach one half. PTS: 1 REF: Pages 152-153 17. When a child inherits a disease that is autosomal recessive, it is inherited from: a. Father c. Both parents b. Mother d. Grandparent ANS: C In most cases of recessive diseaseN, bUoRtShINpaGrTeBn.tCsOoMf affected individuals are heterozygous carriers. PTS: 1 REF: Pages 155-156 18. People diagnosed with neurofibromatosis have varying degrees of the condition because of the genetic principle of: a. Penetrance c. Dominance b. Expressivity d. Recessiveness ANS: B Expressivity is the extent of variation in phenotype associated with a particular genotype. If expressivity of a disease is variable, then the penetrance may be complete but the severity of the disease can vary greatly. A well-known example of variable expressivity in an autosomal dominant disease is type 1 neurofibromatosis. The correct option is the only one that accurately describes the presence of varying degrees of symptomatologic characteristics. PTS: 1 REF: Page 154 19. Which genetic disease has been linked to a mutation of the tumor-suppressor gene? a. Hemochromatosis c. Familial breast cancer b. Retinoblastoma d. Hemophilia A ANS: B The gene responsible for retinoblastoma has been mapped to the long arm of chromosome 13, and its DNA sequence has been extensively studied. This gene is known as a tumor-suppressor gene; the normal function of its protein product is to regulate the cell cycle so that cells do not grow uncontrollably. The correct option is the only one that accurately identifies a disease resulting from a mutation of the tumor-suppressor gene. PTS: 1 REF: Page 154 20. Cystic fibrosis is caused by what type of gene? a. X-linked dominant c. Autosomal dominant b. X-linked recessive d. Autosomal recessive ANS: D The most common lethal autosomal recessive disease in white children, cystic fibrosis, occurs in approximately 1 in 2500 births. The correct option is the only one that accurately identifies the gene type responsible for cystic fibrosis. PTS: 1 REF: Page 154 21. Which is an important criterion for discerning autosomal recessive inheritance? a. Consanguinity is sometimes present. b. Females are affected more than males. c. The disease is observed in both the parents, as well as in the siblings. d. On average, one half of the offspring of the carrier will be affected. ANS: A Consanguinity is often an important characteristic of pedigrees for recessive diseases; relatives share a certain proportionNUofRgSIeNnGesTBre.CceOiMved from a common ancestor. The correct option is the only one that accurately identifies a required factor in autosomal recessive genetic inheritance. PTS: 1 REF: Page 155 22. Consanguinity refers to the mating of persons: a. Who are unrelated b. When one has an autosomal dominant disorder c. Having common family relations d. When one has a chromosomal abnormality ANS: C Consanguinity refers to the mating of two related individuals, and the offspring of such matings are said to be inbred. The correct option is the only one that accurately identifies consanguinity as it relates to human mating. PTS: 1 REF: Page 155 23. Males, having only one X chromosome (as is expected), are said to be: a. Homozygous c. Hemizygous b. Heterozygous d. Ambizygous ANS: C Males, having only one X chromosome, are said to be hemizygous for genes on this chromosome. The correct option is the only one that accurately identifies the term for males having only one X chromosome. PTS: 1 REF: Page 156 24. Males are more often affected by which type of genetic disease? a. Sex-linked dominant c. Sex-linked b. Sex-influenced d. Sex-linked recessive ANS: D Males are more frequently affected by X-linked recessive diseases, with the difference becoming more pronounced as the disease becomes rarer. The correct option is the only one that is a characteristic of a male-dominate disease. PTS: 1 REF: Page 156 25. An X-linked recessive disease can skip generations because: a. Females are hemizygous for the X chromosome. b. The disease can be transmitted through female carriers. c. Mothers cannot pass X-linked genes to their sons. d. These diseases need only one copy of the gene in females. ANS: B Skipped generations are often observed in X-linked recessive disease pedigrees because the gene can be transmitted through female carriers. Males are hemizygous for genes on the X chromosome. Fathers cannot pass X-linked genes to their sons. X-linked recessive diseases are observed significantlyNUmRoSrIeNoGfTteBn.CinOMmales than in females, because males need only one copy of the gene to express the disease. PTS: 1 REF: Page 157 26. The presence of a zygote having one chromosome with the normal complement of genes and one with a missing gene is characteristic of which genetic disorder? a. Cri du chat c. Klinefelter syndrome b. Down syndrome d. Turner syndrome ANS: A This description is only accurate for Cri du chat syndrome. PTS: 1 REF: Pages 148-149 27. A child with which genetic disorder has a characteristic cry? a. Down syndrome c. Turner syndrome b. Klinefelter syndrome d. Cri du chat ANS: D Cri du chat, which literally means “cry of the cat,” describes the characteristic cry of the affected child. The correct option is the only one with the characteristic cry. PTS: 1 REF: Page 148 28. Which statement is true regarding X-linked recessive conditions? a. Such diseases use males as phenotypical carriers. b. These conditions are passed from affected father to all of his female children. c. 25% of an affected individual’s grandsons will be affected. d. Cystic fibrosis is an example of such a condition. ANS: B X-linked recessive conditions are passed from an affected father to all of his daughters, who, as phenotypically normal carriers, transmit it to approximately one half of their sons, who are affected. Cystic fibrosis is an autosomal dominant disease. PTS: 1 REF: Page 158 29. DNA formation occurs in which of the cell’s structures? a. Nucleus c. Organelle b. Cytoplasm d. Membrane ANS: A DNA is formed and replicated only in the cell nucleus. PTS: 1 REF: Pages 135-141 30. What is the risk for the recurrence of autosomal dominant diseases? a. 10% c. 50% b. 30% d. 70% ANS: C The recurrence risk for autosomal dominant diseases is usually 50%. NURSINGTB.COM PTS: 1 REF: Pages 152-153 31. An individual’s genetic makeup is referred to as his or her: a. Phenotype c. Heterozygous locus b. Genotype d. Homozygous locus ANS: B An individual’s genotype is his or her genetic makeup. The correct option is the only one that accurately defines a person’s genetic makeup. PTS: 1 REF: Page 151 MULTIPLE RESPONSE 32. Which disorders have similar modes of inheritance? (Select all that apply.) a. Cri du chat syndrome b. Duchenne muscular dystrophy c. Polycystic kidney disease d. Down syndrome e. Becker muscular dystrophy ANS: B, E Becker muscular dystrophy and Duchenne muscular dystrophy are the only options that are X-linked recessive disorders. PTS: 1 REF: Page 158 33. The key to accurate DNA replication depends on which complementary base pairs? (Select all that apply.) a. Adenine with thymine b. Adenine with guanine c. Guanine with cytosine d. Cytosine with thymine e. Guanine with thymine ANS: A, C The consistent pairing of adenine with thymine and of guanine with cytosine, known as complementary base pairing, is the key to accurate DNA replication. The correct options are the only ones that accurately identify complementary base pairs. PTS: 1 REF: Page 137 34. Chromosomal abnormalities are the leading known cause of: (Select all that apply.) a. Mental illness b. Mental retardation c. Fetal miscarriage d. Cardiovascular disease e. Respiratory disorders ANS: B, C NURSINGTB.COM Chromosome abnormalities are the leading known cause of mental retardation and miscarriage. The correct options are the only ones that accurately identify disorders that related to chromosomal abnormalities. PTS: 1 REF: Page 143 35. Examples of prenatal diagnostic studies include: (Select all that apply.) a. Chorionic villus sampling (CVS) b. Amniocentesis c. Carrier screening d. Preimplantation genetic diagnosis (PGD) e. Drug-sensitivity testing ANS: A, B, D Prenatal testing is conducted before or during the pregnancy but not once labor begins. Such diagnostic studies include amniocenteses, CVS, and PGD. The correct options are the only ones that are exclusively used during the prenatal period. PTS: 1 REF: Page 136 | Box 4-1 36. The advantage derived from human genome sequencing on genetic disorders focuses on: (Select all that apply.) a. Identification of the mutated gene b. Reversal of the mutation c. Diagnosis of the existing disorder d. Appropriate treatment e. Prevention of the disorder ANS: A, C, D The complete human genome sequence will facilitate gene identification, diagnosis, and disease treatment. The reversal of a mutation is not possible. The mapping has no effect on preventing a genetic disorder. PTS: 1 REF: Page 160 MATCHING Match the genetic terms with the corresponding diseases. Terms may be used more than once. A. Autosomal dominant B. Autosomal recessive C. X-linked dominant D. X-linked recessive 37. Cystic fibrosis 38. Duchenne muscular dystrophy 39. Sickle cell disease 40. Huntington disease 37. ANS: B PTS: 1 REF: Page 154 MSC: Is an important example of anNaUuRtoSsIoNmGaTlBre.CceOsMsive disease is cystic fibrosis. 38. ANS: D PTS: 1 REF: Page 158 MSC: Is an important example of an X-linked recessive disease is Duchenne muscular dystrophy. 39. ANS: B PTS: 1 REF: Page 136 | Box 4-1 MSC: Is an important example of an autosomal recessive disease is sickle cell disease. 40. ANS: A PTS: 1 REF: Page 154 MSC: Another well-known autosomal dominant disease is Huntington disease. Chapter 5: Genes, Environment-Lifestyle, and Common Diseases MULTIPLE CHOICE 1. The data reporting that sickle cell disease affects approximately 1 in 600 American blacks is an example of which concept? a. Incidence c. Ratio b. Prevalence d. Risk ANS: B Prevalence rate is the proportion of the population affected by a disease at a specific point in time. Thus both the incidence rate and the length of the survival period in affected individuals determine prevalence. The incidence rate is the number of new cases of a disease reported during a specific period (typically 1 year), divided by the number of individuals in the population. A numerical expression representing a part of a larger whole or proportion is considered a ratio. Any factor that increases the chance of disease or injury is considered a risk. PTS: 1 REF: Page 165 2. The ratio of the disease among the exposed population to the disease rate in an unexposed population is referred to as what type of risk? a. Attributable c. Causal b. Contingency d. Relative ANS: D A common measure of the effect oNfUaRsSpINecGiTfiBc.CriOskMfactor is the relative risk. Assuming a factor is the cause of a disease, attributable risk is the amount of risk that is due to that factor. A future event or circumstance that is possible but cannot be predicted with certainty is a contingency risk. The probability of the outcome is termed a causal risk factor. PTS: 1 REF: Page 165 3. Empirical risks for most multifactorial diseases are based on: a. Chromosomal testing c. Liability thresholds b. Direct observation d. Relative risks ANS: B For most multifactorial diseases, empirical risks (i.e., risks based on direct observation of data) have been derived. The other options are not the basis for determining the empirical risk of most multifactorial diseases. PTS: 1 REF: Page 167 4. What is the cause of familial hypercholesterolemia (FH)? a. Diet high in saturated fats b. Increased production of cholesterol by the liver c. Reduction in the number of low-density lipoprotein (LDL) receptors on cell surfaces d. Abnormal function of lipoprotein receptors circulating in the blood ANS: C A reduction in the number of functional LDL receptors on cell surfaces causes FH. Lacking the normal number of LDL receptors, cellular cholesterol uptake is reduced and circulating cholesterol levels increase (see Box 5-3). The other options are not the basis for developing familial FH. PTS: 1 REF: Page 173 | Box 5-3 5. Which risk factor for hypertension is influenced by genetic factors and lifestyle? a. Sodium intake c. Psychosocial stress b. Physical inactivity d. Obesity ANS: D The most important environmental risk factors for hypertension are increased sodium intake, decreased physical activity, psychosocial stress, and obesity. However, obesity is, itself, influenced by genes and the environment. PTS: 1 REF: Page 172 6. What percentage of all cases of breast cancer are identified as an autosomal dominant form? a. 5 c. 15 b. 10 d. 20 ANS: A An autosomal dominant form of breast cancer accounts for approximately 5% of breast cancer cases in the United States. NURSINGTB.COM PTS: 1 REF: Page 172 | Page 174 7. When a woman has one first-degree relative with breast cancer, her risk of developing breast cancer is how many times greater? a. 2 c. 6 b. 3 d. 10 ANS: A If a woman has one affected first-degree relative, her risk of developing breast cancer doubles. PTS: 1 REF: Page 172 8. Adoption studies have shown that the offspring of an alcoholic parent when raised by nonalcoholic parents have what amount of an increased risk of developing alcoholism? a. Twofold c. Fourfold b. Threefold d. Tenfold ANS: C Adoption studies have shown that the offspring of an alcoholic parent, even when raised by nonalcoholic parents, have a fourfold increased risk of developing the disorder. PTS: 1 REF: Page 179 9. Studies have identified several genes that play a role in the prevention of obesity by affecting what? a. Regulation of appetite c. Absorption of fat b. Metabolizing of fat d. Altering the sense of satiety ANS: A Clinical trials using recombinant leptin have demonstrated moderate weight loss in a subset of obese individuals. In addition, leptin participates in important interactions with other components of appetite control, such as neuropeptide Y and –melanocyte-stimulating hormone and its receptor, the melanocortin-4 receptor (MC4R). Currently, no research supports the other options as being genetically regulated. PTS: 1 REF: Page 178 10. The BRCA1 and BRCA2 mutations increase the risk of which cancer in women? a. Ovarian c. Uterine b. Lung d. Pancreatic ANS: A BRCA1 mutations increase the risk of ovarian cancer among women (20% to 50% lifetime risk), and BRCA2 mutations also confer an increased risk of ovarian cancer (10% to 20% lifetime prevalence). BRCA1 and BRCA2 mutations are not currently believed to be linked with risks of lung, uterine, or pancreatic cancers. PTS: 1 REF: Page 174 11. Blood pressure variations are assoNcUiaRteSdINwGiTthB:.COM a. ß1-adrenergic receptors to increase heart rate b. The release of an antidiuretic hormone (ADH) that increases water reabsorption c. The renin-angiotensin system’s effect on vasoconstriction d. Serum bradykinin, causing vasodilation ANS: C Significant research is now focused on specific components that may influence blood pressure variation, such as the renin-angiotensin system (involved in sodium reabsorption and vasoconstriction). The other options are not related to hypertension. PTS: 1 REF: Page 172 12. The two most important risk factors for type 2 diabetes are: a. Autoantibodies and human leukocyte antigen associations b. Autoantibodies and obesity c. Obesity and positive family history d. HLA associations and positive family history ANS: C The two most important risk factors for type 2 diabetes are positive family history and obesity. The other options are not believed to be important risk factors for this form of diabetes. PTS: 1 REF: Pages 177-178 13. A major characteristic of type 1 diabetes mellitus is that there is: a. Partial insulin secretion c. Insulin resistance b. An autoimmune cause factor d. Obesity as a common risk factor ANS: B A strong association between type 1 diabetes and the presence of several human leukocyte antigen (HLA) class II alleles indicate that type 1 diabetes mellitus is an autoimmune disease. The remaining options are associated with type 2 diabetes. PTS: 1 REF: Page 174 14. Obesity acts as an important risk factor for type 2 diabetes mellitus by: a. Reducing the amount of insulin the pancreas produces b. Increasing the resistance to insulin by cells c. Obstructing the outflow of insulin from the pancreas d. Stimulating the liver to increase glucose production ANS: B People with type 2 diabetes mellitus suffer from insulin resistance (i.e., their cells have difficulty using insulin). The other options are not associated with the effect of obesity regarding insulin production. PTS: 1 REF: Pages 177-178 15. Traits caused by the combined effects of multiple genes are referred to by which term? a. Polygenic c. Modifiable b. Multifocal ANS: A NURSINGTdB..COInMvoluntary Traits in which variation is thought to be caused by the combined effects of multiple genes are polygenic, meaning many genes. Multifocal means relating to or arising from many points. Modifiable refers to the changeability of something. Involuntary suggests being out of the control of someone or something. PTS: 1 REF: Page 165 16. Regarding type 2 diabetes, obesity is considered to be what type of risk? a. Genetic c. Relative b. Empirical d. Modifiable ANS: D Obesity is a modifiable risk factor for many diseases including heart disease, stroke, hypertension, and type 2 diabetes. The other terms do not apply. PTS: 1 REF: Pages 177-178 17. Which disease form is identified on the basis of empirical risk observation? a. Polygenic c. Monozygotic b. Multifactorial d. Genetic ANS: B For most multifactorial diseases, empirical risks (i.e., risks based on direct observation of data) have been derived. Traits in which variation is thought to be caused by the combined effects of multiple genes are polygenic. Monozygotic is a term that refers to identical twins. Genetic refers to issues related to genes and their influence on the body. PTS: 1 REF: Page 167 18. The number of persons living with a specific disease at a specific point in time is referred to by which term? a. Relativity c. Prevalence b. Survivability d. Incidence ANS: C The prevalence rate is the proportion of the population affected by a disease at a specific point in time. Thus both the incidence rate and the length of the survival period in affected individuals determine prevalence. The description in the question does not relate to any of the other options. PTS: 1 REF: Page 164 19. Which type of cancer is said to aggregate among families? a. Breast c. Skin b. Lung d. Brain ANS: A Breast cancer appears to aggregate strongly in families. The other cancers are not believed to be familial in nature. NURSINGTB.COM PTS: 1 REF: Page 172 20. Which dietary lifestyle choice has been associated with a decreased risk for developing colon cancer? a. Increased consumption of dairy produces b. Increased consumption of foods containing vitamin C c. Decreased consumption of foods high in fat d. Decreased consumption of artificial food coloring ANS: C A low-fat, high-fiber diet is thought to decrease the risk of colon cancer. PTS: 1 REF: Page 165 21. It is currently believed that the risk for developing Alzheimer disease: a. Is not directly related to genetic predisposition. b. Is higher among men than it is among women. c. Occurs less among Hispanics than in Asians. d. Doubles among those with an affected first-degree relative. ANS: D The risk of developing Alzheimer disease doubles in individuals who have an affected first-degree relative. The other statements are not true. PTS: 1 REF: Page 178 22. The number of new cases of a disease reported during a specific period divided by the number of individuals in the population is defined as which characteristic of a disease? a. Prevalence rate c. Relative risk b. Incidence rate d. Frequency ANS: B The incidence rate is the number of new cases of a disease reported during a specific period (typically 1 year) divided by the number of individuals in the population. The description provided in the question does not describe any of the other options. PTS: 1 REF: Page 164 MULTIPLE RESPONSE 23. Cancers that cluster strongly in families include: (Select all that apply.) a. Breast b. Colon c. Ovarian d. Lung e. Brain ANS: A, B, C Although breast, ovarian, and colon cancers have shown a strong familial tendency, lung and brain cancers have not. PTS: 1 REF: Page 17N2URSINGTB.COM 24. Which genes are responsible for an autosomal dominant form of breast cancer? (Select all that apply.) a. LCAT b. CHK1 c. CHK2 d. BRCA1 e. BRCA2 ANS: D, E Women who inherit a mutation in BRCA1 or BRCA2 experience a 50% to 80% lifetime risk of developing breast cancer. The other options do not carry this risk. PTS: 1 REF: Page 174 25. Lifestyle modifications that affect health-related risk factors include: (Select all that apply.) a. Diet b. Exercise c. Education d. Finances e. Stress reduction ANS: A, B, E Lifestyle modification (e.g., diet, exercise, stress reduction) can often reduce health risks significantly. Education and finances have not been shown to have an effect on health in a way that involves lifestyle modifications. PTS: 1 REF: Page 180 26. What factors are typically considered when assessing an individual’s risk for developing such common diseases as hypertension? (Select all that apply.) a. Age b. Diet c. Exercise habits d. Family history e. Spiritual beliefs ANS: A, B, C, D Many factors influence the risk of acquiring a common disease, such as cancer, diabetes, or hypertension. These factors can include age, gender, diet, exercise, and family history of the disease. Current research does not support a connection between spiritual beliefs and the development of hypertension. PTS: 1 REF: Page 165 27. Examples of multifactorial diseases associated with adults include: (Select all that apply.) a. Breast cancer b. Coronary heart disease c. Emphysema d. Diabetes mellitus e. Schizophrenia ANS: A, B, D, E NURSINGTB.COM Multifactorial diseases in adults include coronary heart disease, hypertension, breast cancer, colon cancer, diabetes mellitus, obesity, Alzheimer disease, alcoholism, schizophrenia, and bipolar affective disorder. Emphysema is not considered multifactorial. PTS: 1 REF: Pages 171-180 MATCHING Match the terms with the corresponding descriptions. A. Incidence rate effect of multiple genes B. Gene-environment interaction C. Prevalence rate at which some diseases occur D. Obesity E. Polygenic F. Empirical risk G. Relative risk 28. Traits caused by the combined effects of multiple genes 29. Number of persons living with the disease 30. Yields an increased risk for some diseases 31. Modifiable risk factor for many diseases 32. Number of new cases (persons) detected with the disease 33. Measure of the effect of a specific risk factor 34. Risks based on direct observation of data 28. ANS: E PTS: 1 REF: Page 165 MSC: Traits in which variation is thought to be caused by the combined effects of multiple genes are polygenic (i.e., many genes). 29. ANS: C PTS: 1 REF: Page 164 MSC: The prevalence rate is the proportion of the population affected by a disease at a specific point in time. Thus both the incidence rate and the length of the survival period in affected individuals determine prevalence. 30. ANS: B PTS: 1 REF: Page 171 MSC: In some cases, a genetic predisposition may interact with an environmental factor to increase the risk of disease to a significantly higher level than would either factor acting alone. A good example of a gene-environment interaction is given by 1-antitrypsin deficiency, a genetic condition that causes pulmonary emphysema and is greatly exacerbated by cigarette smoking (see Box 5-2). 31. ANS: D PTS: 1 REF: Pages 177-178 MSC: Obesity is a modifiable risk factor for many diseases including heart disease, stroke, hypertension, and type 2 diabetes. 32. ANS: A PTS: 1 REF: Page 164 MSC: The incidence rate is the number of new cases of a disease reported during a specific period (typically 1 year), divided by the number of individuals in the population. 33. ANS: G PTS: 1 REF: Page 165 MSC: A common measure of the effect of a specific risk factor is the relative risk. 34. ANS: F PTS: 1 REF: Page 167 MSC: For most multifactorial diseases, empirical risks (i.e., risks based on direct observation of data) have been derived. NURSINGTB.COM Chapter 6: Epigenetics and Disease MULTIPLE CHOICE 1. What genetic process is likely responsible for the occurrence of asthma in only one of a pair of identical twins? a. Epigenetic modifications c. Transgenerational inheritance b. Genomic imprinting d. Methylation ANS: A Epigenetic modifications can cause individuals with the same deoxyribonucleic acid (DNA) sequences (such as identical twins) to have different disease profiles. The correct option is the only one that accurately identifies the genetic process likely responsible for the occurrence of asthma in only one of a pair of twins. PTS: 1 REF: Page 183 | Page 185 2. Prader-Willi syndrome causes a chromosomal defect that is: a. Initiated by postnatal exposure to a virus b. Inherited from the father c. Related to maternal alcohol abuse d. Transferred from mother to child ANS: B Prader-Willi syndrome can be caused by a 4 Mb deletion of chromosome 15q when inherited from the father. The other options do not accurately identify the reason for the chromosomal damage that causesNPUraRdSeIrN-GWTiBll.iCsOyMndrome. PTS: 1 REF: Pages 187-188 3. A malfunction in DNA methylation can lead to: a. Hypothyroidism c. Cancer b. Blindness d. Diabetes mellitus ANS: C Aberrant methylation can lead to silencing of tumor-suppressor genes in the development of cancer. No research supports a connection between hypothyroidism, blindness, or diabetes mellitus to a malfunctioning of DNA methylation. PTS: 1 REF: Page 183 | Page 186 4. Which statement is true regarding the embryonic development of stem cells? a. They are already differentiated. b. They are referred to as housekeeping genes. c. They already demonstrate DNA sequencing. d. They are said to be pluripotent. ANS: D Early in embryonic development, all cells of the embryo have the potential to become any type of cell in the fetus or adult. These embryonic stem cells are said to be pluripotent. The remaining options are not true statements regarding embryonic stem cell development. PTS: 1 REF: Page 184 5. When microRNA (miRNA) are methylated their messenger RNA (mRNA) targets are over-expressed, the resulting effect on existing cancer would be: a. Cell death c. Remission b. Metastasis d. Relapse ANS: B When miRNA genes are methylated, their mRNA targets are over-expressed, and this over-expression has been associated with metastasis. of the described effect on mRNA targets on existing cancer does not result in any of the other options. PTS: 1 REF: Page 186 6. The difference between DNA sequence mutations and epigenetic modifications is: a. DNA sequence mutations can be directly altered. b. Leukemia is a result of only DNA sequence mutation. c. Epigenetic modifications can be reversed. d. No known drug therapies are available for epigenetic modifications. ANS: C Unlike DNA sequence mutations, which cannot be directly altered, epigenetic modifications can be reversed. The remaining options are not true statements regarding the difference between DNA sequence mutations and epigenetic modifications. PTS: 1 REF: Page 186 NURSINGTB.COM 7. Which term refers to the silenced gene of a gene pair? a. Activated c. Mutated b. Altered d. Imprinted ANS: D Gene silencing, a process during which genes are predictably silenced, depending on which parent transmits them, is known as imprinting; the transcriptionally silenced genes are then said to be imprinted. The remaining options do not accurately identify this process. PTS: 1 REF: Page 187 8. The shape of the face of a child diagnosed with Russell-Silver syndrome is likely to be: a. Round c. Triangular b. Square d. Elongated ANS: C Growth retardation, proportionate short stature, leg-length discrepancy, and a small, triangular-shaped face characterizes Russell-Silver syndrome. The other face shapes are not characteristic of Russell- Silver syndrome. PTS: 1 REF: Page 188 9. Genes responsible for the maintenance of all cells are referred to as: a. Universal c. Housekeeping b. Managerial d. Executive ANS: C A small percentage of genes, termed housekeeping genes, are necessary for the function and maintenance of all cells. The remaining options do not accurately refer to these cells. PTS: 1 REF: Page 184 10. What is the belief regarding twins who adopt dramatically different lifestyles? a. They may experience very different aging processes. b. They will retain very similar methylation patterns. c. They will experience identical phenotypes throughout their lifespans. d. They may never demonstrate similar DNA sequences of their somatic cells. ANS: A Twins with significant lifestyle differences (e.g., smoking versus nonsmoking), accumulate large numbers of differences in their methylation patterns. The twins, despite having identical DNA sequences, become more and more different as a result of epigenetic changes, which in turn affect the expression of genes. These results, along with findings generated in animal studies, suggest that changes in epigenetic patterns may be an important part of the aging process. They will not experience identical phenotypes throughout their lifespans. PTS: 1 REF: Page 185 11. Hypomethylation and the resulting effect on oncogenes result in a(an): a. Decrease in the activity of theNoUnRcoSgINeGneT,Bt.hCuOsMsuppressing cancer development b. Deactivation of MLH1 to halt DNA repair c. Increase in tumor progression from benign to malignant d. Over-expression of microRNA, resulting in tumorigenesis ANS: C Tumor cells typically exhibit hypomethylation (decreased methylation), which can increase the activity of oncogenes. Hypomethylation increases as tumors progress from benign neoplasms to malignancy. Only the correct option accurately describes hypomethylation and its resulting effects. PTS: 1 REF: Page 186 12. When a chromosome lacking 4 Mb is inherited from the mother, the child is at risk for developing which syndrome? a. Prader-Willi c. Beckwith-Wiedemann b. Angelman d. Russell-Silver ANS: B This anomaly illustrates the inheritance pattern of Angelman syndrome, which can be caused by a 4 Mb deletion of chromosome 15q when inherited from the mother. The anomaly is not the cause of any of the other options. PTS: 1 REF: Page 187 MULTIPLE RESPONSE 13. A child’s diagnosis of Beckwith-Wiedemann syndrome is supported by the presence of: (Select all that apply.) a. An omphalocele b. Neonatal hypoglycemia c. Creased earlobes d. Low birth weight e. A large tongue ANS: A, B, C, E Beckwith-Wiedemann syndrome is usually identifiable at birth because the child exhibits a large size for gestational age, neonatal hypoglycemia, a large tongue, creases on the earlobe, and omphalocele. PTS: 1 REF: Page 188 14. A diagnosis of Angelman syndrome in a child is supported by which assessment findings? (Select all that apply.) a. Small feet and hands b. Profound cognitive dysfunction c. Obesity d. Ataxic gait e. History of seizures ANS: B, D, E NURSINGTB.COM A child diagnosed with Angelman syndrome demonstrates a characteristic posture, bouts of uncontrolled laughter, severe mental retardation, seizures, and an ataxic gait. PTS: 1 REF: Page 187 Chapter 7: Innate Immunity: Inflammation MULTIPLE CHOICE 1. Which action is a purpose of the inflammatory process? a. To provide specific responses toward antigens b. To lyse cell membranes of microorganisms c. To prevent infection of the injured tissue d. To create immunity against subsequent tissue injury ANS: C If the epithelial barrier is damaged, then a highly efficient local and systemic response (inflammation) is mobilized to limit the extent of damage, to protect against infection, and to initiate the repair of damaged tissue. The other options do not accurately identify a purpose of the inflammatory process. PTS: 1 REF: Page 191 2. How do surfactant proteins A through D provide innate resistance? a. Initiate the complement cascade. c. Secrete mucus. b. Promote phagocytosis. d. Synthesize lysosomes. ANS: B The lung produces and secretes a family of glycoproteins, collectins, which includes surfactant proteins A through D and mannose-binding lectin. Collectin binding facilitates macrophages to recognize the microorganism, enhancing macrophage attachment, phagocytosis, and killing. The othNerUoRpStIiNoGnTs Bd.oCOnoMt accurately identify how surfactant proteins provide innate resistance. PTS: 1 REF: Page 194 3. Which secretion is a first line of defense against pathogen invasion that involves antibacterial and antifungal fatty acids, as well as lactic acid? a. Optic tears c. Sweat gland perspiration b. Oral saliva d. Sebaceous gland sebum ANS: D Sebaceous glands in the skin secrete sebum that is made up of antibacterial and antifungal fatty acids and lactic acid that provide the first-line barrier against pathogen invasion. PTS: 1 REF: Pages 192-193 4. Which bacterium grows in the intestines after prolonged antibiotic therapy? a. Lactobacillus c. Clostridium difficile b. Candida albicans d. Helicobacter pylori ANS: C Prolonged antibiotic treatment can alter the normal intestinal flora, decreasing its protective activity and leading to the overgrowth of other microorganisms, such as the yeast C. albicans or the bacterium C. difficile. The other options do not accurately identify intestinal bacterium whose growth is a result of prolonged antibiotic therapy. PTS: 1 REF: Page 194 5. What causes the edema that occurs during the inflammatory process? a. Vasodilation of blood vessels c. Endothelial cell contraction b. Increased capillary permeability d. Emigration of neutrophils ANS: B The increased flow and capillary permeability result in a leakage of plasma from the vessels, causing swelling (edema) in the surrounding tissue and is solely responsible for inflammation-induced edema. PTS: 1 REF: Page 195 6. What process causes heat and redness to occur during the inflammatory process? a. Vasodilation of blood vessels c. Decreased capillary permeability b. Platelet aggregation d. Endothelial cell contraction ANS: A The increased blood flow as a result of vasodilation and increasing concentration of red cells at the site of inflammation cause locally increased warmth and redness. The other options do not accurately identify the process that results in inflammatory redness and heat. PTS: 1 REF: Page 195 7. Activation of the classical pathway begins with: a. Viruses NURSINGTcB..COMMast cells b. Antigen-antibody complexes d. Macrophages ANS: B Activation of the classical pathway begins only with the activation of protein C1 and is preceded by the formation of a complex between an antigen and an antibody to form an antigen-antibody complex (immune complex) (see Chapter 8). PTS: 1 REF: Page 197 8. What plasma protein system forms a fibrinous meshwork at an inflamed site? a. Complement c. Kinin b. Coagulation d. Fibrinolysis ANS: B The coagulation (clotting) system is a group of plasma proteins that form a fibrinous meshwork at an injured or inflamed site. This protein system (1) prevents the spread of infection to adjacent tissues, (2) traps microorganisms and foreign bodies at the site of inflammation for removal by infiltrating cells (e.g., neutrophils and macrophages), (3) forms a clot that stops the bleeding, and (4) provides a framework for future repair and healing. The other options do not accurately identify such a protein system. PTS: 1 REF: Page 199 | Page 201 9. Which component of the plasma protein system tags pathogenic microorganisms for destruction by neutrophils and macrophages? a. Complement cascade c. Kinin system b. Coagulation system d. Immune system ANS: A C3b (a component of the complement cascade) adheres to the surface of a pathogenic microorganism and serves as an efficient opsonin. Opsonins are molecules that tag microorganisms for destruction by cells of the inflammatory system, primarily neutrophils and macrophages. The other options do not accurately identify a component capable of tagging pathogenic microorganisms. PTS: 1 REF: Pages 197-199 10. What is the vascular effect of histamine released from mast cells? a. Platelet adhesion c. Vasodilation b. Initiation of the clotting cascade d. Increased endothelial adhesiveness ANS: C C2b affects smooth muscle, causing vasodilation and increased vascular permeability. C3a, C5a, and, to a limited extent, C4a are anaphylatoxins; that is, they induce rapid mast cell degranulation (i.e., release of granular contents) and the release of histamine, causing vasodilation and increased capillary permeability. The other options do not accurately describe the vascular effect of histamine released from mast cells? PTS: 1 REF: Pages 198-199 11. What is an outcome of the compleNmUeRnStINcaGsTcBad.CeO?M a. Activation of the clotting cascade b. Prevention of the spread of infection to adjacent tissues c. Inactivation of chemical mediators such as histamine d. Lysis of bacterial cell membranes ANS: D The complement cascade can be activated by at least three different means, and its products have four functions: (1) anaphylatoxic activity, resulting in mast cell degranulation, (2) leukocyte chemotaxis, (3) opsonization, and (4) cell lysis. The other options do not accurately describe an outcome of the complement cascade. PTS: 1 REF: Page 199 12. The function of opsonization related to the complement cascade is to: a. Tag of pathogenic microorganisms for destruction by neutrophils and macrophages. b. Process pathogenic microorganisms so that activated lymphocytes can be created for acquired immunity. c. Destroy glycoprotein cell membranes of pathogenic microorganisms. d. Promote anaphylatoxic activity, resulting in mast cell degranulation. ANS: A C3b adheres to the surface of a pathogenic microorganism and serves as an efficient opsonin. Opsonins are molecules that tag microorganisms for destruction by cells of the inflammatory system, primarily neutrophils and macrophages. The other options do not accurately describe the function of opsonization related to the complement cascade. PTS: 1 REF: Page 199 13. In the coagulation (clotting) cascade, the intrinsic and the extrinsic pathways converge at which factor? a. XII c. X b. VII d. V ANS: C The coagulation cascade consists of the extrinsic and intrinsic pathways that converge only at factor X. PTS: 1 REF: Page 201 14. Which chemical interacts among all plasma protein systems by degrading blood clots, activating complement, and activating the Hageman factor? a. Kallikrein c. Bradykinin b. Histamine d. Plasmin ANS: D Only plasmin regulates clot formation by degrading fibrin and fibrinogen, and it can activate the complement cascade through components C1, C3, and C5. Plasmin can activate the plasma kinin cascade by activating the Hageman factor (factor XII) and producing prekallikrein activator.NURSINGTB.COM PTS: 1 REF: Page 201 15. The chemotactic factor affects the inflammatory process by: a. Causing vasodilation around the inflamed area b. Stimulating smooth muscle contraction in the inflamed area c. Directing leukocytes to the inflamed area d. Producing edema around the inflamed area ANS: C Two chemotactic factors, neutrophil chemotactic factor (NCF) and eosinophil chemotactic factor of anaphylaxis (ECF-A), are released during mast cell degranulation. NCF attracts neutrophils (a type of leukocytes), and ECF-A attracts eosinophils to the site of inflammation. The other options do not accurately describe the affect chemotactic factors have on the inflammatory process. PTS: 1 REF: Page 207 16. What affect does the process of histamine binding to the histamine-2 (H2) receptor have on inflammation? a. Inhibition c. Acceleration b. Activation d. Termination ANS: A Binding of histamine to the H1 receptor is essentially proinflammatory; that is, it promotes inflammation. On the other hand, binding histamine to the H2 receptor is generally antiinflammatory because it results in the suppression of leukocyte function. The other options do not accurately describe the affect histamine binding to the H2 receptor has on inflammation. PTS: 1 REF: Page 206 17. Frequently when H1 and H2 receptors are located on the same cells, they act in what fashion? a. Synergistically c. Antagonistically b. Additively d. Agonistically ANS: C Both types of receptors are distributed among many different cells and are often present on the same cells and may act in an antagonistic fashion. For instance, neutrophils express both types of receptors, with stimulation of H1 receptors resulting in the augmentation of neutrophil chemotaxis and H2 stimulation resulting in its inhibition. The other options do not accurately describe the relationship between H1 and H2 receptors. PTS: 1 REF: Page 207 18. Some older adults have impaired inflammation and wound healing because of which problem? a. Circulatory system cannot adequately perfuse tissues. b. Complement and chemotaxis are deficient. c. Underlying chronic illness(es) exists. d. Number of mast cells is insuffNicUieRnStI.NGTB.COM ANS: C In some cases, impaired healing is not directly associated with aging, in general, but can instead be linked to a chronic illness such as cardiovascular disease or diabetes mellitus. The other problems are not related to the aging process. PTS: 1 REF: Page 220 19. Which chemical mediator derived from mast cells retracts endothelial cells to increase vascular permeability and to cause leukocyte adhesion to endothelial cells? a. Leukotrienes c. Platelet-activating factor b. Prostaglandin E d. Bradykinin ANS: C The biologic activity of platelet-activating factor is virtually identical to that of leukotrienes; namely, it causes endothelial cell retraction to increase vascular permeability, leukocyte adhesion to endothelial cells, and platelet activation. The other options do not accurately identify the chemical mediator derived from the process described in the question. PTS: 1 REF: Page 207 20. What is the inflammatory effect of nitric oxide (NO)? a. Increases capillary permeability, and causes pain. b. Increases neutrophil chemotaxis and platelet aggregation. c. Causes smooth muscle contraction and fever. d. Decreases mast cell function, and decreases platelet aggregation. ANS: D Effects of NO on inflammation include vasodilation by inducing relaxation of vascular smooth muscle, a response that is local and short lived, and by suppressing mast cell function, as well as platelet adhesion and aggregation. The other options do not accurately identify the effect of NO on the process of inflammation. PTS: 1 REF: Page 209 21. What is the correct sequence in phagocytosis? a. Engulfment, recognition, fusion, destruction b. Fusion, engulfment, recognition, destruction c. Recognition, engulfment, fusion, destruction d. Engulfment, fusion, recognition, destruction ANS: C Once the phagocytic cell enters the inflammatory site, the only correct sequence of phagocytosis involves the following steps: (1) opsonization, or recognition, of the target and adherence of the phagocyte to it; (2) engulfment, or ingestion or endocytosis, and the formation of phagosome; (3) fusion with lysosomal granules within the phagocyte (phagolysosome); and (4) destruction of the target. PTS: 1 REF: Pages 210-211 22. When considering white blood celNlUdRifSfIeNreGnTtBia.lCsO, aMcute inflammatory reactions are related to elevations of which leukocyte? a. Monocytes c. Neutrophils b. Eosinophils d. Basophils ANS: C Only neutrophils are the predominant phagocytes in the early inflammatory site, arriving within 6 to 12 hours after the initial injury, they ingest (phagocytose) bacteria, dead cells, and cellular debris at the inflammatory site. PTS: 1 REF: Pages 208-209 23. In the later stages of an inflammatory response, which phagocytic cell is predominant? a. Neutrophils c. Chemokines b. Monocytes d. Eosinophils ANS: B Only monocytes and macrophages perform many of the same functions as neutrophils but for a longer time and in a later stage of the inflammatory response. PTS: 1 REF: Page 209 | Page 212 24. In regulating vascular mediators released from mast cells, the role of eosinophils is to release: a. Arylsulfatase B, which stimulates the formation of B lymphocytes b. Histaminase, which limits the effects of histamine during acute inflammation c. Lysosomal enzymes, which activate mast cell degranulation during acute inflammation d. Immunoglobulin E, which defends the body against parasites ANS: B Eosinophil lysosomes contain several enzymes that degrade vasoactive molecules, thereby controlling the vascular effects of inflammation. These enzymes include histaminase, which mediates the degradation of histamine, and arylsulfatase B, which mediates the degradation of some of the lipid-derived mediators produced by mast cells. The other options do not accurately describe the role of eosinophils. PTS: 1 REF: Page 209 25. What is the role of a natural killer (NK) cells? a. Initiation of the complement cascade b. Elimination of malignant cells c. Binding tightly to antigens d. Proliferation after immunization with antigen ANS: B The main function of NK cells is to recognize and eliminate cells infected with viruses, although they are also somewhat effective at eliminating other abnormal host cells, specifically cancer cells. The other options do not accurately identify the role of a NK cell. PTS: 1 REF: Page 213 26. Which cytokine is produced and rNelUeRasSeINd GfrToBm.CvOiMrally infected host cells? a. IL-1 c. TNF- b. IL-10 d. IFN- ANS: D Only interferons (IFNs) are produced and released by virally infected cells in response to viral double-stranded ribonucleic acid (RNA). IFN- and IFN- induce the production of antiviral proteins, thereby conferring protection on uninfected cells. IFN- or IFN- is released from virally infected cells and attaches to a receptor on a neighboring cell. IFNs also enhance the efficiency of developing an acquired immune response. PTS: 1 REF: Pages 204-205 27. IFN- is secreted from which cells? a. Virally infected cells c. Macrophages b. Bacterial infected cells d. Mast cells ANS: C Different kinds of interferons (IFNs) are produced by different types of cells—macrophages are the primary producers of both IFN- and IFN-. The other options do not accurately identify cells secreted by IFN-. PTS: 1 REF: Pages 204-205 28. Which manifestation of inflammation is systemic? a. Formation of exudates c. Redness and heat b. Fever and leukocytosis d. Pain and edema ANS: B The only three primary systemic changes associated with the acute inflammatory response are fever, leukocytosis (a transient increase in circulating leukocytes), and increased levels in circulating plasma proteins. PTS: 1 REF: Page 213 29. The acute inflammatory response is characterized by fever that is produced by the hypothalamus being affected by: a. Endogenous pyrogens c. Antigen-antibody complexes b. Bacterial endotoxin d. Exogenous pyrogens ANS: A Fever-causing cytokines are known as endogenous pyrogens. These pyrogens act directly on the hypothalamus, which is the portion of the brain that controls the body’s thermostat. The other options do not accurately describe the cause of fever related to the effects on the hypothalamus. PTS: 1 REF: Page 213 30. What occurs during the process of repair after tissue damage? a. Nonfunctioning scar tissue replaces destroyed tissue. b. Regeneration occurs; the original tissue is replaced. c. Resolution occurs; tissue is regenerated. d. Epithelialization replaces destNroUyReSdINtiGssTuBe..COM ANS: A Repair is the replacement of destroyed tissue with scar tissue. Scar tissue is primarily made up of collagen, which fills in the lesion and restores tensile strength but cannot carry out the physiologic functions of the destroyed tissue. The other options do not accurately describe the process of repair after tissue damage. PTS: 1 REF: Pages 215-216 31. The role of fibroblasts during the reconstructive phase of wound healing is to: a. Generate new capillaries from vascular endothelial cells around the wound. b. Establish connections between neighboring cells and contract their fibers. c. Synthesize and secrete collagen and the connective tissue proteins. d. Provide enzymes that débride the wound bed of dead cells. ANS: C Fibroblasts are the most important cells during the reconstructive phase of wound healing because they synthesize and secrete collagen and other connective tissue proteins. Macrophage-derived transforming growth factor–beta (TGF-ß) stimulates fibroblasts. The other options do not accurately describe the role of fibroblasts in the reconstructive phase of wound healing. PTS: 1 REF: Pages 216-218 32. A keloid is the result of which dysfunctional wound healing response? a. Epithelialization c. Collagen matrix assembly b. Contraction d. Maturation ANS: C An imbalance between collagen synthesis and collagen degradation, during which synthesis is increased relative to degradation, causes both keloids and hypertrophic scars. The other options are not involved in keloids production. PTS: 1 REF: Page 219 33. Which solution is best to use when cleaning a wound that is healing by epithelialization? a. Normal saline c. Hydrogen peroxide b. Povidone-iodine d. Dakin solution ANS: A Normal saline is the most innocuous solution that can be used to cleanse or irrigate a wound that is primarily healing by epithelialization and is the only correct answer for this question. PTS: 1 REF: Page 220 34. Many neonates have a transient depressed inflammatory response as a result of which condition? a. The circulatory system is too immature to perfuse tissues adequately. b. Complement and chemotaxis are deficient. c. Mast cells are lacking. d. The respiratory system is too iNmUmRaStIuNrGeTtBo.CdeOlMiver oxygen to tissues. ANS: B Neonates commonly have transiently depressed inflammatory and immune function partially as a result of a deficiency in components of the alternative pathway. For example, neutrophils and perhaps monocytes may not be capable of efficient chemotaxis. The other options do not accurately explain the common cause of a transient depressed inflammatory response in neonates. PTS: 1 REF: Page 220 35. During phagocytosis, what is occurring during the step referred to as opsonization? a. Phagocytes recognize and adhere to the bacteria. b. Microorganisms are ingested. c. Microorganisms are killed and digested. d. An intracellular phagocytic vacuole is formed. ANS: A During phagocytosis, opsonization involves only the recognition and adherence of phagocytes to bacteria. PTS: 1 REF: Page 210 36. Fusion is the step in phagocytosis during which: a. Microorganisms are killed and digested. b. An intracellular phagocytic vacuole is formed. c. Lysosomal granules enter the phagocyte. d. Microorganisms are ingested. ANS: C Fusion occurs with lysosomal granules entering the phagocyte (phagolysosome). The remaining options do not accurately describe fusion as a step in phagocytosis. PTS: 1 REF: Page 210 37. During the process of endocytosis, the phagosome step results in: a. Microorganisms are ingested. b. Microorganisms are killed and digested. c. Phagocytes recognize and adhere to bacteria. d. An intracellular phagocytic vacuole is formed. ANS: D Small pseudopods that extend from the plasma membrane and surround the adherent microorganism, forming an intracellular phagocytic vacuole or phagosome, carry out engulfment (endocytosis). The membrane that surrounds the phagosome consists of inverted plasma membrane. After the formation of the phagosome, lysosomes converge, fuse with the phagosome, and discharge their contents, creating a phagolysosome. PTS: 1 REF: Pages 210-211 38. When cellular damage occurs and regeneration is minor with no significant complications, the process of returning the cells to preinjury function is referred to as: a. Restoration NURSINGTcB..CORMegrowth b. Resolution d. Replacement ANS: B If damage is minor with no complications and destroyed tissues are capable of regeneration, then returning the injured tissues to an approximation of their original structure and physiologic function is possible. This restoration is called resolution. The other terms are not used to describe this process. PTS: 1 REF: Page 215 39. Newborns often have deficiencies in collectin-like proteins, making them more susceptible to what type of infection? a. Cardiac c. Respiratory b. Urinary d. Gastrointestinal ANS: C Neonates may also be deficient in some of the collectins and collectin-like proteins. This deficiency is especially true of preterm neonates. Some preterm infants with respiratory distress syndrome are deficient in at least one collectin, which negatively affects its innate defense against respiratory infections. The other options are not necessarily related to collectin deficiencies. PTS: 1 REF: Page 220 40. Which cell is the body’s primary defense against parasite invasion? a. Eosinophil c. T lymphocytes b. Neutrophils d. B lymphocytes ANS: A Eosinophils serve as the body’s primary defense against parasites. T lymphocytes and B lymphocytes are involved in acquired immunity. Neutrophils are the predominant phagocytes in the early inflammatory site. PTS: 1 REF: Page 209 MULTIPLE RESPONSE 41. Which chemical mediators induce pain during an inflammatory response? (Select all that apply.) a. Prostaglandins b. Leukotrienes c. Tryptase d. Phospholipase e. Bradykinin ANS: A, E The only chemical mediators that induce pain during an inflammatory response are the prostaglandins and bradykinin. PTS: 1 REF: Page 201 42. Sebaceous glands protect the bodyNUfrRoSmINinGfTeBc.tCioOnMby secreting: (Select all that apply.) a. Antibacterial fatty acids b. Antifungal fatty acids c. Ascorbic acid d. Lactic acid e. Hydrochloric acid ANS: A, B, D Sebaceous glands secrete only antibacterial and antifungal fatty acids and lactic acid. PTS: 1 REF: Pages 192-193 43. Which body fluid has the ability to attack the cell walls of gram-positive bacteria? (Select all that apply.) a. Perspiration b. Semen c. Tears d. Saliva e. Urine ANS: A, C, D Only perspiration, tears, and saliva contain an enzyme (lysozyme) that attacks the cell walls of gram-positive bacteria. PTS: 1 REF: Page 193 44. The main function of NK cells includes: (Select all that apply.) a. Recognizing virus-infected cells b. Eliminating virus-infected cells c. Recognizing bacteria-infected cells d. Eliminating bacteria-infected cells e. Eliminating previously identified cancer cells ANS: A, B, E The main functions of NK cells are recognizing and eliminating cells infected with viruses, not bacteria. They are also somewhat effective at eliminating other abnormal host cells, specifically cancer cells. PTS: 1 REF: Page 213 45. Normal bacterial flora found in the intestines produce vitamin K to assist in the absorption of which of the following? (Select all that apply.) a. Calcium b. Fatty acids c. Large polysaccharides d. Iron e. Magnesium ANS: A, D, E The flora’s production of vitamin K is needed to absorb various ions, such as calcium, iron, and magnesium. Normal intestinal flora is responsible for digesting fatty acids, large polysaccharides, and other dietary substance, but such digestion is not reliant on vitamin K. NURSINGTB.COM PTS: 1 REF: Page 194 46. An individual’s acquired immunity is dependent on the function of which cells? (Select all that apply.) a. T lymphocytes b. B lymphocytes c. Macrophages d. Opsonins e. Neutrophils ANS: A, B, C T lymphocytes, B lymphocytes, macrophages, and dendritic cells are involved in acquired immunity. Opsonins are molecules that tag microorganisms for destruction by cells of the inflammatory system; these cells are primarily neutrophils. PTS: 1 REF: Page 192 | Table 7-1 47. An example of a pathogen capable of surviving and even multiplying inside a macrophage is known as: (Select all that apply.) a. Mycobacterium tuberculosis (tuberculosis) b. Mycobacterium leprae (leprosy) c. Salmonella typhi (typhoid fever) d. Clostridium difficile e. Brucella abortus (brucellosis) ANS: A, B, C, E Several bacteria are resistant to killing by granulocytes and can even survive inside macrophages. Microorganisms such as M. tuberculosis (tuberculosis), M. leprae (leprosy), S. typhi (typhoid fever), and B. abortus (brucellosis) can remain dormant or even multiply inside the phagolysosomes of macrophages. C. difficile is said to be resistant to antibiotics, making it difficult to control. PTS: 1 REF: Pages 212-213 48. An older adult is particularly susceptible to infections of which body parts? (Select all that apply.) a. Lungs b. Skin c. Liver d. Eyes e. Bladder ANS: A, B, E Older adults have increased susceptibility to bacterial infections of the lungs, urinary tract, and skin. Other infections may occur but on an individualized basis. PTS: 1 REF: Page 220 MATCHING NURSINGTB.COM Match each step of phagocytosis with its function. A. Opsonization B. Engulfment C. Phagosome D. Fusion E. Destruction 49. Microorganisms are ingested. 50. Microorganisms are killed and digested. 51. Phagocytes gain enhanced recognition and adherence of bacteria. 52. Lysosomal granules enter the phagocyte. 53. Intracellular phagocytic vacuole is formed. 49. ANS: B PTS: 1 REF: Pages 210-211 MSC: Engulfment is the ingestion of phagosomes. 50. ANS: MSC: E PTS: 1 REF: Pages 210-211 Destruction is the step during which microorganisms are killed and digested. 51. ANS: MSC: A PTS: 1 REF: Pages 210-211 Opsonization is the recognition and adherence of phagocytes to bacteria. 52. ANS: MSC: D PTS: 1 REF: Pages 210-211 Fusion occurs with lysosomal granules within the phagocyte (phagolysosome). 53. ANS: MSC: C PTS: 1 REF: Pages 210-211 Small pseudopods that extend from the plasma membrane and surround the adherent microorganism, forming an intracellular phagocytic vacuole or phagosome, carry out engulfment (endocytosis). NURSINGTB.COM Chapter 8: Adaptive Immunity MULTIPLE CHOICE 1. Which primary characteristic is unique for the immune response? a. The immune response is similar each time it is activated. b. The immune response is specific to the antigen that initiates it. c. The response to a specific pathogen is short term. d. The response is innate, rather than acquired. ANS: B Unlike inflammation, which is nonspecifically activated by cellular damage and pathogenic microorganisms, the immune response is primarily designed to afford long-term specific protection (i.e., immunity) against particular invading microorganisms; that is, it has a memory function. The other options are not unique characteristics of the immune response. PTS: 1 REF: Page 225 2. In which structure does B lymphocytes mature and undergo changes that commit them to becoming B cells? a. Thymus gland c. Bone marrow b. Regional lymph nodes d. Spleen ANS: C B lymphocytes mature and become B cells in specialized (primary) lymphoid organs—the thymus gland for T cells and the bNoUnRe SmINaGrrToBw.CfOorMB cells. Neither regional lymph nodes nor the spleen are involved in changing B lymphocytes into B cells. PTS: 1 REF: Page 225 3. What is the term for the process during which lymphoid stem cells migrate and change into either immunocompetent T cells or immunocompetent B cells? a. Clonal diversity c. Clonal selection b. Clonal differentiation d. Clonal competence ANS: A The process is called the generation of clonal diversity and occurs in specialized (primary) lymphoid organs—the thymus gland for T cells and the bone marrow for B cells. The other options do not accurately identify the process described in the question. PTS: 1 REF: Page 225 4. Which type of immunity is produced by an individual after either natural exposure to the antigen or after immunization against the antigen? a. Passive-acquired immunity c. Passive-innate immunity b. Active-acquired immunity d. Active-innate immunity ANS: B An individual produces active-acquired immunity (active immunity) after natural exposure to an antigen or after immunization, whereas passive-acquired immunity (passive immunity) does not involve the host’s immune response at all. The innate immune system, also known as nonspecific immune system and the first line of defense, is composed of the cells and mechanisms that defend the host from infection by other organisms in a nonspecific manner, which means that the cells of the innate system recognize and respond to pathogens in a generic way. PTS: 1 REF: Page 227 5. What type of immunity is produced when an immunoglobulin crosses the placenta? a. Passive-acquired immunity c. Passive-innate immunity b. Active-acquired immunity d. Active-innate immunity ANS: A Passive-acquired immunity (passive immunity) does not involve the host’s immune response at all. Rather, passive immunity occurs when preformed antibodies or T lymphocytes are transferred from a donor to the recipient. This transfer can occur naturally, as in the passage of maternal antibodies across the placenta to the fetus, or artificially, as in a clinic using immunotherapy for a specific disease. The remaining options do not produce immunity via immunoglobulin transfer across the placenta. PTS: 1 REF: Page 227 6. The portion of the antigen that is configured for recognition and binding is referred to as what type of determinant? a. Immunotope c. Epitope b. Paratope ANS: C NURSINGTdB..COAMntigenitope The precise portion of the antigen that is configured for recognition and binding is called its antigenic determinant or epitope. The other options are not used to identify this portion of the antigen. PTS: 1 REF: Page 228 7. Which characteristic is the most important determinant of immunogenicity when considering the antigen? a. Size c. Complexity b. Foreignness d. Quantity ANS: B Foremost among the criteria for immunogenicity is the antigen’s foreignness. A self-antigen that fulfills all of these criteria except foreignness does not normally elicit an immune response. Thus most individuals are tolerant of their own antigens. The immune system has an exquisite ability to distinguish self (self-antigens) from nonself (foreign antigens). The other options are considered when determining immunogenicity. PTS: 1 REF: Page 229 8. When antigens are administered to produce immunity, why are different routes of administration considered? a. Different routes allow the speed of onset of the antigen to be varied, with the intravenous route being the fastest. b. Some individuals appear to be unable to respond to an antigen by a specific route, thus requiring the availability of different routes for the same antigen. c. Antigen-presenting cells are highly specialized and thus require stimulation by different routes. d. Each route stimulates a different lymphocyte-containing tissue, resulting in different types of cellular and humoral immunity. ANS: D Each route preferentially stimulates a different set of lymphocyte-containing (lymphoid) tissues and therefore results in the induction of different types of cell-mediated or humoral immune responses. The other options do not accurately explain the use of different routes when administering antigens to produce immunity. PTS: 1 REF: Page 229 9. The functions of the major histocompatibility complex (MHC) and CD1 molecules are alike because both: a. Are antigen-presenting molecules. b. Bind antigens to antibodies. c. Secrete interleukins during the immune process. d. Are capable of activating cytotoxic T lymphocytes. ANS: A MHC and CD1 molecules are both antigen presenting molecules (APCs). The other options do not accurately describe the common function of these cells. NURSINGTB.COM PTS: 1 REF: Page 233 | Page 235 10. Where are antibodies produced? a. Helper T lymphocytes c. Plasma cells b. Thymus gland d. Bone marrow ANS: C An antibody or immunoglobulin is a serum glycoprotein produced only by plasma cells in response to a challenge by an immunogen. PTS: 1 REF: Page 229 11. Which immunoglobulin is present in blood, saliva, breast milk, and respiratory secretions? a. IgA c. IgG b. IgE d. IgM ANS: A IgA can be divided into two subclasses, IgA1 and IgA2. IgA1 molecules are predominantly found in the blood, whereas IgA2 is the predominant class of antibody found in normal body secretions. The other options are not found in the substances identified in the question. PTS: 1 REF: Page 229 12. Which antibody initially indicates a typical primary immune response? a. IgG c. IgA b. IgM d. IgE ANS: B Typically, IgM is produced first (primary immune response), followed by IgG against the same antigen. The other options are not involved. PTS: 1 REF: Page 247 13. An individual is more susceptible to infections of mucous membranes when he or she has a seriously low level of which immunoglobulin antibody? a. IgG c. IgA b. IgM d. IgE ANS: C The IgA molecules found in bodily secretions are dimers anchored together through a J-chain and secretory piece. This secretory piece is attached to the IgA antibodies inside the mucosal epithelial cells and may function to protect these immunoglobulin antibodies against degradation by enzymes also found in the secretions, thus decreasing the risk of infections in the mucous membrane. The other options do not accurately identify the immunoglobulin antibody involved in mucous membrane infections. PTS: 1 REF: Page 229 14. The B-cell receptor (BCR) complex functions uniquely by: a. Communicating information about the antigen to the helper T cell b. Secreting chemical signals to cNoUmRmSIuNnGiTcBat.eCObeMtween cells c. Recognizing the antigen on the surface of the B lymphocyte d. Communicating information about the antigen to the cell nucleus ANS: D The role of the BCR is to recognize the antigen; however, unlike circulating antibodies, the receptor must communicate that information to the cell’s nucleus. The other options are not unique to the function of the BCR complex. PTS: 1 REF: Page 232 15. The generation of clonal diversity occurs primarily during which phase of life? a. Fetal c. Infancy b. Neonatal d. Puberty ANS: A Generation of clonal diversity primarily occurs in the fetus and probably continues to a low degree throughout most of adult life. PTS: 1 REF: Pages 236-237 16. The generation of clonal diversity includes a process that: a. Involves antigens that select those lymphocytes with compatible receptors. b. Allows the differentiation of cells into antibody-secreting plasma cells or mature T cells. c. Takes place in the primary (central) lymphoid organs. d. Causes antigens to expand and diversify their populations. ANS: C This process occurs in central lymphoid organs—the thymus gland for T cells and bone marrow for B cells. The other options do not accurately describe the processes included in clonal diversity. PTS: 1 REF: Pages 236-237 17. Which statement is true concerning clonal selection? a. Clonal selection is driven by hormones and does not require foreign antigens. b. This theory involves antigens that select those lymphocytes with compatible receptors. c. Clonal selection takes place in the primary (central) lymphoid organs. d. This process generates immature but immunocompetent T and B cells with receptors. ANS: B Clonal selection, a process during which antigens select those lymphocytes with compatible receptors, expands their population and causes differentiation into antibody-secreting plasma cells or mature T cells (see Table 8-6). The other statements are not true regarding clonal selection. PTS: 1 REF: Page 236 18. Which is an example of an endogenous antigen? a. Yeast NURSINGTcB..COBMacteria b. Cancer cells d. Fungus ANS: B Of the options provided, endogenous antigens include only those uniquely produced by cancerous cells. PTS: 1 REF: Page 244 19. Which cytokine is needed for the maturation of a functional helper T cell? a. IL-1 c. IL-4 b. IL-2 d. IL-12 ANS: B Of the options provided, IL-2 production is critical for the Th cell to mature efficiently into a functional helper cell. PTS: 1 REF: Page 245 20. Th2 cells produce IL-4 and suppress which cells? a. B lymphocytes c. Th1 cells b. Cytotoxic T lymphocytes d. Memory T lymphocytes ANS: C Th2 cells produce IL-4, which suppresses only Th1 and Th17 cells through their IL-4 receptors. PTS: 1 REF: Pages 246-247 21. Which statement is believed to be true concerning Th1 cells? a. Th1 cells are induced by antigens derived from allergens. b. They are induced by antigens derived from cancer cells. c. Th1 cells produce IL-4, IL-5, IL-6, and IL-13. d. They assist in the development of humoral immunity. ANS: B Antigens derived from viral or bacterial pathogens and those derived from cancer cells are hypothesized to induce a greater number of Th1 cells relative to Th2 cells. The other statements are not true regarding Th1 cells. PTS: 1 REF: Pages 246-247 22. Which statement is believed to be true concerning Th2 cells? a. Th2 cells are induced by antigens derived from allergens. b. They are induced by antigens derived from cancer cells. c. Th2 cells produce IL-2, TNF-ß, and IFN- . d. They assist in the development of cell-mediated immunity. ANS: A Antigens derived from multicellular parasites and allergens are hypothesized to be involved in the production of more Th2 cells. The other statements are not true regarding Th2 cells. PTS: 1 REF: Pages 2N4U6-R2S4I7NGTB.COM 23. When a person is exposed to most antigens, antibodies can be usually detected in his or her circulation within: a. 12 hours c. 3 days b. 24 hours d. 6 days ANS: D After only approximately 5 to 7 days is an IgM antibody specific for that antigen detected in the circulation. PTS: 1 REF: Page 247 24. Vaccinations are able to provide protection against certain microorganisms because of the: a. Strong response from IgM c. Memory cells for IgE b. Level of protection provided by IgG d. Rapid response from IgA ANS: B IgG production is considerably increased, making it the predominant antibody class of the secondary response. IgG is often present in concentrations several times larger than those of IgM, and levels of circulating IgG specific for that antigen may remain elevated for an extended period. The other options are not relevant to how vaccinations protect against certain microorganisms. PTS: 1 REF: Page 247 25. Why is the herpes virus inaccessible to antibodies after the initial infection? a. The virus does not circulate in the blood. b. It does not have antibody receptors. c. It resists agglutination. d. The virus is a soluble antigen. ANS: A Many viruses (e.g., measles, herpes) are inaccessible to antibodies after the initial infection only because these viruses do not circulate in the bloodstream; rather, they remain inside infected cells, spreading by direct cell-to-cell contact. PTS: 1 REF: Page 252 26. Increased age may cause which change in lymphocyte function? a. Increased production of antibodies against self-antigens b. Decreased number of circulating T cells c. Decreased production of autoantibodies d. Increased production of helper T cells ANS: A B-cell function is altered with age as shown by decreases in specific antibody production in response to antigenic challenge, with concomitant increases in circulating immune complexes and in circulating autoantibodies (antibodies against self-antigens). Aging does not play a role in either decreasing T cells circulation or increasing helper T cells production. PTS: 1 REF: Pages 2N5U7-R2S5I8NGTB.COM 27. How do antibodies protect the host from bacterial toxins? a. Lysing the cell membrane of the toxins b. Binding to the toxins to neutralize their biologic effects c. Inhibiting the synthesis of DNA proteins needed for growth d. Interfering with the DNA enzyme needed for replication ANS: B To cause disease, most toxins must bind to surface molecules on the individual’s cells. Protective antibodies can bind to the toxins, prevent their interaction with cells, and neutralize their biologic effects. The other options fail to explain how antibodies protect the host from bacterial toxins. PTS: 1 REF: Page 252 28. Which T cell controls or limits the immune response to protect the host’s own tissues against an autoimmune response? a. Cytotoxic T cells c. Th2 cells b. Th1 cells d. Regulatory T (Treg) cells ANS: D The regulatory T (Treg) cell is the only option whose role is to control or limit the immune response to protect the host’s own tissues against autoimmune reactions. PTS: 1 REF: Page 257 29. Evaluation of umbilical cord blood can confirm that which immunoglobulin level is near adult levels? a. IgA c. IgM b. IgG d. IgE ANS: B At birth, the total IgG level in the umbilical cord is the only immunoglobulin that is near adult levels (see Figure 8-30). PTS: 1 REF: Page 257 30. Which statement is true concerning the IgM? a. IgM is the first antibody produced during the initial response to an antigen. b. IgM mediates many common allergic responses. c. IgM is the most abundant class of immunoglobulins. d. IgM is capable of crossing the human placenta. ANS: A Typically, IgM is produced first (primary immune response), followed by IgG against the same antigen. The other options are not true statements regarding IgM. PTS: 1 REF: Page 247 31. Which cell has the ability to recognize antigens presented by the MHC class I molecules? a. T cytotoxic c. CD 8 b. CD 4 ANS: C NURSINGTdB..COTMhelper CD8 cells recognize antigens presented by the major histocompatibility complex (MHC) class I molecules and become mediators of cell-mediated immunity and directly kill other cells (T-cytotoxic cells). CD4 cells tend to recognize antigen presented by MHC class II molecules and develop into helpers in the later clonal selection process (T-helper cells) PTS: 1 REF: Page 240 32. Which cell has a role in developing cell-mediated immunity? a. Th1 c. CD8 b. CD4 d. Th2 ANS: A Only Th1 cells help develop cellular immunity. PTS: 1 REF: Pages 245-246 33. How does the aging process of the T-cell activity affect older adults? a. Poor heat regulation abilities b. Increased risk for bone fractures c. Tendency to develop various infections d. Likelihood of experiencing benign skin lesions ANS: C T-cell activity is deficient in older adults, and a shift in the balance of T-cell subsets is observed. These changes may result in increased susceptibility to infection. The other issues are not related to T-cell activity. PTS: 1 REF: Pages 257-258 34. Which statement is true regarding maternal antibodies provided to the neonate? a. The antibodies enter into the fetal circulation by means of active transport. b. The antibodies are transferred to the fetus via the lymphatic system. c. The antibodies are directly related to the mother’s nutritional intake. d. The antibodies reach protective levels after approximately 6 months of age. ANS: A To protect the child against infectious agents both in utero and during the first few postnatal months, a system of active transport facilitates the passage of maternal antibodies into the fetal circulation. The antibodies are transmitted via the placenta and are related to the mother’s immune system. The infant’s own IgG-related antibodies reach protective levels by 6 months of age. PTS: 1 REF: Page 257 35. Antibodies that are associated with mucosal immune system, such as immunoglobulins, function to prevent which type of infections? a. Infections that attack the respiratory system b. Infections that tend to be chronic in nature c. Infections likely to be resistant to antibiotics d. Infections that focus on epithelial surfaces of the body NURSINGTB.COM ANS: D Antibodies of the systemic immune system function throughout the body, whereas antibodies of the secretory (mucosal) immune system—primarily immunoglobulins of the IgA class—are associated with bodily secretions and function to prevent pathogenic infection on epithelial surfaces. The other options are not necessarily true when considering the immunoglobulins. PTS: 1 REF: Pages 252-253 36. Cytokines are vital to a cell’s ability to do which function? a. Excrete c. Metabolize b. Reproduce d. Communicate ANS: D During their interactions, cells must communicate with each other through soluble cytokines. The other options are not so rigidly related to cytokines. PTS: 1 REF: Pages 235-236 MULTIPLE RESPONSE 37. Which is an example of a bacterial toxin that has been inactivated but still retains its immunogenicity to protect the person? (Select all that apply.) a. Poliomyelitis b. Measles c. Tetanus d. Gonorrhea e. Diphtheria ANS: C, E The symptoms of tetanus or diphtheria are mediated by specific toxins. To prevent harming the recipient of the immunization, bacterial toxins are chemically inactivated so that they have lost most of their harmful properties but still retain their immunogenicity. These agents are referred to as toxoids. Tetanus or diphtheria are the only examples of such inactivated toxins. PTS: 1 REF: Page 252 38. Which statements are true concerning the humoral immune response? (Select all that apply.) a. The humoral immune response is divided into major and minor phases. b. The response has IgG and IgM produced during each of its phrases. c. It has a greater presence of IgG than IgM in one of its phases. d. The humoral immune response is produced in reaction to the presence of an antigen. e. Phases differ in their response time as a result of the effect of memory cells. ANS: B, C, D, E The humoral immune response is divided into two phases, primary and secondary. These phases differ in the relative amounts of IgG produced—the secondary response having a significantly higher proportion of NIgUGRSrIeNlaGtTivBe.CtoOMIgM. The two phases also differ in the speed with which each occurs after the antigen challenge—the secondary phases is significantly more rapid than the primary phase because of the presence of memory cells in the secondary phase. PTS: 1 REF: Page 247 39. CD4 is a characteristic surface marker and a result of which of the following? (Select all that apply.) a. Activity in the primary lymphoid organs b. Process of cellular differentiation c. Alterations to T cells d. Changes to B cells e. Clonal selection ANS: A, B, C, D Differentiation of B cells and T cells in the primary lymphoid organs results in the expression of several characteristic surface markers, such as CD4 on helper T cells, CD8 on cytotoxic T cells, and CD21 and CD40 on B cells. Clonal selection is the process during which antigens select those lymphocytes with complementary T-cell receptors (TCRs) or BCRs. PTS: 1 REF: Pages 236-241 | Page 258 40. What are the necessary components of an adaptive immune response? (Select all that apply.) a. Antigen b. Gamma IgG c. Lymphocyte surface receptors d. Crystalline fragment e. Antibody ANS: A, C, E Antigens are the molecules that can react with components of the adaptive immune system, including antibodies and lymphocyte surface receptors. PTS: 1 REF: Page 228 MATCHING Match each immunoglobulin with its characteristic or function. Each immunoglobulin can be used only once. A. IgA B. IgE C. IgG 41. Crosses the placenta. 42. Is predominantly found in the blood and body secretions. 43. Mediates many common allergic responses 41. ANS: C PTS: 1 NURSINRGETFB:.CPOaMge 229 MSC: As a result of selective transport across the placenta, maternal IgG is the major class of antibody found in the blood of the fetus and newborn. 42. ANS: A PTS: 1 REF: Page 229 MSC: IgA can be divided into two subclasses, IgA1 and IgA2. IgA1 molecules are found predominantly in the blood, whereas IgA2 is the predominant class of antibody found in normal body secretions. 43. ANS: B PTS: 1 REF: Page 230 MSC: IgE is the least concentrated of any of the immunoglobulin classes in the circulation. It appears to have very specialized functions as a mediator of many common allergic responses and in the defense against parasitic infections. Chapter 9: Alterations in Immunity and Inflammation MULTIPLE CHOICE 1. Hypersensitivity is best defined as a(an): a. Disturbance in the immunologic tolerance of self-antigens b. Immunologic reaction of one person to the tissue of another person c. Altered immunologic response to an antigen that results in disease d. Undetectable immune response in the presence of antigens ANS: C Hypersensitivity is an altered immunologic response to an antigen that results in disease or damage to the host. The other options are not accurate definitions of hypersensitivity. PTS: 1 REF: Page 262 2. A hypersensitivity reaction that produces an allergic response is called: a. Hemolytic shock c. Necrotizing vasculitis b. Anaphylaxis d. Systemic erythematosus ANS: B Examples of systemic anaphylaxis are allergic reactions to beestings, peanuts, and fish. The other options are not accurate examples of hypersensitivity. PTS: 1 REF: Page 263 3. The common hay fever allergy is NexUpRrSeIsNseGdTBth.CroOuMgh a reaction that is mediated by which class of immunoglobulins? a. IgE c. IgM b. IgG d. T cells ANS: A Type I reactions are mediated by antigen-specific IgE and the products of tissue mast cells (see Figure 9-1). The most common allergies (e.g., pollen allergies) are type I reactions. In addition, most type I reactions occur against environmental antigens and are therefore allergic. The other options do not accurately identify the mediation factor related to hay fever. PTS: 1 REF: Page 263 | Table 9-1 4. Which type of antibody is involved in type I hypersensitivity reaction? a. IgA c. IgG b. IgE d. IgM ANS: B Type I reactions are only mediated by antigen-specific IgE and the products of tissue mast cells (see Figure 9-1). PTS: 1 REF: Page 263 5. Blood transfusion reactions are an example of: a. Autoimmunity c. Homoimmunity b. Alloimmunity d. Hypersensitivity ANS: B Only alloimmunity (also termed isoimmunity) occurs when the immune system of one individual produces an immunologic reaction against tissues of another individual. PTS: 1 REF: Page 262 6. During an IgE-mediated hypersensitivity reaction, which leukocyte is activated? a. Neutrophils c. Eosinophils b. Monocytes d. T lymphocytes ANS: C Of the options provided, only eosinophils are activated during IgE-mediated hypersensitivity reactions. PTS: 1 REF: Page 263 | Page 265 7. During an IgE-mediated hypersensitivity reaction, what causes bronchospasm? a. Bronchial edema caused by the chemotactic factor of anaphylaxis b. Bronchial edema caused by binding of the cytotropic antibody c. Smooth muscle contraction caused by histamine bound to H1 receptors d. Smooth muscle contraction caused by histamine bound to H2 receptors ANS: C During an IgE-mediated hypersensitivity reaction, only smooth muscle contraction caused by histamine bound to H1 receptors results in bronchospasms. NURSINGTB.COM PTS: 1 REF: Page 263 | Page 265 8. During an IgE-mediated hypersensitivity reaction, the degranulation of mast cells is a result of which receptor action? a. Histamine bound to H2 b. Chemotactic factor binding to the receptor c. Epinephrine bound to mast cells d. Acetylcholine bound to mast cells ANS: A Histamine bound to H2 results in the degranulation of mast cells during an IgE-medicated hypersensitivity reaction. The other options do not cause this reaction. PTS: 1 REF: Page 265 9. What characteristic do atopic individuals have that make them genetically predisposed to develop allergies? a. Greater quantities of histamine c. Greater quantities of IgE b. More histamine receptors d. A deficiency in epinephrine ANS: C Atopic individuals tend to produce higher quantities of IgE and to have more crystalline fragment (Fc) receptors for IgE on their mast cells. The other options do not cause this reaction. PTS: 1 REF: Page 271 10. What is the mechanism that results in type II hypersensitivity reactions? a. Antibodies coat mast cells by binding to receptors that signal its degranulation, followed by a discharge of preformed mediators. b. Antibodies bind to soluble antigens that were released into body fluids, and the immune complexes are then deposited in the tissues. c. Cytotoxic T lymphocytes or lymphokine-producing helper T 1 cells directly attack and destroy cellular targets. d. Antibodies bind to the antigens on the cell surface. ANS: D The mechanism that results in a type II hypersensitivity reaction begins with antibody binding to tissue-specific antigens or antigens that have attached to particular tissues. The cell can be destroyed by antibody IgG or IgM and activation of the complement cascade through the classical pathway. PTS: 1 REF: Page 266 11. When mismatched blood is administered causing an ABO incompatibility, the erythrocytes are destroyed by: a. Complement-mediated cell lysis c. Phagocytosis in the spleen b. Phagocytosis by macrophages d. Natural killer cells ANS: A Erythrocytes are destroyed by complement-mediated lysis in individuals with autoimmune hemolytic anemia or as a result ofNaUnRaSlIlNoiGmTmB.uCnOeMreaction to ABO-mismatched transfused blood cells. The other options are not involved in an ABO incompatibility reaction. PTS: 1 REF: Page 266 12. When antibodies are formed against red blood cell antigens of the Rh system, the blood cells are destroyed by: a. Complement-mediated cell lysis b. Phagocytosis by macrophages c. Phagocytosis in the spleen d. Neutrophil granules and toxic oxygen products ANS: C Antibodies against platelet-specific antigens or against red blood cell antigens of the Rh system coat those cells at low density, resulting in their preferential removal by phagocytosis in the spleen, rather than by complement-mediated lysis. The other options do not cause this reaction. PTS: 1 REF: Pages 266-267 13. When soluble antigens from infectious agents enter circulation, tissue damage is a result of: a. Complement-mediated cell lysis b. Phagocytosis by macrophages c. Phagocytosis in the spleen d. Neutrophil granules and toxic oxygen products ANS: D Of the options available, only the components of neutrophil granules damage the tissue. PTS: 1 REF: Page 267 14. How are target cells destroyed in a type II hypersensitivity reaction? a. Complement-mediated cell lysis b. Phagocytosis by macrophages c. Neutrophil granules and toxic oxygen products d. Natural killer cells ANS: D The mechanism that results in a type II hypersensitivity reaction involves a subpopulation of cytotoxic cells that are not antigen specific (natural killer [NK] cells). Antibody on the target cell is recognized by Fc receptors on the NK cells, which releases toxic substances that destroy the target cell. The other options do not cause the destruction of target cells related to a type II hypersensitivity reaction. PTS: 1 REF: Page 267 15. Graves disease (hyperthyroidism) is an example of which type II hypersensitivity reaction? a. Modulation b. Antibody-dependent cell-mediated cytotoxicity c. Neutrophil-mediated damage d. Complement-mediated lysis ANS: A NURSINGTB.COM The antibody reacts with the receptors on the target cell surface and modulates the function of the receptor by preventing interactions with their normal ligands, replacing the ligand and inappropriately stimulating the receptor or destroying the receptor. For example, in the hyperthyroidism (excessive thyroid activity) of Graves disease, autoantibody binds to and activates receptors for thyroid-stimulating hormone (TSH) (a pituitary hormone that controls the production of the hormone thyroxine by the thyroid). The other options are not examples of type II hypersensitivity reactions. PTS: 1 REF: Page 267 | Page 269 16. Type III hypersensitivity reactions are a result of which of the following? a. Antibodies coating mast cells by binding to receptors that signal its degranulation, followed by the discharge of preformed mediators b. Antibodies binding to soluble antigens that were released into body fluids and the immune complexes being deposited in the tissues c. Tc cells or lymphokine-producing Th1 cells directly attacking and destroying cellular targets d. Antibodies binding to the antigen on the cell surface ANS: B Antigen-antibody (immune) complexes that are formed in the circulation and then deposited later in vessel walls or extravascular tissues (see Figure 9-3) cause most type III hypersensitivity diseases. The other options do not cause this type of reaction. PTS: 1 REF: Page 269 17. A type IV hypersensitivity reaction causes which result? a. Antibodies coating mast cells by binding to receptors that signal its degranulation, followed by the discharge of preformed mediators b. Antibodies binding to soluble antigens that were released into body fluids and the immune complexes being deposited in the tissues c. Lymphokine-producing Th1 cells directly attacking and destroying cellular targets d. Antibodies binding to the antigen on the cell surface ANS: C Types I, II, and III hypersensitivity reactions are mediated by antibody, type IV reactions are mediated by T lymphocytes and do not involve antibody. Type IV mechanisms occur through either Tc cells or lymphokine-producing Th1 cells. Tc cells directly attack and destroy cellular targets. PTS: 1 REF: Page 270 18. In a type III hypersensitivity reaction, the harmful effects after the immune complexes that are deposited in tissues are a result of: a. Cytotoxic T cells c. Complement activation b. Natural killer cells d. Degranulation of mast cells ANS: C Complement activation, particularly through the generation of chemotactic factors for neutrophils, causes the harmful effects of immune complex deposition. The neutrophils bind to antibody and C3b containeNdUiRnStIhNeGcToBm.CpOleMxes and attempt to ingest the immune complexes. Type III hypersensitivity reactions as described are not the result of any of the other options. PTS: 1 REF: Page 269 19. Raynaud phenomenon is classified as a type III hypersensitivity reaction and is due to: a. Immune complexes that are deposited in capillary beds, blocking circulation b. Mast cells that are bound to specific endothelial receptors, causing them to degranulate and creating a localized inflammatory reaction that occludes capillary circulation c. Cytotoxic T cells that attack and destroy the capillaries so that they are unable to perfuse local tissues d. Antibodies that detect the capillaries as foreign protein and destroy them using lysosomal enzymes and toxic oxygen species ANS: A Raynaud phenomenon is a condition caused by the temperature-dependent deposition of immune complexes in the capillary beds of the peripheral circulation. None of the other options are involved in causing this condition. PTS: 1 REF: Page 270 20. Deficiencies in which element can produce depression of both B- and T-cell function? a. Iron c. Iodine b. Zinc d. Magnesium ANS: B Of the options available, only deficient zinc intake can profoundly depress T- and B-cell function. PTS: 1 REF: Page 291 21. When the maternal immune system becomes sensitized against antigens expressed by the fetus, what reaction occurs? a. T-cell immunity c. Fetal immunity b. Alloimmunity d. Autoimmunity ANS: B Alloimmunity occurs when an individual’s immune system reacts against antigens on the tissues of other members of the same species. Sensitization against fetal antigens is not the cause of any other available option. PTS: 1 REF: Page 276 22. Tissue damage caused by the deposition of circulating immune complexes containing an antibody against the host DNA is the cause of which disease? a. Hemolytic anemia c. Systemic lupus erythematosus b. Pernicious anemia d. Myasthenia gravis ANS: C Only the deposition of circulating immune complexes containing an antibody against the host DNA produce tissue damage in individuals with systemic lupus erythematosus (SLE). NURSINGTB.COM PTS: 1 REF: Page 277 23. Why does tissue damage occurs in acute rejection after organ transplantation? a. Th1 cells release cytokines that activate infiltrating macrophages, and cytotoxic T cells directly attack the endothelial cells of the transplanted tissue. b. Circulating immune complexes are deposited in the endothelial cells of transplanted tissue, where the complement cascade lyses tissue. c. Receptors on natural killer cells recognize antigens on the cell surface of transplanted tissue, which releases lysosomal enzymes that destroy tissue. d. Antibodies coat the surface of transplanted tissue to which mast cells bind and liberate preformed chemical mediators that destroy tissue. ANS: A The recipient’s lymphocytes interacting with the donor’s dendritic cells within the transplanted tissue usually initiate sensitization, resulting in the induction of recipient Th1 and Tc cells against the donor’s antigens. The Th1 cells release cytokines that activate infiltrating macrophages, and the Tc cells directly attack the endothelial cells in the transplanted tissue. The other options do not accurately describe how acute rejection after organ transplantation results in tissue damage. PTS: 1 REF: Page 280 24. Which blood cell carries the carbohydrate antigens for blood type? a. Platelets c. Lymphocytes b. Neutrophils d. Erythrocytes ANS: D The reaction that causes a blood transfusion recipient’s red blood cells to clump together is related to the ABO antigens located on the surface of only erythrocytes. PTS: 1 REF: Page 278 25. A person with type O blood is likely to have high titers of which anti-antibodies? a. A c. A and B b. B d. O ANS: C Type O individuals have neither A or B antigen but have both anti-A and anti-B antibodies and therefore cannot accept blood from any of the other three types. PTS: 1 REF: Page 278 26. Which class of immunoglobulins forms isohemagglutinins? a. IgA c. IgG b. IgE d. IgM ANS: D Naturally occurring antibodies, called isohemagglutinins, are immunoglobulins of only the IgM class. PTS: 1 REF: Page 278 NURSINGTB.COM 27. Which component of the immune system is deficient in individuals with infections caused by viruses, fungi, or yeast? a. Natural killer cells c. B cells b. Macrophages d. T cells ANS: D Of the available options, deficiencies in T-cell immune responses are suggested when certain viruses (e.g., varicella, vaccinia, herpes, cytomegalovirus), fungi, and yeasts (e.g., Candida, Histoplasma) or certain atypical microorganisms (e.g., Pneumocystis jiroveci) cause recurrent infections. PTS: 1 REF: Page 281 28. In which primary immune deficiency is there a partial-to-complete absence of T-cell immunity? a. Bruton disease c. Reticular dysgenesis b. DiGeorge syndrome d. Adenosine deaminase deficiency ANS: B The principal immunologic defect in DiGeorge syndrome is the partial or complete absence of T-cell immunity. The other options are not the result of either a partial or complete absence of T-cell immunity. PTS: 1 REF: Page 281 29. How many months does it take for the newborn to be sufficiently protected by antibodies produced by its own B cells? a. 1 to 2 c. 6 to 8 b. 4 to 5 d. 10 to 12 ANS: C By 6 to 8 months, the newborn should be efficiently protected by antibodies produced by its own B cells. PTS: 1 REF: Page 290 30. Considering the effects of nutritional deficiencies on the immune system, severe deficits in calories and protein lead to deficiencies in the formation of which immune cells? a. B cells c. Natural killer cells b. T cells d. Neutrophils ANS: B Severe deficits in calorie or protein intake lead to deficiencies in T-cell function and numbers. The other options are not necessarily affected. PTS: 1 REF: Pages 290-291 31. Urticaria are a manifestation of a which type of hypersensitivity reaction? a. IV c. II b. III d. I ANS: D Urticaria, or hives, is a dermal (skNinU)RmSIaNnGifTeBst.aCtOioMn of only type I allergic reactions. PTS: 1 REF: Page 272 32. Graves disease is a result of: a. Increased levels of circulating immunoglobulins b. The infiltration of the thyroid with T lymphocytes c. Autoantibodies binding to thyroid-stimulating hormone (TSH)-receptor sites d. Exposure to acetylates in substances such as rubber ANS: C In the hyperthyroidism (excessive thyroid activity) of Graves disease, autoantibody binds to and activates receptors for TSH (a pituitary hormone that controls the production of the hormone thyroxine by the thyroid). The other options do not accurately describe the cause of Graves disease. PTS: 1 REF: Page 267 | Page 269 33. Raynaud phenomenon is an example of which type of hypersensitivity? a. IV c. II b. III d. I ANS: B The characteristics of serum sickness are observed in only systemic type III autoimmune diseases such as Raynaud phenomenon. PTS: 1 REF: Page 270 34. Which statement is true concerning an atopic individual? a. They tend to produce less IgE. b. They tend to produce more Fc receptors. c. They tend to attract very few mast cells. d. They tend to produce very high levels of IgM. ANS: B Atopic individuals tend to produce higher quantities of IgE and have more Fc receptors for IgE on their mast cells. The other available options are not true. PTS: 1 REF: Page 271 35. Which statement is true regarding immunodeficiency? a. Immunodeficiency is generally not present in other family members. b. Immunodeficiency is never acquired; rather, it is congenital. c. Immunodeficiency is almost immediately symptomatic. d. Immunodeficiency is a result of a postnatal mutation. ANS: A Generally, the mutations are sporadic and not inherited; a family history exists in only approximately 25% of individuals. The sporadic mutations occur before birth, but the onset of symptoms may be early or later, depending on the particular syndrome. The immunodeficiency can be either congenital or acquired. PTS: 1 REF: Page 28N1URSINGTB.COM 36. A person with type O blood is considered to be the universal blood donor because type O blood contains which of the following? a. No antigens c. Both A and B antigens b. No antibodies d. Both A and B antibodies ANS: A Because individuals with type O blood lack both types of antigens, they are considered universal donors, meaning that anyone can accept their red blood cells. Type O individuals, who have neither A or B antigen but have both anti-A and anti-B antibodies, cannot accept blood from any of the other three types. PTS: 1 REF: Page 278 37. Immunoglobulin E (IgE) is associated with which type of hypersensitivity reaction? a. I c. III b. II d. IV ANS: A Hypersensitivity reactions have been divided into four distinct types: type I (IgE-mediated) hypersensitivity reactions, type II (tissue-specific) hypersensitivity reactions, type III (immune complex–mediated) hypersensitivity reactions, and type IV (cell-mediated) hypersensitivity reactions. PTS: 1 REF: Page 263 38. Graves disease is an autoimmune disease that results in which maternal antibody? a. Binding with receptors for neural transmitters on muscle cells, causing neonatal muscular weakness b. Affecting the receptor for TSH, causing neonatal hyperthyroidism c. Inducing anomalies in the fetus or causing pregnancy loss d. Destroying platelets in the fetus and neonate ANS: B Graves disease is an autoimmune disease in which maternal antibody against the receptor for TSH causes neonatal hyperthyroidism. Myasthenia gravis is an autoimmune disease in which maternal antibody binds with receptors for neural transmitters on muscle cells (acetylcholine receptors), causing neonatal muscular weakness. Systemic lupus erythematosus is an autoimmune disease in which diverse maternal autoantibodies induce anomalies (e.g., congenital heart defects) in the fetus or cause pregnancy loss. Immune thrombocytopenic purpura causes both autoimmune and alloimmune variants to occur, during which maternal antiplatelet antibody destroys platelets in the fetus and neonate. PTS: 1 REF: Page 277 MULTIPLE RESPONSE 39. When a tuberculin skin test is positive, the hard center and erythema surrounding the induration are a result of which of the following? (Select all that apply.) a. Histamine d. Products of complement b. T lymphocytes c. Immune complexes ANS: B, E NURSINGTeB..COMMacrophages The reaction site is infiltrated with only T lymphocytes and macrophages, resulting in a clear hard center (induration) and a reddish surrounding area (erythema). PTS: 1 REF: Page 270 40. Exposure to which of the following could result in a type IV hypersensitivity reaction? (Select all that apply.) a. Poison ivy d. Nickel b. Neomycin e. Detergents c. Dairy products ANS: A, B, D, E Allergens that primarily elicit type IV allergic hypersensitivities include plant resins (e.g., poison ivy, poison oak); metals (e.g., nickel, chromium); acetylates and chemicals in rubber, cosmetics, detergents; and topical antibiotics (e.g., neomycin). PTS: 1 REF: Page 271 41. Which disorders are considered autoimmune? (Select all that apply.) a. Crohn disease d. Systemic lupus erythematosus b. Addison disease e. Noninsulin-dependent diabetes c. Rheumatoid arthritis ANS: A, B, C, D Crohn disease, Addison disease, rheumatoid arthritis, and systemic lupus erythematosus are all diseases that result from autoimmune pathologic conditions. Insulin-dependent diabetes is also an autoimmune disorder, but noninsulin-dependent diabetes is not. PTS: 1 REF: Pages 264-265 | Table 9-2 42. Which statements best define acute rejection? (Select all that apply.) a. Acute rejection is a cell-mediated immune response. b. Acute rejection is usually a type III rejection. c. Immunosuppressive drugs delay or lessen the intensity of an acute rejection. d. Acute rejection is associated with the body’s response to an organ transplant. e. Acute rejection is a response against unmatched human leukocyte antigens (HLAs). ANS: A, C, D, E Acute rejection is primarily a cell-mediated immune response that occurs within days to months after transplantation. This type of rejection occurs when the recipient develops an immune response against unmatched HLAs after transplantation. A biopsy of the rejected organ usually shows an infiltration of lymphocytes and macrophages characteristic of a type IV reaction. Immunosuppressive drugs may delay or lessen the intensity of an acute rejection. PTS: 1 REF: Page 280 NURSINGTB.COM Chapter 10: Infection MULTIPLE CHOICE 1. What is a significant cause of morbidity and mortality worldwide? a. Starvation c. Cardiovascular disease b. Traumatic injury d. Infectious disease ANS: D Despite the wide-scale implementation of progressive public health and immunization policies, infectious disease remains a significant cause of morbidity and mortality. The other options are not significant causes. PTS: 1 REF: Page 299 2. What is the first stage in the infectious process? a. Invasion c. Spread b. Colonization d. Multiplication ANS: B From the perspective of the microorganisms that cause disease, the infectious process undergoes four separate stages of progression: (1) colonization, (2) invasion, (3) multiplication, and (4) spread. PTS: 1 REF: Pages 300-301 3. Which type of microorganism repNroUdRuScIeNsGoTnBt.CheOMskin? a. Viruses c. Protozoa and Rickettsiae b. Bacteria and fungi d. Mycoplasma ANS: B Only bacteria and fungi have the capacity to reproduce on the skin. PTS: 1 REF: Page 303 | Table 10-3 4. Phagocytosis involves neutrophils actively attacking, engulfing, and destroying which microorganisms? a. Bacteria c. Viruses b. Fungi d. Yeasts ANS: A Invasion is the direct confrontation with an individual’s primary defense mechanisms against only bacteria, which include the complement system, antibodies, and phagocytes, such as neutrophils and macrophages. PTS: 1 REF: Page 306 5. Once they have penetrated the first line of defense, which microorganisms do natural killer (NK) cells actively attack? a. Bacteria c. Viruses b. Fungi d. Mycoplasma ANS: C NK cells are the principal defenders against only tumor cells or virally infected cells. PTS: 1 REF: Page 320 6. Which statement concerning exotoxins is true? a. Exotoxins are contained in cell walls of gram-negative bacteria. b. Exotoxins are released during the lysis of bacteria. c. Exotoxins are able to initiate the complement and coagulation cascades. d. Exotoxins are released during bacterial growth. ANS: D Exotoxins are proteins released during bacterial growth. The other options are not true of exotoxins. PTS: 1 REF: Page 306 7. Which statement is true concerning a fungal infection? a. Fungal infections occur only on skin, hair, and nails. b. Phagocytes and T lymphocytes control fungal infections. c. Fungal infections release endotoxins. d. Vaccines prevent fungal infections. ANS: B The host defense against fungal infection includes the fungistatic properties of neutrophils and macrophages. T lymphocytes are crucial in limiting the extent of infection and producing cytokines to further activate macrophages. The other options are not true of fungal infections. NURSINGTB.COM PTS: 1 REF: Page 312 8. Cytokines are thought to cause fevers by stimulating the synthesis of which chemical mediator? a. Leukotriene c. Prostaglandin b. Histamine d. Bradykinin ANS: C Cytokines seem to raise the thermoregulatory set point through stimulation of prostaglandin synthesis and turnover in thermoregulatory (brain) and nonthermoregulatory (peripheral) tissues. The other options do not accurately identify the appropriate chemical mediator. PTS: 1 REF: Pages 301-302 9. Considering the hypothalamus, a fever is produced by: a. Endogenous pyrogens acting directly on the hypothalamus. b. Exogenous pyrogens acting directly on the hypothalamus. c. Immune complexes acting indirectly on the hypothalamus. d. Cytokines acting indirectly on the hypothalamus. ANS: A Little evidence suggests that exogenous pyrogens directly cause fever. Such pyrogens indirectly affect the hypothalamus through endogenous pyrogens released by cells of the host. Neither immune complexes nor cytokines are involved in the process. PTS: 1 REF: Page 302 10. Which statement about vaccines is true? a. Most bacterial vaccines contain attenuated organisms. b. Most viral vaccines are made by using dead organisms. c. Vaccines require booster injections to maintain life-long protection. d. Vaccines provide effective protection against most infections. ANS: C In general, vaccine-induced protection does not persist as long as infection-induced immunity, thus booster injections may be necessary to maintain protection throughout life. The other options are not true of vaccines. PTS: 1 REF: Page 332 11. Vaccines against viruses are created from: a. Killed organisms or extracts of antigens b. Live organisms weakened to produce antigens c. Purified toxins that have been chemically detoxified d. Recombinant pathogenic protein ANS: B Most vaccines against viral infections (e.g., measles, mumps, rubella, varicella [chickenpox], rotavirus) contain liNvUeRvSirINusGeTsBth.CaOt Mare weakened (attenuated) to continue expressing the appropriate antigens but are unable to establish more than a limited and easily controlled infection. The other options are not used in virus-focused vaccines. PTS: 1 REF: Page 332 12. Which statement is a characteristic of HIV? a. HIV only infects T-helper (Th) cells. b. HIV is a retrovirus. c. HIV carries genetic information in its DNA. d. HIV has five identified strains. ANS: B HIV is a member of the retrovirus family, which carries genetic information in the form of two copies of RNA (see Figure 10-12). The other statements are not true of HIV. PTS: 1 REF: Page 324 13. What is the role of reverse transcriptase in HIV infection? a. Reverse transcriptase converts single-stranded DNA into double-stranded DNA. b. It is needed to produce integrase. c. It transports the RNA into the cell nucleus. d. It converts RNA into double-stranded DNA. ANS: D One particular family of viruses, retroviruses (e.g., HIV) carries an enzyme, reverse transcriptase, which creates a double-stranded DNA version of the virus. PTS: 1 REF: Page 324 14. After sexual transmission of HIV, a person can be infected yet seronegative for how many months? a. 1 to 2 c. 18 to 20 b. 6 to 14 d. 24 to 36 ANS: B Antibody appears rather rapidly after infection through blood products, usually within 4 to 7 weeks. After sexual transmission, however, the individual can be infected yet seronegative for 6 to 14 months or, in at least one case, for years. PTS: 1 REF: Page 326 15. Which cells are primary targets for HIV? a. CD4+ Th cells only b. CD4+ Th cells, macrophages, and natural killer cells c. CD8-positive cytotoxic T (Tc) cells and plasma cells d. CD8-positive Tc cells only ANS: B The primary cellular targets for HIV include CD4+ Th cells, macrophages, and NK cells. The other options are not the primary target cells of HIV. PTS: 1 REF: Page 32N5URSINGTB.COM 16. What area in the body may act as a reservoir in which HIV can be relatively protected from antiviral drugs? a. Central nervous system c. Thymus gland b. Bone marrow d. Lungs ANS: A HIV may persist in regions where the antiviral drugs are not as effective, such as the central nervous system (CNS). The other options are not as protected from antiviral drugs. PTS: 1 REF: Page 327 17. AIDS produces a striking decrease in the number of which cells? a. Macrophages c. CD4+ Th cells b. CD8+ T cells d. Memory T cells ANS: C The major immunologic finding in AIDS is the striking decrease in the number of CD4+ Th cells (see Figure 10-15). This finding is not true of the other options. PTS: 1 REF: Page 325 18. HIV antibodies appear within how many weeks after infection through blood products? a. 1 to 2 c. 10 to 12 b. 4 to 7 d. 20 to 24 ANS: B Antibody appears rather rapidly after infection through blood products, usually within 4 to 7 weeks. PTS: 1 REF: Page 326 19. What is the final stage of the infectious process? a. Colonization c. Multiplication b. Invasion d. Spread ANS: D From the perspective of the microorganisms that cause disease, the infectious process undergoes four separate stages of progression: (1) colonization, (2) invasion, (3) multiplication, and (4) spread. PTS: 1 REF: Page 300 20. Toxigenicity is defined as the: a. Ability of the pathogen to invade and multiply in the host b. Pathogen’s ability to produce disease by the production of a soluble toxin c. Ability of an agent to produce disease d. Potency of a pathogen measured in terms of the number of microorganisms required to kill the host ANS: B Toxigenicity is the ability of a pathogen to produce soluble toxins or endotoxins, which are NURSINGTB.COMfactors that greatly influence the of virulence. The other options do not accurately define toxigenicity. PTS: 1 REF: Page 302 21. The ability of the pathogen to invade and multiply in the host is referred to as: a. Infectivity c. Pathogenicity b. Toxigenicity d. Virulence ANS: A Infectivity is the ability of the pathogen to invade and multiply in the host. The other options do not accurately denote the pathogen’s ability to invade and multiply in the host. PTS: 1 REF: Page 302 22. Some bacterial surface proteins bind with the crystalline fragment (Fc) portion of an antibody to: a. Hide in cells to avoid triggering an immune response b. Form self-protecting toxins c. Make staining possible for microscopic observation d. Produce a protective “self” protein ANS: D Some bacterial surface proteins (protein A of Staphylococcus aureus, protein G of Streptococcus pyogenes) bind the Fc portion of the individual’s antibody, thus forming a protective coat of “self” protein. The other options do not accurately define the role of bacterial surface proteins as they bind with the Fc portion on an antibody. PTS: 1 REF: Page 308 23. Which organism is a common sexually transmitted bacterial infection? a. Staphylococcus aureus c. Helicobacter pylori b. Clostridium perfringens d. Treponema pallidum ANS: D Treponema pallidum (spirochete, syphilis) is a sexually transmitted disease. Staphylococcus aureus is commonly ingested, causing food poisoning; Clostridium perfringens (gas gangrene) is a skin or wound infection; and Helicobacter pylori (gastritis, peptic ulcers) is found in the gastrointestinal tract. PTS: 1 REF: Pages 304-305 | Table 10-4 24. Which disease is an example of a rickettsial infection? a. Cholera c. Sleeping sickness b. Candida d. Rocky Mountain spotted fever ANS: D Rocky Mountain spotted fever is a result of a rickettsiae. Cholera is a bacterial infection, candida is a fungal infection, and sleeping sickness is a protozoal infection. PTS: 1 REF: Page 30N2URSINGTB.COM MULTIPLE RESPONSE 25. Which secretion transmits HIV? (Select all that apply.) a. Semen b. Urine c. Saliva d. Breast milk e. Sweat ANS: A, D HIV is a blood-borne pathogen present in body fluids (e.g., blood, vaginal fluid, semen, breast milk). PTS: 1 REF: Page 322 26. Which infection is fungal? (Select all that apply.) a. Ringworm b. Candida c. Cholera d. Athlete’s foot e. Aspergillus ANS: A, B, D, E Infection with a fungus is called mycosis and includes dermatophytes (e.g., tineas, which refers to several skin mycoses including ringworm, athlete’s foot, and others) or yeasts (e.g., Candida, Aspergillus, Cryptococcus). Cholera is a bacterial infection. PTS: 1 REF: Page 311 27. Which statement is true regarding the development of HIV symptoms? (Select all that apply.) a. Symptoms generally appear in the clinical latency stage. b. Symptoms are generally observable within 5 years of the initial infection. c. T cells levels, particularly those of memory T cells, progressively decrease. d. Untreated infected individuals may remain asymptomatic for up to10 years. e. Secondary lymphoid organs experience damage and resulting malfunction. ANS: C, D, E Individuals during the early stages of HIV (early stage disease or clinical latency) are usually asymptomatic. The early stage may last as long as 10 years in untreated people, during which the viral load increases and the numbers of CD4+ cells progressively decrease. As a result of these processes, the level of T cells decreases (particularly memory T cells, which seem more susceptible to HIV infection); thymic production of new T cells is decreased; and the secondary lymphoid organs (particularly the lymph nodes) are damaged. PTS: 1 REF: Pages 326-327 28. Which statements are true regarding endotoxins? (Select all that apply.) a. Endotoxins are lipopolysaccharides. b. Endotoxins are located in the wNUalRlsSIoNfGbTaBc.tCerOiaM. c. Endotoxins are created during the process of lysis. d. Endotoxins are found in gram-negative microorganisms. e. Endotoxins are released during the destruction of its host. ANS: A, B, D, E Endotoxins are lipopolysaccharides (LPSs) contained in the cell walls of gram-negative bacteria and released during lysis (or destruction) of the bacteria. PTS: 1 REF: Page 306 29. Which statements are true regarding viruses? (Select all that apply.) a. Viruses are very complex microorganisms. b. Viruses are referred to as eukaryotes. c. Viruses are capable of producing messenger RNA (mRNA). d. Viruses penetrate plasma membranes via endocytosis. e. Viruses are capable of uncoating cytoplasmic nucleocapsid. ANS: C, D, E Viruses are extremely simple microorganisms and do not possess any of the metabolic organelles found in prokaryotes (e.g., bacteria) or eukaryotes (e.g., human cells). Once bound, the virus can penetrate the plasma membrane by receptor-mediated endocytosis. Within the cytoplasm, the virus uncoats the protective nucleocapsid and releases viral genetic information. Most RNA viruses directly produce mRNA, which is translated into viral proteins, and genomic RNA, which is eventually packaged into new viruses. PTS: 1 REF: Pages 317-319 30. Which of the following play a role in the control of fungal infections? (Select all that apply.) a. Cytokines b. Macrophages c. Natural killer cells d. Neutrophils e. T lymphocytes ANS: A, B, D, E The host defense against fungal infection includes the fungistatic properties of neutrophils and macrophages. T lymphocytes are crucial in limiting the extent of infection and producing cytokines to further activate macrophages. Natural killer cells are a component of innate immune system. PTS: 1 REF: Page 312 31. Complications of AIDS include: (Select all that apply.) a. Kaposi sarcoma b. Helicobacter pylori c. Cytomegalovirus retinitis d. Herpes simplex infection e. Legionella pneumophila ANS: A, C, D NURSINGTB.COM Kaposi sarcoma, cytomegalovirus retinitis, and herpes simplex infection are clinical complications characteristically observed in patients with AIDS. Neither Helicobacter pylori nor Legionella pneumophila are considered classic AIDS opportunistic diseases. PTS: 1 REF: Page 328 | Figure 10-16 MATCHING Match each term with its definition. A. Toxigenicity B. Infectivity C. Pathogenicity D. Virulence 32. Ability of the pathogen to invade and multiply in the host 33. Capacity of a pathogen to cause severe disease 34. An important factor in determining a pathogen’s ability to produce disease by the production of a soluble toxin 35. Ability of an agent to produce disease 32. ANS: B PTS: 1 REF: Page 302 MSC: Infectivity is the ability of the pathogen to invade and multiply in the host. 33. ANS: D PTS: 1 REF: Page 302 MSC: Virulence is the capacity of a pathogen to cause severe disease. 34. ANS: A PTS: 1 REF: Page 302 MSC: Toxigenicity is the ability to produce soluble toxins or endotoxins, factors that greatly influence the pathogen's degree of virulence. 35. ANS: C PTS: 1 REF: Page 302 MSC: Pathogenicity is the ability of an agent to produce disease. NURSINGTB.COM Chapter 11: Stress and Disease MULTIPLE CHOICE 1. Exhaustion occurs if stress continues when which stage of the general adaptation syndrome is not successful? a. Flight or fight c. Adaptation b. Alarm d. Arousal ANS: C Exhaustion occurs if stress continues and adaptation is not successful, ultimately causing impairment of the immune response, heart failure, and kidney failure, leading to death. The other stages occur before the adaptation stage. PTS: 1 REF: Page 339 2. Which organ is stimulated during the alarm phase of the general adaptation syndrome (GAS)? a. Adrenal cortex c. Anterior pituitary b. Hypothalamus d. Limbic system ANS: B The alarm phase of the GAS begins when a stressor triggers the actions of the hypothalamus and the sympathetic nervous system (SNS) (see Figure 11-1). The other organs are not stimulated by the alarm phase of GAS. PTS: 1 REF: Page 33N9URSINGTB.COM 3. During an anticipatory response to stress, the reaction from the limbic system is stimulated by the: a. Retronucleus of the anterior pituitary b. Anterior nucleus of the hippocampus c. Paraventricular nucleus of the hypothalamus d. Prefrontal nucleus of the amygdala ANS: C The paraventricular nucleus (PVN) of the hypothalamus must be stimulated to cause the limbic system to be stimulated. The other options are not involved in the stimulation of the limbic system. PTS: 1 REF: Page 341 4. Which hormone prompts increased anxiety, vigilance, and arousal during a stress response? a. Norepinephrine b. Epinephrine c. Cortisol d. Adrenocorticotropic hormone (ACTH) ANS: A Only the release of norepinephrine promotes arousal, increased vigilance, increased anxiety, and other protective emotional responses. PTS: 1 REF: Page 343 5. Perceived stress elicits an emotional, anticipatory response that begins where? a. Prefrontal cortex c. Limbic system b. Anterior pituitary d. Hypothalamus ANS: C Perceived stressors elicit an anticipatory response that begins in the limbic system of the brain, the only option responsible for emotions and cognition. PTS: 1 REF: Page 343 6. During a stress response, the helper T (Th) 1 response is suppress by which hormone? a. ACTH c. Prolactin b. Cortisol d. Growth hormone ANS: B Stress can activate an excessive immune response and, through cortisol and catecholamines, suppress the Th1 response, causing a Th2 shift. This response is not active by any of the other options. PTS: 1 REF: Page 349 7. What is the effect that low-serum albumin has on the central stress response? a. Impaired circulation of epinepNhUriRnSeIaNnGdTnBo.CreOpMinephrine b. Impaired wound healing c. Lessened circulation of cortisol d. Diminished oncotic pressure ANS: A Low-serum albumin impairs circulation of both epinephrine and norepinephrine since both bind to plasma protein albumin. The other options do not accurately describe the effect of low-serum albumin. PTS: 1 REF: Page 345 8. Stress-age syndrome directly results in depressed function of which system? a. Respiratory c. Digestive b. Endocrine d. Immune ANS: D Of the available options, immunodepression is the only characteristic change observed in stress-age syndrome. PTS: 1 REF: Page 358 9. Stress-induced sympathetic stimulation of the adrenal medulla causes the secretion of: a. Epinephrine and aldosterone c. Epinephrine and norepinephrine b. Norepinephrine and cortisol d. Acetylcholine and cortisol ANS: C The sympathetic nervous system is aroused during the stress response and causes the medulla of the adrenal gland to release catecholamines (80% epinephrine and 20% norepinephrine) into the bloodstream. The stress-induced efforts on the adrenal medulla do not include any of the other options. PTS: 1 REF: Pages 344-345 10. Stress-induced norepinephrine results in: a. Pupil constriction c. Increased sweat gland secretions b. Peripheral vasoconstriction d. Decreased blood pressure ANS: B During stress, norepinephrine raises blood pressure by constricting peripheral vessels; it dilates the pupils of the eye, causes piloerection, and increases sweat gland action in the armpits and palms. PTS: 1 REF: Page 345 11. Released stress-induced cortisol results in the stimulation of gluconeogenesis by affecting which structure? a. Adrenal cortex c. Liver b. Pancreas d. Anterior pituitary ANS: A One of the primary effects of cortisol is the stimulation of gluconeogenesis through stimulation of the adrenal cortex. The other options do not produce a stimulation of gluconeogenesis when exposed toNcUoRrtSisINolG.TB.COM PTS: 1 REF: Page 346 12. What is the effect of increased secretions of epinephrine, glucagon, and growth hormone? a. Hyperglycemia c. Bronchodilation b. Hypertension d. Pupil dilation ANS: A Cortisol enhances the elevation of blood glucose promoted by other hormones, such as epinephrine, glucagon, and growth hormone. This effect is not true of the other options. PTS: 1 REF: Page 346 13. Which hormone increases the formation of glucose from amino acids and free fatty acids? a. Epinephrine c. Cortisol b. Norepinephrine d. Growth hormone ANS: C One of the primary effects of cortisol is the stimulation of gluconeogenesis or the formation of glucose from noncarbohydrate sources, such as amino or free fatty acids in the liver. Neither reaction is a result of the effects of any of the other options. PTS: 1 REF: Page 346 14. What effect do androgens have on lymphocytes? a. Suppression of B-cell responses and enhancement of T-cell responses b. Suppression of T-cell responses and enhancement of B-cell responses c. Suppression of B- and T-cell responses d. Enhancement of B- and T-cell responses ANS: C Androgens suppress T- and B-cell responses. The other options do not occur in response to androgens. PTS: 1 REF: Page 353 15. Which gland regulates the immune response and mediates the apparent effects of circadian rhythms on immunity? a. Anterior pituitary c. Basal ganglia b. Adrenal d. Pineal ANS: D Of the options available, only the pineal gland regulates the immune response and mediates the apparent effects of circadian rhythm on immunity. PTS: 1 REF: Page 354 16. Which cytokines initiate the production of corticotropin-releasing hormone (CRH)? a. IL–1 and IL-6 c. IFN and IL-12 b. IL-2 and TNF- d. TNF-ß and IL-4 ANS: A NURSINGTB.COM Although a number of stress factors initiate the production of CRH, of the options available, only high levels of IL-1 and IL-6 initiate such a response. PTS: 1 REF: Pages 348-349 17. The release of which cytokines is triggered by bacterial or viral infections, cancer, and tissue injury that, in turn, initiate a stress response? a. IL-1 and IL-2 b. IL-12, TNF-, and colony-stimulating factor c. IFN, TNF-ß, and IL-6 d. IL-4 and IL-24 ANS: C Of the options offered, only the release of immune inflammatory mediators IL-6, TNF-ß, and IFN is triggered by bacterial or viral infections, cancer, and tissue injury that, in turn, initiates a stress response through the hypothalamic-pituitary-adrenal (HPA) pathway. PTS: 1 REF: Page 349 18. The action of which hormone helps explain increases in affective anxiety and eating disorders, mood cycles, and vulnerability to autoimmune and inflammatory diseases in women as a result of stimulation of the CRH gene promoter and central norepinephrine system? a. Progesterone c. Estrogen b. Cortisol d. Prolactin ANS: C Of the options provided, only estrogen directly stimulates the CRH gene promoter and the central noradrenergic (norepinephrine) system, which may help explain adult women’s slight hypercortisolism, increases in affective anxiety and eating disorders, mood cycles, and vulnerability to autoimmune and inflammatory disease, all of which follow estradiol fluctuations. PTS: 1 REF: Page 350 19. What effect does estrogen have on lymphocytes? a. Depression of B-cell functions and enhancement of T-cell functions b. Depression of T-cell functions and enhancement of B-cell functions c. Depression of B- and T-cell functions d. Enhancement of B- and T-cell functions ANS: B Estrogens generally are associated with only a depression of T-cell–dependent immune functions and an enhancement of B-cell functions. PTS: 1 REF: Page 353 20. Which statement is true concerning the differences between stress-induced hormonal alterations of men and women? a. After injury, women produce more proinflammatory cytokines than men, a profile that is associated with poor outcomes. b. Androgens appear to induce aNgUreRaStIeNr GdTegBr.CeeOMof immune cell apoptosis after injury, creating greater immunosuppression in injured men than in injured women. c. Psychologic stress associated with some types of competition decreases both testosterone and cortisol, especially in athletes older than 45 years of age. d. After stressful stimuli, estrogen is increased in women, but testosterone is decreased in men. ANS: B Androgens appear to induce a greater degree of immune cell apoptosis after injury, a mechanism that may elicit a greater immunosuppression in injured men versus injured women. The other options are not true statements concerning the differences between how the genders are affected by stress-induced hormones. PTS: 1 REF: Page 353 21. Diagnostic blood work on individuals who perceive themselves to be in a chronic stress state will likely demonstrate: a. Decreased Th lymphocytes c. Decreased Tc cells b. Increased erythrocytes d. Increased platelets ANS: C Illustrating the influence of chronic stress appraisal on the physiologic processes, a meta-analysis of the relationships between stressors and immunity found that a higher perception of stress was associated with reduced T cytotoxic (Tc)-cell cytotoxicity, although not with levels of circulating Th or Tc lymphocytes. Research has substantiates the other options. PTS: 1 REF: Page 355 22. What are the signs that a patient is in the adaptive stage of the general adaptation syndrome? a. He or she begins to experience elevated heart and respiratory rates. b. He or she finds it difficult to concentrate on a solution for the stress. c. The patient perceives his or her only options are to run away or fight back. d. The patient has exceeded his or her ability to cope with the current situation. ANS: C Fight-or-flight behaviors are characteristic of the more advanced adaptive stage, whereas the remaining options are noted in the initial alarm stage. PTS: 1 REF: Page 339 23. The most influential factor in whether a person will experience a stress reaction is his or her: a. General state of physical health c. Intellectual abilities b. Spiritual belief system d. Ability to cope ANS: D A person does not have a stress reNacUtRioSnINuGnTleBs.sCOthMe stress exceeds his or her coping abilities. The other options do not have the same degree of influence, as does a person’s ability to cope. PTS: 1 REF: Page 339 24. A reduction is an individual’s number of natural killer (NK) cells appears to correlate with an increased risk for the development of: a. Depression b. Type 1 diabetes c. Obsessive compulsive disorder (OCD) d. Gastroesophageal reflux disorder (GERD) ANS: A A meta-analysis of studies shows a relationship between depression and the reduction in lymphocyte proliferation and natural killer cell activity. Currently, no research supports the other options. PTS: 1 REF: Page 355 25. A nurse is providing care to a terminally ill adult who has been with his life partner for over 56 years. Research supports the nurse’s assessment of the life partner for signs of: a. Suicidal ideations c. Severe stress reaction b. Cardiac dysrhythmia d. Anorexia induced weight loss ANS: C The results of a Harvard study showed evidence that a spouse’s illness or death can increase a partner’s mortality by causing severe stress and removing a primary source of emotional, psychologic, practical, and financial support. Although the other options may exist, research does not currently support them as having the stated correlation. PTS: 1 REF: Page 357 | What's New box MULTIPLE RESPONSE 26. The effect epinephrine has on the immune system during the stress response is to increase which cells? (Select all that apply.) a. NK cells b. Immunoglobulins c. Cytokines d. T cells e. Th cells ANS: A, D The injection of epinephrine into healthy human beings is associated with a transient increase of the number of lymphocytes (e.g., T cells, natural killer (NK) cells) in the peripheral blood. This association is not true of the other options. PTS: 1 REF: Page 346 27. Which immune cells are suppressed by the corticotropin-releasing hormone (CRH)? (Select all that apply.) a. Monocyte-macrophage cells b. Cytokines c. Tc cells d. Th cells e. B cells ANS: A, D NURSINGTB.COM Direct suppressive effects of CRH have been reported on two immune cell types possessing CRH receptors—the monocyte and macrophage and CD4 (T helper) lymphocytes. CRH does not suppress the remaining options. PTS: 1 REF: Pages 353-354 28. The increased production of proinflammatory cytokines is associated with which considerations? (Select all that apply.) a. Chronic respiratory dysfunction b. Elevated anxiety levels c. Immune disorders d. Age and gender e. Dementia ANS: B, C Increased levels of proinflammatory cytokines has been shown to have a possible link between stress and immune function. The other options are not as directly linked to cytokine levels. PTS: 1 REF: Page 353 29. Which statements are true regarding lymphocytes? (Select all that apply.) a. Lymphocytes are involved in the production of the human growth hormone. b. Elevated catecholamine levels influence lymphocytes. c. Lymphocytes are synthesized in the anterior pituitary gland. d. Lymphocytes have receptors for the hormone prolactin. e. Lymphocytes produce endorphins in large amounts. ANS: A, B, D GH is synthesized from the anterior pituitary gland and is produced by lymphocytes and mononuclear phagocytic cells. Several classes of lymphocytes have receptors for prolactin, suggesting a direct effect of prolactin on immune function. Although the effects of acute elevation of catecholamines on the alteration of lymphocyte function are real, they are short lived, lasting only approximately 2 hours. The other statements regarding lymphocytes are not true. PTS: 1 REF: Page 350 | Page 352 MATCHING Match the hormone with its effects during a stress response. Hormones may be used more than once. A. Epinephrine B. Norepinephrine C. Cortisol NURSINGTB.COM 30. Constricts peripheral vessels to increase blood pressure. 31. Increases cardiac output by increasing heart rate and myocardial contractility. 32. Increases gastric secretions. 30. ANS: B PTS: 1 REF: Page 345 MSC: Norepinephrine regulates blood pressure by constricting smooth muscle in all blood vessels. 31. ANS: A PTS: 1 REF: Page 345 MSC: Epinephrine enhances myocardial contractility (inotropic effect), increases the heart rate (chronotropic effect), and increases venous return to the heart, all of which increase cardiac output and blood pressure. 32. ANS: C PTS: 1 REF: Page 347 MSC: Cortisol promotes gastric secretion in the gastrointestinal tract. Chapter 12: Cancer Biology MULTIPLE CHOICE 1. Which cancer originates from connective tissue? a. Osteogenic sarcoma c. Multiple myeloma b. Basal cell carcinoma d. Adenocarcinoma ANS: A Cancers arising from connective tissue usually have the suffix -sarcoma. The remaining options are not cancers that originate in the connective tissue and, in addition, are lacking the common suffix. PTS: 1 REF: Page 364 2. Carcinoma refers to abnormal cell proliferation originating from which tissue origin? a. Blood vessels c. Connective tissue b. Epithelial cells d. Glandular tissue ANS: B Only cancers arising from epithelial cells are called carcinomas. PTS: 1 REF: Page 364 3. Carcinoma in situ is characterized by which changes? a. Cells have broken through the local basement membrane. b. Cells have invaded immediateNsUuRrrSoIuNnGdTinBg.CtOisMsue. c. Cells remain localized in the glandular or squamous cells. d. Cellular and tissue alterations indicate dysplasia. ANS: C Carcinoma in situ (CIS) refers to preinvasive epithelial malignant tumors of glandular or squamous cell origin. These early stage cancers are localized to the epithelium and have not broken through the local basement membrane or invaded the surrounding tissue. Dysplasia refers to changes in mature cell structure. PTS: 1 REF: Page 364 4. Which term is used to describe a muscle cell showing a reduced ability to form new muscle while appearing highly disorganized? a. Dysplasia c. Myoplasia b. Hyperplasia d. Anaplasia ANS: D Anaplasia is defined as the loss of cellular differentiation, irregularities of the size and shape of the nucleus, and the loss of normal tissue structure. In clinical specimens, anaplasia is recognized by a loss of organization and a significant increase in nuclear size with evidence of ongoing proliferation. The remaining options refer to specific changes in the cell. PTS: 1 REF: Pages 368-369 5. What are tumor cell markers? a. Hormones, enzymes, antigens, and antibodies that are produced by cancer cells b. Receptor sites on tumor cells that can be identified and marked c. Cytokines that are produced against cancer cells d. Identification marks that are used in administering radiation therapy ANS: A Tumor (biologic) markers are substances produced by both benign and malignant cells that are found either in or on the tumor cells or in the blood, spinal fluid, or urine. Tumor markers may include hormones, enzymes, genes, antigens, and antibodies. The other options do not accurately describe examples of tumor markers and their function. PTS: 1 REF: Pages 365-366 6. The function of the tumor cell marker is to: a. Provide a definitive diagnosis of cancer. b. Treat certain types of cancer. c. Predict where cancers will develop. d. Screen individuals at high risk for cancer. ANS: D Screening and identifying individuals at high risk for cancer are ways tumor markers can be used. These markers are not used to definitively diagnosis or treat cancer and are not useful in predicting specific sites of cancer development. PTS: 1 REF: Page 366 NURSINGTB.COM 7. Which statement supports the hypothesis that intestinal polyps are benign neoplasms and the first stage in the development of colon cancer? a. Cancer cells accumulate slower than noncancer cells. b. An accumulation of mutations in specific genes is required for the development of cancer. c. Tumor invasion and metastasis progress more slowly in the gastrointestinal tract. d. Apoptosis is triggered by diverse stimuli, including excessive growth. ANS: B Multiple genetic mutations are required for the evolution of full-blown cancer. The remaining options do not address the progression of benign to metastatic tumors. PTS: 1 REF: Pages 372-373 8. Autocrine stimulation is the ability of cancer cells to: a. Stimulate angiogenesis to create their own blood supply. b. Encourage secretions that turn off normal growth inhibitors. c. Secrete growth factors that stimulate their own growth. d. Divert nutrients away from normal tissue for their own use. ANS: C Cancer cells must have mutations that enable them to proliferate in the absence of external growth signals. To achieve this, some cancers acquire the ability to secrete growth factors that stimulate their own growth, a process known as autocrine stimulation. The remaining options do not describe autocrine stimulation. PTS: 1 REF: Page 380 9. Apoptosis is a(an): a. Normal mechanism for cells to self-destruct when growth is excessive b. Antigrowth signal activated by the tumor-suppressor gene Rb c. Mutation of cell growth stimulated by the TP53 gene d. Transformation of cells from dysplasia to anaplasia ANS: A Normal cells have a mechanism that causes them to self-destruct when growth is excessive and cell cycle checkpoints have been ignored. Diverse stimuli, including normal development and excessive growth, trigger this self-destruct mechanism, called apoptosis. The remaining options do not describe apoptosis. PTS: 1 REF: Page 381 10. Many cancers create a mutation of ras. ras is a(an): a. Tumor-suppressor gene b. Growth-promoting gene c. Intracellular-signaling protein that regulates cell growth d. Cell surface receptor that allows signaling to the nucleus concerning cell growth ANS: C NURSINGTB.COM Up to one-third of all cancers have an activating mutation in the gene for an intracellular signaling protein called ras. This mutant ras stimulates cell growth even when growth factors are missing. The remaining options do not describe ras. PTS: 1 REF: Page 380 11. Oncogenes are genes that are capable of: a. Undergoing mutation that directs the synthesis of proteins to accelerate the rate of tissue proliferation b. Directing synthesis of proteins to regulate growth and to provide necessary replacement of tissue c. Encoding proteins that negatively regulate the synthesis of proteins to slow or halt the replacement of tissue d. Undergoing mutation that directs malignant tissue toward blood vessels and lymph nodes for metastasis ANS: A Oncogenes are mutant genes that, before mutation, direct synthesis of proteins that positively regulate (accelerate) proliferation. The remaining options do not describe oncogenes. PTS: 1 REF: Page 374 12. Burkitt lymphomas designate a chromosome that has a piece of chromosome 8 fused to a piece of chromosome 14. This is an example of which mutation of normal genes to oncogenes? a. Point mutation c. Gene amplification b. Chromosome translocation d. Chromosome fusion ANS: B Chromosome translocations, in which a piece of one chromosome is translocated to another chromosome, can activate oncogenes. One of the best examples is the t(8;14) translocation found in many Burkitt lymphomas; t(8;14) designates a chromosome that has a piece of chromosome 8 fused to a piece of chromosome 14. The remaining options are not best depicted by a Burkitt lymphoma. PTS: 1 REF: Pages 375-376 13. In childhood neuroblastoma, the N-myc oncogene undergoes which type of mutation of normal gene to oncogene? a. Point mutation c. Gene amplification b. Chromosome fusion d. Chromosome translocation ANS: C Amplifications are the result of the duplication of a small piece of a chromosome over and over again; consequently, instead of the normal two copies of a gene, tens or even hundreds of copies are present (see Chapter 4). The N-myc oncogene is amplified in 25% of childhood neuroblastoma. PTS: 1 REF: Page 376 NURSINGTB.COM 14. What aberrant change causes the abnormal growth in retinoblastoma? a. Proto-oncogenes are changed to oncogenes. b. The tumor-suppressor gene is turned off. c. Genetic amplification causes the growth. d. Chromosomes 9 and 21 are fused. ANS: B One of the first discovered tumor-suppressor genes, the retinoblastoma (Rb) gene, normally strongly inhibits the cell division cycle. When it is inactivated, the cell division cycle can proceed unchecked. The Rb gene is mutated in childhood retinoblastoma. The remaining options do not describe the abnormal growth in retinoblastoma. PTS: 1 REF: Page 376 15. Two “hits” are required to inactivate tumor-suppressor genes because: a. Each allele must be altered, and each person has two copies, or alleles, of each gene, one from each parent. b. The first hit stops tissue growth, and the second hit is needed to cause abnormal tissue growth. c. Tumor-suppressor genes are larger than proto-oncogenes, requiring two hits to effect carcinogenesis. d. The first hit is insufficient to cause enough damage to cause a mutation. ANS: A A single genetic event can activate an oncogene, acting in a dominant manner in the cell. However, each person has two copies, or alleles, of each gene, one from each parent. Therefore two hits are required to inactivate the two alleles of a tumor-suppressor gene, allowing the process to become active. The remaining options do not describe the reason two hits are required. PTS: 1 REF: Page 376 | Page 378 16. The ras gene converts from a proto-oncogene to an oncogene by: a. Designating a chromosome that has a piece of one chromosome fused to a piece of another chromosome b. Duplicating a small piece of a chromosome, repeatedly making numerous copies c. Altering one or more nucleotide base pairs d. Promoting proliferation of growth signals by impairing tumor-suppressor genes ANS: C A point mutation is the alteration of one or a few nucleotide base pairs. This type of mutation can have profound effects on the activity of proteins. A point mutation in the ras gene converts it from a regulated proto-oncogene to an unregulated oncogene, an accelerator of cellular proliferation. The remaining options do not describe point mutation as it affects the conversion of a ras gene. PTS: 1 REF: Page 375 17. How do cancer cells use the enzyme telomerase? a. To repair the telomeres to restore somatic cell growth b. As an intracellular signaling chemical to stimulate cell division c. To switch off the telomerase tNo UenRaSbINleGcTeBl.lCs OtoMdivide indefinitely d. To switch on the telomerase to enable cells to divide indefinitely ANS: D Cancer cells, when they reach a critical age, somehow activate telomerase to restore and maintain their telomeres and thereby make it possible for cells to divide over and over again. The remaining options do not describe how cancer cells use telomerase. PTS: 1 REF: Page 382 18. What are characteristics of benign tumors? a. Benign tumors invade local tissues. b. Benign tumors spread through the lymph nodes. c. Benign tumors cause systemic symptoms. d. Benign tumors include the suffix -oma. ANS: D Benign tumors are usually encapsulated and well-differentiated. They retain some normal tissue structure and do not invade the capsules surrounding them or spread to regional lymph nodes or distant locations. Benign tumors are generally named according to the tissues from which they arise and include the suffix -oma. Benign tumors do not cause systemic symptoms. PTS: 1 REF: Page 364 19. Which terms represent the correct nomenclature for benign and malignant tumors of adipose tissue, respectively? a. Liposarcoma, lipoma c. Adisarcoma, adipoma b. Lipoma, liposarcoma d. Adipoma, adisarcoma ANS: B In general, cancers are named according to the cell type from which they originate (e.g., lip for cancers that originate in adipose or fat tissue), whereas benign tumors use the suffix -oma. Cancers arising from connective tissue usually have the suffix sarcoma. PTS: 1 REF: Page 364 | Page 367 | Table 12-2 20. What is the major virus involved in the development of cervical cancer? a. Herpes simplex virus type 6 c. Human papillomavirus b. Herpes simplex virus type 2 d. Human immunodeficiency virus ANS: C Infection with specific subtypes of human papillomavirus (HPV) cause virtually all cervical cancers. The remaining options are not known to be associated with cervical cancer. PTS: 1 REF: Pages 382-383 21. The Papanicolaou (Pap) test is used to screen for which cancer? a. Ovarian c. Cervical b. Uterine d. Vaginal ANS: C NURSINGTB.COM The Pap test, an examination of cervical epithelial scrapings, readily detects early oncogenic human papillomavirus (HPV)infection. The Pap test is not used for screening the other cancer sites listed. PTS: 1 REF: Page 382 22. What is the skin-related health risk induced by some types of chemotherapy? a. Infection c. Pain b. Ultraviolet damage d. Erythema ANS: A Decreased renewal rates of the epidermal layers in the skin may lead to skin breakdown and dryness, altering the normal barrier protection against infection. Radiation therapy may cause skin erythema (redness). Pain and ultraviolet damage is not related to chemotherapies. PTS: 1 REF: Page 396 | Box 12-2 23. Which cancers are all associated with chronic inflammation? a. Brain, muscle, and endocrine b. Colon, thyroid gland, and urinary bladder c. Bone, blood cells, and liver d. Eye, tracheal, and kidney ANS: B Some organs appear to be more susceptible to the oncogenic effects of chronic inflammation; for example, the GI tract, prostate, thyroid gland, pancreas, urinary bladder, pleura, and skin. One large study found a 66% increase in the risk of lung cancer among women with chronic asthma, an inflammatory disease of the airways. At present, no research supports a link between the remaining options and chronic inflammation. PTS: 1 REF: Pages 383-384 24. Chronic inflammation causes cancer by: a. Increasing vasodilation and permeability that alter cellular response to DNA damage b. Liberating lysosomal enzymes when cells are damaged that initiates mutations c. Releasing compounds such as reactive oxygen species that promote mutations d. Increasing the abundance of leukotrienes that are associated with some cancers ANS: C Inflammatory cells release compounds, such as reactive oxygen species (ROS) and other reactive molecules, that can promote mutations and block the cellular response to DNA damage. At present, no research supports the other options as factors related to inflammation causing cancer. PTS: 1 REF: Page 384 25. Inherited mutations that predispose to cancer are almost invariably what kind of gene? a. Proto-oncogenes c. Tumor-suppressor genes b. Oncogenes d. Growth-promoting genes ANS: C NURSINGTB.COM Inherited mutations that predispose to cancer are almost invariably in tumor-suppressor genes. At present, no research supports the other options as factors related to how inherited mutations cause cancer. PTS: 1 REF: Page 379 26. What is the consequence for cells when the functioning TP53 gene is lost as a result of mutation? a. Cells undergo apoptosis. c. Cells receive less oxygen. b. Cells escape apoptosis. d. Cells adhere more readily. ANS: B The most common mutations conferring resistance to apoptosis occur in the TP53 gene. The remaining options do not accurately describe the effect when the functioning TP53 gene is lost as a result of mutation. PTS: 1 REF: Page 381 27. Which gastrointestinal tract condition can be an outcome of both chemotherapy and radiation therapy? a. Increased cell turnover c. Stomatitis b. Constipation d. Bloody stool ANS: C Chemotherapy and radiation therapy may cause a decreased cell turnover, thereby leading to oral ulcers (stomatitis), malabsorption, and diarrhea. None of the other options accurately describe related conditions resulting from chemotherapy and/or radiation therapies. PTS: 1 REF: Page 396 | Box 12-2 28. What is the role of vascular endothelial growth factor (VEGF) and basic fibroblast growth factor (bFGF) in cell metastasis? a. To stimulate growth of nearby tumor cells b. To develop new blood vessels to feed cancer cells c. To prevent cancer cells from escaping apoptosis d. To act as a chemical gradient to guide cells to blood vessels ANS: B By recruiting new vascular endothelial cells and initiating the proliferation of existing blood vessel cells, the angiogenic factors, such as VEGF and growth factor bFGF, allow small cancers to become large cancers. None of the other options accurately describe the role of the various stated factors on cell metastasis. PTS: 1 REF: Page 381 29. It has been determined that a tumor is in stage 2. What is the meaning of this finding? a. Cancer is confined to the organ of origin. b. Cancer has spread to regional structures. c. Cancer is locally invasive. d. Cancer has spread to distant sites NURSINGTB.COM ANS: C Cancer confined to the organ of origin is stage 1; cancer that is locally invasive is stage 2; cancer that has spread to regional structures, such as the lymph nodes, is stage 3; and cancer that has spread to distant sites, such as a liver cancer spreading to the lung or a prostate cancer spreading to bone, is stage 4. PTS: 1 REF: Pages 393-394 | Figure 12-25 30. Which statement is true regarding pain and cancer? a. Pain is primarily a result of pressure caused by the tumor. b. Pain indicates the metastasis of a cancer. c. Pain is usually the initial symptom of cancer. d. Pain is generally associated with late-stage cancer. ANS: D Pain is generally associated with the late stages of cancer. Pressure, obstruction, invasion of a structure sensitive to pain, stretching, tissue destruction, and inflammation can cause pain. Pain is not the initial symptom of cancer nor does it indication that the cancer has metastasized. PTS: 1 REF: Page 399 31. Which cancer may be treated with radiation delivered by brachytherapy? a. Lung c. Cervical b. Colon d. Brain ANS: C Radiation sources, such as small 125I-labeled capsules (also called seeds), can also be temporarily placed into body cavities, a delivery method termed brachytherapy. Brachytherapy is useful in the treatment of cervical, prostate, and head and neck cancers. Brachytherapy is not used in the treatment of the other cancers. PTS: 1 REF: Pages 397-398 32. The survival rate for stage IV Hodgkin disease can be as high as: a. 99% c. 40% b. 70% d. 20% ANS: B Survival rates for Hodgkin disease is 99% for stage I and 70% for stage IV. PTS: 1 REF: Page 394 | Table 12-10 33. What is the cause of anemia in a patient diagnosed with pancreatic cancer? a. Impaired pancreatic function c. Chronic bleeding b. Malnutrition d. Malabsorption of iron ANS: D Iron is malabsorbed in individuals with gastric, pancreatic, or upper intestinal cancer. Commonly associated with malignancy, mechanisms of anemia include chronic bleeding (resulting in iron deficiency), severe malnutrition, cytotoxic chemotherapy, and malignancy in blood-forming organs. The pancreas is not involved in the formation of NURSINGTB.COMblood components. Chronic blee ncy can accompany colorectal or genitourinary malignancies. PTS: 1 REF: Page 396 | Box 12-2 34. By what process do cancer cells multiply in the absence of external growth signals? a. Proto-oncogene c. Reliance on caretaker genes b. Autocrine stimulation d. Pleomorphology ANS: B Cancer cells must have mutations that enable them to proliferate in the absence of external growth signals. To achieve this, some cancers acquire the ability to secrete growth factors that stimulate their own growth, a process known as autocrine stimulation. The other options are not involved in the proliferation of cancer cells in the absence of external growth signals. PTS: 1 REF: Page 380 35. What is the role of caretaker genes? a. Maintenance of genomic integrity c. Secretion of growth factors b. Proliferation of cancer cells d. Restoration of normal tissue structure ANS: A Caretaker genes are responsible for the maintenance of genomic integrity. The other options are not roles assumed by caretaker genes. PTS: 1 REF: Page 379 36. In a normal, nonmutant state, an oncogene is referred to as a: a. Basal cell c. Caretaker gene b. Target cell d. Proto-oncogene ANS: D In its normal nonmutant state, an oncogene is referred to as a proto-oncogene. The other options are not terms used to identify a nonmutant oncogene. PTS: 1 REF: Page 374 37. Which statement is true regarding pleomorphic cells? a. Pleomorphic cells are similar in size. b. They share a common shape. c. They are a result of anaplasia. d. Pleomorphic cells differentiate uniformly. ANS: C In contrast to normal cells, which are uniform in size and shape, anaplastic cells are of variable size and shape and abnormally differentiate, making them pleomorphic. PTS: 1 REF: Pages 368-369 38. What is the most commonly reported symptom of cancer treatment? a. Nausea c. Hair loss b. Fatigue ANS: B NURSINGTdB..COWMeight loss Fatigue is the most frequently reported symptom of cancer and cancer treatment. Although patients report the other options, they are not as frequently experienced as fatigue. PTS: 1 REF: Page 396 | Box 12-2 39. The most common site of metastasis for a patient diagnosed with prostate cancer is which location? a. Bones c. Bladder b. Brain d. Kidney ANS: A The bone, especially the lumbar spine area, is the most common metastasis site for prostate cancer. PTS: 1 REF: Page 391 | Table 12-8 40. Which statement concerning benign tumors is true? a. The resulting pain is severe. c. Benign tumors are fast growing. b. Benign tumors are not encapsulated. d. The cells are well-differentiated. ANS: D A benign tumor is well-differentiated with its tissue appearing similar to the tissue from which it arose. The other options are characteristic of a malignant tumor. PTS: 1 REF: Page 364 | Table 12-1 MULTIPLE RESPONSE 41. Normally, which cells are considered immortal (never die)? (Select all that apply.) a. Germ b. Stem c. Blood d. Epithelial e. Muscle ANS: A, B Usually, germ cells (those that generate sperm and eggs) and stem cells are the only cells in the body that are immortal. Other cells in the body are not immortal and can divide only a limited number of times. The remaining options do not identify the appropriate cells. PTS: 1 REF: Page 382 42. What is the most common route for distant metastasis? (Select all that apply.) a. Seeding b. Blood c. Lymphatic vessels d. Invasion e. Proliferation ANS: B, C To transition from local to distantNmUeRtSasINtaGsTisB, .tChOeMcancer cells must also be able to invade local blood and lymphatic vessels. The remaining options are not directly related to distant metastasis. PTS: 1 REF: Page 387 43. What cellular characteristics are affected by anaplasia? (Select all that apply.) a. Size b. Ability to differentiate c. Life expectancy d. Tissue structure e. Shape ANS: A, B, D, E Anaplasia is defined as the loss of cellular differentiation, irregularities of the size and shape of the nucleus, and loss of normal tissue structure. Life expectancy is not generally included in this term. PTS: 1 REF: Page 364 44. What are the most common causes of nosocomial infections among patients with cancer? (Select all that apply.) a. Indwelling medical devices b. Suppressed immune system c. Visitor-introduced microorganisms d. Poor appetite e. Inadequate wound care ANS: A, C, E Hospital-acquired (nosocomial) infections increase because of indwelling medical devices, inadequate wound care, and the introduction of microorganisms from visitors and other individuals. A suppressed immune system and a poor appetite are possible causes of infections but they are not nosocomial in nature. PTS: 1 REF: Page 396 | Box 12-2 45. Which statements concerning aging and the occurrence of cancer are true? (Select all that apply.) a. Decline in immunologic functions b. Predisposition to nutritional inadequacies c. Unwillingness to access health care services d. Reluctance to engage in cancer screenings e. Effects of immobility on the immune system ANS: A, B, E Many common malignancies occur mostly in older age as a result of immunologic functions declining with age. Older persons are predisposed to nutritional inadequacies, and malnutrition impairs immunocompetence. Far-advanced cancer often results in immobility and general debility that worsens with age. No research supports a correlation between aging and a reluctance to seek health care, in general, or cancer screenings, in particular. NURSINGTB.COM PTS: 1 REF: Page 397 | Table 12-12 If needing more Test Banks, just let me know: [email protected] MATCHING Match the phrases with the corresponding terms. A. Is the process of cancer cell growth. B. Is used to kill cancer cells while minimizing damage to normal structures. C. Is guided by molecular analysis in specific diseases. D. Takes advantage of specific vulnerabilities in specific cancer cells. E. Provides a framework to determine treatment. 46. Chemotherapy 47. Radiation 48. Staging 49. Angiogenesis 50. Target agent 46. ANS: D PTS: 1 REF: Page 396 MSC: All chemotherapeutic agents take advantage of specific vulnerabilities in target cancer cells. 47. ANS: B PTS: 1 REF: Pages 397-398 MSC: Radiation therapy is used to kill cancer cells while minimizing damage to normal structures. 48. ANS: E PTS: 1 REF: Pages 393-394 MSC: Staging may alter the choice of therapy, with more aggressive therapy being delivered to more invasive disease (advanced staging). 49. ANS: A PTS: 1 REF: Page 381 MSC: Angiogenesis is the process of growth and proliferation of cancer cells. 50. ANS: C PTS: 1 REF: Pages 397-398 MSC: The newest highly targeted agents that are used to treat cancer exploit specific vulnerabilities uncovered by molecular analysis in specific diseases. MICS: The newest highly targeted agents that are used to treat cancer exploit specific vulnerabilities uncovered by molecular analysis in specific diseases. Match the organism factor with the cancer it causes. A. HPV B. Human herpesvirus (HHV) 8 C. Hepatis B virus (HBV) D. Helicobacter pylori 51. Cervical cancer 52. Kaposi sarcoma 53. Liver cancer 54. Stomach cancer NURSINGTB.COM 51. ANS: A PTS: 1 REF: Pages 382-383 MSC: HPV infection causes human cervical cancer. 52. ANS: B PTS: 1 REF: Page 383 MSC: HHV-8 infection causes Kaposi sarcoma. 53. ANS: C PTS: 1 REF: Page 383 MSC: Chronic hepatitis infection with HBV or hepatis C virus (HCV) is the leading cause of liver cancer. 54. ANS: D PTS: 1 REF: Page 384 MSC: Chronic H. pylori-associated inflammation causes stomach cancer. Chapter 13: Cancer Epidemiology MULTIPLE CHOICE 1. Which characteristic among women correlates with a high morbidity of cancer of the colon, uterus, and kidney? a. Women older than 45 years of age b. Women who have never had children c. Women who have a high body mass index D. Woman who have smoked for more than 10 years ANS: C A recent hypothesis states that the observed increased incidence of such cancers as breast, endometrium, colon, liver, kidney, and adenomas of the esophagus may be associated with obesity. No current research supports the remaining options. PTS: 1 REF: Pages 417-418 | Figure 13-11 2. Which substance has been shown to increase the risk of cancer when used in combination with tobacco smoking? a. Alcohol c. Antihistamines b. Steroids d. Antidepressants ANS: A Alcohol interacts with smoke, increasing the risk of malignant tumors, possibly by acting as a solvent for the carcinogenic chemicals in smoke products. No current research supports the remaining options asNhUavRiSnIgNGanTBin.CcOreMased effect on the incidence of cancer when used in combination with tobacco smoking. PTS: 1 REF: Page 420 | Page 422 3. Which cancers pose the highest risk for radiologists? a. Thyroid c. Leukemia b. Breast d. Brain ANS: C Ionizing radiation exposure places radiologists at risk for the development of leukemia, lymphoma, and skin cancers. PTS: 1 REF: Page 424 | Table 13-8 4. Research has shown a link between cancer and which sexually transmitted disease? a. Syphilis c. Human papillomavirus b. Gonorrhea d. Pelvic inflammatory disease ANS: C Human papillomavirus (HPV) is the most common sexually transmitted virus in the United States. High-risk, or oncogenic, HPVs can cause cancer. A persistence of infection with high-risk HPV is a prerequisite for the development of cervical intraepithelial neoplasia (CIN) lesions and invasive cervical cancers. No research supports such a link between the remaining options and cancer. PTS: 1 REF: Pages 423-424 5. Research has supported that alcohol consumption has a convincing impact on increasing the risk of which cancer? a. Lung c. Pharynx b. Breast d. Pancreas ANS: C Research had shown that alcohol consumption has a convincing positive impact on the risk of developing pharynx cancer. No research supports such a connection with the remaining options. PTS: 1 REF: Page 422 | Table 13-5 6. Research supports the premise that exercise has a probable impact on reducing the risk of which cancer? a. Liver c. Stomach b. Endometrial d. Colon ANS: D The World Cancer Research Fund summarizes the effects as convincing for cancers of the colon and probable for postmenopausal breast cancer and endometrial cancer. The relationship is not supported for the remaining options. PTS: 1 REF: Page 422 | Table 13-6 7. The World Health Organization (WNUHROSI)NdGeTfiBn.eCsOgMrade 1 (overweight) as a BMI of: a. 18.5 to 24.9 c. 30 to 39.9 b. 25 to 29.9 d. 40 to 50.9 ANS: B A BMI of 25 to 29.9 kg/m2 is considered a grade 1 (overweight) classification. A BMI of 18.5 to 24.9 kg/m2 is considered normal range, whereas 30 to 39.9 kg/m2 is a grade 2 (severe overweight) classification, and a BMI higher than 40 kg/m2 is considered grade 3 (morbidly overweight). PTS: 1 REF: Page 418 | Table 13-4 MULTIPLE RESPONSE 8. Tobacco smoking is associated with which cancers? (Select all that apply) a. Lung b. Skin c. Bladder d. Kidney e. Pancreas ANS: A, C, D, E Besides lung cancer, tobacco increases the risk for cancers of the mouth, lips, nasal cavity and sinuses, larynx, pharynx, esophagus, stomach, pancreas, kidney, bladder, uterus, cervix, colon and rectum, ovary, and acute leukemia. No current research links tobacco smoking to skin cancer. PTS: 1 REF: Page 413 9. The significant risk factors associated with the development of basal cell carcinoma include: (Select all that apply.) a. Light-colored hair b. Pale eyes c. Fair skin d. Freckles e. History of acne ANS: A, B, C, D Risk factors associated with a high risk of melanoma include light-colored hair, eyes, and skin; an inability to tan; and a tendency to freckle, sunburn, and develop nevi. No current research supports acne as a risk factor. PTS: 1 REF: Page 431 10. Which environmental factors have been shown to have greatest significant effect on the frequency of genetic mutations in humans? (Select all that apply.) a. Tobacco use b. Radiation exposure c. Obesity d. Severe allergies e. Stressful occupation ANS: A, B, C NURSINGTB.COM The number of environmental factors can alter the frequency and consequences of genetic mutations. The most significant factors include smoking, radiation, obesity, and a few oncogenic viruses. Allergies and stress have not been strongly linked as risk factors for genetic mutations. PTS: 1 REF: Pages 413-433 11. Research data support the relationship between the exposure of ultraviolet light (UVL) and the development of: (Select all that apply.) a. Basal cell carcinoma b. Squamous cell carcinoma c. Hodgkin lymphoma d. Non-Hodgkin lymphoma e. Soft-tissue sarcoma ANS: A, B Exposure to UVL can cause basal cell carcinoma and squamous cell carcinoma. No current research supports a causative relationship between the remaining options and UVL exposure. PTS: 1 REF: Page 431 12. Alcohol consumption has been proven to be a consistent risk factor in the development of which major cancer? (Select all that apply.) a. Colorectal b. Kidney c. Breast d. Esophageal e. Pancreatic ANS: A, C, D Research supports a consistent link between alcohol consumption and an increased risk for breast, colorectal, and esophageal cancers; no such data have yet been determined for the remaining options. PTS: 1 REF: Page 420 13. Nutritional counseling regarding cancer stem cell repopulation must stress the importance of including sufficient quantities of what in a daily diet? (Select all that apply.) a. Vitamin A b. Vitamin D c. Protein d. Vitamin C e. Water ANS: A, B Cancer stem cells use several developmental mechanisms for the self-renewal, and these mechanisms appear to be fundameNnUtaRlStIoNGthTeBi.nCiOtiMation and recurrence of tumors. Adequate consumption of specific food compounds, including vitamin A and D, genistein, green tea epigallocatechin gallate (EGCG), sulforaphane, theanine, curcumin, and choline may suppress cancer stem renewal. No current research supports such a connection with the other options. PTS: 1 REF: Page 416 14. Which personal lifestyle choices will likely have a positive impact on reducing an individual’s risk for developing cancer? (Select all that apply.) a. Avoiding crowds b. Wearing a hat while golfing c. Eating a strict vegetarian diet d. Having regular health screenings e. Staying indoors during smog alerts ANS: B, E Avoiding exposure to ultraviolet radiation and air pollutants will help decrease the risk of developing skin and lung cancer. Although avoiding crowds may reduce the risk of contracting infections, no research suggests such a decision will reduce the risk of cancer. Eating a vegetarian diet will reduce the consumption of red meat, but no research suggests that such a diet is a factor in reducing the risk of cancer. Health screening, although important, can detect the possible presence of a specific disease, but it does not prevent diseases. PTS: 1 REF: Page 431 | Page 433 15. Which individual is at risk for developing lung and respiratory tract cancers because of exposure to known cancer-inducing environmental factors? (Select all that apply.) a. A patient whose partner smokes 1 1/2 packs of cigarettes a day b. A 10-year employee of an asbestos fiber manufacturer c. A farmer whose equipment is primarily driven by diesel engines d. An older adult with a family history of lung cancer and chronic obstructive pulmonary disease (COPD) e. A 10-year delivery employee for a paper manufacturing company ANS: A, B, C, E Numerous environmental factors can cause inflammation and include, for example, inhaling tobacco smoke, asbestos fibers, or fine particles in the air from diesel engine exhaust and industrial sources. Sulfur dioxide is produced by power plants burning oil and coal, copper smelting, and paper mills. These sources are major factors in lung and other respiratory tract cancers. A family history is not considered a risk factor since these cancers are generally affected by environmental rather than genetic factors. PTS: 1 REF: Page 403 | Page 433 16. Which primary prevention strategy has a substantial likelihood of reducing the risk of developing cancer? (Select all that apply.) a. Offering free smoking cessation classes to college students b. Establishing a support group for patients receiving chemotherapy c. Providing a breast cancer screening at a senior center twice a year d. Supplying low sugar, low-fat sNnUaRckSIsNaGt TaBn.CafOteMr-school tutoring program e. Funding a support service that offers transportation to cancer treatment centers ANS: A, C, D Targeted interventions can substantially reduce the incidences of cancers worldwide and should include primary prevention strategies to decrease or eliminate certain lifestyle factors including tobacco avoidance and the cessation of smoking, a reduction in obesity and alcohol consumption, an increase in physical activity, and the implementation of vaccination programs for liver and cervical cancer, as well as effective early detection programs for colorectal, breast, and cervical cancer. Support groups and transportation to cancer centers are secondary strategies. PTS: 1 REF: Page 409 | What's New box 17. When considering topics for a series of community discussions related to cancer prevention strategies, the nurse will include: (Select all that apply.) a. Healthy eating b. Tobacco avoidance c. Importance of early diagnosis d. Impact of obesity on one’s health e. Age-appropriate exercise routines ANS: A, B, D, E Eliminating smoking, decreasing obesity, participating in appropriate exercises, eliminating infections, and avoiding an unhealthy diet are all essential for preventing cancer. Screening is not a prevention strategy. PTS: 1 REF: Page 412 | What's New box NURSINGTB.COM Chapter 14: Cancer in Children MULTIPLE CHOICE 1. What congenital malformation is commonly linked to acute leukemia in children? a. Down syndrome c. Retinoblastoma b. Wilms tumor d. Neuroblastoma ANS: A Trisomy 21 (Down syndrome) is the most common genetic defect linked to the development of acute leukemia. Children with Down syndrome have a 10- to 20-fold increased risk of developing acute lymphoblastic and myelogenous leukemia and a higher risk for developing acute megakaryocytic leukemia. No current research supports a link between the other stated congenital malformations and acute leukemia. PTS: 1 REF: Page 444 2. When are childhood cancers most often diagnosed? a. During infancy c. After diagnosis of a chronic illness b. At peak times of physical growth d. After an acute illness ANS: B Childhood cancers are most often diagnosed during peak times of physical growth. No current research supports the other options. PTS: 1 REF: Page 443 NURSINGTB.COM 3. Prenatal exposure to diethylstilbestrol (DES) can result in which type of cancer? a. Breast cancer c. Vaginal cancer b. Leukemia d. Lymphoma ANS: C DES has been identified as a transplacental chemical carcinogen; a small percentage of the daughters of women who took DES during pregnancy developed adenocarcinomas of the vagina. No current research supports a link between the drug and other cancers. PTS: 1 REF: Page 445 4. Currently, what percentage of children with cancer can be cured? a. 40% c. 60% b. 50% d. 80% ANS: D Currently, more than 80% of children diagnosed with cancer are cured. PTS: 1 REF: Page 445 5. Most childhood cancers arise from the: a. Epithelium c. Embryologic ectodermal layer b. Mesodermal germ layer d. Viscera ANS: B Most childhood cancers originate from the mesodermal germ layer that gives rise to connective tissue, bone, cartilage, muscle, blood, blood vessels, gonads, kidney, and the lymphatic system. The other options are not common sites from which cancers originate. PTS: 1 REF: Page 443 6. Which form of cancer is linked to congenital malformation syndromes? a. Wilms tumor c. Osteosarcoma b. Retinoblastoma d. Rhabdomyosarcoma ANS: A Wilms tumors are linked with other genetically linked childhood cancers. It is the only form among the available options that is associated with congenital malformation syndromes. PTS: 1 REF: Page 443 | Table 14-1 7. Research data support a carcinogenic relationship in children resulting from exposure to which virus? a. Herpes simplex virus c. Varicella zoster virus b. Influenza d. Epstein-Barr virus ANS: D The strongest association between viruses and the development of cancer in children has been the Epstein-Barr virus (EBV), Burkitt lymphoma, nasopharyngeal carcinoma, and Hodgkin disease. No current research supports a link between the remaining options and childhood cancer. NURSINGTB.COM PTS: 1 REF: Page 445 8. A child diagnosed with acquired immunodeficiency syndrome (AIDS) has an increased risk of developing: a. Non-Hodgkin lymphoma c. Epstein-Barr b. Retinoblastoma d. Leukemia ANS: A Children with AIDS have an increased risk of developing non-Hodgkin lymphoma and Kaposi sarcoma. Leukemia is linked to retinoblastoma. Epstein-Barr has been linked to the development of some cancers, whereas no known link has been found between AIDS and the development of leukemia. PTS: 1 REF: Page 445 9. Which intervention has the greatest affect on a child’s mortality rate when diagnosed with cancer? a. Age at the time of diagnosis b. Participation in clinical trials c. Proximity to a major cancer treatment center d. Parental involvement in the treatment planning ANS: B Mortality rates have significantly declined in the past 40 years largely as a result of advances in treatment and participation in clinical trials. Although important, the other options are not believed to have an affect on the decline of the mortality rate of childhood cancer. PTS: 1 REF: Page 445 10. Which statement is likely true regarding children being treated for cancer with radiation therapy? a. They will most likely have a successful remission of tumor growth. b. They seldom require follow-up maintenance treatments. c. They are prone to experience severe developmental delays. d. They are at increased risk for developing adult cancers. ANS: D Although the need exists for long-term studies, research has shown a correlation between radiation-induced malignancies from radiotherapy (as in cancer treatment) or radiation exposure from diagnostic imaging; both have shown to increase the risk of developing cancer during adulthood. The other options are not necessarily true regarding the outcomes of radiation therapy for childhood cancers. PTS: 1 REF: Page 445 11. How should the nurse reply when a parent questions why a computed tomographic (CT) scan of the head was not ordered for their 5-year-old child after a minor fall? a. Physicians are cautious about ordering CT scan on children younger than 10 years of age. b. CT scans are seldom conclusivNeUwRShIeNnGuTsBe.dCOtoMdiagnosis head injuries in young children. c. The child’s symptoms will determine whether a CT scan is necessary and worth the expense. d. Research suggests that repeated CT scans can increase the risk of developing brain cancer. ANS: D Findings from a recent study of 176,587 children suggest that those who have two or three CT scans of the head before the age of 22 years are three times more likely to develop brain cancer as those in the general population, and the risk of developing leukemia is three times as great in those who received five to ten CT scans. The other options do not represent the logic behind not ordering a CT scan in relationship to minor head trauma. PTS: 1 REF: Page 445 | What's New box MULTIPLE RESPONSE 12. Childhood exposure to which risk factors increases the susceptibility for developing cancers? (Select all that apply.) a. Low birth weight b. Chemotherapy c. Ionizing radiation d. Cigarette smoke e. Hodgkin disease ANS: B, C, E Childhood exposure to ionizing radiation, drugs, or existing cancer-causing viruses has been associated with the risk of developing cancer. Although unhealthy, no current research identifies cigarette smoke or low birth weight as risk factors for childhood cancer. PTS: 1 REF: Page 445 | Table 14-3 13. Which statement is true concerning the difference between adult and childhood cancers? (Select all that apply.) a. Numerous differences exist between these two categories of cancer. b. Childhood cancers are far less likely to be associated with genetic mutations. c. Environmental risks are strongly associated with childhood cancers. d. Exposure to pesticides is a minor risk for the development of adult cancers. e. Maternal exposure to carcinogenic substances presents little risk to the fetus. ANS: A, B Overall, cancers in children are very different than adult cancers and are associated with far fewer genetic mutations. Research does not support the other options. PTS: 1 REF: Page 442 | Page 444 | What's New box 14. Most childhood cancers originate from the mesodermal germ layer that ultimately produces which of the following? (Select all that apply.) a. Connective tissue b. Digestive system c. Muscles d. Kidneys e. Blood ANS: A, C, D, E NURSINGTB.COM Most childhood cancers originate from the mesodermal germ layer, which gives rise to connective tissue, bone cartilage, muscle, blood, blood vessels, gonads, kidney, and the lymphatic system. Cancers of the digestive tract do not originate in the mesodermal germ layer. PTS: 1 REF: Page 443 15. Which statements are true regarding cancers that develop in children? (Select all that apply.) a. Childhood cancers tend to be fast growing. b. Childhood cancers are diagnosed during growth spurts. c. Childhood cancer generally metastasizes by the time of diagnosis. d. Childhood cancer is typically at low risk for aggressive metastasis. e. Childhood cancers respond well to standardized treatment modalities. ANS: A, B, C Childhood cancers are most often diagnosed during peak times of physical growth. In general, they are extremely fast growing, with 80% having distant spread (metastases) at diagnosis. The other options are not true regarding childhood cancers. PTS: 1 REF: Page 443 16. The nurse is preparing a discussion on cancer and its occurrence among college-aged students. Which cancers will the nurse include in the discussion? (Select all that apply.) a. Colorectal b. Testicular c. Thyroid d. Breast e. Lung ANS: A, B, C, D The most common cancers among the adolescent and young adult population (15 to 39 years of age) are Hodgkin lymphoma, leukemia, germ-cell tumors (particularly testicular), central nervous system tumors, non-Hodgkin lymphoma, thyroid cancer, melanoma, sarcomas, and breast, cervical, liver, thyroid and colorectal cancers. Lung cancer generally develops after chronic inhalation of nicotine-containing products. PTS: 1 REF: Page 442 NURSINGTB.COM Chapter 15: Structure and Function of the Neurologic System MULTIPLE CHOICE 1. Which pathway carries sensory information toward the central nervous system (CNS)? a. Ascending c. Somatic b. Descending d. Efferent ANS: A Peripheral nerve pathways can be afferent (ascending) pathways that carry sensory impulses toward the CNS. The remaining options do not carry sensory information to the CNS. PTS: 1 REF: Page 448 2. Which type of axon transmits a nerve impulse at the highest rate? a. Large nonmyelinated c. Small nonmyelinated b. Large myelinated d. Small myelinated ANS: B If the myelin layer is tightly wrapped many times around the axon and is forming the nodes of Ranvier, then conduction velocity increases and the neuron is referred to as myelinated. The increased diameter of the myelinated axons allows for the transmission of impulses at a faster rate. The other options do not affect nervous impulse transmission rates. PTS: 1 REF: Pages 4N4U8-R4S5I0NGTB.COM 3. Which nerves are capable of regeneration? a. Nerves within the brain and spinal cord b. Peripheral nerves that are cut or severed c. Myelinated nerves in the peripheral nervous system d. Unmyelinated nerves of the peripheral nervous system ANS: C Nerve regeneration is limited to only myelinated fibers and generally occurs only in the peripheral nervous system. PTS: 1 REF: Pages 450-451 4. The neurotransmitter, norepinephrine, is secreted in the: a. Somatic nervous system c. Sympathetic postganglion b. Parasympathetic preganglion d. Parasympathetic postganglion ANS: C Most postganglionic sympathetic fibers release norepinephrine (adrenaline). The remaining options do not reflect the correct site of norepinephrine secretion. PTS: 1 REF: Pages 473-474 5. Both oligodendroglia and Schwann cells share the ability to: a. Form a myelin sheath c. Transport nutrients b. Remove cellular debris d. Line the ventricles ANS: A The function of oligodendroglia (oligodendrocytes) is to deposit myelin within the central nervous system (CNS). Oligodendroglia are the CNS counterpart of Schwann cells. The remaining options are not reflective of the common function of these structures. PTS: 1 REF: Pages 449-450 6. During a synapse, what change occurs after the neurotransmitter binds to the receptor? a. The permeability of the presynaptic neuron changes; consequently, its membrane potential is changed as well. b. The permeability of the postsynaptic neuron changes; consequently, its membrane potential is changed as well. c. The postsynaptic cell prevents any change in permeability and destroys the action potential. d. The presynaptic cell synthesizes and secretes additional neurotransmitters. ANS: B The binding of the neurotransmitter at the receptor site changes the permeability of the postsynaptic neuron and, consequently, its membrane potential. The remaining options do not accurately describe the occurrence. PTS: 1 REF: Pages 452-453 7. What name is given to a large network of neurons within the brainstem that is essential for maintaining wakefulness? NURSINGTB.COM a. Midbrain c. Medulla oblongata b. Reticular activating system d. Pons ANS: B The reticular activating system is essential for maintaining wakefulness. The remaining options are not essential to this function. PTS: 1 REF: Page 454 8. Thought and goal-oriented behaviors are functions of which area of the brain? a. Cerebellum c. Prefrontal lobe b. Limbic system d. Occipital lobe ANS: C The prefrontal area is responsible for goal-oriented behavior (i.e., ability to concentrate), short-term or recall memory, and the elaboration of thought and inhibition on the limbic (emotional) areas of the CNS. The remaining options are not involved in these functions. PTS: 1 REF: Page 456 9. The region responsible for the motor aspects of speech is located in the: a. Wernicke area in the temporal lobe c. Wronka area in the parietal lobe b. Broca area in the frontal lobe d. Barlow area in the occipital lobe ANS: B Broca speech area is the only region responsible for the motor aspects of speech. PTS: 1 REF: Page 457 10. Parkinson and Huntington diseases are associated with defects in which area of the brain? a. Thalamus c. Cerebellum b. Medulla oblongata d. Basal ganglia ANS: D Parkinson and Huntington diseases are conditions associated with defects of the basal ganglia. No current research supports the role of any of the other options in these diseases. PTS: 1 REF: Page 457 11. Maintenance of a constant internal environment and the implementation of behavioral patterns are main functions of which area of the brain? a. Thalamus c. Subthalamus b. Epithalamus d. Hypothalamus ANS: D Hypothalamic function falls into two major areas: (1) maintenance of a constant internal environment, and (2) implementation of behavioral patterns. The remaining options do not address these functions. PTS: 1 REF: Page 459 12. The ability of the eyes to track moving objects through a visual field is primarily a function of which colliculi? NURSINGTB.COM a. Inferior c. Mid b. Superior d. Posterior ANS: B The superior colliculi are involved with voluntary and involuntary visual motor movements (e.g., the ability of the eyes to track moving objects in the visual field). Tracking moving objects is not the primary function of the remaining options. PTS: 1 REF: Page 459 13. What parts of the brain mediate the expression of affect, both emotional and behavioral states? a. Hypothalamus and subthalamus c. Limbic system and prefrontal cortex b. Parietal and frontal lobes d. Basal ganglia and medulla oblongata ANS: C Extensive connections with the limbic system and prefrontal cortex mediate the expression of affect, both emotional and behavioral states. The remaining options are not involved in these expressions. PTS: 1 REF: Page 457 14. Reflex activities concerned with heart rate, blood pressure, respirations, sneezing, swallowing, and coughing are controlled by which area of the brain? a. Pons c. Cerebellum b. Midbrain d. Medulla oblongata ANS: D The medulla oblongata makes up the myelencephalon and is the lowest portion of the brainstem. Reflex activities, such as heart rate, respiration, blood pressure, coughing, sneezing, swallowing, and vomiting, are controlled only in this area. PTS: 1 REF: Page 460 15. From which part of the midbrain do cranial nerves V to VIII emerge? a. Midbrain c. Medulla oblongata b. Pons d. Lateral colliculi ANS: B The nuclei of cranial nerves V through VIII (see Table 15-6 for discussion) are located only in the pons. PTS: 1 REF: Pages 459-460 16. From which part of the midbrain do cranial nerves IX to XII emerge? a. Midbrain c. Medulla oblongata b. Pons d. Lateral colliculi ANS: C The nuclei of cranial nerves IX through XII (see Table 15-6 for discussion) are located only in the medulla oblongata. PTS: 1 REF: Page 46N0URSINGTB.COM 17. Which area of the brain assumes the responsibility for conscious and unconscious muscle synergy and for maintaining balance and posture? a. Cerebrum c. Diencephalon b. Cerebellum d. Brainstem ANS: B The cerebellum is responsible for conscious and unconscious muscle synergy and for maintaining balance and posture. This role is not assumed by any of the remaining options. PTS: 1 REF: Page 459 18. Which statement is true regarding upper motor neurons? a. Upper motor neurons directly influence muscles. b. They modify spinal reflex arcs. c. Upper motor neurons are located in the gray matter of the spinal cord. d. They extend their dendritic processes out of the CNS. ANS: B Upper motor neurons (i.e., corticospinal tract) are the classification of motor pathways completely contained within the CNS. Their primary roles include directing, influencing, and modifying reflex arcs, lower-level control centers, and motor and some sensory neurons. The remaining options do not accurately describe the characteristic functions of upper motor neurons. PTS: 1 REF: Page 462 19. The membrane that separates the brain’s cerebellum from its cerebrum is the: a. Tentorium cerebelli c. Arachnoid membrane b. Falx cerebri d. Falx cerebelli ANS: A The tentorium cerebelli is a membrane that separates the cerebellum below from the cerebral structures above. The remaining options do not perform the function described in the stem. PTS: 1 REF: Page 464 20. The function of arachnoid villi is to: a. Produce cerebrospinal fluid b. Provide nutrients to the choroid plexuses c. Transmit impulses within the meninges d. Absorb cerebrospinal fluid into the cerebral venous sinuses ANS: D CSF is reabsorbed by means of a pressure gradient between the arachnoid villi and the cerebral venous sinuses. The remaining options do not accurately describe the function of arachnoid villi. PTS: 1 REF: Page 466 21. Where is the cerebrospinal fluid pNroUdRuScIeNdG?TB.COM a. Arachnoid villi c. Ependymal cells b. Choroid plexuses d. Pia mater ANS: B The choroid plexuses are the structures that produce CSF; they arise from the pia mater. The remaining options do not have a role in CSF production. PTS: 1 REF: Pages 465-466 22. Which of the meninges closely adheres to the surface of the brain and spinal cord and follows the sulci and fissures? a. Dura mater c. Pia mater b. Arachnoid d. Inner dura ANS: C The delicate pia mater closely adheres to the surface of the brain and spinal cord and even follows the sulci and fissures. The remaining options are not represented in the description provided in the stem. PTS: 1 REF: Pages 464-465 23. Norepinephrine produces what primary response? a. Increased contractility of the heart c. Vasoconstriction b. Release of renin from the kidney d. Pupillary dilation ANS: C The primary response from norepinephrine is the stimulation of the 1-adrenergic receptors that cause vasoconstriction. The remaining options do not accurately reflect the primary response of norepinephrine. PTS: 1 REF: Page 474 24. What is an effect of the sympathetic nervous system? a. Stimulation of peristalsis c. Reduction in heart rate b. Increased blood sugar levels d. Pupillary constriction ANS: B In general, sympathetic stimulation promotes responses that are concerned with the protection of the individual, which include increased blood sugar levels, temperature, and blood pressure. The responses noted in the remaining options are not a result of sympathetic stimulation. PTS: 1 REF: Page 476 25. The brain receives approximately what percentage of the cardiac output? a. 80% c. 20% b. 40% d. 10% ANS: C The brain receives approximately 20% of the cardiac output, or 800 to 1000 ml of blood flow per minute. PTS: 1 REF: Page 45N4URSINGTB.COM 26. The collateral blood flow to the brain is provided by the: a. Carotid arteries c. Circle of Willis b. Basal artery d. Vertebral arteries ANS: C The arterial circle (circle of Willis) (see Figure 15-20) is a structure credited with the ability to compensate for reduced blood flow from any one of the major contributors (collateral blood flow). The remaining options are not considered sources of collateral blood flow. PTS: 1 REF: Page 467 27. The nurse recognizes that a patient’s diagnosis of a viral infection of the brain’s meningeal layer is supported by which diagnostic laboratory result? a. Chloride <110 mEq/L c. Protein <10 mg/dl b. Leukocytes >10/mm3 d. Glucose <40 mg/dl ANS: B Viral infections causing meningitis will produce a white blood cell (leukocyte) count greater than 10/mm 3. The chloride level is observed in tuberculous meningitis, the protein level has little clinical significance, and the glucose level is observed in patients with acute bacterial meningitis. PTS: 1 REF: Page 479 | Table 15-8 MULTIPLE RESPONSE 28. Which cranial nerves contain parasympathetic nerves? (Select all that apply.) a. I (olfactory) d. IX (glossopharyngeal) b. III (oculomotor) e. X (vagus) c. VII (facial) ANS: B, C, D, E All but cranial nerve I (olfactory) contain parasympathetic nerves. PTS: 1 REF: Page 473 29. The sympathetic nervous system primarily serves to protect an individual by doing which of the following? (Select all that apply.) a. Decreasing mucous production d. Decreasing sweat excretion b. Increasing blood sugar levels e. Increasing blood pressure c. Increasing body temperature ANS: B, C, E In general, sympathetic stimulation promotes responses that are concerned with the protection of the individual, which include increasing blood sugar levels, temperature, and blood pressure. The remaining options are not protective measures resulting from initiatives made by the sympathetic nervous system. PTS: 1 REF: Page 476 NURSINGTB.COM 30. The aging process brings about what changes to human cells? (Select all that apply.) a. Increased neurofibrillary tangles d. Decreased myelin presence b. Imbalance of neurotransmitters e. Altered dendrite structure c. Increased neuron production ANS: A, B, D, E Principal cellular changes associated with aging include a decrease in the number of neurons, decreased myelin, decreased number of dendritic processes and synaptic connections, intracellular neurofibrillary tangles, and an imbalance in the amount and distribution of neurotransmitters. The aging process does not bring about an increase in neuron production. PTS: 1 REF: Page 478 | Box 15-5 31. Which statement is true regarding the blood-brain barrier (BBB)? (Select all that apply.) a. The BBB is dependent on astrocytes. b. It uses the meningeal layers of the brain. c. It restricts the flow of large molecules, such as potassium. d. Naturally occurring inflammatory mediators affect the BBB. e. The BBB appears to play a role in slowing down the onset of degenerative brain disease. ANS: A, C, D, E The BBB is a term used to describe cellular structures that selectively inhibit certain substances in the blood from entering the interstitial spaces of the brain or CSF. This term emphasizes the impermeability of the nervous system to large and potentially harmful molecules. Astrocytes wrap their foot processes around the epithelial cells of brain capillaries, thereby contributing to the formation of the BBB. Tight junctions between capillary endothelial cells form a barrier that regulates the passage of ions (e.g., sodium, potassium) that could interfere with nerve transmission, prevent toxins from entering the brain, and promote transport of nutrients and the removal of metabolites. Hormones, neurotransmitters, and inflammatory mediators can affect BBB permeability. Inhibiting these endogenous chemicals with drug therapy may reduce brain edema and slow the onset of degenerative brain diseases. The statement regarding the meningeal layers of the brain is incorrect. PTS: 1 REF: Page 469 | Box 15-4 32. A nurse caring for an older adult patient would expect which functional changes to occur? (Select all that apply.) a. Increased risk for falls b. Increased risk for falls c. Loss of muscle in the arms and legs d. Decrease in the need for social interaction e. Increased agitation and frustration levels ANS: A, B, C Functional changes with aging include skeletal muscle atrophy, progressive deficits in taste and smell, and a decrease in neuromuscular control with changes in gait and posture. Neither increased agitation and fruNsUtrRaStiIoNnGnToBr.CaOdMecreased need for social interaction is considered an expected change resulting from the aging process. PTS: 1 REF: Page 478 | Box 15-5 33. What evidence does the nurse expect to see when a patient experiences trauma to the hypothalamus? (Select all that apply.) a. Uneven expression of mood b. Unstable blood glucose levels c. Poor regulation of body temperature d. Visual disturbances such as blurred vision e. Nausea, vomiting, and symptoms of gastroesophageal reflux disease ANS: A, B, C The hypothalamus forms the base of the diencephalon. Hypothalamic function controls autonomic nervous system (ANS) function, regulation of body temperature, endocrine function (e.g., unstable glucose levels), and regulation of emotional expression. Visual and gastrointestinal symptoms would not likely be related to hypothalamus function. PTS: 1 REF: Page 459 | Box 15-3 34. A Schwann cell: (Select all that apply.) a. Can form the myelin sheath. b. Is also referred to as a neurolemmocyte. c. Affects the function of the nodes of Ranvier. d. Is located in the peripheral nervous system. e. Is responsible of decreasing conduction velocity. ANS: A, B, C, D The Schwann cell, or neurolemmocyte, is a glial cell that wraps around and covers axons in the peripheral nervous system. Schwann cells form and maintain the myelin sheath, and the nodes of Ranvier form the spaces on either side of the Schwann cell. If the myelin layer is tightly wrapped many times around the axon, forming nodes of Ranvier, then it increases conduction velocity and the neuron is referred to as myelinated. PTS: 1 REF: Page 450 MATCHING Match the function with the appropriate cranial nerve. A. Fibers emerge from the posterior midbrain and exit from the skull to run to the eye. B. Provides motor and sensory functions to the face, mouth, nose, and eyes. C. Fibers emerge from the midbrain, exit from the skull, and extend to the eye. D. Controls motor functions to the pharynx and salivary glands and sensory functions from the pharynx and tongue. E. Innervates muscles that move the eye laterally. F. Affects control over the motor fibers to the muscles of tongue and sensory impulses from the tongue to the brain. G. Is purely sensory and carries impulses for the sense of smell. H. Is made up of parasympathetic motor fibers that supply the smooth muscles of the abdominal oNrgUaRnSsI.NGTB.COM I. Transmits impulses for the sense of hearing. J. Carries sensory and motor fibers to the pharynx and larynx. 35. Glossopharyngeal nerve 36. Oculomotor nerve 37. Trochlear nerve 38. Abducens nerve 39. Trigeminal nerve 40. Hypoglossal nerve 41. Vagus nerve 42. Spinal accessory nerve 43. Olfactory nerve 44. Vestibulocochlear nerve 35. ANS: D PTS: 1 REF: Page 472 | Table 15-6 MSC: The glossopharyngeal nerve affects control over the motor fibers that serve the pharynx (throat) and salivary glands, as well as the sensory fibers that carry impulses from the pharynx, posterior tongue (taste buds), and pressure receptors of the carotid artery. 36. ANS: C PTS: 1 REF: Page 472 | Table 15-6 MSC: The oculomotor nerve fibers emerge from the midbrain, exit from the skull, and extend to the eye. These fibers control: (1) the motor fibers to the inferior oblique, superior, inferior, and medial rectus extraocular muscles that direct the eyeball; (2) the levator muscles of the eyelid; (1) the smooth muscles of the iris and ciliary body; and (4) proprioception (sensory) to brain from the extraocular muscles. 37. ANS: A PTS: 1 REF: Page 472 | Table 15-6 MSC: The trochlear nerve fibers emerge from posterior midbrain and exit from skull to affect control over the proprioceptor and motor fibers for the superior oblique muscle of eye (extraocular muscle). 38. ANS: E PTS: 1 REF: Page 472 | Table 15-6 MSC: The abducens nerve affects control over the motor fibers to the lateral rectus muscle and the proprioceptor fibers from the same muscle to the brain, allowing the eyes to move laterally. 39. ANS: B PTS: 1 REF: Page 472 | Table 15-6 MSC: The trigeminal nerve affects control over both the motor and sensory functions to the face; conducts sensory impulses from the mouth, nose, surface of eye, and dura mater; it also contains motor fibers that stimulate the chewing muscles. 40. ANS: F PTS: 1 REF: Page 472 | Table 15-6 MSC: The hypoglossal nerve affects control over the motor fibers to the muscles of tongue and sensory impulses from the tongue to the brain. 41. ANS: H PTS: 1 REF: Page 472 | Table 15-6 MSC: The vagus nerve affects control over the fibers that carry sensory and motor impulses for the pharynx. A large part of this nerve is parasympathetic motor fibers that supply smooth muscles of the abdominal organs and is capable of receiving sensory impulses from the viscera. 42. ANS: J PTS: 1 REF: Page 472 | Table 15-6 MSC: The spinal accessory nerve affects control over the sensory and motor fibers for the sternocleidomastoid and trapezius muscles and the muscles of the soft palate, pharynx, and larynx. 43. ANS: G PTS: 0 REF: Page 472 | Table 15-6 MSC: The olfactory nerve is purely sensory and carries impulses for the sense of smell. 44. ANS: I PTS: 0 REF: Page 472 | Table 15-6 MSC: The vestibulocochlear nerve is purely sensory; the vestibular branch transmits impulses for the sense of equilibrium; the cochlear branch transmits impulses for the sense of hearing. NURSINGTB.COM Chapter 16: Pain, Temperature Regulation, Sleep, and Sensory Function MULTIPLE CHOICE 1. Pricking one’s finger with a needle would cause minimal pain, whereas experiencing abdominal surgery would produce more pain. This distinction is an example of which pain theory? a. Gate control theory c. Specificity theory b. Intensity theory d. Pattern theory ANS: C According to the specificity theory, a direct relationship exists between the intensity of pain and the extent of tissue injury. The remaining options are not related to the intensity of perceived pain. PTS: 1 REF: Page 485 2. Which pain theory proposes that a balance of impulses conducted from the spinal cord to the higher centers in the central nervous system (CNS) modulates the transmission of pain? a. GCT c. Specificity theory b. Pattern theory d. Neuromatrix theory ANS: A Only the gate control theory (GCT) explains that a balance of impulses conducted to the spinal cord, where cells in the substantia gelatinosa function as a spinal gate, regulates pain transmission to higher centerNs UinRtShINe GCTNBS.C. OM PTS: 1 REF: Page 485 3. Which type of nerve fibers transmits pain impulses? a. A-alpha (A) fibers c. A-delta (A) fibers b. A-beta (A) fibers d. B fibers ANS: C Of the available options, only medium-sized A fibers transmit pain impulses. PTS: 1 REF: Page 486 4. Where are the primary-order pain transmitting neurons located within the spinal cord? a. Lateral root ganglia c. Anterior root ganglia b. Dorsal root ganglia d. Medial root ganglia ANS: B The cell bodies of the primary-order neurons, or pain-transmitting neurons, reside only in the dorsal root ganglia just lateral to the spine along the sensory pathways that penetrate the posterior part of the cord. PTS: 1 REF: Page 487 5. The gate in the GCT of pain is located in the: a. Substantia gelatinosa c. Nucleus proprius b. Marginal layer d. Dorsolateral tract of Lissauer ANS: A The synaptic connections between the cells of the primary- and secondary-order neurons located in the substantia gelatinosa and other Rexed laminae function as a pain gate. The remaining options do not act in this function. PTS: 1 REF: Page 487 6. Which spinal tract carries the most nociceptive information? a. Archeospinothalamic c. Dorsal spinothalamic b. Paleospinothalamic d. Lateral spinothalamic ANS: D Most nociceptive information travels by means of ascending columns in the lateral spinothalamic tract (also called the anterolateral funiculus). The other tract options do not carry the most nociceptive information. PTS: 1 REF: Page 487 7. The major relay station of sensory information is located in the: a. Basal ganglia c. Thalamus b. Midbrain d. Hypothalamus ANS: C Although the organization of all of the ascending tracts is complex, the principal target for nociceptive afferents is the thalamus, which, in general, is the major relay station of NURSINGTB.COMsensory information. The re fulfill this objective. PTS: 1 REF: Page 487 8. Where in the CNS does a person’s learned pain response occur? a. Cerebral cortex c. Thalamus b. Frontal lobe d. Limbic system ANS: A The cognitive-evaluative system overlies the individual’s learned behavior concerning the experience of pain and can modulate the perception of pain and is mediated only through the cerebral cortex. PTS: 1 REF: Page 487 9. Massage therapy relieves pain by closing the pain gate with the stimulation which fibers? a. A c. B b. A d. C ANS: A Massaging stimulates different A fibers to close the pain gate. The remaining options do not fulfill this objective. PTS: 1 REF: Page 485 10. What part of the brain provides the emotional response to pain? a. Limbic system c. Thalamus b. Parietal lobe d. Hypothalamus ANS: A The limbic and reticular tracts are involved in alerting the body to danger, initiating arousal of the organism, and emotionally processing the perceived afferent signals, not just as stimuli, but also as pain. The remaining options do not fulfill this objective. PTS: 1 REF: Page 487 11. Which neurotransmitters inhibit pain in the medulla and pons? a. Norepinephrine and serotonin b. Gamma-aminobutyric acid (GABA) and aspartate c. Glutamate and tumor necrosis factor–alpha d. Neurokinin A and nitric oxide ANS: A Norepinephrine and serotonin (5-hydroxytryptamine) contribute to pain modulation (inhibition) in the medulla and pons. The remaining options do not fulfill this objective. PTS: 1 REF: Page 490 12. Which endogenous opioid is located in the hypothalamus and pituitary and is a strong -receptor agonist? a. Enkephalins c. Dynorphins b. Endorphins d. Endomorphins NURSINGTB.COM ANS: B The synthesis and activity of -endorphin is concentrated in the hypothalamus and the pituitary gland and act as strong µ-receptor agonist. The remaining options do not fulfill this objective. PTS: 1 REF: Page 491 13. What is the term that denotes the duration of time or the intensity of pain that a person will endure before outwardly responding? a. Tolerance c. Threshold b. Perception d. Dominance ANS: A Pain tolerance is the duration of time or the intensity of pain that an individual will endure before initiating overt pain responses. The other options are not related to the duration or intensity of pain endured before the pain is recognized. PTS: 1 REF: Page 491 14. Pain that warns of actual or impending tissue injury is referred to as what? a. Chronic c. Acute b. Psychogenic d. Phantom ANS: C Acute pain is a protective mechanism that alerts the individual to a condition or experience that is immediately harmful to the body. The remaining options do not fulfill this objective. PTS: 1 REF: Pages 491-492 15. Which description characterizes visceral pain? a. Is sharp and well-defined when transmitted by A-delta (A) fibers. b. Is perceived as poorly localized and is transmitted by the sympathetic nervous system. c. Arises from connective tissue, muscle, bone, or skin. d. Is perceived as dull, aching, and poorly localized when transmitted by C fibers. ANS: B Of the options provided, only visceral pain refers to pain in internal organs and the abdomen and is transmitted by sympathetic afferents. Visceral pain is poorly localized because of fewer nociceptors in the visceral structures. PTS: 1 REF: Page 492 16. When caring for a person who has experienced pain for 3 days, anxiety is likely to produce which physical signs that a nurse would expect to find? a. Fever and muscle weakness or reports of fatigue b. Irritability and depression or reports of constipation c. Decreased blood pressure or reports of fatigue d. Increased heart rate and respiratory rate with diaphoresis ANS: D NURSINGTB.COM Anxiety is common in acute pain states and is usually apparent in the alterations of vital signs and can include elevation of blood sugar levels, decreases in gastric acid secretion and intestinal motility, and a general decrease in blood flow to the viscera and skin. Nausea occasionally occurs. The other symptoms are not generally associated with an anxiety response to acute pain. PTS: 1 REF: Pages 491-492 17. Enkephalins and endorphins act to relieve pain by which process? a. Inhibiting cells in the substantia gelatinosa b. Stimulating the descending efferent nerve fibers c. Attaching to opiate receptor sites d. Blocking transduction of nociceptors ANS: C Enkephalins and endorphins are neurohormones that act as neurotransmitters by binding to one or more G protein–coupled opioid receptors and thus relieving pain. The other options are not accurate descriptions of how enkephalins and endorphins relieve pain. PTS: 1 REF: Pages 490-491 18. What is a long-term complication of rewarming as a treatment for hypothermia? a. Acidosis c. Shock b. Dysrhythmias d. Renal failure ANS: D Rewarming can result in long-term complications that include congestive heart failure, hepatic and renal failure, abnormal erythropoiesis, myocardial infarction, pancreatitis, and neurologic dysfunctions. Short-term complications of rewarming include acidosis, rewarming shock, and dysrhythmias. PTS: 1 REF: Page 501 19. How does the release (increase) of epinephrine raise body temperature? a. The release of epinephrine causes shivering. b. It affects muscle tone. c. It raises the metabolic rate. d. It increases and strengthens the heart rate. ANS: C Epinephrine and norepinephrine produce a rapid transient increase in heat production by raising the body’s basal metabolic rate. The other options are not correct descriptions of the effects of epinephrine on body heat. PTS: 1 REF: Page 496 20. Using a fan to reduce body temperature is an example of which mechanism of heat loss? a. Evaporation c. Convection b. Radiation d. Conduction ANS: C NURSINGTB.COM Only convection causes the transfer of heat through currents of gases or liquids. PTS: 1 REF: Page 497 21. Up to how many liters of fluid per hour may be lost by sweating? a. 2 c. 6 b. 4 d. 8 ANS: A Sweating may cause as much as 2.2 L of fluid per hour to be lost. PTS: 1 REF: Page 497 22. Heat loss from the body via radiation occurs by: a. Emanations of electromagnetic waves b. Transfer of heat through currents of liquids or gas c. Dilation of blood vessels bringing blood to skin surfaces d. Direct heat loss from molecule-to-molecule transfer ANS: A Radiation refers to heat loss through electromagnetic waves. None of the other options accurately describes heat loss via radiation. PTS: 1 REF: Page 497 23. Which cytokines are endogenous pyrogens? a. IL-3, IL-10, and IL-18 b. IL-2, IL-8, and IFN- c. IL-4, IL-12, colony-stimulating factor, and IFN- d. IL-1, IL-6, TNF-, and IFN- ANS: D Endogenous pyrogens include prostaglandin E2 (PGE2), interleukin-1 (IL-1), IL-6, tumor necrosis factor–alpha (TNF-), and interferon-gamma (IFN-). The other options are not endogenous pyrogens. PTS: 1 REF: Page 498 24. Which hormones help diminish the febrile response? a. Arginine vasopressin (AVP), melanocyte-stimulating hormone-alpha (-MSH), and corticotropin-releasing factor b. Adrenocorticotropic hormone (ACTH), thyroid-stimulating hormone, and thyroxine (T4) c. Antidiuretic hormone, growth hormone, and aldosterone d. None; hormones only facilitate the increase of core body temperature. ANS: A During fever, AVP, -MSH, and corticotropin-releasing factors are released from the brain, and antiinflammatory cytokines (e.g., IL-1, IL-10) can act as endogenous cryogens or antipyretics to help diminish the febrile response. The other options are not hormones that diminish the febrile response. PTS: 1 NURSINGTB.COM REF: Page 498 25. Prolonged high environmental temperatures that produce dehydration, decreased plasma volumes, hypotension, decreased cardiac output, and tachycardia cause which disorder of temperature regulation? a. Heat cramps c. Malignant hyperthermia b. Heat stroke d. Heat exhaustion ANS: D Of the options presented, only heat exhaustion, or collapse, is a result of prolonged high core or environmental temperatures resulting in dehydration, decreased plasma volumes, hypotension, decreased cardiac output, and tachycardia. PTS: 1 REF: Page 500 26. In acute hypothermia, what physiologic change shunts blood away from the colder skin to the body core in an effort to decrease heat loss? a. Hypotension c. Voluntary muscle movements b. Peripheral vasoconstriction d. Shivering ANS: B Tissue hypothermia slows the rate of chemical reactions (tissue metabolism), increases the viscosity of the blood, slows blood flow through microcirculation, facilitates blood coagulation, and stimulates profound vasoconstriction. The remaining options do not fulfill this objective. PTS: 1 REF: Page 501 27. A heat stroke is characterized by: a. Core temperatures usually reaching approximately 39.9° C (103.9° F) b. Sweat production on the face occurring even during dehydration c. A rapidly decreasing core temperature as heat loss from the evaporation of sweat ceases d. Symptoms caused by the loss of sodium and prolonged sweating ANS: B When the core temperature reaches or exceeds 40.5° C (104.9° F), the brain may be preferentially cooled by maximal blood flow through the veins of the head and face, specifically the forehead. Sweat production on the face is maintained even during dehydration. The remaining options do not occur during heat stroke. PTS: 1 REF: Page 500 28. Which medication is used to reverse the effects of malignant hyperthermia? a. Propranolol c. Dantrolene sodium b. Diazepam d. Sodium carbonate ANS: C Treatment includes the withdrawaNl UoRf SthINe GpTroBv.CoOkiMng agents and the administration of dantrolene sodium (a skeletal relaxant that inhibits calcium release during muscle contraction). The other options are not effective in the treatment of malignant hyperthermia. PTS: 1 REF: Pages 500-501 29. The major sleep center is located in which section of the brain? a. Thalamus c. Frontal lobe b. Brainstem d. Hypothalamus ANS: D A small group of hypothalamic nerve cells, the suprachiasmatic nucleus (SCN), controls the timing of the sleep-wake cycle and coordinates this cycle with circadian rhythms (24-hour rhythm cycles) in areas of the brain and other tissues. The remaining options do not fulfill this objective. PTS: 1 REF: Pages 502-503 30. Which neuropeptide promotes wakefulness? a. Prostaglandin D2 c. Hypocretins b. L-tryptophan d. Growth factors ANS: C The hypothalamus, as a major sleep center, secretes hypocretins (orexins), which are neuropeptides that promote wakefulness and rapid eye movement (REM) sleep, as well as appetite, energy consumption, and pleasure or reward. The remaining options do not fulfill this objective. PTS: 1 REF: Page 503 31. Which term is also used to refer to paradoxic sleep? a. Non-REM c. REM b. Light d. Delta wave ANS: C REM sleep is also known as paradoxic sleep because the electroencephalographic (EEG) pattern is similar to the normal awake pattern. None of the other terms are used to identify paradoxic sleep. PTS: 1 REF: Page 503 32. The sudden apparent arousal in which a child expresses intense fear or another strong emotion while still in a sleep state characterizes which sleep disorder? a. Night terrors c. Somnambulism b. Insomnia d. Enuresis ANS: A Three types of parasomnias include arousal disorders such as confusional arousals, sleepwalking (somnambulism), and night terrors (dream anxiety attacks). The remaining options do not involve a sense or expression of fear or any other strong emotion. NURSINGTB.COM PTS: 1 REF: Page 505 33. Coronary artery disease is most affected by which component of sleep? a. Non-REM c. REM b. Light d. Delta wave ANS: C Coronary artery disease is most affected during rapid eye movement (REM) sleep. During this component of sleep, dreams may provoke nocturnal angina, increased heart rate, and electrocardiographic (ECG) changes. The other options are not associated with coronary artery disease. PTS: 1 REF: Page 506 34. Rapid eye movement (REM) sleep occurs in cycles approximately every: a. 45 minutes c. 120 minutes b. 90 minutes d. 150 minutes ANS: B REM sleep accounts for 20% to 25% of sleep time and is characterized by desynchronized, low-voltage, fast activity that occurs for 5 to 60 minutes approximately every 90 minutes, beginning after 1 to 2 hours of non-REM sleep. PTS: 1 REF: Page 503 35. Loud snoring, a decrease in oxygen saturation, fragmented sleep, chronic daytime sleepiness, and fatigue are clinical manifestations of which sleep disorder? a. Obstructive sleep apnea c. Somnambulism b. Upper airway resistance syndrome d. Narcolepsy ANS: A Obstructive sleep apnea is characterized by repetitive increases in resistance to airflow within the upper airway with loud snoring, gasping, intervals of apnea lasting from 10 to 30 seconds, fragmented sleep, and chronic daytime sleepiness and fatigue, as well as a decrease in oxygen saturation. The remaining options do not exhibit the signs and symptoms listed in the stem. PTS: 1 REF: Page 504 36. What are the expected changes in sleep patterns of older adults? a. Older adults experience difficulty falling asleep with less time spent in REM sleep. b. They experience sound sleep during the night with approximately 50% of the time spent in REM sleep and dreaming. c. Older men commonly experience interrupted sleep patterns later in life than do older women. d. Older adults awaken often but with a rapid return to sleep; they awaken refreshed but often later in the morning. ANS: A The sleep pattern of the older adult differs from the younger adult in that total sleep time is decreased, and the older individual takes longer to initiate and maintain sleep. Older adults tend to go to sleep earlier in the evNeUnRinSgINaGnTdBa.CwOaMken more frequently during the night and earlier in the morning. Rapid eye movement (REM) and slow-wave sleep decreases. The alteration in sleep pattern typically appears approximately 10 years later in women than it does in men. PTS: 1 REF: Page 504 37. Pinkeye is characterized by inflammation of which structure? a. Eyelids c. Meibomian glands b. Sebaceous glands d. Conjunctiva ANS: D Acute bacterial conjunctivitis (pinkeye) is an inflammation of the conjunctiva (mucous membrane covering the front part of the eyeball). The other structures are not affected by this inflammation. PTS: 1 REF: Page 507 38. Open-angle glaucoma occurs because of: a. Decreased production of aqueous humor b. Increased production of vitreous humor c. Obstructed outflow of aqueous humor d. Excessive destruction of vitreous humor ANS: C Open-angle glaucoma occurs because of an obstruction of the outflow of aqueous humor at trabecular meshwork or Schlemm canal. The remaining options fail to accurately describe the cause of open-angle glaucoma. PTS: 1 REF: Page 511 | Table 16-5 39. How can glaucoma cause blindness? a. Infection of the cornea b. Pressure on the optic nerve c. Opacity of the lens d. Obstruction of the venous return from the retina ANS: B Loss of visual acuity as a result of pressure on the optic nerve is the only reason glaucoma can result in blindness. PTS: 1 REF: Pages 510-511 40. When comparing the effects of acute and chronic pain on an individual, chronic pain is more often: a. The external event that results in a sense of fear b. Viewed as being meaningful but undesirable c. A factor that contributes to depression d. A sense of internal unease ANS: C Chronic pain is often associated with a sense of hopelessness and helplessness as relief becomes more elusive and the timNeUfrRamSINe GmToBr.eCpOrMotracted. The pain is perceived as meaningless, and depression is often a concomitant finding, as either a result of the chronic pain state or as a contributor to its development. Individuals often psychologically respond to acute pain with fear (e.g., fear of diagnosis, fear of continued pain), anxiety, and a general sense of unpleasantness or unease. PTS: 1 REF: Page 492 41. When considering the risk factors for the development of phantom limb pain, the nurse recognizes which as a primary contributing factor? a. Age, with adolescent patients being at a higher risk than adults b. Presence of pain in the limb before amputation c. Patient’s previous experience with managing pain d. Cultural views regarding the acceptance of pain ANS: B Phantom limb pain is pain that an individual feels in an amputated limb after the stump has completely healed. It is more likely to appear in individuals who experienced pain in the limb before amputation. The other options would not be considered a primary contributing factors. PTS: 1 REF: Page 494 42. Based on an understanding of the physiologic process of nociceptors, the nurse expects which surgical procedure to create more pain? a. Repair of several crushed fingers b. External fixation of a dislocated shoulder c. Cyst removal on the internal surface of an ovary d. Repair of a ruptured spleen ANS: A The variable nature and distribution of nociceptors affect the relative sensitivity to pain in different areas of the body; the tips of the fingers have more nociceptors than the skin on the back, and all skin has many more nociceptors than the internal organs including bone. PTS: 1 REF: Pages 485-486 43. The basis of the specificity theory of pain is that: a. Injury to specific organs results in specific types of pain. b. Chronic pain is generally less intense than acute pain. c. The greater the tissue injury, the greater the pain. d. Acute pain is specific only to certain injuries. ANS: C According to the specificity theory, a direct relationship exists between the intensity of pain and the extent of tissue injury. The remaining options are not accurate statements regarding this pain theory. PTS: 1 REF: Page 485 44. Which statement is true regarding the gate control theory (GCT) of pain? a. The pain gate is located in the brain. b. A closed gate increases pain pNerUcReSpItNioGnT. B.COM c. The brain primarily controls the pain gate. d. An open gate facilitates the brain in processing the pain. ANS: D The open gate in the spinal cord regulates the transmission of pain impulses that ascend to the brain for further processing and interpretation, thus leading to the management of pain. The remaining statements are not true when discussing the GCT of pain. PTS: 1 REF: Page 485 MULTIPLE RESPONSE 45. Which factors contribute to sensorineural hearing loss? (Select all that apply.) a. Ménière disease b. Aging c. Diabetes mellitus d. Noise exposure e. Outer ear trauma ANS: A, B, C, D Impairment of the organ of Corti or its central connections causes a sensorineural hearing loss. The hearing loss may be gradual or sudden. Conditions that commonly cause sensorineural hearing loss include congenital and hereditary factors, noise exposure, aging, Ménière disease, ototoxicity, and systemic disease (e.g., syphilis, Paget disease, collagen diseases, diabetes mellitus). Outer ear trauma is not a typical cause of sensorineural hearing loss. PTS: 1 REF: Page 517 46. Why are children more susceptible to heat stroke than are adults? (Select all that apply.) a. Children produce more metabolic heat when exercising. b. They have more surface area–to-mass ratio. c. Children have less sweating capacity. d. They an underdeveloped hypothalamus. e. Children have an overdeveloped ability to perceive heat. ANS: A, B, C Children are more susceptible to heat stroke than are adults because (1) they produce more metabolic heat when exercising, (2) they have a greater surface area–to-mass ratio, and (3) their sweating capacity is less than that of adults. The remaining options are not true of a child. PTS: 1 REF: Page 500 47. Heat exhaustion results in: (Select all that apply.) a. Profuse sweating b. Profound vasodilation c. A need to ingest warm liquidsNURSINGTB.COM d. Permanent damage to the hypothalamus e. An increased risk for future heat exhaustion ANS: A, B, C Internally high temperatures cause the appropriate hypothalamic response of profound vasodilation and profuse sweating. The individual should be encouraged to drink warm fluids to replace fluid lost through sweating. Heat exhaustion is a result of prolonged high core or environmental temperatures that are unique to each incidence. PTS: 1 REF: Page 500 48. It is true that a fever: (Select all that apply.) a. Is a complex cascade involving several different systems. b. Can be a result of a dysfunctional hypothalamus. c. Should be eliminated as quickly as possible. d. Triggers endocrine responses. e. Is in response to a pyrogen. ANS: A, B, D, E Fever is a complex, integrated cascade of behavioral, neurologic, and endocrine responses to an immune challenge initiated by endogenous pyrogens or disorders of the hypothalamus. Fever production aids responses to infectious processes through several mechanisms and should be interrupted only when it might present an additional risk to the individual. PTS: 1 REF: Pages 498-500 MATCHING Match the types of chronic pain with its description. Types of pain may be used more than once. A. Myofascial pain syndrome B. Neuropathic pain C. Deafferentation pain D. Sympathetically maintained pain 49. Pain that results from tumor infiltration of nerve tissue, from trauma or chemical injury to the nerve, or from damage from radiation, chemotherapy, or surgical sectioning of the nerve 50. Pain that is thought to be caused by trauma or disease of nerves and leads to abnormal processing of sensory information by the peripheral and central nervous systems 51. Pain that occurs after peripheral nerve injury and is described as continuous with severe sensations and a burning quality 52. Pain that is the result of muscle spasms, tenderness, and stiffness and leads to muscle guarding that limits muscle motion NURSINGTB.COM 49. ANS: C PTS: 1 REF: Page 494 MSC: Deafferentation pain results from trauma or chemical injury to the peripheral nervous system, from tumor infiltration of nerve tissue, or from damage from radiation, chemotherapy, or surgical sectioning of a nerve with the loss of sensory input to the central nervous system. 50. ANS: B PTS: 1 REF: Page 494 MSC: Neuropathic pain is the result of trauma or disease of nerves and leads to abnormal processing of sensory information by the peripheral and central nervous systems. 51. ANS: D PTS: 1 REF: Pages 494-495 MSC: Sympathetically maintained pain (SMP) is another type of neuropathic pain that occurs after peripheral nerve or extremity injury and is characterized as continuous and severe with a burning quality. 52. ANS: A PTS: 1 REF: Pages 492-493 MSC: Myofascial pain syndrome (MPS) is associated with injury to muscle, fascia, and tendons. Chapter 17: Alterations in Cognitive Systems, Cerebral Hemodynamics, and Motor Function MULTIPLE CHOICE 1. Cognitive operations cannot occur without the effective functioning of the brain’s: a. Pons c. Reticular activating system b. Medulla oblongata d. Cingulate gyrus ANS: C Cognitive cerebral functions require a functioning reticular activating system (RAS). Cognitive operations are not managed by any of the other options. PTS: 1 REF: Page 528 2. Which intracerebral disease process is capable of producing diffuse dysfunction? a. Closed head trauma with bleeding c. Neoplasm b. Subdural pus collections d. Infarct emboli ANS: D Disorders within the brain substance (intracerebral)—bleeding, infarcts emboli, and tumors—primarily functioning as masses may cause diffuse dysfunction. Such localized destructive processes directly impair functioning of the thalamic or hypothalamic activating systems. Disorders outside the brain but within the cranial vault (extracerebral), including neoplasms, closed-head trauma with subsequent bleeding, and subdural empyema (accumulation of pus), can cause similar dysfunction. NURSINGTB.COM PTS: 1 REF: Page 528 3. What is the most common infratentorial brain disease process that results in the direct destruction of the reticulating activation system (RAS)? a. Cerebrovascular disease c. Neoplasms b. Demyelinating disease d. Abscesses ANS: A Infratentorial disorders produce a decline in arousal through a direct destruction of the RAS and its pathways. The most common cause of direct destruction is cerebrovascular disease, but demyelinating diseases, neoplasms, granulomas, abscesses, and head injury also may cause brainstem destruction by tissue compression. PTS: 1 REF: Page 528 4. What stimulus causes posthyperventilation apnea (PHVA)? a. Changes in PCO2 levels c. Damage to the forebrain b. Changes in PaCO2 levels d. Any arrhythmic breathing pattern ANS: B With normal breathing, a neural center in the forebrain (cerebrum) produces a rhythmic breathing pattern. When consciousness decreases, lower brainstem centers regulate the breathing pattern by responding only to changes in PaCO2 levels. This irregular breathing pattern is called PHVA. The other options are not responsible for PHVA. PTS: 1 REF: Pages 529-530 | Table 17-4 5. Posthyperventilation apnea (PHVA) ceases and rhythmic breathing is resumed when levels of arterial: a. Carbon dioxide increase c. Oxygen increase b. Carbon dioxide become normal d. Oxygen decrease ANS: B Rhythmic breathing returns when the PCO2 level returns to normal. None of the remaining options would affect normal rhythmic breathing after PHVA. PTS: 1 REF: Page 530 | Table 17-4 6. Cheyne-Stokes respirations are described as a: a. Sustained deep rapid but regular pattern of breathing b. Crescendo-decrescendo pattern of breathing, followed by a period of apnea c. Prolonged inspiratory period, gradually followed by a short expiratory period d. Completely irregular breathing pattern with random shallow, deep breaths and irregular pauses ANS: B Cheyne-Stokes respiration is an abnormal rhythm of breathing (periodic breathing) that alternates between hyperventilation and apnea. Cheyne-Stokes respirations do not include a sustained deep respiratory rate. Altered inspiratory and expiratory periods are not characteristic of Cheyne-Stokes respirations. Random, irregular breathing patterns are not observed during Cheyne-Stokes respirations. NURSINGTB.COM PTS: 1 REF: Pages 529-530 | Page 532 | Table 17-4 7. Vomiting is associated with central nervous system (CNS) injuries that compress which of the brain’s anatomic locations? a. Vestibular nuclei in the lower brainstem b. Floor of the third ventricle c. Any area in the midbrain d. Diencephalon ANS: A Vomiting, yawning, and hiccups are complex reflexlike motor responses that are integrated by neural mechanisms in the lower brainstem. Vomiting often accompanies CNS injuries that involve the vestibular nuclei. The remaining options will not trigger vomiting when compressed. PTS: 1 REF: Page 533 8. Which midbrain dysfunction causes pupils to be pinpoint size and fixed in position? a. Diencephalon dysfunction b. Oculomotor cranial nerve dysfunction c. Dysfunction of the tectum d. Pontine dysfunction ANS: D Pinpoint fixed pupils are a result of pontine dysfunction. The diencephalon, oculomotor cranial nerve, and tectum are not involved in such a pupil reaction. PTS: 1 REF: Page 532 9. What characteristic is a medical criterion of brain death? a. Akinetic mutism c. Apnea b. Coma d. Locked-in syndrome ANS: C Apnea is viewed as a criterion of brainstem death, whereas the remaining options reflect cerebral death. PTS: 1 REF: Pages 533-534 10. A clinical manifestation caused by damage to the lower pons includes an abnormal: a. Flexion with or without extensor response of the lower extremities b. Extension response of the upper and lower extremities c. Extension response of the upper extremities and flexion response of the lower extremities d. Flaccid response in the upper and lower extremities ANS: B A flaccid state with little or no motor response to stimuli is characteristic of damage to the pons. None of the other responses are considered a clinical manifestation of damage to the lower pons. PTS: 1 REF: Page 53N4U|RTSaIbNleG1T7B-.6COM 11. Which person is at the greatest risk for developing delirium? a. An individual with diabetes celebrating a 70th birthday b. A depressed Hispanic woman c. An individual on the second day after hip replacement d. A man diagnosed with schizophrenia ANS: C Delirium is associated with autonomic nervous system overactivity and typically develops in 2 to 3 days, most commonly in critical care units, postsurgically, or during withdrawal from CNS depressants (e.g., alcohol, narcotic agents). Age, gender, and chronic illnesses are not generally associated with delirium triggers. PTS: 1 REF: Page 545 12. A sudden, explosive, disorderly discharge of cerebral neurons is termed: a. Reflex c. Epilepsy b. Seizure d. Convulsion ANS: B A sudden, explosive, disorderly discharge of cerebral neurons describes a seizure. This description is not accurate for the other options. PTS: 1 REF: Page 550 13. A complex partial seizure is described as: a. Alternating of tonic and clonic movements b. Impairment of both consciousness and the ability to react to exogenous stimuli c. Focal motor movement without loss of consciousness d. One seizure followed by another in less than 1 minute ANS: B A complex partial seizure results is impaired consciousness, as well as the inability to respond to exogenous stimuli. None of the other options accurately describe a complex partial seizure. PTS: 1 REF: Page 552 14. Status epilepticus is considered a medical emergency because of the: a. Loss of consciousness b. Development of cerebral hypoxia c. Possibility of a head injury during seizures d. Decrease in brain metabolism ANS: B Status epilepticus is a true medical emergency because a single seizure can last longer than 30 minutes, resulting in hypoxia of the brain. The other options are not the criteria used to consider status epilepticus. PTS: 1 REF: Page 553 15. The most critical aspect in correctNlyUdRiSaIgNnGoTsBin.CgOaMseizure disorder and establishing its cause is: a. Computed tomographic (CT) scan c. Skull x-ray studies b. Cerebrospinal fluid analysis d. Health history ANS: D Although the history may be supplemented with the remaining options, it remains the pivotal tool for establishing the cause of a seizure disorder. PTS: 1 REF: Page 555 16. What type of seizure starts in the fingers and progressively spreads up the arm and extends to the leg? a. Complex-psychomotor seizure c. Generalized seizures b. Focal (partial) Jacksonian seizure d. Atonic-drop seizure ANS: B Focal (partial) Jacksonian seizures most often begin in the face and fingers and then progressively spread to other body parts. The other options do not begin and spread in the fashion described. PTS: 1 REF: Page 551 | Table 17-16 17. What area of the brain mediates the executive attention functions? a. Limbic c. Parietal b. Prefrontal d. Occipital ANS: B The prefrontal areas mediate several cognitive functions, called executive attention functions (e.g., planning, problem solving, setting goals). The remaining options are not areas involved with the mediation of executive attention functions. PTS: 1 REF: Page 538 18. What term describes the loss of the comprehension or production of language? a. Agnosia c. Akinesia b. Aphasia d. Dysphasia ANS: B Aphasia is the loss of the comprehension or production of language. The remaining options are not terms used to describe this loss of function. PTS: 1 REF: Page 539 19. With receptive dysphasia (fluent), the individual is able to: a. Respond in writing, but not in speech. b. Produce verbal speech, but not comprehend language. c. Comprehend speech, but not verbally respond. d. Neither respond verbally nor comprehend speech. ANS: C The individual experiencing receptive dysphasia may be able to produce verbal language, but language is meaningless because of a disturbance in understanding all language. The NURSINGTB.COMremaining options do not desc sia. PTS: 1 REF: Page 543 | Table 17-10 20. What is the normal intracranial pressure (in mm Hg)? a. 5 to 15 c. 12 to 14 b. 7 to 20 d. 80 to 120 ANS: A Intracranial pressure is normally 5 to 15 mm Hg or 60 to 180 cm water (H2O). The remaining options reflect increased intracranial pressure. PTS: 1 REF: Pages 555-556 21. Cerebral edema is an increase in the fluid content of the brain’s: a. Ventricles c. Neurons b. Tissue d. Meninges ANS: B Cerebral edema is an increase in the fluid content of brain tissue; that is, a net accumulation of water within the brain. Cerebral edema is not noted in the brain’s ventricles, neurons, or meninges. PTS: 1 REF: Page 557 22. What type of cerebral edema occurs when permeability of the capillary endothelium increases after injury to the vascular structure? a. Cytotoxic c. Vasogenic b. Interstitial d. Ischemic ANS: C Increased permeability of the capillary endothelium of the brain after injury to the vascular structure causes vasogenic edema. The remaining options are not consistent with this description. PTS: 1 REF: Pages 557-558 23. A communicating hydrocephalus is caused by an impairment of the: a. Cerebrospinal fluid flow between the ventricles b. Cerebrospinal fluid flow into the subarachnoid space c. Blood flow to the arachnoid villi d. Absorption of cerebrospinal fluid ANS: D Hydrocephalus from impaired absorption outside the ventricles is called communicating (extraventricular) hydrocephalus. The other options do not accurately describe the cause of a communicating hydrocephalus. PTS: 1 REF: Page 558 24. Which edema is most often observed with noncommunicating hydrocephalus? a. Metabolic c. Vasogenic b. Interstitial ANS: B NURSINGTdB..COIsMchemic Interstitial edema is observed most often with noncommunicating hydrocephalus. Noncommunicating hydrocephalus is not the cause of any of the other options. PTS: 1 REF: Page 558 25. Which dyskinesia involves involuntary movements of the face, trunk, and extremities? a. Paroxysmal c. Hyperkinesia b. Tardive d. Cardive ANS: B Tardive dyskinesia is the involuntary movement of the face, trunk, and extremities. The other terms do not describe involuntary movements of the face, trunk, and extremities. PTS: 1 REF: Page 562 26. Antipsychotic drugs cause tardive dyskinesia by mimicking the effects of increased: a. Dopamine c. Norepinephrine b. Gamma-aminobutyric acid d. Acetylcholine ANS: A The antipsychotic drugs cause denervation hypersensitivity, which mimics the effect of too much dopamine. None of the other options produce such an affect. PTS: 1 REF: Page 562 27. The existence of regular, deep, and rapid respirations after a severe closed head injury is indicative of neurologic injury to the: a. Lower midbrain c. Supratentorial b. Pontine area d. Cerebral area ANS: A Central reflex hyperpnea, which is a sustained deep and rapid but regular respiratory pattern that is the result of central nervous system (CNS) damage or disease, involves the lower midbrain and upper pons. This neurologic injury is observed after increased intracranial pressure and blunt head trauma. Damage to any of the other areas listed would not produce this breathing pattern. PTS: 1 REF: Page 530 | Table 17-4 28. What type of posturing exists when a person with a severe closed head injury has all four extremities in rigid extension with the forearms in hyperpronation and the legs in plantar extension? a. Decorticate c. Spastic b. Decerebrate d. Cerebellar ANS: B Decerebrate posturing includes opisthotonos (hyperextension of the vertebral column) with clenching of the teeth; extension, abduction, and hyperpronation of the arms; and extension of the lower extremities including plantar extension. The other options do not describe such posturing. NURSINGTB.COM PTS: 1 REF: Page 534 | Page 575 | Table 17-6 29. Since his cerebrovascular accident, a man has been denying his left hemiplegia. What term is used to describe this finding? a. Visual agnosia c. Amusia agnosia b. Anosognosia d. Agraphia agnosia ANS: B Anosognosia is ignorance or denial of the existence of disease. None of the remaining options describes such denial. PTS: 1 REF: Page 542 | Table 17-9 30. After a cerebrovascular accident, a man is unable to either feel or identify a comb with his eyes closed. This is an example of: a. Agraphia c. Anosognosia b. Tactile agnosia d. Prosopagnosia ANS: B Tactile agnosia is the inability to recognize objects by touch. None of the other options define the inability to recognize objects by touch. PTS: 1 REF: Page 542 | Table 17-9 31. Most dysphasias are associated with cerebrovascular accidents involving which artery? a. Anterior communicating c. Circle of Willis b. Posterior communicating d. Middle cerebral ANS: D Dysphasias are usually associated with a cerebrovascular accident involving the middle cerebral artery or one of its many branches. Damage to or occlusion of any of the other options does not cause dysphasias. PTS: 1 REF: Pages 539-540 32. Tactile agnosia is related to injury of which area of the brain? a. Frontotemporal c. Temporal b. Parietal d. Broca area ANS: B Tactile agnosia (astereognosis) is the inability to recognize objects by touch as a result of damage to the parietal lobe. Tactile agnosia is not related to an injury to any of the other options. PTS: 1 REF: Page 542 | Table 17-9 33. Neurofibrillary tangles characterize which neurologic disorder? a. Dementia syndrome c. Alzheimer disease b. Delirium d. Parkinson disease ANS: C Amyloid plaques, neurofibrillary tangles, as well as neuronal and synaptic losses in the NURSINGTB.COMbrain, characterize Alzheimer PTS: 1 REF: Pages 546-549 | Table 17-13 34. The body compensates for a rise in intracranial pressure by first displacing the: a. Cerebrospinal fluid c. Venous blood b. Arterial blood d. Cerebral cells ANS: A A rise in intracranial pressure necessitates an equal reduction in the volume of the other contents. The most readily displaced content of the cranial vault is cerebrospinal fluid (CSF), not any of the other options. PTS: 1 REF: Pages 555-556 35. Stage 1 intracranial hypertension is caused by the: a. Loss of autoregulation that normally maintains constant blood flow during changes in cerebral perfusion pressure b. Displacement of cerebrospinal fluid, followed by compression of the cerebral venous system c. Vasoconstriction of the cerebral arterial system with reciprocal increase in systemic blood pressure d. Compression of the medulla oblongata in the brainstem by herniation of the cerebral cortex ANS: B If intracranial pressure remains high after cerebrospinal fluid (CSF) displacement out of the cranial vault, then cerebral blood volume is altered, resulting in stage 1 intracranial hypertension. Vasoconstriction and external compression of the venous system occur in an attempt to further decrease the intracranial pressure. None of the remaining options accurately describe the cause of stage 1 intracranial hypertension. PTS: 1 REF: Page 556 36. Dilated and sluggish pupils, widening pulse pressure, and bradycardia are clinical findings evident of which stage of intracranial hypertension? a. 1 c. 3 b. 2 d. 4 ANS: C Stage 3 intracranial hypertension exhibits clinical manifestations that include decreasing levels of arousal, Cheyne-Stokes respiration or central neurogenic hyperventilation, pupils that become sluggish and constricted, widened pulse pressure, and bradycardia. These responses are not characteristic of any other stage. PTS: 1 REF: Page 556 37. Dilation of the ipsilateral pupil, following uncal herniation, is the result of pressure on which cranial nerve (CN)? a. Optic (CN I) c. Oculomotor (CN III) b. Abducens (CN VI) d. Trochlear (CN IV) ANS: C NURSINGTB.COM The oculomotor CN (III) is involved in this manifestation of pupil dilation. None of the other options would result in pupil dilation when subjected to pressure. PTS: 1 REF: Page 557 | Box 17-4 38. Which characteristic is the most critical index of nervous system dysfunction? a. Size and reactivity of pupils c. Motor response b. Pattern of breathing d. Level of consciousness ANS: D Level of consciousness is the most critical clinical index of nervous system function or dysfunction. An alteration in consciousness indicates either improvement or deterioration of a person’s condition. No other option is used as the critical index of nervous system. PTS: 1 REF: Page 529 39. Diagnostic criteria for a persistent vegetative state include: a. Absence of eye opening b. Lack of subcortical responses to pain stimuli c. Roving eye movements with visual tracking d. Return of autonomic functions such as gastrointestinal function ANS: D Diagnostic criteria for vegetative state (VS) include the return of professed vegetative (autonomic) functions, including sleep-wake cycles and normalization of respiratory and digestive system functions. Only the correct option appropriately describes the diagnostic criteria for a VS. PTS: 1 REF: Page 534 40. Uncal herniation occurs when: a. The hippocampal gyrus shifts from the middle fossa through the tentorial notch into the posterior fossa. b. The diencephalon shifts from the middle fossa straight downward through the tentorial notch into the posterior fossa. c. The cingulate gyrus shifts under the falx cerebri. d. A cerebellar tonsil shifts through the foramen magnum. ANS: A Uncal herniation (i.e., hippocampal herniation, lateral mass herniation) occurs when the uncus or hippocampal gyrus (or both) shifts from the middle fossa through the tentorial notch into the posterior fossa. This shift results in the compression of the ipsilateral third cranial nerve (CN), impairing parasympathetic function. This impairment is carried on in the periphery of the nerve, then in the contralateral third CN, and finally in the mesencephalon, inducing coma. The other options do not appropriately describe when uncal herniation occurs. PTS: 1 REF: Page 557 | Box 17-4 41. Which assessment finding marks the end of spinal shock? a. Return of blood pressure and hNeUaRrtSrIaNtGeTtoB.nCoOrMmal b. Gradual return of spinal reflexes c. Return of bowel and bladder function d. Evidence of diminished deep tendon reflexes and flaccid paralysis ANS: B A gradual return of spinal reflexes marks the end of spinal shock. The other options are not an indication of the cessation of spinal shock. PTS: 1 REF: Pages 570-571 42. Characteristics of primary motor neuron atrophy include: a. Loss of sensation in distal, proximal, or midline muscles b. Fasciculations and muscle cramps c. Flaccid paralysis with paresthesia d. Spastic paralysis with increased deep tendon reflexes ANS: B Fasciculations are particularly associated with primary motor neuron injury, and muscle cramps are common. The other options do not describe characteristics of primary motor neuron atrophy. PTS: 1 REF: Pages 571-572 43. The weakness resulting from the segmental paresis and paralysis characteristic of anterior horn cell injury is difficult to recognize because: a. Upper motor neurons are involved. b. The injury is microscopic. c. Two or more nerve roots supply each muscle. d. The person is unable to feel the involved muscles. ANS: C The paresis and paralysis associated with anterior horn cell injury are segmental; however, because two or more roots supply each muscle, the segmental character of the weakness may be difficult to recognize. The reason this pathophysiologic condition is difficult to recognize is not appropriately explained by any of the other options. PTS: 1 REF: Page 571 44. Parkinson disease is a degenerative disorder of the brain’s: a. Hypothalamus c. Frontal lobe b. Anterior pituitary d. Basal ganglia ANS: D Parkinson disease is a commonly occurring degenerative disorder of the basal ganglia and not of any of the other brain structures. PTS: 1 REF: Pages 564-565 45. Clinical manifestations of Parkinson disease are caused by a deficit in which of the brain’s neurotransmitters? a. Gamma-aminobutyric acid NURSINGTcB..CONMorepinephrine b. Dopamine d. Acetylcholine ANS: B Parkinson disease is a commonly occurring degenerative disorder involving deficits of dopamine, not of any of the other options. PTS: 1 REF: Pages 565-566 46. Tremors at rest, rigidity, akinesia, and postural abnormalities are a result of the atrophy of neurons in the brain’s: a. Caudate that produces serotonin b. Putamen that produces gamma-aminobutyric acid c. Substantia nigra that produces dopamine d. Hypothalamus that produces acetylcholine ANS: C The hallmark characteristics of Parkinson disease (PD) are a result of a loss of dopaminergic-pigmented neurons in the substantia nigra pars compacta with dopaminergic deficiency in the putamen portion of the striatum (the striatum includes the putamen and caudate nucleus). The remaining options are not characteristics of PD. PTS: 1 REF: Pages 565-566 MULTIPLE RESPONSE 47. Dementia is commonly characterized by the deterioration in which abilities? (Select all that apply.) a. Sociability b. Balance c. Memory d. Speech e. Decision making ANS: C, D, E Dementia is the progressive failure (an acquired deterioration) of many cerebral functions that include impairment of intellectual function with a decrease in orienting, memory, language, executive attentional functions, and alterations in behavior. Loss of the need for social contact and impaired balance are not associated with dementia, although a person with such a diagnosis may exhibit these deficiencies. PTS: 1 REF: Pages 545-546 48. The clinical manifestations of Parkinson disease include: (Select all that apply.) a. Fragmented sleep b. Drooping eyelids c. Depression d. Muscle stiffness e. Bradykinesia ANS: A, C, D, E The classic motor manifestations of Parkinson disease (PD) are bradykinesia, tremor at rest (resting tremor), rigidity (musNcUleRsStIiNffGnTesBs.C),OaMnd postural abnormalities. Nonmotor symptoms associated with PD include hyponosmia, fatigue, pain, autonomic dysfunction, sleep fragmentation, depression, and dementia with or without psychosis. Drooping eyelids are not characteristics of PD. PTS: 1 REF: Page 566 49. In Parkinson disease the basal ganglia influence the hypothalamic function to produce which clinical manifestations? (Select all that apply.) a. Inappropriate diaphoresis b. Gastric retention c. Vomiting d. Diarrhea e. Urinary retention ANS: A, B, E The basal ganglia influence hypothalamic function (autonomic and neuroendocrine) through pathways connecting the hypothalamus with the basal ganglia and cerebral cortex. Common autonomic symptoms in Parkinson disease include inappropriate diaphoresis, gastric retention, constipation, and urinary retention. Neither vomiting nor diarrhea would be clinical manifestation observed under these circumstances. PTS: 1 REF: Pages 567-568 Chapter 18: Disorders of the Central and Peripheral Nervous Systems and the Neuromuscular Junction MULTIPLE CHOICE 1. Diffuse axonal injuries (DAIs) of the brain often result in: a. Reduced levels of consciousness c. Fine motor tremors b. Mild but permanent dysfunction d. Visual disturbances ANS: A Focal brain injuries account for more than two-thirds of head injury deaths; DAIs accounts for less than one third. However, more severely disabled survivors, including those in an unresponsive state or reduced level of consciousness, have DAIs. The other options do not appropriately complete the stem. PTS: 1 REF: Page 582 2. What event is most likely to occur to the brain in a classic cerebral concussion? a. Brief period of vital sign instability b. Cerebral edema throughout the cerebral cortex c. Cerebral edema throughout the diencephalon d. Disruption of axons extending from the diencephalon and brainstem ANS: A Transient cessation of respiration can occur with brief periods of bradycardia, and a decrease in blood pressure occurs, lasting 30 seconds or less. Vital signs stabilize within a few seconds to within normal limiNtsU.RTShINe GoTthBe.rCoOpMtions do not accurately describe an event associated with a classic cerebral concussion. PTS: 1 REF: Page 588 3. Which disorder has clinical manifestations that include decreased consciousness for up to 6 hours, as well as retrograde and posttraumatic amnesia? a. Mild concussion c. Cortical contusion b. Classic concussion d. Acute subdural hematoma ANS: B Evidence of a classic concussion is the immediate loss of consciousness, which lasts less than 6 hours. Retrograde and anterograde (posttraumatic) amnesia is also present. The other options do not apply. PTS: 1 REF: Page 588 4. What group is most at risk of spinal cord injury from minor trauma? a. Children c. Adults b. Adolescents d. Older adults ANS: D Because of preexisting degenerative vertebral disorders, older adults are particularly at risk for minor trauma, resulting in serious spinal cord injury, especially from falls. The risk to the other age groups is less than that of the older adult. PTS: 1 REF: Page 634 5. The edema of the upper cervical cord after spinal cord injury is considered life threatening because of which possible outcome? a. Hypovolemic shock from blood lost during the injury b. Breathing difficulties from an impairment to the diaphragm c. Head injury that likely occurred during the injury d. Spinal shock immediately after the injury ANS: B In the cervical region, spinal cord swelling may be life threatening because of the possibility of resulting impairment of the diaphragm function (phrenic nerves exit C3-C5). The other options do not appropriately explain the threat. PTS: 1 REF: Page 591 6. What indicates that spinal shock is terminating? a. Voluntary movement below the level of injury b. Reflex emptying of the bladder c. Paresthesia below the level of injury d. Decreased deep tendon reflexes and flaccid paralysis ANS: B Indications that spinal shock is terminating include the reappearance of reflex activity, hyperreflexia, spasticity, and reflex emptying of the bladder. Termination of a spinal cord injury is not evidenced by any of the other options. NURSINGTB.COM PTS: 1 REF: Page 592 7. What term is used to describe the complication that can result from a spinal cord injury above T6 that is producing paroxysmal hypertension, as well as piloerection and sweating above the spinal cord lesion? a. Craniosacral dysreflexia c. Autonomic hyperreflexia b. Parasympathetic dysreflexia d. Retrograde hyperreflexia ANS: C Individuals most likely to be affected have lesions at the T6 level or above. Paroxysmal hypertension (up to 300 mm Hg systolic), a pounding headache, blurred vision, sweating above the level of the lesion with flushing of the skin, nasal congestion, nausea, piloerection caused by pilomotor spasm, and bradycardia (30 to 40 beats/minute) characterize autonomic hyperreflexia. No other options appropriately describe this complication. PTS: 1 REF: Pages 593-594 8. Why does a person who has a spinal cord injury experience faulty control of sweating? a. The hypothalamus is unable to regulate body heat as a result of damage to the sympathetic nervous system. b. The thalamus is unable to regulate body heat as a result of damage to the sympathetic nervous system. c. The hypothalamus is unable to regulate body heat as a result of damage to the parasympathetic nervous system. d. The thalamus is unable to regulate body heat as a result of damage to spinal nerve roots. ANS: A A spinal cord injury results in disturbed thermal control because the hypothalamus is unable to regulate a damaged sympathetic nervous system. This damage causes faulty control of sweating and radiation through capillary dilation. The other options do not appropriately describe the process that causes faulty control of sweating. PTS: 1 REF: Page 592 9. Autonomic hyperreflexia–induced bradycardia is a result of stimulation of the: a. Sympathetic nervous system to ß-adrenergic receptors to the sinoatrial node b. Carotid sinus to the vagus nerve to the sinoatrial node c. Parasympathetic nervous system to the glossopharyngeal nerve to the atrioventricular node d. Bundle branches to the -adrenergic receptors to the sinoatrial node ANS: B The intact autonomic nervous system reflexively responds with an arteriolar spasm that increases blood pressure. Baroreceptors in the cerebral vessels, the carotid sinus, and the aorta sense the hypertension and stimulate the parasympathetic system. The heart rate decreases, but the visceral and peripheral vessels do not dilate because efferent impulses cannot pass through the cord. The process is not appropriately described by the other options. PTS: 1 REF: Pages 5N9U3-R5S9I5N|GFTigBu.CreO1M8-13 10. A herniation of which disk will likely result in motor and sensory changes of the lateral lower legs and soles of the feet? a. L2-L3 c. L5-S1 b. L3-L5 d. S2-S3 ANS: C Clinical manifestations of posterolateral protrusions include radicular pain exacerbated by movement and straining (medial calf suggests L5; lateral calf suggests S1 root compression). Herniation of any of the other vertebrae will not result in the described symptoms. PTS: 1 REF: Pages 595-596 | Figure 18-14 11. Which condition poses the highest risk for a cerebrovascular accident (CVA)? a. Insulin-resistant diabetes mellitus c. Polycythemia b. Hypertension d. Smoking ANS: B Hypertension is the single greatest risk factor for stroke. The other options are recognized risk factors but do not carry the intensity of hypertension. PTS: 1 REF: Page 598 12. A right hemisphere embolic CVA has resulted in left-sided paralysis and reduced sensation of the left foot and leg. Which cerebral artery is most likely affected by the emboli? a. Middle cerebral c. Posterior cerebral b. Vertebral d. Anterior cerebral ANS: D Symptoms of an embolic stroke in only the right anterior cerebral artery would include left-sided contralateral paralysis or paresis (greater in the foot and thigh) and mild upper extremity weakness with mild contralateral lower extremity sensory deficiency with loss of vibratory and/or position sense and loss of two-point discrimination. PTS: 1 REF: Page 600 | Table 18-5 13. Atrial fibrillation, rheumatic heart disease, and valvular prosthetics are risk factors for which type of stroke? a. Hemorrhagic c. Embolic b. Thrombotic d. Lacunar ANS: C High-risk sources for the onset of embolic stroke are atrial fibrillation (15% to 25% of strokes), left ventricular aneurysm or thrombus, left atrial thrombus, recent myocardial infarction, rheumatic valvular disease, mechanical prosthetic valve, nonbacterial thrombotic endocarditis, bacterial endocarditis, patent foramen ovale, and primary intracardiac tumors. These are not risk factors for the other options provided. PTS: 1 REF: Page 599 NURSINGTB.COM 14. Microinfarcts resulting in pure motor or pure sensory deficits are the result of which type of stroke? a. Embolic c. Lacunar b. Hemorrhagic d. Thrombotic ANS: C A lacunar stroke (lacunar infarct) is a microinfarct smaller than 1 cm in diameter. Because of the subcortical location and small area of infarction, these strokes may have pure motor and sensory deficits. The other options would not result in the complications described. PTS: 1 REF: Page 599 15. Which vascular malformation is characterized by arteries that feed directly into veins through vascular tangles of abnormal vessels? a. Cavernous angioma c. Arteriovenous angioma b. Capillary telangiectasia d. Arteriovenous malformation ANS: D In only an arteriovenous malformation (AVM), do arteries feed directly into veins through a vascular tangle of malformed vessels. PTS: 1 REF: Pages 604-605 16. Which clinical finding is considered a diagnostic indicator for an arteriovenous malformation (AVM)? a. Systolic bruit over the carotid artery b. Decreased level of consciousness c. Hypertension with bradycardia d. Diastolic bruit over the temporal artery ANS: A A systolic bruit over the carotid in the neck, the mastoid process, or (in a young person) the eyeball is almost always diagnostic of an AVM. The other options are not as indicative as the systolic bruit. PTS: 1 REF: Page 605 17. Which cerebral vascular hemorrhage causes meningeal irritation, photophobia, and positive Kernig and Brudzinski signs? a. Intracranial c. Epidural b. Subarachnoid d. Subdural ANS: B Assessment findings related to only a subarachnoid hemorrhage include meningeal irritation and inflammation, causing neck stiffness (nuchal rigidity), photophobia, blurred vision, irritability, restlessness, and low-grade fever. A positive Kernig sign, in which straightening the knee with the hip and knee in a flexed position produces pain in the back and neck regions, and a positive Brudzinski sign, in which passive flexion of the neck produces neck pain and increased rigidity, may appear. PTS: 1 REF: Pages 6N0U5-R6S0I6NGTB.COM 18. In adults, most intracranial tumors are located: a. Infratentorially c. Laterally b. Supratentiorially d. Posterolaterally ANS: A Approximately 70% to 75% of all intracranial tumors diagnosed in adults are located supratentorially (above the tentorium cerebella). The other options are not primary locations for intracranial tumors in adults. PTS: 1 REF: Page 626 19. In children, most intracranial tumors are located: a. Infratentorially c. Laterally b. Supratentiorially d. Posterolaterally ANS: B Approximately 70% of all intracranial tumors in children are located infratentorially (below the tentorium cerebelli) and not in the locations provided by the other options. PTS: 1 REF: Page 626 20. The most common primary central nervous system (CNS) tumor is the: a. Microglioma c. Astrocytoma b. Neuroblastoma d. Neuroma ANS: C Astrocytomas are the most common primary CNS tumors (50% of all brain and spinal cord tumors). The other options do not occur as frequently. PTS: 1 REF: Page 629 21. Meningiomas characteristically compress from: a. Within neural tissues c. Outside the spinal cord b. Outside spinal nerve roots d. Within the subarachnoid space ANS: C Extramedullary spinal cord tumors produce dysfunction by compression of adjacent tissue, not by direct invasion. The pathologic characteristic of meningiomas is not appropriately described by the other options. PTS: 1 REF: Page 633 22. What is the central component of the pathogenic model of multiple sclerosis? a. Myelination of nerve fibers in the peripheral nervous system (PNS) b. Demyelination of nerve fibers in the CNS c. Development of neurofibrillary tangles in the CNS d. Inherited autosomal dominant trait with high penetrance ANS: B Multiple sclerosis (MS) is an autoimmune disorder diffusely involving the degeneration of CNS myelin and loss of axons. The other options are not central components of the pathogenic model of MS. NURSINGTB.COM PTS: 1 REF: Pages 618-619 23. A blunt force injury to the forehead would result in a coup injury to which region of the brain? a. Frontal c. Parietal b. Temporal d. Occipital ANS: A Coup injuries occur directly below the point of impact. Objects striking the front of the head usually produce only coup injuries (contusions and fractures) because the inner skull in the occipital area is smooth. A coup injury is not nearly as likely when other portions of the brain are affected. PTS: 1 REF: Page 583 24. A blunt force injury to the forehead would result in a contrecoup injury to which region of the brain? a. Frontal c. Parietal b. Temporal d. Occipital ANS: A The focal injury produces a contrecoup (on the pole opposite the site of impact) injury. The frontal portion of the brain is opposite of the site of impact. Objects striking the back of the head usually result in both coup and contrecoup injuries because of the irregularity of the inner surface of the frontal bones. A contrecoup injury is not nearly as likely when other portions of the brain are affected. PTS: 1 REF: Page 583 25. Spinal cord injuries most likely occur in which region? a. Cervical and thoracic c. Lumbar and sacral b. Thoracic and lumbar d. Cervical and thoracic-lumbar ANS: D Vertebral injuries most often occur at vertebrae C1-C2 (cervical), C4-C7, and T1-L2 (thoracic lumbar). None of the other options are applicable. PTS: 1 REF: Pages 590-591 26. The most likely rationale for body temperature fluctuations after cervical spinal cord injury is that the person has: a. Developed bilateral pneumonia or a urinary tract infection. b. Sustain sympathetic nervous system damage resulting in disturbed thermal control. c. Sustained a head injury that damaged the hypothalamus’s ability to regulate temperature. d. Developed septicemia from posttrauma infection. ANS: B Spinal cord injuries result in distuNrbUeRdStIhNeGrmTBa.lCcOoMntrol because the sympathetic nervous system is damaged. None of the remaining options explain this complication. PTS: 1 REF: Page 592 27. A man who sustained a cervical spinal cord injury 2 days ago suddenly develops severe hypertension and bradycardia. He reports severe head pain and blurred vision. The most likely explanation for these clinical manifestations is that he is: a. Experiencing acute anxiety b. Developing spinal shock c. Developing autonomic hyperreflexia d. Experiencing parasympathetic areflexia ANS: C Autonomic hyperreflexia is the only option that is characterized by paroxysmal hypertension (up to 300 mm Hg systolic), a pounding headache, blurred vision, sweating above the level of the lesion with flushing of the skin, nasal congestion, nausea, piloerection caused by pilomotor spasm, and bradycardia (30 to 40 beats per minute). PTS: 1 REF: Pages 593-594 28. The type of vascular malformation that most often results in hemorrhage is: a. Cavernous angioma c. Capillary telangiectasia b. Venous angioma d. Arteriovenous malformation ANS: D In an arteriovenous malformation (AVM), arteries feed directly into veins through a vascular tangle of malformed vessels, causing venous hemorrhaging. The other options are not as likely to result in a hemorrhage. PTS: 1 REF: Pages 604-605 29. Atheromatous plaques are most commonly found: a. In larger veins c. At branches of arteries b. Near capillary sphincters d. On the venous sinuses ANS: C Over 20 to 30 years, atheromatous plaques (stenotic lesions) tend to form at branchings and curves in the cerebral circulation, not at any of the other options provided. PTS: 1 REF: Page 598 30. Multiple sclerosis is best described as a(an): a. Central nervous system demyelination, possibly from an immunogenetic virus b. Inadequate supply of acetylcholine at the neurotransmitter junction as a result of an autoimmune disorder c. Depletion of dopamine in the central nervous system as a result of a virus d. Degenerative disorder of lower and upper motor neurons caused by viral-immune factors ANS: A Multiple sclerosis (MS) is an autoimmune disorder diffusely involving the degeneration of central nervous system (CNS) myNelUinRSaInNdGlToBss.CoOfMaxons. MS is described as occurring when a previous infectious insult to the CNS has occurred in a genetically susceptible individual with a subsequent abnormal immune response in the CNS. The other options do not adequately describe MS. PTS: 1 REF: Pages 618-619 31. What is the most common opportunistic infection associated with acquired immunodeficiency syndrome (AIDS)? a. Non-Hodgkin lymphoma c. Toxoplasmosis b. Kaposi sarcoma d. Cytomegalovirus ANS: C Toxoplasmosis is the most common opportunistic infection and occurs in approximately one third of individuals with AIDS. Cytomegalovirus encephalitis is common in those with AIDS but is often not diagnosed while the person is alive. Other neoplasms associated with human immunodeficiency virus (HIV) include systemic non-Hodgkin lymphoma and metastatic Kaposi sarcoma. PTS: 1 REF: Page 617 32. It is true that Guillain-Barré syndrome (GBS): a. Is preceded by a viral illness. c. Results in asymmetric paralysis. b. Involves a deficit in acetylcholine. d. Is an outcome of HIV. ANS: A GBS is considered to be an autoimmune disease triggered by a preceding bacterial or viral infection. None of the other options are true of GBS. PTS: 1 REF: Pages 622-623 33. It is true that myasthenia gravis: a. Is an acute autoimmune disease. c. May result in adrenergic crisis. b. Affects the nerve roots. d. Causes muscle weakness. ANS: D Exertional fatigue and weakness that worsens with activity, improves with rest, and recurs with resumption of activity characterizes myasthenia gravis. None of the other options are true of myasthenia gravis. PTS: 1 REF: Page 626 34. In which disorder are acetylcholine receptor antibodies (IgG antibodies) produced against acetylcholine receptors? a. Guillain-Barré syndrome c. Myasthenia gravis b. Multiple sclerosis d. Parkinson disease ANS: C The main defect of myasthenia gravis is the formation of autoantibodies (an immunoglobulin G [IgG] antibody) against receptors at the Ach-binding site on the postsynaptic membrane. This defect is not found in any of the other options. PTS: 1 REF: Pages 6N2U5-R6S2I6NGTB.COM 35. Multiple sclerosis and Guillain-Barré syndrome are similar in that they both: a. Result from demyelination by an immune reaction. b. Cause permanent destruction of peripheral nerves. c. Result from inadequate production of neurotransmitters. d. Block acetylcholine receptor sites at the myoneuronal junction. ANS: A Acute inflammatory demyelinating polyneuropathy (AIDP) accounts for most occurrences of Guillain- Barré syndrome (GBS). Multiple sclerosis (MS) is an autoimmune disorder diffusely involving degeneration of CNS myelin and loss of axons. Only the correct option accurately describes the similarity between MS and GBS PTS: 1 REF: Page 618 | Pages 622-623 MULTIPLE RESPONSE 36. Which clinical manifestation is characteristic of cluster headaches? (Select all that apply.) a. Preheadache aura b. Severe unilateral tearing c. Gradual onset of a tight band around the head d. Significant unilateral, temporal pain e. Pain lasting from 30 to 120 minutes ANS: B, C, E The cluster headache attack usually begins without warning and is characterized by severe, unilateral tearing, burning, periorbital, and retrobulbar or temporal pain lasting 30 minutes to 2 hours. Neither preheadache aura nor significant unilateral, temporal pain is clinical manifestation characteristic of cluster headaches. PTS: 1 REF: Pages 608-609 37. What are the initial clinical manifestations immediately noted after a spinal cord injury? (Select all that apply.) a. Headache b. Bladder incontinence c. Loss of deep tendon reflexes d. Hypertension e. Flaccid paralysis ANS: B, C, E A complete loss of reflex function in all segments below the level of the lesion characterizes a spinal cord injury. Severe impairment below the level of the lesion is obvious; it includes paralysis and flaccidity in muscles, absence of sensation, loss of bladder and rectal control, transient drop in blood pressure, and poor venous circulation. Neither headache nor hypertension is an initial clinical manifestation related to a spinal cord injury. PTS: 1 REF: Page 592 MATCHING NURSINGTB.COM Match the terms with the corresponding descriptions. A. Complication of mastoiditis B. Opportunistic infection C. CNS manifestation of tuberculosis D. Mosquito-borne viral infection E. Tick-borne bacterial infection 38. Meningitis 39. Encephalitis 40. Cryptococcus neoformans 41. Brain abscess 42. Lyme disease 38. ANS: C PTS: 1 REF: Pages 611-612 MSC: Tubercular meningitis is the most common and serious form of central nervous system (CNS) tuberculosis. 39. ANS: D PTS: 1 REF: Page 614 MSC: Encephalitis is an acute febrile illness, usually of viral origin, with nervous system involvement. Arthropod-borne (mosquito-borne) viruses and herpes simplex cause the most common encephalitides. 40. ANS: B PTS: 1 REF: Page 617 MSC: Opportunistic infections may be bacterial, fungal, protozoal, or viral in origin and produce nervous system disease. Cryptococcus neoformans is an example of such an infection. 41. ANS: A PTS: 1 REF: Page 613 MSC: Abscesses may occur in association with a contiguous spread of infection, such as the middle ear, mastoid cells, nasal cavity, and nasal sinuses. 42. ANS: E PTS: 1 REF: Page 617 MSC: Lyme disease, a tick-borne spirochete bacterial infection, is a common arthropod-borne infection in the United States. NURSINGTB.COM Chapter 19: Neurobiology of Schizophrenia, Mood Disorders, and Anxiety Disorders MULTIPLE CHOICE 1. Which would be considered a positive symptom of schizophrenia? a. Blunted affect c. Poverty of speech b. Auditory hallucinations d. Lack of social interaction ANS: B Positive symptoms frequently occur during a psychotic episode, when an individual loses touch with reality and experiences something that should be absent (e.g., hallucinations). The remaining options are classified as negative symptoms. PTS: 1 REF: Pages 644-646 2. The onset of schizophrenia can be triggered by which prenatal occurrence? a. Viral infection c. Maternal smoking b. Maternal depression d. Exposure to toxic waste ANS: A A leading hypothesis for the cause of schizophrenia suggests that the illness results from neurodevelopmental defects that occur in fetal life. Several early environmental factors have been suggested to increase the risk of developing schizophrenia, including viral infection during pregnancy, prenatal nutritional deficiencies, and perinatal complications, such as birth defects and neonatal hypoxia. No current research supports the theory that any of the other options are prenatNaUl RtrSigINgGerTsBo.Cf OscMhizophrenia. PTS: 1 REF: Page 642 3. Which neurotransmitter is reduced in people with schizophrenia? a. Dopamine c. Acetylcholine b. Gamma-aminobutyric acid d. Serotonin ANS: B In the dorsal prefrontal cortex of schizophrenic brains, glutamic acid decarboxylase, the major enzyme in gamma-aminobutyric acid GABA biosynthesis, is diminished, which likely impairs synaptic performance and cognitive and behavioral functions associated with this brain region. The other neurotransmitters mentioned in the option choices are not related to the presentation of schizophrenia. PTS: 1 REF: Page 643 4. Alterations in which part of the brain are linked to hallucinations, delusions, and thought disorders associated with schizophrenia? a. Parietal lobe c. Temporal lobe b. Limbic system d. Hypothalamus ANS: C . Only temporal lobe alterations may be responsible for the production of positive schizophrenic symptoms, such as hallucinations, delusions, thought disorders, and bizarre behavior. PTS: 1 REF: Page 642 5. Antipsychotic drugs block which neurotransmitter receptor? a. Norepinephrine c. Serotonin b. Gamma-aminobutyric acid d. Dopamine ANS: D The dopamine hypothesis initially suggested that abnormal elevation in dopaminergic transmission contributes to the onset of schizophrenia. This hypothesis was based on pharmacologic studies showing that antipsychotic drugs are potent blockers of brain dopamine receptors; therefore the other options are incorrect. PTS: 1 REF: Page 643 6. What data confer the link between bipolar disorders and schizophrenia? a. Individuals with bipolar disorder who exhibit psychotic behaviors have deficits in reelin expression linked to genetic loci located on chromosome 22. b. Individuals with schizophrenia who exhibit psychotic behaviors have deficits in serotonin linked to genetic loci located on chromosome 16. c. Individuals with bipolar disorder who exhibit psychotic behaviors have deficits in gamma-aminobutyric acid (GABA) linked to genetic loci located on chromosome 20. d. Individuals with schizophrenia who exhibit psychotic behaviors have deficits in reelin expression linked to genNeUtiRcSlIoNcGi TloBc.CatOeMd on chromosome 18. ANS: A Interestingly, loci on chromosomes 18 and 22 have been linked to bipolar disorder and schizophrenia. Individuals with bipolar disorder, who may exhibit psychotic behavior, have deficits in reelin expression linked to genetic loci located on chromosome 22, which confers susceptibility to schizophrenia. The remaining options do not appropriately describe the link between bipolar disorder and schizophrenia. PTS: 1 REF: Pages 647-648 7. Hypothalamic-pituitary-adrenal (HPA) system abnormalities exist in a large percentage of individuals with: a. Schizophrenia c. Mania b. Major depression d. Panic disorder ANS: B Excessive activation of the HPA system resulting in elevated glucocorticoid secretion is found in a large percentage (30% to 70%) of people with major depression, suggesting that mechanisms responsible for HPA hormone alterations contribute to the pathophysiologic condition of depression. HPA system abnormalities do not necessarily exit in individuals represented by the remaining options. PTS: 1 REF: Page 648 8. The common property among the three types of medications used to treat depression is that they: a. Increase neurotransmitter levels within the synapse. b. Increase neurotransmitter levels in the presynapse. c. Decrease neurotransmitter levels in the postsynapse. d. Decrease neurotransmitter levels within the synapse. ANS: A All available antidepressants share the common property, albeit through different mechanisms, that increasing monoamine neurotransmitter levels within the synapse is the basis for their antidepressant effects. The processes by which antidepressants affect depression make the remaining options untrue. PTS: 1 REF: Page 648 9. The link between major depression and cortisol secretion is that individuals with depression: a. Show suppression of plasma cortisol when given dexamethasone. b. Have a decreased plasma cortisol level, despite the administration of exogenous corticosteroids. c. Show that persistently elevated plasma cortisol levels can result in inflammation that is believed to trigger depression. d. Have normal plasma cortisol levels throughout the day when they take antidepressant medication as prescribed. ANS: C . Persistent elevations in cortisol mNayURalSsIoNiGnTdBu.cCeOiMmmunosuppression that compromises the body’s immune systems to contain inflammation and infectious diseases. Increasing evidence suggests that inflammation is another risk factor that triggers the onset of depression. The options related to dexamethasone and exogenous corticosteroids are not true as they apply to depression and cortisol secretion. PTS: 1 REF: Page 648 10. A decrease in receptor binding for which neurotransmitter is found in individuals with depression? a. Norepinephrine c. Dopamine b. Serotonin d. Acetylcholine ANS: B Postmortem and/or brain imaging studies of individuals with depression reveal a widespread decrease in serotonin 5-HT1A-receptor subtype binding in frontal, temporal, and limbic cortex, as well as serotonin-transporter binding in cerebral cortex and hippocampus. A decrease in receptor binding is not observed in the other neurotransmitters. PTS: 1 REF: Page 650 11. When treating individuals with depression, the result produced by electroconvulsive therapy (ECT) is believed to be an alteration in: a. The monoamine systems c. Norepinephrine b. Serotonin d. The limbic system ANS: A Although the mechanism of action of ECT is not clear, the procedure is known to produce alterations in only the monoamine systems. PTS: 1 REF: Page 652 12. Which electrolyte imbalance contributes to lithium toxicity? a. Hypernatremia c. Hyperkalemia b. Hyponatremia d. Hypokalemia ANS: B Lithium toxicity can be a result of hyponatremia. Lithium is normally removed from the kidneys; however, when the body is sodium depleted, the kidneys reabsorb sodium along with lithium. Lithium toxicity does not result from any of the remaining options. PTS: 1 REF: Pages 652-653 13. Which neurotransmitter is inhibited in panic disorders? a. Norepinephrine c. Dopamine b. Serotonin d. Gamma-aminobutyric acid (GABA) ANS: D Panic disorder also may involve the GABA-benzodiazepine (BZ) receptor system. The other options are more related to depression. PTS: 1 REF: Page 65N4URSINGTB.COM 14. A criterion for a diagnosis of generalized anxiety disorder (GAD) is a period of excessive worrying that lasts for at least how many months? a. 3 c. 9 b. 6 d. 12 ANS: B GAD is diagnosed when an individual spends at least 6 months worrying excessively and exhibits at least three of the six symptoms. Although 3 months is not sufficient time, the remaining options are excessive. PTS: 1 REF: Page 655 15. Individuals who eat aged cheese and avocado when taking monoamine oxidase inhibitors (MAOIs) may experience: a. Kidney damage c. Orthostatic hypotension b. Hypertensive crisis d. Weight gain ANS: B MAOIs also may induce acute and heightened elevations in blood pressure (e.g., hypertensive crisis) after the intake of tyramine-rich foods, such as aged cheeses, sour cream, pods of broad beans, pickled herring, liver, canned figs, raisins, and avocados. The ingestion of tyramine-rich foods is not a trigger for the remaining options. PTS: 1 REF: Page 651 16. Electroconvulsive therapy (ECT) is used to treat depression: a. In pregnant women b. By enhancing the effect of antidepressants c. As a first-line treatment before medications d. That is unipolar ANS: A ECT is used to treat depression when individuals fail to respond to antidepressants or when they are severely depressed, pregnant, suicidal, or psychotic. The other options are not true regarding the conditions of this treatment. PTS: 1 REF: Page 652 17. A notable complication of panic disorder is: a. Avolition c. Alogia b. Anhedonia d. Agoraphobia ANS: D Of the available options, the only notable complication of panic disorder is the development of agoraphobia or phobic avoidance of places or situations where escape or help is not readily available. PTS: 1 REF: Page 654 18. Persistent symptoms of posttraumatic stress disorder (PTSD) include: a. Depression NURSINGTcB..CONMightmares b. Hypertension d. Poor nutrition ANS: C . In PTSD, the individual re-experiences the traumatic event as intrusive collections or flashbacks during the day and during persistent nightmares. Nightmares replicate the traumatic experiences and often prevent sleep. The remaining options are not typically attributed to PTSD. PTS: 1 REF: Page 656 19. Hallucinations, delusions, and thought disorders occur with alterations to which part of the brain? a. Temporal lobe c. Hypothalamus b. Parietal lobe d. Cerebral cortex ANS: A Only temporal lobe alterations may be responsible for the production of positive schizophrenic symptoms, such as hallucinations, delusions, and thought disorders. PTS: 1 REF: Page 642 MULTIPLE RESPONSE 20. Significant numbers of individuals with depression have problems related to: (Select all that apply.) a. Sleep cycles b. Weight stabilization c. Eating patterns d. Thyroid function e. Cognitive skills ANS: A, B, C, D Insomnia, loss of appetite and body weight, and reduced interest in pleasurable activities and interpersonal relationships frequently accompany depression. Approximately 20% to 30% of persons with unipolar depression have an altered hypothalamic-pituitary-thyroid (HPT) system. Impaired cognitive skills are not necessarily associated with depression. PTS: 1 REF: Pages 648 | Pages 650-651 21. Which neurotransmitter is inhibited in generalized anxiety disorder (GAD)? (Select all that apply.) a. Acetylcholine b. Serotonin c. Dopamine d. Norepinephrine e. Epinephrine ANS: B, D Abnormalities in the norepinephrine and serotonin systems were reported in GAD. The other options are more related to depression. NURSINGTB.COM PTS: 1 REF: Page 655 22. What are the most common side effects of selective serotonin reuptake inhibitors (SSRIs)? (Select all that apply.) a. Orthostatic hypotension b. Dry mouth c. Sleep disturbances d. Agitation e. Nausea ANS: C, E Common side effects of SSRIs include sleep disturbances (e.g., insomnia) and nausea. The remaining options are not common side effects of SSRIs. PTS: 1 REF: Page 652 MATCHING Match the terms with the corresponding descriptions. A. Delusions B. Formal thought disorder C. Anhedonia D. Hallucinations E. Alogia F. Avolition 23. Perceptions are experienced without external stimulation of the sense organs. 24. Persistent beliefs are contrary to the educational and cultural background of the individual. 25. Fluent speech is difficult to comprehend. 26. Spontaneous speech is absent. 27. Condition is characterized by an inability to have emotional experiences. 28. Condition is characterized by a deficit in spontaneous or goal-directed activities. 23. ANS: D PTS: 1 REF: Page 644 MSC: A hallucination is a perception experienced without external stimulation of the sense organs. 24. ANS: A PTS: 1 REF: Page 646 MSC: A delusion is a persistent belief contrary to the educational and cultural background of the individual. 25. ANS: B PTS: 1 REF: Page 646 MSC: A common form of disorganized speech is formal thought disorder, which involves fluent speech that is difficult to comprehend. 26. ANS: E PTS: 1 REF: Page 646 MSC: Alogia is the absence of spontaneous speech production for the purpose of answering questions or expressing oneself. 27. ANS: C PTS: 1 REF: Page 646 MSC: In anhedonia, individuals are unable to experience emotions such as pleasure or pain; they report a sense of detachment from the environment. 28. ANS: F PTS: 1 REF: Page 646 MSC: Avolition is a deficit in spontaneous or goal-directed behavior in which an individual may sit for prolonged periods and must beNpUrRodSdINedGiTnBto.CcOoMmpleting simple daily tasks. Chapter 20: Alterations of Neurologic Function in Children MULTIPLE CHOICE 1. The neural groove closes dorsally during which week of gestational life? a. Second c. Eighth b. Fourth d. Twelfth ANS: B During the fourth gestational week, the neural groove deepens, its folds develop laterally, and it closes dorsally to form the neural tube, epithelial tissue that ultimately becomes the central nervous system (CNS). The second week is too early, and the other options represent times periods after the groove closes. PTS: 1 REF: Page 660 2. Which nutritional deficiency in a pregnant woman is associated with neural tube defect (NTD)? a. Iron c. Zinc b. Vitamin C d. Folate ANS: D Maternal folate deficiency is associated with NTDs, but the specific mechanism that relates to how folate supplements prevent these anomalies is unknown. The other options are not thought to cause such a defect. PTS: 1 REF: Page 66N3URSINGTB.COM 3. Which defect of neural tube closure is most common? a. Anterior c. Lateral b. Posterior d. Midline ANS: B Posterior defects are most common. The other options are not the most common. PTS: 1 REF: Page 663 4. What is the anomaly in which the soft bony component of the skull and much of the brain is missing? a. Anencephaly c. Cranial meningocele b. Myelodysplasia d. Hydrocephaly ANS: A Anencephaly is an anomaly in which the soft, bony component of the skull and much of the brain are missing. The other options do not represent this condition. PTS: 1 REF: Pages 663-664 5. The most common cause of obstructive hydrocephalus in infants is: a. Obstructed arachnoid villi b. Stenosis of the aqueduct of Sylvius c. Excessive production of cerebrospinal fluid d. Impaired cerebrospinal fluid circulation in the subarachnoid space ANS: B Congenital aqueduct stenosis most commonly causes obstructive hydrocephalus. The other options do not represent the most common cause of this condition. PTS: 1 REF: Page 669 6. What is the term for a herniation or protrusion of brain and meninges through a defect in the skull? a. Encephalocele c. Arachnoidocele b. Meningocele d. Cephacephalocele ANS: A Encephalocele refers to a herniation or protrusion of brain and meninges through a defect in the skull, resulting in a saclike structure. The other terms are not used to describe an encephalocele. PTS: 1 REF: Page 664 7. What is the result of a Chiari type II malformation associated with a myelomeningocele? a. Upward displacement of the cerebellum into the diencephalon b. Motor and sensory lesions below the level of the myelomeningocele c. Downward displacement of the cerebellum, brainstem, and fourth ventricle d. Generalized cerebral edema and hydrocephalus ANS: C NURSINGTB.COM One serious, potentially life-threatening problem associated with myelomeningocele is the Chiari type II malformation. This deformity involves the downward displacement of the cerebellum, cerebellar tonsils, brainstem, and fourth ventricle. The other options do not appropriately describe the pathologic characteristics resulting from a Chiari type II malformation. PTS: 1 REF: Page 665 8. Prompt surgical repair of a myelomeningocele is critical to best prevent: a. Infection c. Mental retardation b. Paralysis d. Additional nervous system damage ANS: D Until the myelomeningocele is surgically closed, cerebrospinal fluid (CSF) may accumulate, resulting in further dilation and enlargement of the sac, which may risk more damage to the nervous system. The other options are included as possible damage caused by the increased leaking of CSF. PTS: 1 REF: Page 665 9. Which body system is the largest site for human immunodeficiency virus (HIV) infection in infants and children? a. Central nervous system c. Integumentary system b. Gastrointestinal system d. Musculoskeletal system ANS: A A particularly vulnerable site of HIV-1 infection in infants and children is the CNS. This vulnerability cannot be proven regarding the other provided options. PTS: 1 REF: Page 677 10. An infant diagnosed with hydrocephalus is observed to demonstrate: a. Shrunken ventricles b. Bulging fontanels c. Retarded head growth d. Decreased production of cerebrospinal fluid ANS: B During the early weeks of life, the head begins to grow at an abnormal rate. Significant dilation of the ventricles may occur before an abnormal increase in head growth develops. The fontanels enlarge and become full and bulging. Cerebrospinal fluid production does not decrease. PTS: 1 REF: Pages 668-670 11. Gait disturbances and instability are characteristic of which form of cerebral palsy? a. Spastic c. Ataxic b. Dyskinetic d. Biochemical ANS: C Ataxic cerebral palsy causes damage to the cerebellum and exhibits gait disturbances and instability. This description is not accurate of the other options. NURSINGTB.COM PTS: 1 REF: Page 672 12. Children with phenylketonuria (PKU) are unable to synthesize: a. Essential amino acid, phenylalanine, to tyrosine b. Renin, erythropoietin, and antidiuretic hormone c. Aldosterone, cortisol, and androgens d. Neurotransmitters gamma-aminobutyric acid (GABA) and acetylcholine ANS: A PKU is an inborn error of metabolism characterized by the inability of the body to convert the essential amino acid, phenylalanine, to tyrosine. PKU does not affect synthesis of the other options. PTS: 1 REF: Page 673 13. Benign febrile seizures are characterized by: a. A temperature lower than 39° C b. Respiratory or ear infections c. Onset after the fifth year of life d. Episodes lasting 30 minutes or longer ANS: B An acute respiratory or ear infection is usually present. Simple febrile seizures are rare in infants before 9 months of age or in children older than 5 years of age. The convulsion occurs with a rise in temperature higher than 39° C (102.2 °F). The convulsion is short (15 minutes or less). PTS: 1 REF: Page 679 14. What is the most common general symptom of a localized childhood brain tumor? a. Poor bonding b. Increased intracranial pressure c. Delayed extinction of newborn reflexes d. Failure to thrive ANS: B Symptoms of brain tumors may be generalized or localized. The most common general symptom is increased intracranial pressure, which may cause headaches, irritability, vomiting, somnolence, and bulging of fontanels. The remaining options are not the most common symptom of a localized childhood tumor. PTS: 1 REF: Page 685 15. The tonic neck reflex observed in a newborn should no longer be obtainable by: a. 2 years c. 10 months b. 1 year d. 5 months ANS: D The tonic neck reflex should be unobtainable by 5 months of age. NURSINGTB.COM PTS: 1 REF: Page 663 | Table 20-1 16. What term is used to describe a hernial protrusion of a saclike cyst that contains meninges, spinal fluid, and a portion of the spinal cord through a defect in a posterior arch of a vertebra? a. Encephalocele c. Spina bifida occulta b. Meningocele d. Myelomeningocele ANS: D Myelomeningocele is a hernial protrusion of a saclike cyst containing meninges, spinal fluid, and a portion of the spinal cord with its nerves through a defect in the posterior arch of a vertebra. The remaining options are not appropriate terms to identity the described condition. PTS: 1 REF: Pages 664-665 17. What test is performed on amniotic fluid and maternal blood to test for neural tube defect? a. Total protein c. -fetoprotein b. Culture d. C-reactive protein ANS: C The presence of a neural tube defect (NTD) may result in an elevated amniotic fluid -fetoprotein (AFP) level and subsequent maternal serum AFP levels. The tests presented in the remaining options are not used to diagnose an NTD. PTS: 1 REF: Page 666 18. The clinical manifestations of dyskinetic cerebral palsy include: a. Increased muscle tone and prolonged primitive reflexes b. Exaggerated deep tendon reflexes, clonus, and rigidity of extremities c. Scoliosis, contractures, and stiffness of trunk muscles d. Jerky uncontrolled and abrupt fine musculoskeletal movements ANS: D Dyskinetic cerebral palsy is associated with extreme difficulty in fine motor coordination and purposeful movements. Movements are jerky, uncontrolled, and abrupt, resulting from injury to the basal ganglia or thalamus. The symptoms presented by the other options are not associated with this form of cerebral palsy. PTS: 1 REF: Page 672 MULTIPLE RESPONSE 19. Which musculoskeletal deformities are associated with myelomeningocele? (Select all that apply.) a. Extra digits on feet b. Clubfoot c. Dislocation of the hips d. Scoliosis e. Kyphosis ANS: B, C, D, E NURSINGTB.COM Several musculoskeletal deformities are related to myelomeningocele, including clubfoot, dislocation of hip or hips, and poor spinal alignment. Spinal deformities, such as scoliosis and kyphosis, are also common. Extra digits are not associated with myelomeningocele. PTS: 1 REF: Page 665 20. True microcephaly can be caused by: (Select all that apply.) a. Autosomal gene alterations b. Prenatal physical abuse of the mother c. X-linked gene alterations d. Toxic-induced chromosomal defects e. Maternal anorexia ANS: A, C, D Genetic alterations, including autosomal dominant, autosomal recessive or X-linked genes, or various chromosomal abnormalities can cause true (primary) microcephaly (present at birth). Environmental causes include toxin exposure during the period of induction and major cell migration. Radiation, intrauterine infection, or chemical exposure may be the initiating factor. Secondary microcephaly (developing postnatally) is associated with a variety of causes including infection, trauma, metabolic disorders, maternal anorexia experienced during the third trimester of pregnancy, and the presence of other genetic syndromes. PTS: 1 REF: Pages 667-668 21. Which statements regarding the term myelodysplasia are true? (Select all that apply.) a. Myelodysplasia is used to define a defect in the formation of the spinal cord. b. It can be used to refer to brain anomalies involving missing brain tissue. c. Myelodysplasia correctly includes encephaloceles. d. It is used to describe a herniation of brain and meninges through a defect in the skull. e. Myelodysplasia can be used to refer to a form of spina bifida. ANS: A, E Although myelodysplasia is defined as a defect in the formation of the spinal cord, the term is also used to refer to anomalies of both the vertebral column and the spinal cord. Birth defects in which the vertebrae fail to close are known as spina bifida (split spine). Anencephaly is an anomaly in which the soft, bony component of the skull and much of the brain are missing. Encephalocele refers to a herniation or protrusion of various amounts of brain and meninges through a defect in the skull, resulting in a saclike structure. PTS: 1 REF: Page 663 MATCHING Match the tumor name with its site of development. A. Medulloblastoma B. Ependymoma C. Cerebellar astrocytNomURaSINGTB.COM D. Craniopharyngioma E. Neuroblastoma 22. Originates in the neural crest that normally forms the sympathetic ganglia and adrenal medulla. 23. Develops in the fourth ventricle. 24. Develops in the vermis of the cerebellum and may extend to the fourth ventricle. 25. Originates from the pituitary or hypothalamus. 26. Causes unilateral symptoms such as nystagmus. 22. ANS: E PTS: 1 REF: Page 684 MSC: A neuroblastoma is an embryonal aggressive tumor that originates in the neural crest cells that normally give rise to the sympathetic nervous system (sympathetic ganglia and the adrenal medulla). 23. ANS: B PTS: 1 REF: Page 683 MSC: The ependymoma develops in the fourth ventricle and arises from the ependymal cells that line the ventricular system. 24. ANS: A PTS: 1 REF: Page 683 MSC: A medulloblastoma is an embryonal tumor and the most common childhood malignant tumor. It occurs as an invasive tumor that develops in the vermis of the cerebellum and may extend into the fourth ventricle. 25. ANS: D PTS: 1 REF: Page 683 MSC: The area of the sella turcica, the structure containing the pituitary gland, is the site of several childhood brain tumors, including craniopharyngioma (the most common). These tumors may originate from the pituitary gland or the hypothalamus. 26. ANS: C PTS: 1 REF: Page 683 MSC: Cerebellar astrocytomas are located on the surface of the right or left cerebellar hemisphere and cause unilateral symptoms (occurring on the same side of the tumor), such as head tilt, limb ataxia, and nystagmus when the eyes are turned toward the tumor. NURSINGTB.COM Chapter 21: Mechanisms of Hormonal Regulation MULTIPLE CHOICE 1. What imbalance lessens the rate of secretion of parathyroid hormone (PTH)? a. Increased serum calcium levels b. Decreased serum magnesium levels c. Decreased levels of thyroid-stimulating hormone d. Increased levels of thyroid-stimulating hormone ANS: A The overall effect of parathyroid hormone (PTH)is to increase serum calcium and to decrease serum phosphate concentration. The other presented imbalances will not affect PTH in the described fashion. PTS: 1 REF: Pages 702-703 2. Regulation of the release of catecholamines from the adrenal medulla is an example of which type of regulation? a. Negative feedback c. Neural b. Positive feedback d. Physiologic ANS: C The release of hormones occurs either in response to an alteration in the cellular environment or in the process of maintaining a regulated level of certain hormones or certain substances. Several different mechanisms, one of which is neural control (e.g., stress-induced release of catecholaNmUiRnSeIsNfGroTmB.CthOeMadrenal medulla), regulate the release of hormones. The remaining options do not accurately describe the example given. PTS: 1 REF: Page 690 3. How does a faulty negative-feedback mechanism result in a hormonal imbalance? a. Hormones are not synthesized in response to cellular and tissue activities. b. Decreased hormonal secretion is a response to rising hormone levels. c. Too little hormone production is initiated. d. Excessive hormone production results from a failure to turn off the system. ANS: D Negative-feedback systems are important in maintaining hormones within physiologic ranges. The lack of negative-feedback inhibition on hormonal release often results in pathologic conditions. Excessive hormone production, which is the result of the failure to turn off the system, can cause various hormonal imbalances and related conditions. The correct option is the only accurate description of this hormonal function. PTS: 1 REF: Pages 690-691 4. Which substance is a water-soluble protein hormone? a. Thyroxine c. Follicle-stimulating hormone b. Aldosterone d. Insulin ANS: D Peptide or protein hormones, such as insulin, pituitary, hypothalamic, and parathyroid, are water soluble and circulate in free (unbound) forms. All the remaining options are fat-soluble hormones. PTS: 1 REF: Page 691 5. Which of the following is a lipid-soluble hormone? a. Cortisol c. Epinephrine b. Oxytocin d. Growth hormone ANS: A Cortisol and adrenal androgens are lipid-soluble hormones and are primarily bound to a carrier or transport protein in circulation. The other options are water-soluble hormones. PTS: 1 REF: Page 691 6. Most protein hormones are transported in the bloodstream and are: a. Bound to a lipid-soluble carrier b. Free in an unbound, water-soluble form c. Bound to a water soluble–binding protein d. Free because of their lipid-soluble chemistry ANS: B Peptide or protein hormones, such as insulin, pituitary, hypothalamic, and parathyroid, are water soluble and circulate in free (unbound) forms. The other options are not true statements related to the transport of protein hormones. PTS: 1 REF: Page 69N1URSINGTB.COM 7. When insulin binds its receptors on muscle cells, an increase in glucose uptake by the muscle cells is the result. This is an example of what type of effect by a hormone? a. Pharmacologic c. Synergistic b. Permissive d. Direct ANS: D Direct effects are the obvious changes in cell function that specifically result from the stimulation by a particular hormone. The other options are not used to identify the described effect. PTS: 1 REF: Page 696 8. Thyroid-stimulating hormone (TSH) is released to stimulate thyroid hormone (TH) and is inhibited when plasma levels of TH are adequate. This is an example of: a. Positive feedback c. Neural regulation b. Negative feedback d. Physiologic regulation ANS: B Feedback systems provide precise monitoring and control of the cellular environment. Negative feedback occurs because the changing chemical, neural, or endocrine response to a stimulus negates the initiating change that triggered the release of the hormone. Thyrotropin-releasing hormone (TRH) from the hypothalamus stimulates TSH secretion from the anterior pituitary. Secretion of TSH stimulates the synthesis and secretion of THs. Increasing levels of T4 and triiodothyronine (T3) then generate negative feedback on the pituitary and hypothalamus to inhibit TRH and TSH synthesis. The described example is not accurately identified by any of the other options. PTS: 1 REF: Page 690 9. Lipid-soluble hormone receptors are located: a. Inside the plasma membrane in the cytoplasm b. On the outer surface of the plasma membrane c. Inside the mitochondria d. On the inner surface of the plasma membrane ANS: A Lipid-soluble hormone receptors are located inside the plasma membrane and easily diffuse across the plasma membrane to bind to either cytosolic or nuclear receptors. The other options are not true statements. PTS: 1 REF: Pages 694-695 10. Which second messenger is stimulated by epinephrine binding to a -adrenergic receptor? a. Calcium b. Inositol triphosphate (IP3) c. Diacylglycerol (DAG) NURSINGTB.COM d. Cyclic adenosine monophosphate (cAMP) ANS: D Second-messenger molecules are the initial link between the first signal (hormone) and the inside of the cell (see Table 21-3). For example, the binding of epinephrine to a  adrenergic–receptor subtype activates (through a stimulatory G protein) the enzyme, adenylyl cyclase. Adenylyl cyclase catalyzes the conversion of adenosine triphosphate (ATP) to the second messenger, 3', and 5'-cAMP. The remaining messengers are not stimulated by epinephrine to bind as described. PTS: 1 REF: Page 694 11. Which hormone does the second messenger calcium (Ca++) bind to activate phospholipase C through a G protein? a. Angiotensin II c. Estrogen b. Thyroxine d. Testosterone ANS: A Ca++ is considered an important second messenger that facilitates the binding of a hormone (e.g., norepinephrine, angiotensin II) to a surface receptor, activating the enzyme phospholipase C through a G protein inside the plasma membrane. None of the other options acts on its target cell via a second messenger. PTS: 1 REF: Page 694 12. The control of calcium in cells is important because it: a. Is controlled by the calcium negative-feedback loop. b. Is continuously synthesized. c. Acts as a second messenger. d. Carries lipid-soluble hormones in the bloodstream. ANS: C In addition to being an important ion that participates in a multitude of cellular actions, Ca++ is considered an important second messenger. The other options are not true statements related to the control of calcium within the cells. PTS: 1 REF: Page 694 13. The portion of the pituitary that secretes oxytocin is: a. Posterior c. Anterior b. Inferior d. Superior ANS: A Only the posterior pituitary secretes oxytocin. PTS: 1 REF: Page 698 14. Antidiuretic hormone acts to cause vasoconstriction when: a. Urine output is less than 20 ml/hr. b. Serum osmolality is increased. c. Osmotic and oncotic pressuresNaUrReSiInNcGreTaBs.eCdO. M d. Vasopressin is pharmacologically administered. ANS: D ADH was originally named vasopressin because, in extremely high doses, it causes vasoconstriction and a resulting increase in arterial blood pressure. However, significant vasoconstriction may only be achieved pharmacologically. Antidiuretic hormone induced vasoconstriction is not a result of the other options. PTS: 1 REF: Pages 698-699 15. What is the target tissue for prolactin-releasing factor? a. Hypothalamus c. Mammary glands b. Anterior pituitary d. Posterior pituitary ANS: B Prolactin-releasing factor targets the anterior pituitary gland to stimulate the secretion of prolactin. The other remaining options are incorrect. PTS: 1 REF: Page 698 | Table 21-4 16. Where is antidiuretic hormone (ADH) synthesized, and where does it act? a. Hypothalamus; renal tubular cells c. Anterior pituitary; posterior pituitary b. Renal tubules; renal collecting ducts d. Posterior pituitary; loop of Henle ANS: A Once synthesized in the hypothalamus, ADH acts on the vasopressin 2 (V2) receptors of the renal duct cells to increase their permeability. This information helps eliminate the other options. PTS: 1 REF: Pages 696-698 17. Where is oxytocin synthesized? a. Hypothalamus c. Anterior pituitary b. Paraventricular nuclei d. Posterior pituitary ANS: D ADH and oxytocin are synthesized in hypothalamic neurons but are stored and secreted by the posterior pituitary. The other options do not synthesize oxytocin. PTS: 1 REF: Page 713 18. The releasing hormones that are made in the hypothalamus travel to the anterior pituitary via the: a. Vessels of the zona fasciculata c. Hypophyseal stalk b. Infundibular stem d. Portal hypophyseal blood vessels ANS: D Releasing and inhibitory hormones are synthesized in the hypothalamus and are secreted into the portal blood vessels through which they travel to the anterior pituitary hormones. The releasing hormones do not enter the anterior pituitary gland via any of the other options. PTS: 1 REF: Pages 6N9U6-R6S9I7NGTB.COM 19. Which mineral is needed for thyroid-stimulating hormone (TSH) to stimulate the secretion of thyroid hormone (TH)? a. Iron c. Iodide b. Zinc d. Copper ANS: C TSH, which is synthesized and stored in the anterior pituitary, stimulates secretion of TH by activating intracellular processes, including the uptake of iodine necessary for the synthesis of TH. This supports the elimination of the other options. PTS: 1 REF: Page 713 20. Which hormone triggers uterine contractions? a. Thyroxine c. Growth hormone b. Oxytocin d. Insulin ANS: B Oxytocin is responsible for the contraction of the uterus and milk ejection in lactating women and may affect sperm motility in men. The remaining options are not capable of triggering uterine contractions. PTS: 1 REF: Page 699 21. What effect does hyperphosphatemia have on other electrolytes? a. Increases serum calcium. c. Decreases serum magnesium. b. Decreases serum calcium. d. Increases serum magnesium. ANS: B Hyperphosphatemia leads to hypocalcemia; the other options are incorrect. PTS: 1 REF: Page 702 22. Insulin transports which electrolyte in the cell? a. Potassium c. Sodium b. Calcium d. Phosphorus ANS: A Insulin facilitates the intracellular transport of potassium, phosphate, and magnesium. Insulin does not facilitate the transport of the other electrolytes. PTS: 1 REF: Pages 705-706 23. A person who has experienced physiologic stresses will have increased levels of which hormone? a. Adrenocorticotropic hormone c. Somatostatin b. Thyroid hormone d. Alpha endorphin ANS: A Stress has been shown to increase adrenocorticotropic hormone secretion. The other options are not increased with stress. PTS: 1 REF: Page 70N8URSINGTB.COM 24. What is the action of calcitonin? a. Increases metabolism. c. Increases serum calcium. b. Decreases metabolism. d. Decreases serum calcium. ANS: D Calcitonin, also called thyrocalcitonin, acts to lower serum calcium levels by inhibiting bone-resorbing osteoclasts, making the other options incorrect. PTS: 1 REF: Page 702 25. Which hormone is involved in the regulation of serum calcium levels? a. Parathyroid hormone (PTH) b. Thyroxine (T4) c. Adrenocorticotropic hormone (ACTH) d. Triiodothyronine (T3) ANS: A The parathyroid glands produce PTH, a regulator of serum calcium; therefore the other options are incorrect. PTS: 1 REF: Page 702 26. Which hormone inhibits the secretion of growth hormone (GH)? a. Somatostatin c. Thyroid-stimulating hormone b. Thyroxine d. Calcitonin ANS: C Two hormones from the hypothalamus control GH secretion: (1) GH-releasing hormone (GHRH), which increases GH secretion; and (2) somatostatin, which inhibits it. The other options would not produce the necessary effect. PTS: 1 REF: Page 700 27. Target cells for parathyroid hormone are located in the: a. Tubules of nephrons c. Glomeruli of nephrons b. Thyroid gland d. Smooth and skeletal muscles ANS: A Parathyroid hormone acts on its plasma membrane receptor only in the distal and proximal tubules of the kidney’s nephron. PTS: 1 REF: Page 703 28. Which compound or hormone is secreted by the adrenal medulla? a. Cortisol c. Androgens b. Epinephrine d. Aldosterone ANS: B Of the available options, the catecholamine epinephrine (adrenaline) is the only major product secreted. PTS: 1 REF: Page 71N0URSINGTB.COM 29. The secretion of adrenocorticotropic-stimulating hormone (ACTH) will result in the increased level of which hormone? a. Thyroxine c. Cortisol. b. Insulin d. Antidiuretic hormone ANS: C Psychologic and physiologic stress (e.g., hypoxia, hypoglycemia, hyperthermia, exercise) increases ACTH secretion, leading to increased cortisol levels. Only cortisol describes the appropriate feedback loop. PTS: 1 REF: Page 708 30. Which human physiologic system is believed to be an integral factor in unhealthy aging? a. Cardiac c. Central nervous b. Endocrine d. Urinary ANS: B Because the endocrine system is integral to health, changes in endocrine function have been used as biomarkers for unhealthy aging. The other systems, although contributing to the unhealthy aging, are not as integral. PTS: 1 REF: Page 711 31. What are the effects of aging on pancreatic cells? a. Pancreatic cells are replaced by fat cells. b. Increased insulin production is typical. c. Beta cell production increases. d. No appreciable change occurs on pancreatic cells. ANS: A With aging, the pancreatic cells are increasingly replaced with fat tissue. Dysfunction of the pancreas with decreased insulin secretion of the beta cells, insulin receptors, and insulin resistance have all been documented. PTS: 1 REF: Page 711 32. Aldosterone directly increases the reabsorption of: a. Magnesium c. Sodium b. Calcium d. Water ANS: C In the kidney, aldosterone primarily acts on the epithelial cells of the nephron-collecting duct to increase sodium ion reabsorption. This action cannot be said of the other options. PTS: 1 REF: Page 709 33. Which is an expected hormonal change in an older patient? a. Thyroid-stimulating hormone secretion below normal b. Triiodothyronine level below normal c. Cortisol level below normal d. Adrenocorticotropic hormoneNleUvReSl IaNbGoTvBe.CnoOrMmal ANS: A Aging causes overall thyroid-stimulating hormone secretion to diminish but does not bring about the other changes. PTS: 1 REF: Page 711 MULTIPLE RESPONSE 34. What are actions of glucocorticoids? (Select all that apply.) a. Decreasing muscle cell reuptake of glucose b. Fat storage c. Decreased blood glucose d. Carbohydrate metabolism e. Liver gluconeogenesis ANS: A, D, E The term glucocorticoid refers to those steroidal hormones that have direct effects on carbohydrate metabolism. These hormones increase blood glucose concentration by promoting gluconeogenesis in the liver and by decreasing uptake of glucose into muscle cells, adipose cells, and lymphatic cells. Glucocorticoids are not capable of fat storage. PTS: 1 REF: Pages 706-707 MATCHING Match the anatomic structure with its hormone. Names of structures may be used more than once. A. Anterior pituitary B. Posterior pituitary C. Thyroid D. Adrenal cortex E. Adrenal medulla 35. Calcitonin 36. Cortisol 37. Oxytocin 38. Glycoproteins 39. Epinephrine 35. ANS: C PTS: 1 REF: Page 702 MSC: The thyroid gland secretes calcitonin. 36. ANS: D PTS: 1 REF: Page 706 MSC: The adrenal cortex secretes several steroid hormones, including the glucocorticoids (mainly cortisol), the mineralocorticoids (mainly aldosterone), and the adrenal androgens and estrogens. 37. ANS: B PTS: 1 REF: Page 698 MSC: The posterior pituitary secretes two polypeptide hormones: (1) ADH, also called arginine vasopressin, and (2) oxytocin. 38. ANS: A PTS: 1 REF: Pages 699-700 MSC: The anterior pituitary releasesNsUixRmSIaNjoGrTsBti.mCOulMatory hormones. They can be grouped into three categories: (1) corticotropin-related hormones (ACTH, B-lipoprotein, melanocyte-stimulating hormone [MSH], and related endorphins), (2) glycoproteins (LH, FSH, and TSH), and (3) somatomammotropins (GH and prolactin). 39. ANS: E PTS: 1 REF: Page 710 MSC: The major products secreted by the adrenal medulla are the catecholamines epinephrine (adrenaline) and norepinephrine. Chapter 22: Alterations of Hormonal Regulation MULTIPLE CHOICE 1. The effects of the syndrome of inappropriate antidiuretic hormone (SIADH) secretion include solute: a. Retention and water retention c. Dilution and water retention b. Retention and water loss d. Dilution and water loss ANS: C The symptoms of SIADH secretion are a result of dilutional hyponatremia and water retention. This information supports the elimination of the other options. PTS: 1 REF: Page 719 2. The common cause of elevated levels of antidiuretic hormone (ADH) secretion is: a. Ectopically produced ADH c. Posterior pituitary tumor b. Inflammation of the hypothalamus d. Inflammation of the nephrons ANS: A A common cause of elevated levels of ADH secretion is ectopically produced ADH, which makes the other options incorrect. PTS: 1 REF: Page 718 3. Which laboratory value would the nurse expect to find if a person is experiencing syndrome of inappropriate antidiuNreUtRicShINoGrmTBo.nCeO(MSIADH)? a. Hypernatremia and urine hypoosmolality b. Serum potassium (K+) level of 5 mEq/L and urine hyperosmolality c. Serum sodium (Na+) level of 120 mEq/L and serum hypoosmolality d. Hypokalemia and serum hyperosmolality ANS: C A diagnosis of SIADH requires a serum sodium level of less than 135 mEq/L, serum hypoosmolality less than 280 mOsm/kg, and urine hyperosmolarity. Potassium levels are not considered a factor. PTS: 1 REF: Page 719 4. Diabetes insipidus is a result of: a. Antidiuretic hormone hyposecretion c. Insulin hyposecretion b. Antidiuretic hormone hypersecretion d. Insulin hypersecretion ANS: A Of the available options, diabetes insipidus is a result of insufficient antidiuretic hormone. PTS: 1 REF: Page 719 5. A patient who is diagnosed with a closed head injury has a urine output of 6 to 8 L/day. Electrolytes are within normal limits, but his antidiuretic hormone (ADH) level is low. Although he has had no intake for 4 hours, no change in his polyuria level has occurred. These symptoms support a diagnosis of: a. Neurogenic diabetes insipidus b. Syndrome of inappropriate antidiuretic hormone c. Psychogenic polydipsia d. Osmotically induced diuresis ANS: A The stated symptoms are reflective of neurogenic diabetes insipidus and not of the remaining options. PTS: 1 REF: Pages 719-720 6. Diabetes insipidus, diabetes mellitus, and syndrome of inappropriate antidiuretic hormone all exhibit which symptom? a. Polyuria c. Vomiting b. Edema d. Thirst ANS: D Thirst is the only symptom common to all these conditions. PTS: 1 REF: Pages 719-720 | Pages 737-738 7. The cause of neurogenic diabetes insipidus (DI) is related to an organic lesion of the: a. Anterior pituitary c. Posterior pituitary b. Thalamus ANS: C NURSINGTdB..CORMenal tubules Neurogenic DI is a result of dysfunctional antidiuretic hormone synthesis, caused by a lesion of the posterior pituitary, hypothalamus, or pituitary stalk. PTS: 1 REF: Page 719 8. Which form of diabetic insipidus (DI) will result if the target cells for antidiuretic hormone (ADH) in the renal collecting tubules demonstrate insensitivity? a. Neurogenic c. Psychogenic b. Nephrogenic d. Ischemic ANS: B Only nephrogenic DI is associated with an insensitivity of the renal collecting tubules to ADH. PTS: 1 REF: Page 720 9. Which laboratory value is consistently low in a patient with diabetes insipidus (DI)? a. Urine-specific gravity c. Urine protein b. Serum sodium d. Serum total protein ANS: A The basic criteria for diagnosing DI include a low urine-specific gravity while sodium levels are high. Protein levels are not considered. PTS: 1 REF: Page 720 10. Which form of diabetes insipidus (DI) is treatable with exogenous antidiuretic hormone (ADH)? a. Neurogenic c. Nephrogenic b. Psychogenic d. Ischemic ANS: A Neurogenic DI is treated with ADH replacement therapy. The other options are incorrect. PTS: 1 REF: Page 720 11. Which condition may result from pressure exerted by a pituitary tumor? a. Hypothyroidism c. Diabetes insipidus b. Hypercortisolism d. Insulin hyposecretion ANS: A If the tumor exerts sufficient pressure, then thyroid and adrenal hypofunction may occur because of lack of thyroid-stimulating hormone (TSH) and adrenocorticotropic hormone (ACTH). These result in the symptoms of hypothyroidism and hypocortisolism. The remaining options are not associated with the pressure exerted by a pituitary tumor. PTS: 1 REF: Page 722 12. The term used to describe a person who experiences a lack of all hormones associated with the anterior pituitary is: a. Panhypopituitarism b. Adrenocorticotropic hormone deficiency ANS: A NURSINGTcB..COHMypopituitarism d. Anterior pituitary failure Panhypopituitarism is the only available term that is correctly associated with the lack of all anterior pituitary hormones. PTS: 1 REF: Page 721 13. Visual disturbances are a result of a pituitary adenoma because of the: a. Liberation of anterior pituitary hormones into the optic chiasm b. Pituitary hormones clouding the lens of the eyes c. Pressure of the tumor on the optic chiasm d. Pressure of the tumor on the optic and oculomotor cranial nerves ANS: C Of the available options, pressure on the optic chiasm is the only cause for visual disturbances resulting from a pituitary adenoma. PTS: 1 REF: Page 722 14. Which disorder is considered a co-morbid condition of acromegaly? a. Hypotension c. Brain cancer b. Diabetes d. Thyroid cancer ANS: B Symptoms of type 2 diabetes mellitus, such as polyuria and polydipsia, may occur. Acromegaly-associated hypertension is usually asymptomatic until symptoms of heart failure develop. Neither thyroid nor brain cancer has been associated with acromegaly. PTS: 1 REF: Page 724 15. Which disorder is caused by hypersecretion of the growth hormone (GH) in adults? a. Cushing syndrome c. Giantism b. Acromegaly d. Myxedema ANS: B Acromegaly is a term for adults who have been exposed to continuously high levels of GH, whereas the term giantism is reserved for children and adolescents. The other options do not apply to hypersecretion of GH. PTS: 1 REF: Page 722 16. Giantism occurs only in children and adolescents because their: a. Growth hormones are still diminished. b. Epiphyseal plates have not yet closed. c. Skeletal muscles are not yet fully developed. d. Metabolic rates are higher than in adulthood. ANS: B Giantism is related to the effects of growth hormones on the growth of long bones at their epiphyseal plates. This information makes the other options incorrect. NURSINGTB.COM PTS: 1 REF: Page 722 17. Amenorrhea, galactorrhea, hirsutism, and osteoporosis are each caused by a: a. Posterior pituitary adenoma c. Prolactinoma b. Thymoma d. Growth hormone adenoma ANS: C Of the options available, the hallmark of a prolactinoma is the sustained elevation of serum prolactin that is responsible for the symptoms listed in the question. PTS: 1 REF: Page 724 18. Graves disease develops from a(n): a. Viral infection of the thyroid gland that causes overproduction of thyroid hormone b. Autoimmune process during which lymphocytes and fibrous tissue replace thyroid tissue c. Thyroid-stimulating immunoglobulin that causes overproduction of thyroid hormones d. Ingestion of goitrogens that inhibits the synthesis of the thyroid hormones, causing a goiter ANS: C The pathologic features of Graves disease indicates that normal regulatory mechanisms are overridden by abnormal immunologic mechanisms that result in the stimulation of excessive TH. The remaining options are incorrect statements. PTS: 1 REF: Page 726 19. The signs of thyrotoxic crisis include: a. Constipation with gastric distention c. Hyperthermia and tachycardia b. Bradycardia and bradypnea d. Constipation and lethargy ANS: C The systemic symptoms of thyrotoxic crisis include hyperthermia and tachycardia. The remaining options are not associated with this disorder. PTS: 1 REF: Page 728 20. Pathologic changes associated with Graves disease include: a. High levels of circulating thyroid-stimulating immunoglobulins b. Diminished levels of thyrotropin-releasing hormone c. High levels of thyroid-stimulating hormone d. Diminished levels of thyroid-binding globulin ANS: A The only option that correctly describes the changes associated with Graves disease identifies high levels of circulating thyroid-stimulating immunoglobulins that are found in more than 95% of individuals diagnosed with the disease. PTS: 1 REF: Page 72N6URSINGTB.COM 21. The level of thyroid-stimulating hormone (TSH) in individuals with Graves disease is usually: a. High c. Normal b. Low d. In constant flux ANS: B The hyperfunction of the thyroid gland leads to suppression of TSH because of the normal negative feedback mechanism, thus eliminating the other options as being correct. PTS: 1 REF: Page 726 22. Palpation of the neck of a person diagnosed with Graves disease would detect a thyroid that is: a. Left of midline c. Normal in size b. Small with discrete nodules d. Diffusely enlarged ANS: D The only option that characterizes Graves disease is a diffused enlargement of the thyroid gland. PTS: 1 REF: Page 760 23. A deficiency of which chemical may result in hypothyroidism? a. Iron c. Zinc b. Iodine d. Magnesium ANS: B The only cause of hypothyroidism from among the provided options is a deficiency of endemic iodine. PTS: 1 REF: Page 728 24. What are clinical manifestations of hypothyroidism? a. Intolerance to heat, tachycardia, and weight loss b. Oligomenorrhea, fatigue, and warm skin c. Restlessness, increased appetite, and metrorrhagia d. Constipation, decreased heat rate, and lethargy ANS: D The lower levels of thyroid hormone result in decreased energy metabolism, resulting in constipation, bradycardia, and lethargy, thus eliminating the remaining options. PTS: 1 REF: Pages 728-730 | Table 22-3 25. Diagnosing a thyroid carcinoma is best performed with: a. Measurement of serum thyroid levels c. Ultrasonography b. Radioisotope scanning d. Fine-needle aspiration biopsy ANS: D Fine-needle aspiration of a thyroid nodule is generally performed to diagnose this condition; this method is best for early detection, thus eliminating the other options. NURSINGTB.COM PTS: 1 REF: Page 731 26. Renal failure is the most common cause of which type of hyperparathyroidism? a. Primary c. Exogenous b. Secondary d. Inflammatory ANS: B Chronic renal failure is the most common cause of secondary hyperparathyroidism because of the resulting hyperphosphatemia that stimulates parathyroid hormone secretion. Although the other options may occur, they are not the most common types of the disorder. PTS: 1 REF: Page 732 27. The most common cause of hypoparathyroidism is: a. Pituitary hyposecretion c. Parathyroid gland damage b. Parathyroid adenoma d. Autoimmune parathyroid disease ANS: C The most common cause of hypoparathyroidism is damage caused during thyroid surgery, thus eliminating the other options as being correct. PTS: 1 REF: Page 733 28. The most probable cause of low serum calcium after a thyroidectomy is: a. Hyperparathyroidism, secondary to Graves disease b. Myxedema, secondary to surgery c. Hypoparathyroidism caused by surgical injury d. Hypothyroidism caused by the lack of thyroid replacement ANS: C The most common cause of hypoparathyroidism is damage caused during thyroid surgery, resulting in a lack of circulating PTH and causing a depressed level of serum calcium. This information supports the elimination of the other options. PTS: 1 REF: Page 733 29. A patient diagnosed with diabetic ketoacidosis (DKA) has the following laboratory values: arterial pH 7.20; serum glucose 500 mg/dl; positive urine glucose and ketones; serum potassium (K+) 2 mEq/L; serum sodium (Na+) 130 mEq/L. The patient reports that he has been sick with the “flu” for 1 week. What relationship do these values have to his insulin deficiency? a. Increased glucose use causes the shift of fluid from the intravascular to the intracellular space. b. Decreased glucose use causes fatty acid use, ketogenesis, metabolic acidosis, and osmotic diuresis. c. Increased glucose and fatty acids stimulate renal diuresis, electrolyte loss, and metabolic alkalosis. d. Decreased glucose use results in protein catabolism, tissue wasting, respiratory acidosis, and electrolyte loss. ANS: B NURSINGTB.COM Decreased glucose causes fatty acid use, ketogenesis, metabolic acidosis, and osmotic diuresis, which have resulted in the symptoms listed in the question. The relationship between the stated assessment values and insulin deficiency is not effectively described by any of the other options. PTS: 1 REF: Pages 744-745 30. Polyuria occurs with diabetes mellitus because of the: a. Formation of ketones c. Elevation in serum glucose b. Chronic insulin resistance d. Increase in antidiuretic hormone ANS: C Glucose accumulates in the blood and appears in the urine as the renal threshold for glucose is exceeded, producing an osmotic diuresis and the symptoms of polyuria and thirst. None of the other options appropriately describes the pathologic features of diabetes mellitus–induced polyuria. PTS: 1 REF: Page 738 31. Type 2 diabetes mellitus is best described as a(an): a. Resistance to insulin by insulin-sensitive tissues b. Need for lispro instead of regular insulin c. Increase of glucagon secretion from  cells of the pancreas d. Presence of insulin autoantibodies that destroy  cells in the pancreas ANS: A One of the basic pathophysiologic characteristics of type 2 diabetes is the development of insulin-resistant tissue cells. None of the remaining options appropriately describes type 2 diabetes. PTS: 1 REF: Page 739 32. A person diagnosed with type 1 diabetes experiences hunger, lightheadedness, tachycardia, pallor, headache, and confusion. The most probable cause of these symptoms is: a. Hyperglycemia caused by incorrect insulin administration b. Dawn phenomenon from eating a snack before bedtime c. Hypoglycemia caused by increased exercise d. Somogyi effect from insulin sensitivity ANS: C The most likely cause of these symptoms is hypoglycemia, which is often caused by a lack of systemic glucose as a result of muscular activity. None of the remaining options appropriately describes why a person diagnosed with type 1 diabetes experiences the described symptoms. PTS: 1 REF: Page 744 | Table 22-10 33. Which serum glucose level would indicate hypoglycemia in a newborn? a. 28 mg/dl c. 60 mg/dl b. 40 mg/dl d. 80 mg/dl NURSINGTB.COM ANS: A Serum glucose <30 mg/dl in newborn (first 2 to 3 days) and <55 to 60 mg/dl in adults is associated with hypoglycemia. PTS: 1 REF: Page 744 | Table 22-10 34. When comparing the clinical manifestations of both diabetic ketoacidosis (DKA) and hyperglycemic hyperosmolar nonketotic syndrome (HHNKS), which condition is associated with only DKA? a. Fluid loss c. Increased serum glucose b. Weight loss d. Kussmaul respirations ANS: D Kussmaul respirations are only observed in those with DKA. PTS: 1 REF: Pages 745-746 35. Hypoglycemia, followed by rebound hyperglycemia, is observed in those with: a. The Somogyi effect b. The dawn phenomenon c. Diabetic ketoacidosis d. Hyperosmolar hyperglycemic nonketotic syndrome ANS: A Hypoglycemia, followed by rebound hyperglycemia, is observed only in the Somogyi effect. PTS: 1 REF: Page 746 36. The first laboratory test that indicates type 1 diabetes is causing the development of diabetic nephropathy is: a. Dipstick test for urine ketones b. Increase in serum creatinine and blood urea nitrogen c. Protein on urinalysis d. Cloudy urine on the urinalysis ANS: C Microalbuminuria is the first manifestation of this form of renal failure. Although the other options may develop, they occur after protein is found in the urine. PTS: 1 REF: Pages 748-749 37. Which classification of oral hypoglycemic drugs decreases hepatic glucose production and increases insulin sensitivity and peripheral glucose uptake? a. Biguanide (metformin) c. Meglitinides (glinides) b. Sulfonylureas (glyburide) d. -Glycosidase inhibitor (miglitol) ANS: A Only biguanides decrease hepatic glucose production and increase insulin sensitivity and peripheral glucose uptake. PTS: 1 REF: Page 74N3U|RTSaIbNleG2T2B-.9COM 38. What causes the microvascular complications in patients with diabetes mellitus? a. The capillaries contain plaques of lipids that obstruct blood flow. b. Pressure in capillaries increase as a result of the elevated glucose attracting water. c. The capillary basement membranes thicken, and cell hyperplasia develops. d. Fibrous plaques form from the proliferation of subendothelial smooth muscle of arteries. ANS: C Microvascular complications are a result of capillary basement membranes thickening and endothelial cell hyperplasia. None of the remaining options appropriately describes the cause of microvascular complications in patients with diabetes mellitus. PTS: 1 REF: Page 747 39. Retinopathy develops in patients with diabetes mellitus because: a. Plaques of lipids develop in the retinal vessels. b. Pressure in the retinal vessels increase as a result of increased osmotic pressure. c. Ketones cause microaneurysms in the retinal vessels. d. Retinal ischemia and red blood cell aggregation occur. ANS: D Retinopathy appears to be a response to retinal ischemia and red blood cell aggregation. None of the remaining options appropriately describes the relationship between retinopathy and diabetes mellitus. PTS: 1 REF: Page 747 40. A person has acne, easy bruising, thin extremities, and truncal obesity. These clinical manifestations are indicative of which endocrine disorder? a. Hyperthyroidism c. Diabetes insipidus b. Hypoaldosteronism d. Cushing disease ANS: D These symptoms are characteristic of Cushing disease and are caused by excessive ACTH secretion. The symptoms described are not characteristic of any of the other options. PTS: 1 REF: Pages 753-755 MULTIPLE RESPONSE 41. A person may experience which complications as a result of a reduction in parathyroid hormone (PTH)? (Select all that apply.) a. Muscle spasms b. Tonic-clonic seizures c. Laryngeal spasms d. Hyporeflexia e. Asphyxiation ANS: A, B, C, E NURSINGTB.COM Symptoms associated with hypoparathyroidism are related to hypocalcemia. Hypocalcemia causes a lowering of the threshold for nerve and muscle excitation so that a slight stimulus anywhere along the length of a nerve or muscle fiber may initiate a nerve impulse. This creates tetany manifested as muscle spasms, hyperreflexia, tonic-clonic convulsions, laryngeal spasms, and, in severe cases, death from asphyxiation. PTS: 1 REF: Pages 733-734 42. A chronic complication of diabetes mellitus is likely to result in microvascular complications in which areas? (Select all that apply.) a. Eyes b. Coronary arteries c. Renal system d. Peripheral vascular system e. Nerves ANS: A, C, E Of the options provided, the areas most often affected are the retina, kidneys, and nerves. PTS: 1 REF: Page 747 MATCHING Match the phrases with the corresponding terms. A. Acromegaly B. Cushing disease C. Addison disease D. Graves disease E. Myxedema F. Pheochromocytoma 43. Hypersecretion of thyroid hormone (TH) 44. Hypersecretion of adrenocorticotropic hormone (ACTH) 45. Hypersecretion of adrenal medulla hormones 46. Hyposecretion of thyroid hormone (TH) 47. Hyposecretion of adrenal cortex hormones 48. Hypersecretion of growth hormone (GH) 43. ANS: D PTS: 1 REF: Page 726 MSC: Graves disease is caused by the hypersecretion of TH. 44. ANS: MSC: B PTS: 1 REF: Page 753 Cushing disease is caused by the hypersecretion of ACTH. 45. ANS: MSC: F PTS: 1 REF: Page 758 Pheochromocytoma is a tumor that causes hypersecretion of adrenal medulla hormones. 46. ANS: MSC: E PTS: 1 REF: Page 729 Myxedema is the long-standing hyposecretion of TH. 47. ANS: MSC: C PTS: 1 REF: Page 757 Addison disease is a result of hyposecretion of adrenal cortex hormones. 48. ANS: MSC: A PTS: 1 REF: Pages 722-723 Acromegaly is the condition aNssUoRcSiaItNeGd TwBit.Ch OthMe exposure of adults to high levels of GH. Chapter 23: Obesity and Disorders of Nutrition MULTIPLE CHOICE 1. If a patient with multiple sclerosis starts coughing frequently during meals and starts to eat significantly less food than normal, the patient may have a.pneumonia. b.gastroesophageal reflux disease. c.peptic ulcer. d.dysphagia. ANS: D Patients with multiple sclerosis may develop dysphagia; coughing during meals and eating less than usual are common symptoms. Dysphagia may cause pneumonia if food enters the lungs and causes infection. Gastroesophageal reflux disease causes burning and pain but typically not coughing. Peptic ulcer causes intestinal pain but not coughing. 2. For a patient with dysphagia, the food that would be most difficult to swallow is a.applesauce. b.mashed potatoes. c.chocolate pudding. d.chicken noodle soup. ANS: D NURSINGTB.COM Patients with dysphagia often have difficulty swallowing thin liquids, such as chicken noodle soup. Thicker, pureed foods such as applesauce, mashed potatoes, and chocolate pudding are usually easier to swallow. 3. If a patient has difficulty swallowing, the best position for eating is a.propped up in bed with a caregiver by the bedside. b.sitting upright opposite a caregiver. c.sitting at a dining table with social dining companions. d.lying on the left side, with a caregiver by the side. ANS: B The safest eating position for someone who has trouble swallowing is sitting upright; the caregiver should sit opposite to help provide cues and reminders while the patient is eat- ing. It is usually best for patients with dysphagia not to interact socially during mealtimes because talking may make swallowing more difficult. Sitting propped up in bed and lean- ing back would make it easier for food to enter the airway. Lying on ones side while eating is very likely to cause aspiration. 4. The most acute risk for patients with dysphagia is a.constipation. b.dehydration. c.dry mouth. d.panic attacks. ANS: B An acute daily concern for patients with dysphagia is dehydration. Patients are not usually allowed to drink fluids without supervision, and so fluid intake must be monitored throughout the day. Constipation and dry mouth may occur with poor fluid and food intake but are less acute. Patients with dysphagia may experience feelings of panic, but this is not the most acute concern. 5. The most helpful recommendation for a client who often experiences heartburn at night would be to a.avoid eating within 4 hours of going to bed. b.increase the fiber content of the diet. c.decrease the fiber content of the diet. d.take antacid medications before going to bed. ANS: A Gastroesophageal reflux disease (GERD) usually occurs 1 to 4 hours after eating; there- fore, waiting at least 4 hours after eating before going to bed would help prevent heartburn at night. Antacid medications mayNbUeRhSIeNlpGfTuBl .bCuOtMshould not be used on a regular basis be- fore other strategies have been tried. Fiber content of the diet does not affect heartburn. 6. An example of a meal that is likely to relax the lower esophageal sphincter and allow gastroesophageal reflux is a.pasta with marinara sauce and sourdough bread. b.ham with rice pilaf. c.fried chicken and pasta salad. d.chicken and spinach tortilla wrap and salsa. ANS: C High-fat foods and beverages relax the lower esophageal sphincter (allowing stomach contents to back up). Both fried chicken and pasta salad are high in fat and would exacer- bate gastroesophageal reflux. The other meals are relatively low in fat and would be much less likely to cause reflux. 7. Peptic ulcer disease (PUD) would be most like to occur in someone who uses medica- tion to treat a.gastroesophageal reflux disease. b.constipation. c.celiac disease. d.joint pain or arthritis. ANS: D Risk of PUD increases with chronic use of nonsteroidal anti-inflammatory drugs (NSAIDs), often used to treat joint pain or arthritis. Use of antacids to treat GERD would decrease risk of PUD. Laxatives and stool softeners used to treat constipation do not affect risk of PUD. Celiac disease is treated by avoiding dietary sources of gluten, not with med- ications. 8. Nutrition therapy for peptic ulcers should be individualized, depending on a.type of drug treatment. b.location of the ulcer. c.patient tolerance. d.the cause of the ulcer. ANS: C There are no specific dietary recommendations for treatment of peptic ulcer, and so foods eaten depend on the patients symptoms and tolerance. The type of drug treatment and lo- cation of the ulcer do not significantly affect food tolerance. Factors that contribute to formation of peptic ulcers are mostly nonnutritional (stress, skipping meals, chronic use of use of NSAIDs), and so dietary intake is not a significant factor in treatment. 9. If a patient who has undergone NgUasRtrSiIcNbGyTpBa.CssOsMurgery for treatment of extreme obesity experiences sudden sweating, nausea, and stomach cramps after meals, the patient may have a.gastroesophageal reflux. b.pancreatitis. c.peptic ulcer disease. d.dumping syndrome. ANS: D Patients who have undergone gastric bypass may experience dumping syndrome as a large volume of hyperosmolar food is dumped into the small intestine because the stomach res- ervoir is missing or too small. GERD and peptic ulcers are not necessarily more common after gastrectomy. Pancreatitis causes extreme pain and vomiting but is not related to gas- tric bypass. 10. An appropriate meal for someone with dumping syndrome would be a.a small vanilla milkshake. b.cream of mushroom soup. c.half a turkey sandwich. d.popcorn and sugar-free soda. ANS: C Individual tolerances vary, but the turkey half-sandwich would probably be tolerated best. Half a sandwich is probably small enough not to cause symptoms. A milkshake is high in sugar and probably too cold. Cream of mushroom soup is probably too hot, and liquids should be taken between rather than with meals. The carbonation in the soda may increase gastrointestinal distention because of gas. 11. If a patient has esophagitis and also has worn tooth enamel, he or she may have a.a hiatal hernia. b.fluoride deficiency. c.an eating disorder. d.gastroesophageal reflux disease. ANS: C The eating disorder bulimia nervosa causes esophagitis and eroded tooth enamel because of the effects of stomach acid being vomited back into the esophagus and mouth. A hiatal hernia and GERD may cause esophagitis, but stomach contents would enter only the esophagus, not the mouth, and so the teeth would not be worn. Fluoride deficiency may cause loss of tooth enamel but would not cause esophagitis. 12. An example of a meal that may contain gluten is a.baked chicken breast with herbeNdUrRicSeI.NGTB.COM b.pork chop with sweet potatoes and butter. c.grilled steak with baked potato and sour cream. d.roast beef with mashed potatoes and gravy. ANS: D Gravy is often made with wheat flour, which contains gluten. Rice, potatoes, and sweet potatoes do not contain gluten. Baked chicken, pork chops, and grilled steak do not contain gluten unless they are breaded or coated with wheat flour before being cooked. 13. A patient with lactose intolerance would be most likely to experience symptoms if he or she ate a a.cup of pudding. b.turkey sandwich with Swiss cheese. c.biscuit with butter and honey. d.cup of sherbet. ANS: A A cup of pudding would contain a cup of milk; most individuals with lactose intolerance can only tolerate to cup milk at one time. Sherbet, Swiss cheese, biscuits, and butter all have low levels of lactose. 14. Milk may be made suitable for patients with lactose intolerance by treating it with tab- lets that contain a.yogurt. b.lactase enzyme. c.vitamin D and calcium. d.Lactobacillus acidophilus. ANS: B Lactase enzyme tables may be added to milk 24 hours before it is drunk to digest the lac- tose; this makes the milk suitable for patients with lactose intolerance. Lactobacillus aci- dophilus and yogurt are not necessarily better tolerated than regular milk. Vitamin D and calcium do not affect lactose intolerance, although patients with lactose intolerance may benefit from supplements of these nutrients if their intake is low because they avoid milk. 15. For patients with inflammatory bowel disease, nutrition therapy is a.the primary mode of treatment. b.rarely necessary. c.an important adjunct to drugs and surgery. d.highly specialized. ANS: C NURSINGTB.COM Nutrition therapy is important to help maintain nutritional status of patients with inflam- matory bowel disease, but is used in conjunction with drug and surgical treatments; it is not the primary mode of treatment. Nutrition therapy is often needed to maintain weight and micronutrient status. Awareness of deficiencies that occur in these patients is im- portant, but the RD does not need specialized knowledge to work with patients with IBD. 16. Because patients with active Crohns disease have intestinal inflammation, they benefit from high intake of a.fat. b.dietary fiber. c.simple sugars. d.protein. ANS: D Patients with inflammatory bowel disease benefit from a diet high in protein to compen- sate for malabsorption. Fat intake should be moderate to provide adequate kilocalories within a healthful diet; high fat intake may cause steatorrhea. Dietary fiber may irritate the inflamed gut. Simple sugars provide a readily digested and absorbed source of energy but no other nutrients. 17. Patients with inflammatory bowel disease may benefit from a high-fiber diet during a.acute episodes. b.times of remission. c.recovery from surgery. d.preparation for surgery. ANS: B A high-fiber diet may help stimulate peristalsis during times of remission in patients with inflammatory bowel disease. During acute episodes, the diet should be low in fiber and should promote bowel rest. In general, the diet should be high calorie, high protein, and nutrient dense to compensate for malabsorption. 18. Fluid loss is most likely to be a problem for a patient with a.a colostomy. b.a hiatal hernia. c.an ileostomy. d.Crohns disease. ANS: C NURSINGTB.COM Fluid loss is most likely to be a problem for a patient with an ileostomy because food exits the intestines before water is absorbed in the colon. A colostomy may cause some fluid loss, depending on where the colostomy is located and how much of the colon the intesti- nal contents pass through for fluid absorption. A hiatal hernia does not cause fluid loss. Crohns disease may cause some fluid loss during acute periods of inflammation because of malabsorption. 19. The most significant nutritional concern for a patient with an ileostomy is a.fat malabsorption and steatorrhea. b.dumping syndrome. c.loss of fluid and electrolytes. d.malabsorption of protein and carbohydrates. ANS: C For a patient with an ileostomy, digestive output exits the body instead of passing through the colon, the major site of absorption of water and electrolytes. Digestion and absorption of protein, carbohydrate, and fat are not usually affected. Dumping syndrome is caused by a partial or total gastrectomy. 20. For patients with ostomies, a lower amount of output occurs a. if the ostomy is closer to the rectum. b. if the ostomy is closer to the rectum. c.with high fiber intake. d.with high fat intake. ANS: A As effluent progresses through the colon, water is absorbed, and the effluent becomes more solid; therefore, an ostomy closer to the rectum would have a smaller volume. Fiber intake may increase output slightly, but less so than the site of the ostomy; fat intake does not significantly affect the amount of output. 21. If a patient has had a large portion of their small intestine removed because of cancer and is unable to maintain a stable weight, a feeding regimen that would be most appropri- ate is a.enteral feedings into the stomach. b.enteral feedings into the jejunum. c.parenteral nutrition only. d.supplemental parenteral nutrition. ANS: D Patients who have undergone removal of large portions of the small intestine and are una- ble to maintain their weight should receive supplemental parenteral nutrition. Enteral feedings would not be more effectNivUeRtShIaNnGoTrBa.lCfOeMedings because these patients have a lim- ited amount of gut to absorb the nutrients. Parenteral nutrition should not be used alone because some oral feeding helps stimulate recovery of gut function and maintain gut integ- rity. 22. If a patient receiving chemotherapy is struggling with nausea and vomiting, helpful suggestions may include a.drinking acidic fruit juices to stimulate gastric juices. b.being active before and after meals to improve appetite. c.eating frequent small meals throughout the day. d.eating foods with plenty of flavor and aroma. ANS: C Patients receiving chemotherapy who have nausea and vomiting are most likely to main- tain adequate nutritional intake if they eat several small meals throughout the day. Rest before and after meals is more likely than activity to help control nausea. Acidic fruit juic- es may increase feelings of nausea. Bland foods are likely to be better tolerated than fla- vorful and aromatic foods. 23. The best way to prevent formation of diverticula in the colon is to a.eat plenty of fruits, vegetables, and whole grains. b.avoid foods with rough hulls, such as nuts and seeds. c.use stool softeners to promote daily bowel movements. d.avoid foods that cause gas and increase pressure in the colon. ANS: A Diverticula are caused by high pressure in the colon, often in association with straining to have a bowel movement; they can be prevented by intake of high-fiber foods, such as fruits, vegetables, and whole grains. There is no evidence that nuts, seeds, or other foods with hulls increase risk of diverticula. Using stool softeners to promote daily bowel movements may help prevent diverticula, but eating a high-fiber diet is preferable because of other nutritional benefits. Foods that cause gas are not associated with formation of di- verticula. 24. Someone who may benefit from avoiding sugar-free chewing gum is a patient with a.peptic ulcer disease. b.gastroesophageal reflux disease. c. irritable bowel syndrome. d.celiac disease. ANS: C Sugar-free chewing gum is usually sweetened with sorbitol or mannitol, which may exac- erbate irritable bowel syndrome. PNUUDRS, IGNEGRTBD.C, aOnMd celiac disease are not generally affect- ed by chewing gum or intake of sugar alcohols. 25. In consuming a high-fiber diet, it is important to also consume adequate amounts of a.fluid. b.calcium and iron. c.fat-soluble vitamins. d.complex carbohydrates. ANS: A It is important to consume adequate fluid along with a high-fiber diet to prevent fecal im- paction. High-fiber diets generally have high levels of complex carbohydrates. Slightly more calcium and iron may be needed because fiber may decrease the bioavailability of these nutrients. However, diets high in fiber tend to have higher amounts of nutrients in general. Requirements for fat-soluble vitamins are not generally affected by fiber intake. 26. If a client complains of excessive gas, a good question to ask is whether he or she a.drinks coffee and tea. b.has problems with constipation. c.chews gum. d. is under a lot of stress. ANS: C Excessive gas may be caused by swallowing air while chewing gum. Drinking coffee and tea, stress, and constipation do not usually cause excess gas formation. 27. An example of a daily change that may help decrease constipation is a.drinking tomato juice instead of carbonated beverages with lunch. b.eating oatmeal and raisins instead of a bagel for breakfast. c.snacking on pretzels instead of potato chips. d.eating mashed potatoes instead of pasta with dinner. ANS: B Increasing intake of dietary fiber helps prevent constipation; oatmeal and raisins have sig- nificantly higher levels of fiber than does a bagel. In fruit and vegetable juices, most of the fiber is removed during the juicing process; drinking vegetable juice would add nutrients but not fiber. Pretzels have less fat than potato chips, but both have low amounts of fiber. Mashed potatoes contain only slightly more fiber than does pasta. 28. An example of a high-fiber food item to include with lunch is a.pasta with tomato sauce and Parmesan cheese. b.beef tacos with lettuce and tomaNtoUeRsS. INGTB.COM c.an omelet with mushrooms and onions. d.black bean soup. ANS: D Beans such as black beans have especially high amounts of fiber. Tomatoes, lettuce, mushrooms, onions, and white pasta have lower amounts of dietary fiber. Cheese, beef, and eggs contain no dietary fiber. 29. A possible cause of acute diarrhea is a.short bowel syndrome. b.lactose intolerance. c.emotional stress. d.celiac disease. ANS: C Acute diarrhea may be caused by enteritis, medications, dietary changes, or emotional stress. Short bowel syndrome, lactose intolerance, and celiac disease may cause chronic diarrhea by causing various types of malabsorption. 30. Treatment of diarrhea generally begins with a.a high-fiber, low-fat diet. b.removal of the cause of diarrhea. c.adequate fluids to hydrate the patient. d.a low-fat, low-fiber, or low-lactose diet. ANS: C The primary concern in treatment of diarrhea is provision of adequate fluids to hydrate the patient. After taking fluids, affected patients may progress to a low-fat, low-fiber, or low-lactose diet and then to a regular diet as tolerated. If possible, the cause of diarrhea may be removed, but this is not always possible and is not the first priority. A high-fiber, low-fat diet may be beneficial once the diarrhea has resolved. NURSINGTB.COM Chapter 24: Structure and Function of the Reproductive Systems MULTIPLE CHOICE 1. The initial reproductive structures of the male and female embryos appear the same until which week of gestation? a. Third c. Twentieth b. Eighth d. Thirtieth ANS: B Between 6 and 7 weeks’ gestation, the male embryo differentiates under the influence of testes-determining factor (TDF). In the absence of testosterone, a loss of the wolffian system occurs and the two gonads develop into ovaries at 6 to 8 weeks’ gestation. PTS: 1 REF: Page 769 2. The absence of which major hormone is a determinant of sexual differentiation (wolffian system) in utero? a. Estrogen c. Growth hormone b. Progesterone d. Testosterone ANS: D In the absence of testosterone, a loss of the wolffian system occurs and the two gonads develop into ovaries at 6 to 8 weeks’ gestation. Between 6 and 7 weeks’ gestation, the male embryo differentiates under the influence of TDF. The presence of estrogen is a determinating factor. None of the other options function as a major determinant to sexual differentiation in utero. NURSINGTB.COM PTS: 1 REF: Page 769 3. Which gland produces the associated hormones that are found in high levels in a female fetus? a. Posterior pituitary excretes gonadotropin-releasing hormone (GnRH) and luteinizing hormone (LH). b. Hypothalamus excretes luteinizing hormone (LH) and gonadotropin-releasing hormone (GnRH). c. Anterior pituitaryfollicle-stimulating hormone (FSH) and luteinizing hormone (LH). d. Hypothalamus excretes gonadotropin-releasing hormone (GnRH) and follicle-stimulating hormone (FSH). ANS: C In the female fetus, the anterior pituitary excretes high levels of two gonadotropins—FSH and LH. The other sequences are not correct. PTS: 1 REF: Pages 769-770 4. Which hormone is linked to an increase in appetite during puberty? a. Inhibin c. Activin b. Leptin d. Follistatin ANS: B Sensitivity to leptin, which regulates appetite and energy metabolism, increases during puberty; in theory, the adolescent consumes more calories to meet the caloric needs of the pubertal growth spurt. The percent of body fat and leptin levels in girls continue to increase, whereas muscle mass increases in boys. No apparent link exists between increased appetite during puberty and any of the other options. PTS: 1 REF: Page 778 5. The Skene glands are located on either side of which structure? a. Introitus c. Clitoris b. Urinary meatus d. Vestibule ANS: B The ducts of the Skene glands (also called the lesser vestibular or paraurethral glands) are related only to the urinary meatus. PTS: 1 REF: Page 772 6. What is the function of the mucus secreted by the Bartholin glands? a. Enhancement of the motility of sperm b. Lubrication of the urinary meatus and vestibule c. Maintenance of an acid-base balance to discourage proliferation of pathogenic bacteria d. Enhancement of the size of the penis during intercourse ANS: A In response to sexual stimulation,NthUeRBSIaNrGthToBli.nCOgMlands secrete mucus that serves only to lubricate the inner labial surfaces, as well as to enhance the viability and motility of sperm. PTS: 1 REF: Page 772 7. Which change is a result of puberty and defends the vagina from infection? a. The pH stabilizes between 7 and 8. b. A thin squamous epithelial lining develops. c. Vaginal pH becomes more acidic. d. Estrogen levels are low. ANS: C At puberty, the pH becomes more acidic (4 to 5) and the squamous epithelial lining thickens. These changes are maintained until menopause (cessation of menstruation), at which time the pH rises again to more alkaline levels and the epithelium thins out. Therefore protection from infection is greatest during the years when a woman is most likely to be sexually active. Estrogen does not play a role in infection protection. PTS: 1 REF: Page 773 8. What happens to the vagina’s lining at puberty? a. It becomes thinner. c. It assumes a neutral pH. b. It becomes thicker. d. It undergoes atrophy. ANS: B Before puberty, vaginal pH is approximately 7 (neutral) and the vaginal epithelium is thin. At puberty, the pH becomes more acidic (4 to 5) and the squamous epithelial lining thickens. Cell atrophy is not associated with puberty. PTS: 1 REF: Page 773 9. Which structure is lined with columnar epithelial cells? a. Perimetrium c. Myometrium b. Endocervical canal d. Vagina ANS: B Of the available options, only the endocervical canal does not have an endometrial layer; rather, the layer is lined with columnar epithelial cells. PTS: 1 REF: Page 775 10. Where is the usual site of fertilization of an ovum? a. Trumpet end of the fallopian tubes c. Ampulla of the fallopian tubes b. Fimbriae of the fallopian tubes d. Os of the fallopian tubes ANS: C The ampulla, or distal third, of the fallopian tube is the usual site of fertilization (see Figure 23-7). PTS: 1 REF: Page 775 11. Where is the usual site of cervical dysplasia or cancer in situ? a. Squamous epithelium of the ceNrUvRixSImNeGeTtBs .tChOeMcuboidal epithelium of the vagina. b. Columnar epithelium of the cervix meets the squamous epithelium of the uterus. c. Squamous epithelium of the cervix meets the columnar epithelium of the uterus. d. Columnar epithelium of the cervix meets the squamous epithelium of the vagina. ANS: D The point at which the columnar epithelium of the cervix meets the squamous epithelium of the vagina is called the transformation zone or the squamous-columnar junction. The transformation zone is especially susceptible to the oncogenic human papillomavirus (HPV), which leads to cervical dysplasia and, ultimately, cervical cancer; these are the cells sampled during a Papanicolaou (Pap) test. PTS: 1 REF: Page 775 12. Having ejected a mature ovum, the ovarian follicle develops into a(n): a. Atretic follicle c. Corpus luteum b. Thecal follicle d. Functional scar ANS: C Having ejected a mature ovum, the only resulting structure is the corpus luteum. PTS: 1 REF: Page 776 13. The mucosal secretions of the cervix secrete which immunoglobulin? a. IgA c. IgG b. IgE d. IgM ANS: A Mucosal secretions from the cervix contain enzymes and antibodies—predominantly IgA. PTS: 1 REF: Page 775 14. The equivalent to the female gonad is the male: a. Epididymis c. Vas deferens b. Spermatic cord d. Testes ANS: D Between 6 to 7 weeks’ gestation, the male embryo will differentiate under the influence of TDF. TDF stimulates the male gonads to develop into the two testes. The ovaries, the female gonads, are the primary female reproductive organs. PTS: 1 REF: Page 769 15. A surge of which hormone causes the corpus luteum to produce progesterone? a. Follicle stimulating hormone c. Gonadotropin-releasing hormone b. Luteinizing hormone d. Estrogen ANS: B Luteinizing hormone from the anterior pituitary stimulates the corpus luteum to secrete progesterone, the second major female sex hormone. PTS: 1 REF: Page 778 16. What directly causes ovulation duNriUnRgStIhNeGmTBen.CsOtrMual cycle? a. Gradual decrease in estrogen levels c. Sharp rise in progesterone levels b. Sudden increase of LH d. Gradual increase in estrogen levels ANS: B Menstrual cyclicity and regular ovulation are dependent on (1) the activity of the gonadostat (GnRH pulse generator); (2) the pituitary secretion of gonadotropins; and (3) estrogen (estradiol)–positive feedback for the preovulatory LH and FSH surges, oocyte maturation, and corpus luteum formation. PTS: 1 REF: Pages 778-780 17. Which anatomic structure secretes follicle stimulating hormone (FSH) and luteinizing hormone (LH)? a. Hypothalamus c. Anterior pituitary b. Ovaries d. Adrenal cortex ANS: C The anterior pituitary is the gland that secretes FSH and LH. PTS: 1 REF: Pages 780-781 18. During the time that ovulation occurs, which statement concerning basal body temperature (BBT) is true? a. BBT increases. b. BBT decreases. c. BBT fluctuates around 37° C (98° F). d. BBT rises consistently above 37.8° C (100° F). ANS: C During the follicular phase of ovulation, the BBT fluctuates around 37° C (98° F). PTS: 1 REF: Page 782 19. What structure in the male lies posterior to the urinary bladder? a. Seminal vesicles c. Cowper glands b. Prostate glands d. Parabladder glands ANS: A The seminal vesicles are a pair of glands, each measuring approximately 4 to 6 cm long, which lie behind the urinary bladder and in front of the rectum. None of the other structures lie in this location. PTS: 1 REF: Page 788 20. When do penile erections begin? a. Before birth c. Shortly before puberty b. Shortly after birth d. After puberty ANS: A Erections begin in utero and continue throughout life, but ejaculation does not occur until sperm production begins at puberty. PTS: 1 REF: Page 78N7URSINGTB.COM 21. Which statement is true regarding the major difference between male and female sex hormone production? a. Luteinizing hormone has no apparent action in a man. b. In a man, sex hormone production is relatively constant. c. Estradiol is not produced in a man. d. In a man, gonadotropin-releasing hormone does not cause the release of follicle stimulating hormone. ANS: B In men, sex hormone production is relatively constant with some diurnal variation. The other options are not true statements. PTS: 1 REF: Page 789 22. Where in the male body does spermatogenesis occur? a. Epididymis c. Seminiferous tubules b. Rete testes d. Vas deferens ANS: C Spermatogenesis takes place only in the seminiferous tubules of the testes (see Figure 23-14). PTS: 1 REF: Pages 784-785 23. Which immunoglobulin is contained in breast milk? a. IgA c. IgG b. IgE d. IgM ANS: A Not only does breast milk composition change over time to meet the changing digestive capabilities and nutritional requirements of the infant, but it also contains immune cells, specific immunoglobulins, especially IgA, and nonspecific antimicrobial factors, such as lysozymes and lactoferrin, that protect the infant against infection, allergies, and asthma. PTS: 1 REF: Page 784 24. Which hormone promotes the development of the lobular ducts in the breasts? a. Progesterone c. Oxytocin b. Prolactin d. Estrogen ANS: D Only estrogen promotes the increase in the size of the breasts by the formation of a mass of tissue under the areola, which increases the size and pigmentation of the areola and contributes to the development of the lobular ducts. PTS: 1 REF: Page 783 25. What causes the vasomotor flushes (hot flashes) that are associated with declining ovarian function with age? a. Decreased estrogen levels c. Increased estrogen levels b. Absence of estrogen ANS: D NURSINGTdB..CORMapid changes in estrogen levels A rapid change in estrogen levels (withdrawal or increase), rather than low estrogen levels, induces hot flashes. PTS: 1 REF: Pages 794-795 26. When does the male body begin to produce sperm? a. Before birth c. At puberty b. Shortly after birth d. When erection is possible ANS: C Erections begin in utero and continue throughout life, but ejaculation does not occur until sperm production begins at puberty. PTS: 1 REF: Page 787 27. The human zygote has a total of how many chromosomes? a. 23 c. 46 b. 25 d. 50 ANS: C A 23-chromosome female gamete, the ovum, and a 23-chromosome male gamete, the spermatozoon (sperm cell), unite to form a 46-chromosome zygote that is capable of developing into a new individual. PTS: 1 REF: Page 768 28. Which hormone promotes the development of testosterone in both males and females? a. Progesterone c. Oxytocin b. Prolactin d. Estrogen ANS: B Prolactin, a polypeptide synthesized and secreted from the pituitary, helps maintain biosynthesis of testosterone. PTS: 1 REF: Page 791 29. Which hormone stimulates gonads to produce both male and female hormones? a. Gonadotropin-releasing hormone (GnRH) b. Follicle-stimulating hormone (FSH) c. Luteinizing hormone (LH) d. Estrogen ANS: C Extrahypothalamic factors cause the hypothalamus to secrete GnRH, which stimulates the anterior pituitary to secrete gonadotropins—FSH and LH. These hormones, in turn, stimulate the gonads (ovaries or testes) to secrete female or male sex hormones. PTS: 1 REF: Pages 769-771 30. Which hormone relaxes the myometrium and prevents lactation until the fetus is born? a. Gonadotropin-releasing hormoNnUeR(SGINnGRTHB).COM b. Follicle-stimulating hormone (FSH) c. Progesterone d. Estrogen ANS: C Progesterone is sometimes called the hormone of pregnancy. Its effects in pregnancy include (1) maintenance of the thickened endometrium; (2) relaxation of smooth muscle in the myometrium, which prevents premature contractions and helps the uterus expand; (3) thickening of the myometrium, which prepares it for the muscular work of labor; (4) prevention of lactation until the fetus is born; and (5) prevention of additional maturation of ova by way of suppressing FSH and LH, thereby stopping the menstrual cycle. PTS: 1 REF: Page 778 31. A Sims-Huhner test is of particular interest to a patient experiencing which condition? a. Infertility c. Venereal disease b. Possible HIV infection d. Perimenopause ANS: A A Sims-Huhner test evaluates the ability of sperm to penetrate and maintain motility in cervical mucus 2 to 4 hours after coitus approximately 1 day before ovulation. The enzyme-linked immunosorbent assay (ELISA) detects the presence of antibodies to human immunodeficiency virus (HIV). Venereal Disease Research Laboratory (VDRL) is a test for nonspecific venereal diseases. Estradiol levels are associated with erratic or intermittent menstruation. PTS: 1 REF: Page 792 | Table 23-3 MULTIPLE RESPONSE 32. Estrogen has many biological effects on the female body including: (Select all that apply.) a. Maturation of reproductive organs b. Differentiating female physical characteristics c. Postpuberty closure of short bones d. Regulation of the menstrual cycle e. Endometrial regeneration after menstruation ANS: A, B, D, E Estrogen has numerous biologic effects, many of which involve interactions with other hormones. Estrogen is needed for the maturation of the reproductive organs, development of secondary sex characteristics (differentiating male and female physical characteristics that are not directly related to reproduction), closure of long bones after the pubertal growth spurt, regulation of the menstrual cycle, and endometrial regeneration after menstruation. PTS: 1 REF: Page 77N6URSINGTB.COM 33. Which statements about the human papillolmavirus (HPV) and vaccine are true? (Select all that apply.) a. Currently, two HPV vaccines have been approved for use in the United States. b. HPV is believed to be responsible for the majority of the diagnosed cases of cervical cancer. c. A form of the vaccine has been approved for use in males to prevent genital warts. d. The administration of the vaccine is a one-dose intramuscular injection. e. The recommended age for vaccination of girls is between 11 and 12 years of age. ANS: A, B, C, E Two HPV vaccines are currently approved in the United States: (1) quadrivalent HPV recombinant vaccine and (2) bivalent HPV recombinant vaccine. HPV is responsible for 99.7% of cervical cancer cases, 40% of penile cancers, and an estimated 5% of all cancers worldwide. The vaccine is administered by intramuscular injection, and the recommended schedule is a three-dose series with the second and third doses administered 2 and 6 months after the first dose. The recommended age for vaccination of girls is 11 to 12 years. The vaccine can be administered to girls as young as 9 years of age. The quadrivalent vaccine has been approved for males 9 to 26 years of age to prevent genital warts. PTS: 1 REF: Page 775 | What's New box 34. Which statements are true regarding the female menstrual cycle? (Select all that apply.) a. Initial cycles may dramatically vary in length. b. By adulthood, the commonly accepted cycle average is 28 (27 to 30) days. c. The length of a cycle varies among women. d. Up to 8 years before menopause, the intervals of the menstrual cycle begin to lengthen. e. Menopause is achieved when a woman is without a period for 2 years. ANS: A, B, C, D At first, cycles are anovulatory and may vary in length from 10 to 60 days or longer. As adolescence proceeds into adulthood, regular patterns of menstruation and ovulation are established at intervals ranging from 25 to 35 days. The length of the menstrual cycle varies considerably among women. The commonly accepted cycle average is 28 (27 to 30) days, with rhythmic intervals of 21 to 35 days considered normal. Approximately 2 to 8 years before menopause, cycles begin to lengthen again. Menopause is defined as the cessation of menstrual flow for 1 year. PTS: 1 REF: Page 778 35. Testosterone is believed to have a role in: (Select all that apply.) a. Male-patterned baldness b. Libido levels c. Acne development d. Altered cholesterol metabolism e. Thinning of the larynx ANS: A, B, C, D Testosterone is associated with allNtUhRe SoIpNtGioTnBs.CeOxcMept thinning of the larynx; it actually stimulates the growth of the larynx’s cartilage. PTS: 1 REF: Page 789 36. What are normal characteristics of aging of the male reproductive system? (Select all that apply.) a. Reduced sperm count b. Slower, less forceful ejaculations c. Testicular atrophy and softening d. Longer time to achieve full erection e. Decreased levels of testosterone ANS: B, C, D, E The described effects on ejaculation, testes, erection, and testosterone are normal characteristics of male aging. Sperm count remains normal with age, although the semen tends to contain more defective and nonmotile sperm. PTS: 1 REF: Pages 795-796 Chapter 25: Alterations of the Female Reproductive System MULTIPLE CHOICE 1. In 95% of children of delayed puberty, the problem is caused by: a. Disruption in the hypothalamus c. Deficit in estrogen or testosterone b. Disruption of the pituitary d. Physiologic hormonal delays ANS: D In 95% of children with delayed puberty, the delay is physiologic; that is, hormonal levels are normal and the hypothalamic-pituitary-gonadal (HPG) axis is intact, but maturation is slowly happening. This option is the only answer that accurately describes the most common cause of delayed puberty. PTS: 1 REF: Page 802 2. What is the first sign of puberty in girls? a. Breast enlargement c. Menstruation b. Growth of pubic hair d. Vaginal discharge ANS: A Of the options available, the first sign of puberty in girls is usually thelarche or breast development. PTS: 1 REF: Page 802 3. Which type of precocious pubertyNcUaRuSseINs GthTeBc.ChOilMd to develop some secondary sex characteristics of the opposite sex? a. Mixed c. Isosexual b. Incomplete d. Homosexual ANS: A Mixed precocious puberty, which is virilization of a girl or feminization of a boy, causes the child to develop some secondary sex characteristics of the opposite sex. This option is the only answer that accurately identifies the type of precocious puberty described. PTS: 1 REF: Page 804 4. The release of which chemical mediator causes primary dysmenorrhea? a. Leukotrienes c. Bradykinin b. Prostaglandins d. C-reactive protein ANS: B Primary dysmenorrhea is painful menstruation associated with the release of prostaglandins in ovulatory cycles. This option is the only answer that accurately identifies the chemical mediator associated with dysmenorrhea. PTS: 1 REF: Page 804 5. Considering the pathophysiologic characteristics of primary amenorrhea, what anatomic structure is involved in compartment II? a. Ovary c. Hypothalamus b. Anterior pituitary d. Vagina ANS: A Compartment II disorders involve only the ovary. PTS: 1 REF: Page 805 6. Considering the pathophysiologic characteristics of primary amenorrhea, what anatomic structure is involved in compartment IV? a. Vagina c. Ovary b. Hypothalamus d. Anterior pituitary ANS: B Of the options available, only compartment IV disorders include central nervous system (CNS) conditions, in particular hypothalamic disorders. PTS: 1 REF: Page 805 7. Which condition is considered a clinical cause of amenorrhea? a. Disorder in the endometrium c. Lack of physical exercise b. Obstruction of the fallopian tubes d. Failure to ovulate ANS: D Depressed ovarian hormone levels, which are associated with a variety of clinical disorders, also cause amenorrhea by preventing ovulation. This option is the only answer that accurately identifies a clinical cause of cycle irregularities. PTS: 1 REF: Page 80N7URSINGTB.COM 8. Clinical manifestations that include irregular or heavy bleeding, the passage of large clots, and the depletion of iron stores support which diagnosis? a. Premenstrual syndrome c. Polycystic ovary syndrome b. Dysfunctional uterine bleeding d. Primary dysmenorrhea ANS: B Unpredictable and variable bleeding, in terms of amount and duration, characterize dysfunctional uterine bleeding. Especially during perimenopause, dysfunctional bleeding also may involve flooding and the passage of large clots, which often indicate excessive blood loss. Excessive bleeding can lead to iron-deficiency anemia. This option is the only answer that demonstrates the clinical manifestations described. PTS: 1 REF: Page 809 9. What statement concerning the pathogenetic mechanisms of polycystic ovarian syndrome (POS) is true? a. POS causes a decrease in leptin levels; this decrease reduces the hypothalamic pulsatility of gonadotropin-releasing hormone, which reduces the number of follicles that mature. b. POS is a result of a disorder in the anterior pituitary that increases the follicle-stimulating hormone, which reduces the luteinizing hormone released. c. POS is a result of a combination of conditions that include oligo-ovulation or anovulation, elevated levels of androgens, or clinical signs of hyperandrogenism and polycystic ovaries. d. POS inhibits testosterone, which stimulates androgen secretion by the ovarian stroma and indirectly reduces sex hormone–binding globulin. ANS: C POS has at least two of the following conditions: oligo-ovulation or anovulation, elevated levels of androgens, or clinical signs of hyperandrogenism and polycystic ovaries. Of the options available, only this answer accurately defines the pathogenetic mechanisms of POS. PTS: 1 REF: Page 810 10. What is the leading cause of infertility in women? a. Pelvic inflammatory disease c. Salpingitis b. Endometriosis d. Polycystic ovary syndrome ANS: D Polycystic ovary syndrome remains one of the most common endocrine disturbances affecting women, especially young women, and is a leading cause of infertility in the United States. PTS: 1 REF: Page 810 11. Considering the mediating factors of premenstrual syndrome (PMS), which medication may be used either continually or only during the menstrual period as a treatment for the condition? a. NSAIDs NURSINGTcB..COSMSRIs b. Estrogen d. Progesterone ANS: C A selective serotonin reuptake inhibitors (SSRI) (an antidepressant) relieves symptoms in approximately 60% to 90% of women and may be continually administered or only prescribed during the premenstrual period. Oral contraceptive pills that contain estrogen and progesterone also can be continuously used for up to 3 months to decrease the frequency of menstrual periods, PMS, and premenstrual dysphoric disorder (PMDD). Nonsteriodal antiinflammatory drugs (NSAIDs) would not be continually administered. PTS: 1 REF: Page 813 12. Which statement regarding pelvic inflammatory disease (PID) is true? a. An episode of mild PID can decrease the possibility of a successful pregnancy by 80%. b. Such an inflammation results in temporary changes to the ciliated epithelium of the fallopian tubes. c. PID has not been associated with an increased risk of an ectopic pregnancy. d. Contracting this infection increases the risk of uterine cancer. ANS: D PID infection results in permanent changes to the ciliated epithelium of the fallopian or uterine tubes. A recent study has found that one episode of mild, subclinical PID resulted in a 40% decrease in later pregnancy rates, and multiple episodes of PID further increase the risk of infertility. Scarring caused by PID greatly increases the risk of a later ectopic pregnancy by up to tenfold. Scarring and adhesions also can result in chronic pelvic pain and, potentially, an increased risk of later uterine cancer. PTS: 1 REF: Pages 813-814 13. When a woman’s uterus is assessed as protruding through the entrance of the vagina to the hymen, which grade of prolapse does this indicate? a. 0 c. 2 b. 1 d. 3 ANS: C A grade 2 prolapse reaches the hymen (see Figure 24-10). PTS: 1 REF: Page 818 | Box 24-10 14. Which term is used to identify the descent of the posterior bladder and trigone into the vaginal canal? a. Rectocele c. Cystocele b. Vaginocele d. Enterocele ANS: C Cystocele is the only term used to identify the descent of a portion of the posterior bladder wall and trigone into the vaginal canal; the trauma of childbirth is usually the cause. NURSINGTB.COM PTS: 1 REF: Page 819 15. What type of cyst develops when an ovarian follicle is stimulated but no dominant follicle develops and completes the maturity process? a. Follicular c. Corpus albicans b. Corpus luteal d. Benign ovarian ANS: D Only benign cysts of the ovary are produced when a follicle or a number of follicles are stimulated but no dominant follicle develops and completes the maturity process. PTS: 1 REF: Page 820 16. Which term is used to identify benign uterine tumors that develop from smooth muscle cells in the myometrium and are commonly called uterine fibroids? a. Endometrial polyps c. Leiomyomas b. Myometrial polyps d. Myometriomas ANS: C Leiomyomas, commonly called myomas or uterine fibroids, are benign smooth muscle tumors in the myometrium (see Figure 24-14). The other terms do not accurately identify the tumors described. PTS: 1 REF: Page 821 17. What theory is used to describe the cause of endometriosis? a. Obstruction within the fallopian tubes prevents the endometrial tissue from adhering to the lining of the uterus. b. Endometrial tissue passes through the fallopian tubes and into the peritoneal cavity and remains responsive to hormones. c. Inflammation of the endometrial tissue develops after recurrent sexually transmitted diseases. d. Endometrial tissue lies dormant in the uterus until the ovaries produce sufficient hormone to stimulate its growth. ANS: B It has been proposed that endometriosis is caused by the implantation of endometrial cells during retrograde menstruation, during which menstrual fluids move through the fallopian tubes and empty into the pelvic cavity (see Figure 24-16). Similar to normal endometrial tissue, the ectopic (out of place) endometrium responds to the hormonal fluctuations of the menstrual cycle. Of the available options, this answer is the only accepted theory for the cause of endometriosis. PTS: 1 REF: Page 823 18. Which virus is a precursor for developing cervical intraepithelial neoplasia (CIN) and cervical cancer? a. Human papillomavirus (HPV) c. Herpes simplex II virus (HSV) b. Epstein-Barr virus (EBV) d. Cytomegalovirus (CMV) ANS: A Infection with high-risk (oncogenNicU) RtySpINesGoTfBH.CPOVM (predominantly 16 and 18) is a necessary precursor to the development of precancerous dysplasia of the cervix that leads to invasive cancer. The other options are not precursors to CIN and cervical cancer. PTS: 1 REF: Page 825 19. Which description is used when a progressive neoplastic change involves the full epithelial thickness of the cervix? a. Cervical intraepithelial neoplasia c. Cervical carcinoma in situ b. Cervical dysplasia d. Invasive carcinoma of the cervix ANS: C The progressive neoplastic changes of cervical cells are classified on a continuum from cervical intraepithelial neoplasia (dysplasia) to cervical carcinoma in situ (full epithelial thickness of the cervix is involved), which is generally a precursor of invasive carcinoma of the cervix to invasive carcinoma of the cervical tissue. PTS: 1 REF: Page 827 20. Which factor increases the risk for ovarian cancer after the age of 40 years? a. Use of fertility drugs c. Multiple pregnancies b. Oral contraceptive use d. Prolonged lactation ANS: A Ovarian cancer in women older than 40 years of age is associated with early menarche, late menopause, nulliparity, and the use of fertility drugs. The other options are not necessarily related to women older than the age of 40 years. PTS: 1 REF: Page 831 21. Infertility is defined as the inability to conceive after how many months of unprotected intercourse with the same partner? a. 6 c. 18 b. 12 d. 24 ANS: B Infertility is defined as the inability to conceive after 1 year of unprotected intercourse with the same partner. PTS: 1 REF: Page 835 22. Which of the following is not considered a cause of galactorrhea? a. Proliferation of the lactiferous ducts of the breast b. Hypothyroidism, resulting from a decrease in thyroid-releasing hormone c. Excess prolactin secretion from the pituitary d. Drugs such as high-dose oral contraceptives and phenothiazines ANS: A The most common cause of galactorrhea is nonpuerperal hyperprolactinemia, or excessive amounts of prolactin. A variety of exogenous agents (such as drugs) and disorders can trigger one of these three mechanisms, thereby causing hyperprolactinemia. Hypothyroidism causes increasedNsUecRrSeItNioGnToBf.ChOyMpothalamic thyroid-releasing hormone, which stimulates the release of prolactin from the pituitary. The proliferation of lactiferous breast ducts is not associated with galactorrhea. PTS: 1 REF: Pages 836-837 23. Fluid-filled squishy sacs characterize which breast disorder? a. Paget disease c. Nonproliferative breast lesions b. Cysts d. Lobular carcinoma in situ ANS: C Cysts (fluid-filled sacs) are a specific type of lump that commonly occurs in women in their 30s, 40s, and early 50s. Cysts feel squishy when they occur close to the surface of the breast; however, when deeply embedded, cysts can feel hard. The other options do not accurately identify the disorder associated with these symptoms. PTS: 1 REF: Pages 838-839 24. What are typical findings on breast palpation of a woman diagnosed with simple fibroadenoma? a. Painful, round, movable, and fluid-filled mass b. Painless, movable, hard, and irregular mass c. Smooth, solid, mobile, and well-circumscribed mass d. Smooth, nonmovable, irregular, and soft mass ANS: C Only this option accurately identifies the typical palpation results of a simple fibroadenoma. PTS: 1 REF: Pages 840-841 25. Which benign breast tumor affects postmenopausal women and is characterized by the principal lactiferous ducts becoming dilated and filled with cellular debris? a. Mammary duct ectasia c. Phyllodes tumor b. Intraductal papilloma d. Fibroadenoma ANS: A Of the options available, only mammary duct ectasia is associated with the age and the identified pathologic characteristics (see Table 24-9). PTS: 1 REF: Page 838 | Table 24-9 26. The majority of the small percentage of ovarian cancers that are associated with a known pattern of inheritance are associated with: a. Susceptibility of the BRCA1 gene b. Mutations of the BRCA2 gene c. Hereditary nonpolyposis colorectal cancer (HNPCC) syndrome d. Low progesterone levels ANS: A The majority (approximately 90%) of ovarian cancers are sporadic and not associated with a known pattern of inheritance. Of the 5% to 10% that have a familial component, the majority are associated with the bNreUaRstScINanGcTeBr.CsuOsMceptibility gene 1 (BRCA1) and a smaller number with mutations of the BRCA2 or mismatched repair genes (HNPCC syndrome). Low progesterone levels are not associated with ovarian cancers. PTS: 1 REF: Page 831 27. What is usually the first clinical manifestation of breast cancer? a. Nipple dimpling c. Enlargement of one breast b. Nipple discharge d. Painless lump ANS: D Invasive carcinoma of the breast generally exhibits a nontender palpable mass or thickened area. This option is the only answer that accurately describes the first clinical manifestation of breast cancer. PTS: 1 REF: Page 873 MULTIPLE RESPONSE 28. What is a recognized treatment for the symptoms often associated with pelvic organ prolapse? (Select all that apply.) a. Pessary b. Kegel exercises c. Estrogen therapy d. Surgical repair e. Bearing down exercises ANS: A, B, C, D A common first-line treatment is a pessary, which is a removable mechanical device that holds the uterus in position. The pelvic fascia may be strengthened through Kegel exercises (repetitive isometric tightening and relaxing of the pubococcygeal muscles) or by estrogen therapy in menopausal women. Maintaining a healthy body mass index, preventing constipation, and treating chronic cough may help as well. Surgical repair with or without a hysterectomy is the treatment of last resort. Bearing down would likely exacerbate the problem. PTS: 1 REF: Page 818 29. Dysfunctional uterine bleeding (DUB), secondary to ovarian dysfunction, is abnormal uterine bleeding resulting from: (Select all that apply.) a. Endometriosis b. Progesterone deficiency c. Sexually transmitted infections d. Congenital abnormalities in the uterine structure e. Estrogen excess ANS: B, E Of the options available, DUB, secondary to ovarian dysfunction, is a result of either progesterone deficiency or unopposed estrogen excess. PTS: 1 REF: Pages 808-809 NURSINGTB.COM 30. The size of benign uterine tumors, such as leiomyomas, is thought to be caused by the influence of which hormone? (Select all that apply.) a. Progesterone b. Estrogen c. Luteinizing hormone d. Gonadotropin-stimulating hormone e. Growth factors ANS: A, B, E The cause of uterine leiomyomas is unknown, although their size appears to be related to only estrogen, progesterone, growth factors, angiogenesis, and apoptosis. PTS: 1 REF: Page 822 31. What are the common clinical manifestations of endometriosis? (Select all that apply.) a. Back and flank pain b. Infertility c. Dysuria d. Amenorrhea e. Dysmenorrhea ANS: B, E Common clinical manifestations primarily include infertility, dysmenorrhea, dyschezia (pain on defecation), and dyspareunia (pain on intercourse). PTS: 1 REF: Page 824 NURSINGTB.COM Chapter 26: Alterations of the Male Reproductive System MULTIPLE CHOICE 1. In the 95% of those with delayed puberty, the problem is caused by which condition? a. Disruption in the hypothalamus c. Deficit in estrogen or testosterone b. Disruption of the pituitary d. Physiologic hormonal delays ANS: D In 95% of cases, delayed puberty is a physiologic delay; that is, hormonal levels are normal and the hypothalamic-pituitary-gonadal (HPG) axis is intact, but maturation is happening slowly. This selection is the only option that accurately describes 95% of those with delayed puberty. PTS: 1 REF: Page 886 2. What is the first sign of puberty in boys? a. Thickening of the scrotal skin c. Enlargement of the testes b. Growth of pubic hair d. Change in voice ANS: C The first sign of puberty in boys is an enlargement of the testes and a thinning of the scrotal skin. PTS: 1 REF: Page 886 3. Which type of precocious pubertyNcUaRuSseINs GthTeBc.ChOilMd to develop some secondary sex characteristics of the opposite sex? a. Mixed c. Isosexual b. Incomplete d. Homosexual ANS: A Mixed precocious puberty (i.e., virilization of a girl or feminization of a boy) causes the child to develop some secondary sex characteristics of the opposite sex. This selection is the only option that accurately identifies the type of precocious puberty described. PTS: 1 REF: Page 887 4. What term is used to identify a condition in which the foreskin cannot be retracted over the glans penis? a. Paraphimosis c. Prephimosis b. Priapism d. Phimosis ANS: D Phimosis is the only term used to identify the condition in which the foreskin cannot be retracted back over the glans. PTS: 1 REF: Page 888 5. What term is used to identify a fibrotic condition that causes lateral curvature of the penis during erection, which is associated with a local vasculitis-like inflammatory reaction and decreased tissue oxygenation? a. Phimosis c. Lateral paraphimosis b. Lateral phimosis d. Peyronie disease ANS: D Peyronie disease (bent nail syndrome) is a fibrotic condition of the tunica albuginea of the penis, resulting in varying degrees of curvature and sexual dysfunction (see Figure 25-2). Although the exact cause is unknown, a local vasculitis-like inflammatory reaction occurs and decreased tissue oxygenation results in fibrosis and calcification. Peyronie disease is the only term used to identify the pathophysiologic condition described. PTS: 1 REF: Pages 889-890 6. What term is used to identify an inflammation of the glans penis? a. Glanitis c. Priapism b. Balanitis d. Hydrocelitis ANS: B Balanitis is the only term used to identify an inflammation of the glans penis (see Figure 25-4). PTS: 1 REF: Page 890 7. Cryptorchidism can be defined as which of the following? a. Normal developmental state of the testes b. Abnormal state in which the teNsUteRsSaINreGoTvBe.CrdOeMveloped c. Lack of scrotum d. Testicular maldescent ANS: D Cryptorchidism is a condition of testicular maldescent, the only option that accurately defines cryptorchidism. PTS: 1 REF: Page 892 8. What is the most common infectious cause of orchitis and one that usually affects postpubertal boys? a. Herpes c. Mumps b. Escherichia coli d. Cytomegalovirus ANS: C Of the options available, mumps is the most common infectious cause of orchitis and usually affects postpubertal boys. PTS: 1 REF: Pages 894-895 9. The risk of which cancer is greater if the man has a history of cryptorchidism? a. Penile c. Prostate b. Testicular d. Epididymal ANS: B The risk of testicular cancer is 35 to 50 times greater in men with cryptorchidism or in those with a history of cryptorchidism than it is for the general male population. This is not true of the other options. PTS: 1 REF: Pages 893-894 10. What are the clinical manifestations of testicular cancer? a. Firm, nontender testicular mass c. Painful fluid-filled testicular mass b. Painful, mobile, firm testicular mass d. Soft, nontender testicular mass ANS: A Of the options available, a firm, painless testicular enlargement is commonly identified as the first sign of testicular cancer. PTS: 1 REF: Pages 895-896 11. How does the epididymis become infected? a. The pathogenic microorganisms ascend the vasa deferentia from an already infected urethra or bladder. b. The pathogenic microorganisms are attached to sperm that travel through the genital tract. c. The pathogenic microorganisms from the tunica vaginalis are transported to the epididymis. d. The pathogenic microorganisms from the prostate fluid ascend to the epididymis. ANS: A The pathogenic microorganisms usually reach the epididymis by ascending the vasa deferentia from an already infecteNdUuRreStIhNrGaToBr.CblOaMdder. Of the selections available, this is the only option that accurately describes how the epididymis becomes infected. PTS: 1 REF: Page 897 12. Symptoms of benign prostatic hyperplasia (BPH) are a result of which pathophysiologic condition? a. Infection of the prostate c. Ischemia of the urethra b. Obstruction of the urethra d. Compression of the urethra ANS: D BPH becomes problematic as prostatic tissue compresses the urethra, where it passes through the prostate. Of the selections available, only this option accurately describes the pathophysiologic condition behind the symptoms of BPH. PTS: 1 REF: Pages 897-899 13. Which infection has clinical manifestations that include the sudden onset of malaise, low back pain, and perineal pain with high fever and chills, dysuria, nocturia, and urinary retention? a. Orchitis c. Epididymitis b. Balanitis d. Bacterial prostatitis ANS: D Bacterial prostatitis can exhibit common manifestations that include a sudden onset of malaise, low back and perineal pain, high fever (up to 40° C [104° F]), and chills, as well as dysuria, inability to empty the bladder, nocturia, and urinary retention. Myalgia and arthralgia also may occur. This selection is the only option that exhibits the symptoms described. PTS: 1 REF: Pages 899-900 14. Priapism has been associated with the abuse of what substance? a. Marijuana c. Cocaine b. Alcohol d. Heroin ANS: C Of the options available, priapism has been associated with cocaine use. PTS: 1 REF: Page 890 15. Which age group should be targeted for testicular cancer education and screening? a. 15 to 35 year olds c. 30 to 55 year olds b. 20 to 45 year olds d. 45 to 70 year olds ANS: A Overall, testicular cancers are rare, yet they are the most common form of cancer in young men between the ages of 15 and 35 years. PTS: 1 REF: Page 895 16. What is the reason breast cancer inNUmReSnINhGasTBsu.CcOhMa poor prognosis? a. Breast cancer is extremely aggressive in men. b. Treatment is usually delayed as a result of late detection. c. Chemotherapies are not as effective in men. d. Breast tumors tend to be small and hard to isolate. ANS: B Breast cancer is relatively uncommon in men, but it has a poor prognosis because men tend to delay seeking treatment until the disease is advanced. This selection is the only option that accurately identifies the reason breast cancer in men has a poor prognosis. PTS: 1 REF: Page 914 MULTIPLE RESPONSE 17. Which are clinical manifestations of male breast cancer? (Select all that apply.) a. Ulceration present on the breast b. Retraction of breast tissue c. Nipple discharge d. Palpable mass midline of the nipple e. Unilateral solid mass ANS: A, B, C, E The malignant male breast lesion is usually a unilateral solid mass located near the nipple. Because the nipple is commonly involved, crusting and nipple discharge are typical clinical manifestations. Other findings include skin retraction, ulceration of the skin over the tumor, and axillary node involvement. PTS: 1 REF: Page 914 18. Which statements are true regarding urethritis? (Select all that apply.) a. A purulent drainage may be present. b. A clear mucus-like discharge may be present. c. Symptoms include urethral tingling and itching or burning on urination. d. A 24-hour urine test is required to diagnose the disorder. e. Treatment includes appropriate antibiotic therapy. ANS: A, B, C, E Symptoms of urethritis include urethral tingling and itching or a burning sensation on urination (dysuria), frequency, and urgency. The individual may note a purulent or clear mucus-like discharge from the urethra. Nucleic acid detection amplification tests allow easy detection of Neisseria gonorrhoeae and Chlamydia trachomatis in first-void urine. Treatment consists of appropriate antibiotic therapy for infectious urethritis and an avoidance of future chemical or mechanical irritation. PTS: 1 REF: Page 888 19. Which statements are true regarding prostate cancer? (Select all that apply.) a. It ranks second to lung cancer as being most common among American men. b. A familial history of prostate cancer is a risk factor. c. Dietary habits seem to play a rNoUleRiSnINitGsTdBe.vCeOloMpment. d. African-American men have an increased risk for its development. e. Being over 65 years of age increases the risk for developing prostate cancer. ANS: B, C, D, E Prostate cancer is the most commonly diagnosed non–skin cancer in American men, and the incidence varies greatly worldwide. Possible causes include a genetic predisposition, environmental and dietary factors, inflammation, and alterations in levels of hormones (e.g., testosterone, dihydrotestosterone, estradiol) and growth factors. Incidence is greatest among northwestern European and North American men (particularly African Americans) older than 65 years of age. PTS: 1 REF: Pages 900-910 MATCHING Match the description with the disorder. A. Painless diverticulum of the epididymis located between the head of the epididymis and the testis B. Collection of fluid in the tunica vaginalis C. Rotation of a testis, which twists blood vessels of the spermatic cord D. Abnormal dilation of the vein within the spermatic cord 20. Varicocele 21. Hydrocele 22. Testicular torsion 23. Spermatocele 20. ANS: D PTS: 1 REF: Page 891 MSC: A varicocele is an abnormal dilation of a vein within the spermatic cord and is classically described as a bag of worms 21. ANS: B PTS: 1 REF: Page 892 MSC: A hydrocele is a collection of fluid within the tunica vaginalis 22. ANS: C PTS: 1 REF: Page 894 MSC: Torsion of the testis is rotation of a testis, which twists blood vessels in the spermatic cord. 23. ANS: A PTS: 1 REF: Page 892 MSC: A spermatocele is a painless diverticulum of the epididymis located between the head of the epididymis and the testis. NURSINGTB.COM Chapter 27: Sexually Transmitted Infections MULTIPLE CHOICE 1. What unique factor causes adolescent girls to have a high risk for sexually transmitted infections (STIs)? a. They are in an experimental phase with sexual intercourse and believe they are resistant to developing STIs. b. The adolescent cervix is immature and lacks immunity. c. The length of the vaginal canal is short in adolescents, allowing a greater concentration of microorganisms within the internal genitalia. d. In adolescent girls, the anus to the vaginal introitus are in close proximity. ANS: B Partly, perhaps, because of risk-taking behavior (unprotected intercourse or selection of high-risk partners), many adolescents have an increased risk for STI exposure and infection. The unique factor for adolescent women is that they have a physiologically increased susceptibility to infection because of increased cervical immaturity and lack of immunity. The remaining options are not considered legitimate risk factors for STIs. PTS: 1 REF: Pages 918-919 2. How is gonorrhea transmitted from a pregnant woman to her fetus? a. Unbound in the blood via the placenta b. Attached to immunoglobulin G (IgG) via the placenta c. Across amniotic membranes bNyUtRheSIdNiGreTcBt .iCnOoMculation with the fetal scalp electrodes during labor monitoring d. Predominately through infected cervical and secretions during the birth process ANS: D A pregnant woman can transmit gonorrhea to her fetus during the birth process. The infection passes from mother to child predominately through infected cervical and vaginal secretions. The transmission is not associated with the other options. PTS: 1 REF: Page 920 3. Which statement is false about the factors that facilitate the ascent of gonococci into the uterus and fallopian tubes? a. Ascent of gonococci is facilitated because the cervical plug disintegrates during menstruation. b. Ascent of gonococci is facilitated because the vaginal pH decreases to 2 or 3. c. Ascent of gonococci is facilitated because the uterine contractions may cause retrograde menstruation into the fallopian tubes. d. Ascent of gonococci is facilitated because the bacteria may adhere to sperm and be transported to the fallopian tubes. ANS: B Several factors can facilitate the ascent of gonococci into the uterus and the fallopian tubes, where they cause pelvic inflammatory disease (PID). Among these factors are (1) disintegration of the cervical mucous plug and (2) a rise in vaginal pH greater than 4.5 during menstruation. The other options are accurate statements. PTS: 1 REF: Page 921 4. In women, what is the usual site of original gonococcal infection? a. Endocervical canal c. Fallopian tube b. Vagina d. Labia majora ANS: A In women, the endocervical canal (inner portion of the cervix) is the usual site of original gonococcal infection, although urethral colonization and infection of Skene or Bartholin glands also are common. The other options are not usually associated with gonococcal infections. PTS: 1 REF: Page 921 5. What is the primary site for uncomplicated local gonococci infections in men? a. Epididymis c. Urethra b. Lymph nodes d. Prostate ANS: C Uncomplicated local infections are observed primarily as urethral infections in men. PTS: 1 REF: Page 921 NURSINGTB.COM 6. What local complication of a gonococcal infection is diagnosed in approximately 10% of affected women? a. Acute salpingitis c. Vaginitis b. Cystitis d. Cervicitis ANS: A Acute salpingitis, or pelvic inflammatory disease (PID), is the most common local complication in women. Approximately 10% of women with untreated cervical gonorrhea develop PID. PTS: 1 REF: Page 921 7. Which laboratory test is considered adequate for an accurate and reliable diagnosis of gonococcal urethritis in a symptomatic man? a. Ligase chain reaction (LCR) c. Polymerase chain reaction (PCR) b. Gram-stain technique d. DNA testing ANS: B Microscopic evaluation of Gram-stained slides of clinical specimens is deemed positive for Neisseria gonorrhoeae if gram-negative diplococci with the typical “kidney bean” morphologic appearance are found inside polymorphonuclear leukocytes. Such a finding is considered adequate for the diagnosis of gonococcal urethritis in a symptomatic man. The other options are not relevant to the diagnosis of this condition. PTS: 1 REF: Page 922 8. How does an established gonococcal infection usually express itself in newborns? a. Generalized skin rash 4 to 6 days after birth b. Systemic infection with fever c. Bilateral corneal ulceration d. Yellow vaginal or penile discharge approximately 10 days after birth ANS: C Established infection causes bilateral corneal ulceration. The other options are not usual sites for such an infection. PTS: 1 REF: Page 922 9. What is the major concern regarding the treatment of gonococci infections? a. Development of antibiotic resistance c. Changes in pathogenicity b. Changes in virulence d. Mutations into different strains ANS: A Several types of drug-resistant strains have been identified; they are penicillinase-producing Neisseria gonorrhoeae (PPNG), which is resistant to penicillin; tetracycline-resistant N. gonorrhoeae (TRNG), which is resistant to tetracycline; chromosomal control of mechanisms of resistance of N. gonorrhoeae (CMRNG), which is resistant to penicillin and tetracycline; and increasingly a fluoroquinolone-resistant N. gonorrhoeae (QRNG). The other options are not major concerns. PTS: 1 REF: Page 922 NURSINGTB.COM 10. Which sexually transmitted infection frequently coexists with gonorrhea? a. Syphilis c. Chlamydia b. Herpes simplex virus d. Chancroid ANS: C The coexistence of chlamydial infection with gonorrhea frequently occurs. No coexistence exists with the other options. PTS: 1 REF: Page 922 11. During which stage of syphilis do bloodborne bacteria spread to all the major organ systems? a. Primary c. Latent b. Secondary d. Tertiary ANS: B Bloodborne bacteria spread to all major organ systems during only stage II, secondary syphilis (see Box 26-2). PTS: 1 REF: Page 923 12. In which stage of syphilis would the following clinical manifestations be found: destructive skin, bone and soft tissue lesions, aneurysms, heart failure, and neurosyphilis? a. Primary c. Latent b. Secondary d. Tertiary ANS: D Stage IV, tertiary syphilis, is the only stage during which significant morbidity and mortality occur, including destructive skin, bone, and soft-tissue lesions (see Box 26-2). PTS: 1 REF: Page 924 13. Which organism is responsible for the development of syphilis? a. Neisseria syphilis c. Haemophilus ducreyi b. Treponema pallidum d. Chlamydia trachomatis ANS: B T. pallidum is the only cause of syphilis. PTS: 1 REF: Page 923 14. Which is a characteristic lesion of secondary syphilis? a. Condylomata lata c. Chancroid b. Gummas d. Donovan bodies ANS: A The only secondary syphilis lesion is the condylomata lata. PTS: 1 REF: Pages 924-925 15. By which method is the organism that causes syphilis best identified? a. Acid-fast stain c. In vitro culture b. Gram-stained slide ANS: D NURSINGTdB..CODMarkfield microscopy Because Treponema pallidum cannot be cultured in vitro, early definitive diagnosis of primary or secondary syphilis depends on darkfield microscopy of a specimen taken from a chancre, regional lymph node, or other lesion. The remaining options are not relevant. PTS: 1 REF: Page 925 16. When a patient has small, vesicular lesions that last between 10 and 20 days, which sexually transmitted infection is suspected? a. Genital herpes c. Syphilis b. Chancroid d. Chlamydia ANS: A If symptoms occur, the individual may have small (1 to 2 mm), multiple, vesicular lesions that are generally located on the labia minora, fourchette, or penis. They may also appear on the cervix, buttocks, and thighs and are often painful and pruritic. These lesions usually last approximately 10 to 20 days. The other options do not demonstrate these symptoms. PTS: 1 REF: Page 933 17. Which statement is false regarding the risk of transmission of the herpes simplex virus (HSV) from mother to fetus? a. Neonatal infection of HSV rarely occurs in the intrapartum or postpartum period. b. The risk is higher in women who have a primary HSV infection. c. The risk is higher in women who experience ruptured membranes more than 6 hours before delivery. d. The risk is higher when internal fetal monitoring devices are used. ANS: A Neonatal infections can occur in utero or, more commonly, during the intrapartum or postpartum period. The other options are accurate statements. PTS: 1 REF: Page 933 18. During the latent period of a herpes virus infection, where in the host cell is the genome of the virus maintained? a. Mitochondria c. Nucleus b. Lysosomes d. Cytoplasm ANS: C During the latent period, the genome for the virus is maintained in the host cell nucleus without causing the death of the cell. PTS: 1 REF: Page 933 19. During reactivation (release from latency), herpes virus genomes are transported through which nerves to the dermal surface? a. Somatic c. Autonomic b. Peripheral sensory d. Peripheral motor ANS: B NURSINGTB.COM Only during reactivation are the viral genomes transported through the peripheral sensory nerves back to the dermal surface. PTS: 1 REF: Page 933 20. Which statement provides the most accurate information regarding the transmission of herpes simplex virus (HSV)? a. HSV is transmitted only when vesicles are present. b. HSV is transmitted only while lesions are present. c. The use of condoms prevents the transmission of HSV. d. The risk of transmission is present even during latent periods. ANS: D Latent infections can become reactivated and cause a recurrent infection with similar manifestations. The other options are not accurate statements regarding the transmission of HSV. PTS: 1 REF: Pages 933-934 21. Which drug may be prescribed orally for outbreak management of herpes simplex viral (HSV) infections? a. Acyclovir (Zovirax) c. Zidovudine (AZT) (Retrovir) b. 5-Fluorouracil (5-FU) d. Bichloroacetic acid (BCA) ANS: A Although no curative treatment for HSV infection is known, only oral acyclovir, valacyclovir, penciclovir, and famciclovir are used for primary and periodic outbreaks and to prevent recurrences. PTS: 1 REF: Page 934 22. Which of the following causes condylomata acuminata or genital warts? a. Chlamydia c. Human papillomavirus (HPV) b. Adenovirus d. Herpes simplex virus 1 (HSV-1) ANS: C Genital warts are quite contagious and are a result of only HPV. PTS: 1 REF: Pages 934-935 23. Which treatment is used for trichomoniasis? a. Topical application of 5-Fluorouracil (5-FU) b. Topical application of acyclovir c. Systemic metronidazole d. Systemic tetracycline ANS: A The treatment of choice for trichomoniasis is a single 2-gram dose of metronidazole (Flagyl) or tinidazole. The other options are not applicable. PTS: 1 REF: Page 938 24. A woman diagnosed with trichomNoUniRaSsIiNs GasTkBs.CifOhMer sexual partner should be treated as well. What is the appropriate response? a. Sexual partners should be treated only if symptoms are present. b. Sexual partners should be treated even if they are asymptomatic. c. Infections in men are self-limiting; therefore a male sexual partner does not require treatment. d. Sexual partners should be treated to prevent infection. ANS: B Sexual partners, even if asymptomatic, are also treated and examined for coexisting sexually transmitted infections. PTS: 1 REF: Page 938 25. Which microorganism is sexually transmitted, primarily by homosexual men, through infected feces? a. Shigellosis c. Giardia lamblia b. Cytomegalovirus (CMV) d. Entamoeba histolytica ANS: A Only the Shigella infection, termed shigellosis, is transmitted by contact with infected feces particularly among homosexual men. PTS: 1 REF: Page 940 | Table 26-3 26. Which hepatitis virus is known to be sexually transmitted? a. A c. C b. B d. D ANS: B Only hepatitis B virus (HBV) is known to be sexually transmitted. PTS: 1 REF: Page 940 27. Which sexually transmitted disease occasionally causes clinical manifestations of scant intermittent penile discharge, slight pruritus, and mild dysuria? a. Campylobacter enteritis c. Cytomegalovirus enteritis b. Shigellosis d. Trichomoniasis ANS: D Most men with trichomoniasis are asymptomatic but may have scant intermittent discharge, slight pruritus, and mild dysuria. The other options do not demonstration these symptoms. PTS: 1 REF: Page 937 MULTIPLE RESPONSE 28. Which statements are true regarding the hepatitis B infection? (Select all that apply.) a. Hepatitis B poses a significant risk for chronic liver disease. b. Hepatocellular cancer is a common comorbid condition. c. Universal vaccination of infants and children is recommended. d. The preventive method of choNicUeRiSsIiNmGmTBun.CiOzaMtion. e. Universal vaccination of sexually active adults is recommended. ANS: A, B, C, D Hepatitis B infection poses significant health risks including chronic liver disease and hepatocellular cancer. Immunization against hepatitis B is the most effective means of preventing transmission. Universal vaccination of infants and children is recommended, as well as vaccination of high-risk adults. PTS: 1 REF: Page 941 29. What are the common modes of transmission for the hepatitis B virus (HBV)? (Select all that apply.) a. Needle punctures b. Blood transfusions c. Contact with infected body fluids d. Skin cuts e. Ingestion of infected substances ANS: A, B, C, D Transmission of HBV can occur through needle puncture, blood transfusion, cuts in the skin, and contact with infected body fluids. Ingestion is not a recognized transmission mode. PTS: 1 REF: Page 941 30. Which statements are true regarding the parasitic infection referred to as scabies? (Select all that apply.) a. Scabies is spread through skin-to-skin contact. b. The crab lice, Phthirus pubis, cause scabies. c. Severe pruritus is its major clinical manifestation. d. Symptoms worsen at night. e. Treatment is provided through oral medication therapy. ANS: A, C, D Scabies is a common parasitic infection that can be spread by skin-to-skin contact and sexual contact. The scabies mite burrows through the skin, depositing two or three large eggs per day. Intense pruritus, especially at night, is the most pronounced clinical manifestation. Treatment consists of topical application of a pediculicide. Pediculosis pubis (crabs) is commonly transmitted sexually and is caused by the crab louse, P. pubis. PTS: 1 REF: Pages 938-939 31. Which statements are true concerning the sexually transmitted infection (STI) lymphogranuloma venereum? (Select all that apply.) a. Lymphogranuloma venereum is an STI commonly diagnosed in the United States. b. It begins as a skin infection. c. Lymphogranuloma venereum spreads to lymph tissues. d. Primary and secondary lesions are apparent with this STI. e. A 60-day course of oral erythromycin is the recommended treatment. ANS: B, C, D Lymphogranuloma venereum is aNcUhRroSnINicGSTTBI.CuOnMcommon in the United States. The lesion begins as a skin infection and spreads to the lymph tissue, causing inflammation, necrosis, buboes, and abscesses of the inguinal lymph nodes. Primary lesions appear on the penis and scrotum in men and on the cervix, vaginal wall, and labia in women. Secondary lesions involve inflammation and swelling of the lymph nodes with the formation of large blue buboes that rupture and form draining ulcerative lesions. A 21-day or longer course of oral doxycycline or erythromycin is needed for treatment. Treatment of sexual partners is recommended. PTS: 1 REF: Pages 931-932 MATCHING Match the disease with the corresponding causative organism. A. Trichomoniasis B. Granuloma inguinale C. Amebiasis D. Syphilis E. Chancroid 32. Treponema pallidum 33. Haemophilus ducreyi 34. Calymmatobacterium granulomatis 35. Entamoeba histolytica 36. Trichomonas vaginalis 32. ANS: D PTS: 1 REF: Page 923 MSC: T. pallidum is the cause of syphilis. 33. ANS: E PTS: 1 REF: Page 925 MSC: H. ducreyi, a gram-negative bacillus, causes chancroid. 34. ANS: B PTS: 1 REF: Page 929 MSC: Granuloma inguinale (donovanosis) is a chronic, progressively destructive bacterial infection caused by Calymmatobacterium granulomatis. 35. ANS: C PTS: 1 REF: Page 919 | Table 26-1 MSC: E. histolytica is the cause of amebiasis. 36. ANS: A PTS: 1 REF: Page 937 MSC: Trichomoniasis is a result of infection caused by T. vaginalis. NURSINGTB.COM Chapter 28: Structure and Function of the Hematologic System MULTIPLE CHOICE 1. What is the most abundant class of plasma protein? a. Globulin c. Clotting factors b. Albumin d. Complement proteins ANS: B Albumin (approximately 60% of total plasma protein at a concentration of about 4 g/dl) is the most abundant plasma protein. PTS: 1 REF: Page 946 2. What is the effect of low plasma albumin? a. Clotting factors decrease, thus increasing the chance of prolonged bleeding. b. Fewer immunoglobulins are synthesized, thus impairing the immune function. c. Less iron is stored, thus increasing the incidence of iron deficiency anemia. d. Osmotic pressure decreases, thus water moves from the capillaries to the interstitium. ANS: D In the case of decreased production (e.g., cirrhosis, other diffuse liver diseases, protein malnutrition) or excessive loss of albumin (e.g., certain kidney diseases, extensive burns), the reduced oncotic pressure leads to excessive movement of fluid and solutes into the tissues and decreased blood volume. The other options are not accurate descriptions of the effect of low plasma albumin. NURSINGTB.COM PTS: 1 REF: Page 946 3. What is the life span of an erythrocyte (in days)? a. 20 to 30 c. 100 to 120 b. 60 to 90 d. 200 to 240 ANS: C Because it cannot undergo mitotic division, the erythrocyte has a limited life span of approximately 120 days. PTS: 1 REF: Pages 947-948 4. Which statement concerning erythrocytes is true? a. Erythrocytes contain a nucleus, mitochondria, and ribosomes. b. Erythrocytes synthesize proteins. c. Erythrocytes have the ability to change shape to squeeze through microcirculation. d. Erythrocyte colony-stimulating factor (E-CSF) stimulates erythrocytes. ANS: C Reversible deformity enables the erythrocyte to assume a more compact torpedo-like shape, squeeze through the microcirculation, and return to normal. The other options are not accurate statements about erythrocytes. PTS: 1 REF: Page 948 5. Granulocytes that contain granules of vasoactive amines, such as histamine, are called: a. Neutrophils c. Monocytes b. Eosinophils d. Basophils ANS: D Basophils contain cytoplasmic granules that hold an abundant mixture of biochemical mediators, including histamine, chemotactic factors, proteolytic enzymes, and an anticoagulant (heparin) (see Figure 27-3, C). This is not an accurate description of any of the other options. PTS: 1 REF: Page 949 6. Which of the following are formed elements of the blood that are not cells but are disk-shaped cytoplasmic fragments essential for blood clotting? a. Monocytes c. Macrophages b. Platelets d. Erythrocytes ANS: B Platelets (thrombocytes) are not true cells but are disk-shaped cytoplasmic fragments that are essential for blood coagulation and control of bleeding. This description is not accurate for any of the other options. PTS: 1 REF: Pages 950-951 7. Blood cells that differentiate into macrophages are known as: a. Monocytes NURSINGTcB..COEMosinophils b. Neutrophils d. Basophils ANS: A Only monocytes migrate into a variety of tissues and fully mature into tissue macrophages and myeloid dendritic cells (see Table 27-3). PTS: 1 REF: Page 950 8. Without prior exposure to an antigen, which cells are able to destroy some types of tumor cells and some virus-infected cells? a. Lymphocytes c. Megakaryocytes b. Plasma cells d. Natural killer (NK) cells ANS: D NK cells, which resemble large granular lymphocytes, kill some types of tumor cells (in vitro) and some virus-infected cells without being induced by previous exposure to these antigens. This capability is not true of the other options. PTS: 1 REF: Page 950 9. What is the life span of platelets (in days)? a. 10 c. 90 b. 30 d. 120 ANS: A A platelet circulates for approximately 10 days and ages. Macrophages of the mononuclear phagocyte system, mostly in the spleen, remove platelets. PTS: 1 REF: Page 951 10. Fetal hematopoiesis occurs in which structure? a. Gut c. Bone marrow b. Spleen d. Thymus ANS: B The spleen is the largest of the secondary lymphoid organs and the site of fetal hematopoiesis. PTS: 1 REF: Page 951 11. What is the consequence of a splenectomy? a. The level of iron in circulation increases. b. Antibody production increases to improve immune function. c. The number of defective cells in circulation increases. d. The number of clotting factors increases. ANS: C Splenic absence from any cause (e.g., atrophy, traumatic injury, removal because of disease) has several secondary effects on the body, among them an increase in morphologically defective blood cells in the circulation, confirming the spleen’s role in removing old or damaged cells. This description of the consequence of a splenectomy is not accurate for the other options. NURSINGTB.COM PTS: 1 REF: Pages 951-952 12. During an infection, why do lymph nodes enlarge and become tender? a. B lymphocytes proliferate. b. The nodes are inflamed. c. The nodes fill with purulent exudate. d. The nodes are not properly functioning. ANS: A The B lymphocyte proliferation in response to significant antigen (e.g., during infection) may result in lymph node enlargement and tenderness (reactive lymph node). This description is not accurate for the other options. PTS: 1 REF: Page 954 13. Which blood cells are the chief phagocytes involved in the early inflammation process? a. Neutrophils c. Eosinophils b. Monocytes d. Erythrocytes ANS: A Neutrophils are the chief phagocytes of early inflammation. PTS: 1 REF: Page 949 14. Which blood cells are biconcave in shape and have the capacity to be reversibly deformed? a. Neutrophils c. Eosinophils b. Monocytes d. Erythrocytes ANS: D The erythrocyte’s size and shape are ideally suited to its function as a gas carrier. A red blood cell (RBC) is a small disk with two unique properties: (1) a biconcave shape and (2) the capacity to be reversibly deformed. These are characteristics not observed in any of the other options. PTS: 1 REF: Page 948 15. Which hemoglobin is made from oxidized ferric iron (Fe3+) and lacks the ability to bind oxygen? a. Deoxyhemoglobin c. Methemoglobin b. Oxyhemoglobin d. Glycosylated hemoglobin ANS: C Without reactivation by methemoglobin reductase, the Fe3+-containing hemoglobin (methemoglobin) cannot bind oxygen. This capability is not true of the other types of hemoglobin mentioned. PTS: 1 REF: Page 961 16. The absence of parietal cells would prevent the absorption of an essential nutrient necessary to prevent which type of anemia? a. Iron deficiency NURSINGTcB..COFMolic acid deficiency anemia b. Pernicious anemia d. Aplastic anemia ANS: B Dietary vitamin B12 is a large molecule that requires a protein secreted by parietal cells into the stomach (intrinsic factor [IF]) to transport across the ileum. Defects in IF production lead to decreased B12 absorption and pernicious anemia. The other options are not the result of this process. PTS: 1 REF: Page 962 17. Which nutrients are necessary for the synthesis of DNA and the maturation of erythrocytes? a. Protein and niacin c. Cobalamin (vitamin B12) and folate b. Iron and vitamin B6 (pyridoxine) d. Pantothenic acid and vitamin C ANS: C Cobalamin and folate are necessary for the synthesis of DNA and for the maturation of erythrocytes. The remaining options are not necessary for these processes to occur. PTS: 1 REF: Page 962 | Table 27-6 18. Which nutrients are necessary for hemoglobin synthesis? a. Protein and niacin c. Cobalamin (vitamin B12) and folate b. Iron and vitamin B6 (pyridoxine) d. Pantothenic acid and vitamin C ANS: B Iron and B6 (pyridoxine) are necessary for hemoglobin synthesis (see Table 27-6). The remaining options are not necessary for hemoglobin synthesis. PTS: 1 REF: Page 962 | Table 27-6 19. Recycling of iron from erythrocytes is made possible by which of the following? a. Transferrin c. Apoferritin b. Hemosiderin d. Erythropoietin ANS: A Transferrin is recycled (transferrin cycle) in the following manner: (1) the transferrin-iron complex binds to a transferring receptor on the erythroblast’s plasma membrane; (2) the complex moves into the cell by receptor-mediated endocytosis; (3) iron is released (dissociated) from transferrin; and (4) the dissociated transferrin is returned to the bloodstream for reuse. The other options do not present an accurate description of the recycling of erythrocytic iron. PTS: 1 REF: Pages 963-964 20. By which structure are mature erythrocytes removed from the bloodstream? a. Liver c. Thymus b. Lymph nodes d. Spleen ANS: D After approximately 100 to 120 days in the circulation, old erythrocytes are removed by tissue macrophages, primarily in the spleen. NURSINGTB.COM PTS: 1 REF: Page 962 21. Which substance is used to correct the chronic anemia associated with chronic renal failure? a. Iron c. Cobalamin (vitamin B12) b. Erythropoietin d. Folate ANS: B One of the most significant advances in the study of hematopoietic growth factors has been the development of erythropoietin for individuals with chronic renal failure. The other options are not associated with the treatment of chronic anemia. PTS: 1 REF: Pages 960-961 22. What is the role of thromboxane A (TXA2) in the secretion stage of hemostasis? a. Stimulates the synthesis of serotonin. b. Promotes vasodilation. c. Stimulates platelet aggregation. d. Promotes formation of cyclooxygenase. ANS: C Platelet aggregation is primarily stimulated by TXA2 and adenosine diphosphate (ADP), which induce functional fibrinogen receptors on the platelet. The other options do not present an accurate description of the role of thromboxane A. PTS: 1 REF: Page 969 23. Which of the following is the role of nitric oxide (NO) in hemostasis? a. Stimulates the release of fibrinogen to maintain the platelet plug. b. Stimulates the release of clotting factors V and VII. c. Causes vasoconstriction and stimulates platelet aggregation. d. Controls platelet activation through cyclic adenosine monophosphate (cAMP)–mediated signaling. ANS: D Endothelial cell NO synthase produces NO, which controls platelet activation through cAMP-mediated signaling. The other options do not present an accurate description of the role of NO in hemostasis. PTS: 1 REF: Page 966 | Figure 27-18 24. The drug heparin acts in hemostasis by which processes? a. Inhibiting thrombin and antithrombin III (AT-III) b. Preventing the conversion of prothrombin to thrombin c. Shortening the fibrin strands to retract the blood clot d. Degrading the fibrin within blood clots ANS: A Clinically administered heparin or heparin sulfate (on the surface of endothelial cells) binds to AT-III and induces a conformational change that greatly enhances its activity. Under normal conditions, the presence of endothelial cell heparin sulfate and available AT-III in the circulation cooperateNUtoRpSrIoNtGecTtBt.hCeOMvessels from the effects of spontaneously activated thrombin. The other options do not accurately describe the role heparin plays in hemostasis. PTS: 1 REF: Page 970 25. What is plasmin’s role in the clotting process? a. Stimulates platelet aggregation. b. Inhibits platelet adhesion and aggregation. c. Prevents the conversion of prothrombin to degrade the fibrin within blood clots. d. Degrades the fibrin within blood clots. ANS: D Plasmin (also called fibrinase or fibrinolysin) is a serine protease that degrades fibrin polymers in clots. It is not capable of the functions described in the remaining options. PTS: 1 REF: Pages 971-972 26. What does polycythemia at birth indicate? a. Hypoxia in utero c. Congenitally absent spleen b. Dysfunctional bone marrow d. Dehydration in utero ANS: A The hypoxic intrauterine environment stimulates erythropoietin production in the fetus and accelerates fetal erythropoiesis, producing polycythemia (excessive proliferation of erythrocyte precursors) of the newborn. The other options are not related to polycythemia. PTS: 1 REF: Page 975 27. Where are Kupffer cells located? a. Kidneys c. Pancreas b. Liver d. Spleen ANS: B The liver macrophages are the only location for Kupffer cells. PTS: 1 REF: Page 950 | Page 962 | Table 27-3 28. Where are Langerhans cells found? a. Skin c. Kidney b. Intestinal lining d. Thyroid ANS: A Of the available options, only the skin is the location for Langerhans cells. PTS: 1 REF: Page 950 | Table 27-3 29. What is the role of collagen in the clotting process? a. Initiates the clotting cascade. c. Stimulates fibrin. b. Activates platelets. d. Deactivates fibrinogen. ANS: B NURSINGTB.COM In the clotting process, collagen provides a particularly strong stimulus to activate platelets. Collagen does not bring about any of the other options. PTS: 1 REF: Page 969 30. Which form of iron (Fe) can be used in the formation of normal hemoglobin? a. Fe+ c. Fe3+ b. Fe2+ d. Fe4+ ANS: B It is crucial that the iron be correctly charged; only reduced ferrous iron (Fe2+) can bind oxygen in the lungs and release it in the tissues. PTS: 1 REF: Page 961 31. Where are alveolar macrophages found? a. Skin c. Gastrointestinal tract b. Breasts d. Lungs ANS: D The lung is the only location for alveolar macrophages. PTS: 1 REF: Page 950 | Table 27-3 32. What changes to the hematologic system is related to age? a. Platelet adhesiveness decreases. b. Lymphocyte function decreases. c. Cellular immunity increases. d. Erythrocyte reproduction accelerates. ANS: B Blood composition changes little with age. A delay in erythrocyte replenishment may occur after bleeding, presumably because of iron deficiency. Lymphocyte function appears to decrease with age. Particularly affected is a decrease in cellular immunity. Platelet adhesiveness probably increases with age. PTS: 1 REF: Page 975 33. What is the function of erythrocytes? a. Tissue oxygenation c. Infection control b. Hemostasis d. Allergy response ANS: A Erythrocytes are solely responsible for tissue oxygenation. PTS: 1 REF: Pages 947-948 MULTIPLE RESPONSE 34. Which characteristics allow erythrocytes to function as gas carriers? (Select all that apply.) a. Permanent shape b. Compactness c. Reversible deformability NURSINGTB.COM d. Presence of hyperactive mitochondria e. Biconcavity ANS: C, E A red blood cell (RBC) is a small disk with two unique properties: (1) a biconcave shape and (2) the capacity to be reversibly deformed. The other options are not relevant to the function of gas transport. PTS: 1 REF: Page 948 35. Which statements about plasma proteins are correct? (Select all that apply.) a. Provide clotting factors. b. Transport triglycerides. c. Synthesize complement proteins. d. Create hydrostatic pressure. e. Transport cholesterol. ANS: A, B, C, E Plasma proteins do not create hydrostatic pressure. The other options are all accurate statements regarding plasma proteins. PTS: 1 REF: Pages 945-947 36. What are the primary anticoagulant mechanisms? (Select all that apply.) a. Antithrombin III b. Tissue factor pathway inhibitor c. Hematopoiesis d. Protein C e. Phagocytosis ANS: A, B, D The major regulatory factors that control hemostasis reside where the greatest probability of clotting would occur—on the endothelial cell surface. The primary anticoagulant mechanisms include thrombin inhibitors (e.g., antithrombin III), tissue factor inhibitors (e.g., tissue factor pathway inhibitor), and mechanisms for degrading activated clotting factors (e.g., protein C). Hematopoiesis and phagocytosis are processes that are not related to anticoagulation. PTS: 1 REF: Page 970 37. Which statements are true regarding the role of the endothelium in clot formation? (Select all that apply.) a. The surface of the endothelium produces plasma protease inhibitors. b. Plasma protease inhibitors assist in preventing clot formation. c. Thrombomodulin is a protein that is converted on the surface of endothelial cells. d. Protein A binds to thrombomodulin. e. Activated protein C enhances the adhesion ability of neutrophils. ANS: A, B, C The surface of the endothelium produces plasma protease inhibitors to resist clot formation. Thrombomodulin is a mNUemRSbIrNaGneTBth.CroOmMbin-binding protein matter and is converted to activated protein C (see Figure 27-18) on the surface of endothelial cells. Protein C in the circulation binds to thrombomodulin. Activated protein C inhibits the adhesion of neutrophils to the endothelium. PTS: 1 REF: Pages 970-971 38. Which statements characterize albumin? (Select all that apply.) a. Retains sodium to maintain water balance. b. Provides colloid osmotic pressure. c. Is synthesized in the liver. d. Is a carrier for drugs that have low water solubility. e. Is a small molecule ANS: B, C, D Albumin is a plasma protein produced by the liver. It serves as a carrier molecule for the normal components of blood, as well as for drugs that have low solubility in water (e.g., free fatty acids, lipid-soluble hormones, thyroid hormones, bile salts). Albumin molecules are large and do not diffuse freely through the vascular endothelium, thus they maintain the critical colloidal osmotic pressure (or oncotic pressure) that regulates the passage of water and solutes into the surrounding tissues (see Chapters 1 and 3). PTS: 1 REF: Page 946 MATCHING Match the descriptions with the corresponding terms. A. Clotting B. Red blood cell development C. Red blood cell destruction D. Platelet formation E. Blood cell production 39. Endomitosis 40. Hemostasis 41. Hematopoiesis 42. Erythropoiesis 43. Phagocytosis 39. ANS: D PTS: 1 REF: Page 965 MSC: During thrombopoiesis, the megakaryocyte progenitor is programmed to undergo an endomitotic cell cycle called endomitosis, during which DNA replication of platelets occurs. 40. ANS: A PTS: 1 REF: Page 965 MSC: Hemostasis is defined as arrest of bleeding. 41. ANS: E PTS: 1 REF: Page 954 MSC: Blood cell production (hematopoiesis) is ongoing, occurring in the liver and spleen of the fetus and only in bone marrow (medullary hematopoiesis) after birth. 42. ANS: B PTS: 1 REF: Page 959 MSC: It was not until the 1850s that the bone marrow was identified as the site of erythropoiesis, or the development of red blood cells. 43. ANS: C PTS: 1 REF: Page 950 MSC: Monocytes and macrophages NarUeRaScItNivGeTpBh.aCgOocMytes that participate in the immune and inflammatory responses. They also ingest dead or defective host cells, particularly blood cells. Chapter 29: Alterations of Erythrocyte Platelet, Hemostatic Function MULTIPLE CHOICE 1. What term is used to describe the capacity of some erythrocytes to vary in size, especially in relationship to some anemias? a. Poikilocytosis c. Anisocytosis b. Isocytosis d. Microcytosis ANS: C Additional descriptors of erythrocytes associated with some anemias include anisocytosis (assuming various sizes) or poikilocytosis (assuming various shapes) (see Figure 28-1). The remaining terms are not associated with this condition. PTS: 1 REF: Page 982 2. What is the fundamental physiologic manifestation of anemia? a. Hypotension c. Hypoxia b. Hyperesthesia d. Ischemia ANS: C The fundamental physiologic manifestation of anemia is a reduced oxygen-carrying capacity of the blood, resulting in tissue hypoxia. PTS: 1 REF: Page 982 | Page 985 3. The paresthesia that occurs in vitNamURinSIBN1G2TdBe.fCiOciMency anemia is a result of which of the following? a. Reduction in acetylcholine receptors in the postsynaptic nerves b. Myelin degeneration in the spinal cord c. Destruction of myelin in peripheral nerves d. Altered function of neurons in the parietal lobe ANS: B Effects on the nervous system can occur if a vitamin B12 deficiency causes the anemia. Myelin degeneration may occur with the resultant loss of fibers in the spinal cord, producing paresthesia (numbness), gait disturbances, extreme weakness, spasticity, and reflex abnormalities. This selection is the only option that accurately describes the cause of paresthesia in such anemias. PTS: 1 REF: Page 985 | Page 987 4. Which of the following describes how the body compensates for anemia? a. Increasing rate and depth of breathing b. Decreasing capillary vasoconstriction c. Hemoglobin holding more firmly onto oxygen d. Kidneys releasing more erythropoietin ANS: A Tissue hypoxia creates additional demands and compensatory actions on the pulmonary and hematologic systems. The rate and depth of breathing increase in an attempt to increase the availability of oxygen. This selection is the only option that accurately describes the compensation mechanism in such anemias. PTS: 1 REF: Page 985 5. Which of the following is classified as a megaloblastic anemia? a. Iron deficiency c. Sideroblastic b. Pernicious d. Hemolytic ANS: B Pernicious anemia is the most common type of megaloblastic anemia. The remaining options are not classified as megaloblastic anemias. PTS: 1 REF: Pages 987-988 6. Deficiencies in folate and vitamin B12 alter the synthesis of which of the following? a. RNA c. DNA b. Cell membrane d. Mitochondria ANS: C Deficiencies in folate and vitamin B12 result in defective erythrocyte precursor DNA synthesis. These deficiencies are not associated with alterations of the other options. PTS: 1 REF: Page 987 7. The underlying disorder of whichNaUneRmSIiNaGisTBa.rCeOsuMlt of the defective secretion of the intrinsic factor, which is essential for the absorption of vitamin B12? a. Microcytic c. Hypochromic b. Pernicious d. Hemolytic ANS: B Vitamin B12 deficiency causes pernicious anemia, the most common type of megaloblastic anemia. PTS: 1 REF: Pages 987-988 8. After a person has a subtotal gastrectomy for chronic gastritis, which type of anemia will result? a. Iron deficiency c. Folic acid b. Aplastic d. Pernicious ANS: D From the options available, only pernicious anemia is caused by vitamin B12 deficiency, which is often associated with the end-stage type A chronic atrophic gastritis. PTS: 1 REF: Page 988 9. What causes the atrophy of gastric mucosal cells that result in pernicious anemia? a. Erythrocyte destruction c. Vitamin B12 malabsorption b. Folic acid malabsorption d. Poor nutritional intake ANS: C Deficiency in intrinsic factor (IF) secretion may be congenital or may result from adult onset gastric mucosal atrophy and the destruction of parietal cells. In older adults, virtually all vitamin B12-deficiency anemia is caused by a failure of IF-related absorption. This selection is the only option that accurately identifies the cause of gastric mucosal cell atrophy. PTS: 1 REF: Page 988 10. Which statement best describes a Schilling test? a. Administration of radioactive cobalamin and the measurement of its excretion in the urine to test for vitamin B12 deficiency b. Measurement of antigen-antibody immune complexes in the blood to test for hemolytic anemia c. Measurement of serum ferritin and total iron-binding capacity in the blood to test for iron deficiency anemia d. Administration of folate and measurement in 2 hours of its level in a blood sample to test for folic acid deficiency anemia. ANS: A The Schilling test indirectly evaluates vitamin B12 absorption by administering radioactive B12 and measuring excretion in the urine. This selection is the only option that accurately describes a Schilling test. PTS: 1 REF: Page 988 11. What is the treatment of choice for pernicious anemia (PA)? a. Cyanocobalamin by oral intakNeURSINGTB.COM b. Vitamin B12 by injection c. Ferrous fumarate by Z-track injection d. Folate by oral intake ANS: B Replacement of vitamin B12 (cobalamin) is the treatment of choice for PA. Initial injections of vitamin B12 are administered weekly until the deficiency is corrected, followed by monthly injections for the remainder of the individual’s life. The other options are not treatments for PA. PTS: 1 REF: Page 988 12. Which condition resulting from untreated pernicious anemia (PA) is fatal? a. Brain hypoxia c. Heart failure b. Liver hypoxia d. Renal failure ANS: C Of the options available, untreated PA is fatal, usually because of heart failure. PTS: 1 REF: Page 989 13. How is the effectiveness of vitamin B12 therapy measured? a. Reticulocyte count c. Hemoglobin b. Serum transferring d. Serum vitamin B12 ANS: A The effectiveness of cobalamin replacement therapy is determined by a rising reticulocyte count. The other options are not used as indicators of the effectiveness of vitamin B12 therapy PTS: 1 REF: Pages 988-989 14. Which statement about folic acid is false? a. Folic acid absorption is dependent on the enzyme folacin. b. Folic acid is stored in the liver. c. Folic acid is essential for RNA and DNA synthesis within erythrocytes. d. Folic acid is absorbed in the upper small intestine. ANS: A Folic acid absorption is not dependent on the enzyme folacin. The other options are true statements regarding folic acid. PTS: 1 REF: Page 989 15. Which anemia produces small, pale erythrocytes? a. Folic acid c. Iron deficiency b. Hemolytic d. Pernicious ANS: C The microcytic-hypochromic anemias, which include iron deficiency anemia (IDA), are characterized by erythrocytes that are abnormally small and contain abnormally reduced amounts of hemoglobin. This description is not true of the other options. NURSINGTB.COM PTS: 1 REF: Pages 989-990 16. Which type of anemia is characterized by fatigue, weakness, and dyspnea, as well as conjunctiva of the eyes and brittle, concave nails? a. Pernicious c. Aplastic b. Iron deficiency d. Hemolytic ANS: B Early symptoms of iron deficiency anemia (IDA) include fatigue, weakness, and shortness of breath. Pale earlobes, palms, and conjunctivae (see Figure 28-4) are also common signs. Progressive IDA causes more severe alterations, with structural and functional changes apparent in epithelial tissue (see Figure 28-4). The nails become brittle, thin, coarsely ridged, and spoon-shaped or concave (koilonychia) as a result of impaired capillary circulation. The tongue becomes red, sore, and painful. These symptoms are not associated with the other options. PTS: 1 REF: Pages 990-991 17. What is the most common cause of iron deficiency anemia (IDA)? a. Decreased dietary intake c. Vitamin deficiency b. Chronic blood loss d. Autoimmune disease ANS: B The most common cause of IDA in well-developed countries is pregnancy and chronic blood loss. PTS: 1 REF: Page 990 18. Continued therapy of pernicious anemia (PA) generally lasts how long? a. 6 to 8 weeks c. Until the iron level is normal b. 8 to 12 months d. The rest of one’s life ANS: D Because PA cannot be cured, maintenance therapy is a life-long endeavor. PTS: 1 REF: Pages 988-989 19. Sideroblastic anemia can occasionally result from an autosomal recessive transmission inherited from which relative? a. Mother c. Grandfather b. Father d. Grandmother ANS: A An occasional autosomal recessive transmission occurs only with mitochondrial mutations from the mother. PTS: 1 REF: Page 992 20. Clinical manifestations of mild-to-moderate splenomegaly and hepatomegaly, bronze-colored skin, and cardiac dysrhythmias are indicative of which anemia? a. Iron deficiency NURSINGTcB..COSMideroblastic b. Pernicious d. Aplastic ANS: C Of the options available, only sideroblastic anemia exhibits mild-to-moderate enlargement of the spleen (splenomegaly) and liver (hepatomegaly), as well as abnormal skin pigmentation (bronze colored). Heart rhythm disturbances, along with congestive heart failure, are major life-threatening complications related to cardiac iron overload. PTS: 1 REF: Page 992 21. Considering sideroblastic anemia, what would be the expected effect on the plasma iron levels? a. Plasma iron levels would be high. b. Levels would be low. c. Levels would be normal. d. Levels would be only minimally affected. ANS: A Plasma iron levels would be high (see Table 28-3). PTS: 1 REF: Page 986 | Table 28-3 22. In aplastic anemia (AA), pancytopenia develops as a result of which of the following? a. Suppression of erythropoietin to produce adequate amounts of erythrocytes b. Suppression of the bone marrow to produce adequate amounts of erythrocytes, leukocytes, and thrombocytes c. Lack of DNA to form sufficient quantities of erythrocytes, leukocytes, and thrombocytes d. Lack of stem cells to form sufficient quantities of leukocytes ANS: B AA is a critical condition characterized by pancytopenia, which is a reduction or absence of all three blood cell types, resulting from the failure or suppression of bone marrow to produce adequate amounts of blood cells. This selection is the only option that accurately identifies the cause of pancytopenia. PTS: 1 REF: Page 993 23. What is the most common pathophysiologic process that triggers aplastic anemia (AA)? a. Autoimmune disease against hematopoiesis by activated cytotoxic T (Tc) cells b. Malignancy of the bone marrow in which unregulated proliferation of erythrocytes crowd out other blood cells c. Autoimmune disease against hematopoiesis by activated immunoglobulins d. Inherited genetic disorder with recessive X-linked transmission ANS: A Most cases of AA result from an autoimmune disease directed against hematopoietic stem cells. Tc cells appear to be the main culprits. None of the remaining options is considered a common trigger of AA. PTS: 1 REF: Page 995 NURSINGTB.COM 24. An allogenic bone marrow transplantation remains the preferred method for treating which anemia? a. Polycythemia vera c. Sideroblastic b. Aplastic d. Anemia of chronic disease (ACD) ANS: B Of the options available, bone marrow and, most recently, peripheral blood stem cell transplantation from a histocompatible sibling are the preferred treatments for the underlying bone marrow failure observed in aplastic anemias. PTS: 1 REF: Page 995 25. Which statement is true regarding warm autoimmune hemolytic anemia? a. Warm autoimmune hemolytic anemia occurs primarily in men. b. It is self-limiting and rarely produces hemolysis. c. Erythrocytes are bound to macrophages and sequestered in the spleen. d. Immunoglobulin M coats erythrocytes and binds them to receptors on monocytes. ANS: C The immunoglobulin G–coated red blood cells bind to the Fc receptors on monocytes and splenic macrophages and are removed by phagocytosis. The other options are not true when considering this type of anemia. PTS: 1 REF: Page 998 26. When considering hemolytic anemia, which statement is true regarding the occurrence of jaundice? a. Erythrocytes are destroyed in the spleen. b. Heme destruction exceeds the liver’s ability to conjugate and excrete bilirubin. c. The patient has elevations in aspartate transaminase (AST) and alanine transaminase (ALT). d. The erythrocytes are coated with an immunoglobulin. ANS: B Jaundice (icterus) is present when heme destruction exceeds the liver’s ability to conjugate and excrete bilirubin. This selection is the only option that accurately describes the process that affects the occurrence of hemolytic anemia–related jaundice. PTS: 1 REF: Page 1000 27. Erythrocyte life span of less than 120 days, ineffective bone marrow response to erythropoietin, and altered iron metabolism describe the pathophysiologic characteristics of which type of anemia? a. Aplastic c. Anemia of chronic disease b. Sideroblastic d. Iron deficiency ANS: C Anemia of chronic disease results from a combination of (1) decreased erythrocyte life span, (2) suppressed production of erythropoietin, (3) ineffective bone marrow erythroid progenitor response to erythropoietin, and (4) altered iron metabolism and iron sequestration in macrophages. This result is not true of the other options. NURSINGTB.COM PTS: 1 REF: Page 1001 28. What is the primary cause of the symptoms of polycythemia vera? a. Decreased erythrocyte count c. Increased blood viscosity b. Destruction of erythrocytes d. Neurologic involvement ANS: C As polycythemia vera progresses, many of the symptoms are related to the increased blood cellularity and viscosity. No other option is the primary cause of the symptoms of polycythemia vera. PTS: 1 REF: Page 1003 29. Treatment for polycythemia vera involves which of the following? a. Therapeutic phlebotomy and radioactive phosphorus b. Restoration of blood volume by plasma expanders c. Administration of cyanocobalamin d. Blood transfusions ANS: A In low-risk individuals, the recommended therapy is phlebotomy and low-dose aspirin, whereas radioactive phosphorus has been used to suppress erythropoiesis. The other options are not considered in the treatment of polycythemia vera. PTS: 1 REF: Pages 1003-1004 30. Considering iron replacement therapy prescribed for iron deficiency anemia, who is likely to require long-term daily maintenance dosage? a. A woman who has not yet experienced menopause b. A teenager who is involved in strenuous athletics c. A middle-aged man who smokes two packs of cigarettes a day d. An older person demonstrating signs of dementia ANS: A Menstruating women may need daily oral iron replacement therapy (325 mg/day) until menopause as a result of their menstrual blood loss. None of the other options are a chronic source of blood loss. PTS: 1 REF: Page 991 31. Which statement is true regarding the physical manifestations of vitamin B12 deficiency anemia? a. Vitamin B12 deficiency anemia seldom results in neurologic symptoms. b. The chances of a cure are good with appropriate treatment. c. The condition is reversible in 75% of the cases. d. Symptoms are a result of demyelination. ANS: D The neurologic manifestations characteristic of vitamin B12 deficiency anemia result from nerve demyelination that may produce neuronal death. These complications pose a serious threat because they are not reversible, even with appropriate treatment. NURSINGTB.COM PTS: 1 REF: Page 988 MULTIPLE RESPONSE 32. A 2000 ml blood loss will produce which assessment finding? (Select all that apply.) a. Air hunger b. Normal blood pressure in the supine position c. Rapid thready pulse d. Cold clammy skin e. lactic acidosis ANS: A, C, D With a 2000 ml loss of blood, central venous pressure, cardiac output, and arterial blood pressure are below normal, even when at rest and in the supine position. The person commonly has air hunger; a rapid, thready pulse; and cold, clammy skin. With a 1500 ml loss of blood, supine blood pressure and pulse can still be normal. Lactic acidosis is observed with a blood loss of 2500 ml or more. PTS: 1 REF: Page 996 | Table 28-5 33. Which medications are associated with an intermediate increase in a person’s risk for developing aplastic anemia? (Select all that apply.) a. Penicillin b. Chloramphenicol (Chloromycetin) c. Phenytoin (Dilantin) d. Trimethoprim-sulfamethoxazole (Bactrim) e. Thiazides ANS: B, C, D Chloramphenicol (Chloromycetin), phenytoin (Dilantin), and trimethoprim-sulfamethoxazole (Bactrim) are associated with an intermediate increase in the risk of developing aplastic anemia. The other options are not associated with a rare increase in risk. PTS: 1 REF: Page 994 | Table 28-4 34. Which conditions are generally included in the symptoms of pernicious anemia (PA)? (Select all that apply.) a. Weakness b. Weight gain c. Low hemoglobin d. Paresthesias e. Low hematocrit ANS: A, C, D, E When the hemoglobin and hematocrit levels in the blood have significantly decreased, the individual experiences the classic symptoms of PA—weakness, fatigue, paresthesias of the feet and fingers, difficulty in walking, loss of appetite, abdominal pains, and weight loss. PTS: 1 REF: Page 986 | Page 988 | Table 28-3 NURSINGTB.COM 35. What are the clinical manifestations of folate deficiency anemia? (Select all that apply.) a. Constipation b. Flatulence c. Dysphagia d. Stomatitis e. Cheilosis ANS: B, C, D, E Specific symptoms of folate deficiency anemia include severe cheilosis (scales and fissures of the lips and corners of the mouth), stomatitis (inflammation of the mouth), and painful ulcerations of the buccal mucosa and tongue. Gastrointestinal symptoms may be present and include dysphagia (difficulty swallowing), flatulence, and watery diarrhea. PTS: 1 REF: Page 989 36. Which diseases are commonly associated with anemia of chronic disease? (Select all that apply.) a. Rheumatoid arthritis b. Acquired immunodeficiency syndrome (AIDS) c. Polycythemia vera d. Systemic lupus erythematosus e. Chronic hepatitis ANS: A, B, D, E AIDS, rheumatoid arthritis, systemic lupus erythematosus, malaria, acute and chronic hepatitis, and chronic renal failure are commonly associated with anemias of chronic disease. Polycythemia vera is not associated with this form of anemia. PTS: 1 REF: Page 1001 MATCHING Match the phrases with the corresponding terms. Options may be used more than once. A. Normocytic-normochromic anemia B. Microcytic-hypochromic anemia C. Macrocytic-normochromic anemia 37. Pernicious anemia 38. Sideroblastic anemia 39. Aplastic anemia 37. ANS: C PTS: 1 REF: Pages 987-988 MSC: Pernicious anemia, a form of macrocytic-normochromic anemia, is caused by vitamin B12 deficiency. 38. ANS: B PTS: 1 REF: Page 989 MSC: The microcytic-hypochromic anemias include sideroblastic anemia. 39. ANS: A PTS: 1 REF: Page 993 MSC: Normocytic-normochromic anemias, including aplastic anemia, are characterized by erythrocytes that are relatively normal in size but with hemoglobin content that is insufficient in number. NURSINGTB.COM Chapter 30: Alterations of Leukocyte, Lymphoid Function MULTIPLE CHOICE 1. What change is observed in leukocytes during an allergic disorder (type I hypersensitivity) often caused by asthma, hay fever, and drug reactions? a. Neutrophilia c. Eosinophilia b. Basophilia d. Monocytosis ANS: C Eosinophilia is an absolute increase (more than 450/µL) in the total numbers of circulating eosinophils. Allergic disorders (type I hypersensitivity) associated with asthma, hay fever, and drug reactions, as well as parasitic infections (particularly with metazoal parasites), are often cited as causes. This change is not identified by any of other options. PTS: 1 REF: Pages 1010-1011 2. In infectious mononucleosis (IM), what does the Monospot test detect? a. Immunoglobulin E (IgE) c. Immunoglobulin G (IgG) b. Immunoglobulin M (IgM) d. Immunoglobulin A (IgA) ANS: B Heterophile antibodies are a heterogeneous group of IgM antibodies that are agglutinins against nonhuman red blood cells (e.g., sheep, horse) and are detected by qualitative (monospot) or quantitative (heterophile antibody) test methods. This statement is not true of the other options. NURSINGTB.COM PTS: 1 REF: Pages 1012-1013 3. Which description is consistent with acute lymphocytic leukemia (ALL)? a. ALL is a progressive neoplasm defined by the presence of greater than 30% lymphoblasts in the bone marrow or blood. b. Leukocytosis and a predominance of blast cells characterize the bone marrow and peripheral blood. As the immature blasts increase, they replace normal myelocytic cells, megakaryocytes, and erythrocytes. c. B cells fail to mature into plasma cells that synthesize immunoglobulins. d. The translocation of genetic material from genes 9 and 22 create an abnormal, fused gene identified as BCR-ABL. ANS: A ALL is a progressive neoplasm defined by the presence of greater than 30% lymphoblasts in the bone marrow or blood. None of the other options provide an accurate description of ALL. PTS: 1 REF: Page 1016 4. Which description is consistent with chronic myelogenous leukemia (CML)? a. Defects exist in the ras oncogene, TP53 tumor-suppressor gene, and INK4A, the gene encoding a cell-cycle regulatory protein. b. Leukocytosis and a predominance of blast cells characterize the bone marrow and peripheral blood. As the immature blasts increase, they replace normal myelocytic cells, megakaryocytes, and erythrocytes. c. B cells fail to mature into plasma cells that synthesize immunoglobulins. d. The translocation of genetic material from genes 9 and 22 creates an abnormal, fused protein identified as BCR-ABL1. ANS: D The Philadelphia chromosome is present in more than 95% of those with CML, and the presence of the BCR-ABL1 protein is responsible for the initiation of CML. The other options do not accurately describe CML. PTS: 1 REF: Pages 1014-1015 | Page 1021 5. Which description is consistent with chronic lymphocytic leukemia (CLL)? a. Defects exist in the ras oncogene, TP53 tumor-suppressor gene, and INK4A, the gene encoding a cell-cycle regulatory protein. b. Leukocytosis and a predominance of blast cells characterize the bone marrow and peripheral blood. As the immature blasts increase, they replace normal myelocytic cells, megakaryocytes, and erythrocytes. c. B cells fail to mature into plasma cells that synthesize immunoglobulins. d. The translocation of genetic material from genes 9 and 22 creates an abnormal, fused protein identified as BCR-ABL. ANS: C CLL is derived from transformation of a partially mature B cell that has not yet encountered antigen. The other options do not accurately describe CLL. PTS: 1 REF: Pages 1N0U20R-S1I0N2G1TB.COM 6. Which electrolyte imbalance accompanies multiple myeloma (MM)? a. Hyperkalemia c. Hyperphosphatemia b. Hypercalcemia d. Hypernatremia ANS: B Elevated levels of calcium in the blood (hypercalcemia) characterize the common presentation of MM. The other options do not accompany MM. PTS: 1 REF: Page 1032 7. Reed-Sternberg (RS) cells represent malignant transformation and proliferation of which of the following? a. Interleukin (IL)–1, IL-2, IL-5, and IL-6 b. Tumor necrosis factor–beta c. B cells d. T cells ANS: C Although the molecular events that cause malignant transformation remain controversial, RS cells are apparently from B-cell lineage. The other options are not relevant to this process. PTS: 1 REF: Pages 1024-1025 8. Local signs and symptoms of Hodgkin disease–related lymphadenopathy are a result of which of the following? a. Pressure and ischemia c. Inflammation and ischemia b. Pressure and obstruction d. Inflammation and pressure ANS: B Local symptoms caused by pressure and obstruction of the lymph nodes are the result of lymphadenopathy. The other options do not contribute to the lymphadenopathy associated with Hodgkin disease. PTS: 1 REF: Page 1025 9. Which virus is associated with Burkitt lymphoma in African children? a. Cytomegalovirus c. Human papillomavirus b. Adenovirus d. Epstein-Barr virus ANS: D Epstein-Barr virus, found in nasopharyngeal secretions, is associated with Burkitt lymphoma in African children. The other options are not associated with this malignancy. PTS: 1 REF: Page 1029 10. Which term is used to describe a red-purple discoloration caused by diffuse hemorrhage into the skin tissue? a. Petechiae c. Ecchymosis b. Hematoma d. Purpura ANS: D NURSINGTB.COM Diffuse hemorrhage into skin tissues that is visible through the skin causes a red-purple discoloration identified as a purpura. None of the other options are used to identify this symptom. PTS: 1 REF: Page 1037 11. Which statement best describes heparin-induced thrombocytopenia (HIT)? a. Immunoglobulin G immune–mediated adverse drug reaction that reduces circulating platelets b. Hematologic reaction to heparin in which the bone marrow is unable to produce sufficient platelets to meet the body’s needs c. Immunoglobulin E–mediated allergic drug reaction that reduces circulating platelets d. Cell-mediated drug reaction in which macrophages process the heparin and platelet complexes that are then destroyed by activated cytotoxic T cells. ANS: A Heparin is a common cause of drug-induced thrombocytopenia. HIT is an immune-mediated, adverse drug reaction caused by immunoglobulin G antibodies that leads to increased platelet consumption and a decrease in platelet counts. None of the other options accurately describe HIT. PTS: 1 REF: Page 1038 12. Immune thrombocytopenia (ITP) is a(n) condition in adults and a(n) condition in children. a. Acute; acute c. Acute; chronic b. Chronic; chronic d. Chronic; acute ANS: D ITP may be acute or chronic. The acute form is frequently observed in children. Chronic ITP is more commonly observed in adults, with the highest prevalence in women between 20 and 40 years of age. PTS: 1 REF: Pages 1038-1039 13. Vitamin is required for normal clotting factor synthesis by the . a. K; kidneys c. K; liver b. D; kidneys d. D; liver ANS: C Vitamin K, a fat-soluble vitamin, is necessary for the synthesis and regulation of prothrombin, procoagulant factors (VII, IX, X), and anticoagulant regulators (proteins C and S) in the liver. PTS: 1 REF: Pages 1042-1043 14. What is the most common cause of vitamin K deficiency? a. Administration of warfarin (Coumadin) b. Total parenteral nutrition with antibiotic therapy c. An immunoglobulin G–mediaNteUdRaSuINtoGimTBm.CuOneMdisorder d. Liver failure ANS: B The most common cause of vitamin K deficiency is parenteral nutrition in combination with broad-spectrum antibiotics that destroy normal gut flora. None of the other options are commonly associated with vitamin K deficiency. PTS: 1 REF: Page 1043 15. Which disorder is described as an unregulated release of thrombin with subsequent fibrin formation and accelerated fibrinolysis? a. Disseminated intravascular coagulation (DIC) b. Immune thrombocytopenic purpura (ITP) c. Heparin-induced thrombocytopenia (HIT) d. Essential thrombocythemia (ET) ANS: A DIC is an acquired clinical syndrome characterized by widespread activation of coagulation resulting in the formation of fibrin clots in medium and small vessels throughout the body. This description does not accurately identify any of the other options. PTS: 1 REF: Page 1043 16. In disseminated intravascular coagulation (DIC), what activates the coagulation cascade? a. Cytokines, such as platelet-activating factor (PAF), and tumor necrosis factor- alpha (TNF-) b. Thromboxane A, causing platelets to aggregate and consume clotting factors c. Tissue factor (TF) located in the endothelial layer of blood vessels and subcutaneous tissue d. Endotoxins from gram-negative and gram-positive bacteria circulating in the bloodstream ANS: C Direct tissue damage (ischemia and necrosis, surgical manipulation, crushing injury) causes the endothelium to release TF. The common pathway for DIC appears to be excessive and widespread exposure of TF. The other options are not responsible for the activation of the coagulation cascade. PTS: 1 REF: Pages 1043-1044 17. Which proinflammatory cytokines are responsible for the development and maintenance of disseminated intravascular coagulation (DIC)? a. Granulocyte colony-stimulating factor (G-CSF); interleukin (IL)–2, IL-4, and IL-10; and tumor necrosis factor-gamma (IFN-) b. Granulocyte-macrophage colony-stimulating factor (GM-CSF); and IL-3, IL-5, IL-9, and IFN- c. Macrophage colony-stimulating factor (M-CSF); IL-7, IL-11, and IL-14; and PAF d. Tumor necrosis factor-alpha (TNF-); IL-1, IL-6, and IL-8; and platelet-activating factor (PAF) ANS: D NURSINGTB.COM Endotoxin, in particular, triggers the release of multiple cytokines that play a significant role in the development and maintenance of DIC. Proinflammatory cytokines—TNF-; IL-1, IL-6, IL-8; PAF—are responsible for the clinical signs and symptoms associated with the sepsis associated with DIC. None of the other options perform this function. PTS: 1 REF: Page 1044 18. In disseminated intravascular coagulation (DIC), what are the indications of microvascular thrombosis? a. Reduced amplitude in peripheral pulses b. Symmetric cyanosis of fingers and toes c. Numbness and tingling in fingers and toes d. Bilateral pallor and edema of fingers and toes ANS: B Several organ systems are susceptible to microvascular thrombosis that affects their function. Indicators of multisystem failure include changes in the level of consciousness, behavior, and mentation; confusion; seizure activity; oliguria; hematuria; hypoxia; hypotension; hemoptysis; chest pain; and tachycardia. Symmetric cyanosis of the fingers and toes (i.e., “blue finger/toe syndrome”) and, in some instances, of the nose and breasts may be present. The other options are not recognized indicators of microvascular thrombosis. PTS: 1 REF: Page 1046 19. What is the most reliable and specific test for diagnosing disseminated intravascular coagulation (DIC)? a. Prothrombin time (PT) b. Activated partial thromboplastin time (aPTT) c. Fibrin degradation products (FDP) d. D-dimer ANS: D D-dimer testing measures a specific DIC-related product. This statement is not true of the other options. PTS: 1 REF: Page 1047 20. What term is used to identify thrombi that occlude arterioles and capillaries and are made up of platelets with minimal fibrin and erythrocytes? a. Essential (primary) thrombocythemia (ET) b. Acute idiopathic thrombotic thrombocytopenic purpura c. Thrombotic thrombocytopenic purpura (TTP) d. Immune thrombocytopenic purpura (ITP) ANS: C Of the available options, only TTP is characterized by thrombotic microangiopathy in which platelets aggregate and cause occlusion of arterioles and capillaries in the microcirculation. PTS: 1 REF: Page 1040 NURSINGTB.COM 21. Which of the following is characterized by what is referred to as pathognomonic pentad of symptoms? a. Acute idiopathic thrombotic thrombocytopenic purpura b. Essential (primary) thrombocythemia (ET) c. Immune thrombocytopenic purpura (ITP) d. Thrombotic thrombocytopenic purpura (TTP) ANS: A Acute idiopathic thrombotic thrombocytopenic purpura is characterized by a pathognomonic pentad of symptoms that includes extreme thrombocytopenia (fewer than 20,000 platelets/mm3), intravascular hemolytic anemia, ischemic signs and symptoms most often involving the central nervous system (approximately 65% exhibit memory disturbances, behavioral irregularities, headaches, or coma), kidney failure (affecting approximately 65% of individuals), and fever (present in approximately 33% of individuals The other options do not demonstrate these symptoms. PTS: 1 REF: Page 1040 22. Which statement relates to immune thrombocytopenic purpura (ITP)? a. ITP is formed in conditions of low flow and is made up of mostly red cells with larger amounts of fibrin and few platelets. b. An alteration of multipotent stem cells, resulting in an excess production of platelets, causes ITP. c. Mononuclear phagocytes in the spleen remove antibody-coated platelets from circulation. d. Arterial clots are made up of mostly platelet aggregates held together by fibrin strands. ANS: C ITP involves the antigen usually forming immune complexes with circulating antibodies, and it is thought that the immune complexes bind to Fc receptors on platelets, leading to their destruction in the spleen. None of the other options are accurately related to ITP. PTS: 1 REF: Pages 1038-1039 23. When the demand for mature neutrophils exceeds the supply, immature neutrophils are released indicating: a. A shift to the right c. Leukocytosis b. A shift to the left d. Leukemia ANS: B When the demand for circulating mature neutrophils exceeds the supply, the marrow begins to release immature neutrophils (and other leukocytes) into the blood. Premature release of the immature white cells is responsible for the phenomenon known as a shift to the left or leukemoid reaction. None of the remaining options would be used to identify the process described. PTS: 1 REF: Page 1009 24. Hodgkin disease is characterized by the presence of which of the following? a. Philadelphia chromosome NURSINGTcB..COMMicrovascular thrombi b. Virchow triad d. Reed-Sternberg (RS) cells ANS: D Hodgkin disease is characterized by its progression from one group of lymph nodes to another, the development of systemic symptoms, and the presence of RS cells (see Figure 29-8), but not the involvement of the Philadelphia chromosome. Virchow triad is a symptom related to thrombus formation. Disseminated intravascular coagulation is associated with microvascular thrombi. PTS: 1 REF: Page 1024 MULTIPLE RESPONSE 25. Which classic clinical manifestations are symptoms of IM? (Select all that apply.) a. Lymph node enlargement b. Hepatitis c. Pharyngitis d. Edema in the area of the eyes e. Fever ANS: A, C, E At the time of diagnosis, the individual usually has the classic triad of symptoms: fever, pharyngitis, and lymphadenopathy of the cervical lymph nodes. The triad does not include hepatitis or orbital edema. PTS: 1 REF: Page 1012 26. Early detection of acute leukemia would include which of the following symptoms? (Select all that apply.) a. Dizziness b. Paresthesia c. Anorexia d. Bruising e. Bone pain ANS: C, D, E Signs and symptoms related to bone marrow depression include fatigue caused by anemia, bleeding resulting from thrombocytopenia (reduced numbers of circulating platelets), and fever caused by infection. Anorexia can occur in all varieties of acute leukemia and is associated with weight loss. Pain in the bones and joints is thought to result from leukemia infiltration with secondary stretching of the periosteum. The other options are not generally associated with acute leukemia. PTS: 1 REF: Page 1017 | Page 1019 27. What are the most significant risk factors for the development of thrombus formation as referred to by the Virchow triad? (Select all that apply.) a. Endothelial injury to blood vessels b. Turbulent arterial blood flow NURSINGTB.COM c. Rapid coagulation of the blood d. Stagnant venous blood flow e. History of obesity ANS: A, B, C, D The risk for developing spontaneous thrombi is related to several factors, referred to as the Virchow triad: (1) injury to the blood vessel endothelium, (2) abnormalities of blood flow, and (3) hypercoagulability of the blood. Obesity is not associated with the triad. PTS: 1 REF: Pages 1048-1049 28. Which statements are true regarding leukemias? (Select all that apply.) a. A single progenitor cell undergoes a malignant change. b. Leukemia is a result of uncontrolled cellular proliferation. c. Bone marrow becomes overcrowded. d. Leukocytes are under produced. e. Hematopoietic cell production is decreased. ANS: A, B, C, E In the leukemias, a single progenitor cell undergoes malignant transformation. The common feature of all forms of leukemia is an uncontrolled proliferation of malignant leukocytes, causing an overcrowding of bone marrow and decreased production and function of normal hematopoietic cells. PTS: 1 REF: Page 1015 29. The two major forms of leukemia, acute and chronic, are classified by which criteria? (Select all that apply.) a. Predominant cell type b. Rate of progression c. Age of individual when cells differentiation occurs d. Stage of cell development when malignancy first occurs e. Serum level of leukocytes ANS: A, B The current classification of leukemia is based on (1) the predominant cell of origin (either myeloid or lymphoid) and (2) the rate of progression, which usually reflects the degree at which cell differentiation was arrested when the cell became malignant (acute or chronic) (see Figure 29-2). The remaining options are inaccurate statements regarding the classification criteria. PTS: 1 REF: Page 1013 30. What are the clinical manifestations of advanced non-African Burkitt lymphoma? (Select all that apply.) a. Abdominal swelling b. Night sweats c. Fever d. Weight gain e. Dementia ANS: A, B, C NURSINGTB.COM In non-African Burkitt lymphoma, the most common presentation is abdominal swelling. More advanced disease may exhibit night sweats, fever, and weight loss. Dementia is not associated with this disease. PTS: 1 REF: Page 1029 MATCHING Match the causes or diagnostic tests with the hematologic disorders. A. Epstein-Barr virus B. Bence Jones protein C. Diagnosed by the Reed-Sternberg cell D. Diagnosed by the Philadelphia chromosome 31. Infectious mononucleosis 32. Chronic myelogenous leukemia 33. Multiple myeloma 34. Hodgkin lymphoma 31. ANS: A PTS: 1 REF: Page 1011 MSC: The most common etiologic agent is Epstein-Barr virus, a ubiquitous, lymphotrophic, gamma-group herpesvirus. 32. ANS: D PTS: 1 REF: Pages 1014-1015 MSC: The Philadelphia chromosome is present in more than 95% of patients with chronic myelogenous leukemia. 33. ANS: B PTS: 1 REF: Page 1032 MSC: The myeloma may produce free immunoglobulin light chain (Bence Jones protein) that is present in the blood and urine in approximately 80% of patients. 34. ANS: C PTS: 1 REF: Page 1024 MSC: Hodgkin lymphoma is characterized by its progression from one group of lymph nodes to another, the development of systemic symptoms, and the presence of Reed-Sternberg cells NURSINGTB.COM Chapter 31: Alterations of Hematologic Function in Children MULTIPLE CHOICE 1. What is the cause of polycythemia in the fetus? a. Fetal hemoglobin has a greater affinity for oxygen as a result of diphosphoglycerate (DPG). b. The fetus has a different hemoglobin structure of two - and two -chains rather than two -and two -chains. c. Increased erythropoiesis occurs in response to the hypoxic intrauterine environment. d. The lungs of the fetus are undeveloped and unable to diffuse oxygen adequately to the pulmonary capillaries. ANS: C The hypoxic intrauterine environment stimulates erythropoietin production in the fetus. This selection is the only option that accurately explains why the fetus develops polycythemia. PTS: 1 REF: Page 1057 2. Why does fetal hemoglobin have a greater affinity for oxygen than adult hemoglobin? a. The fetus does not have its own oxygen supply and must rely on oxygen from the maternal vascular system. b. The fetus has two -chains on the hemoglobin, rather than two -chains as in the adult. NURSINGTB.COM c. Fetal hemoglobin interacts less readily with diphosphoglycerate (DPG), which inhibits hemoglobin-oxygen binding. d. Fetal hemoglobin production occurs in the vessels and liver rather than in the bone marrow as in the adult. ANS: C Fetal hemoglobin has greater affinity for oxygen than does adult hemoglobin because it interacts less readily with the enzyme, DPG, which inhibits hemoglobin-oxygen binding. This selection is the only option that accurately explains why fetal hemoglobin has a greater affinity for oxygen than adult hemoglobin. PTS: 1 REF: Page 1056 3. Which blood cell type is elevated at birth but decreases to adult levels during the first year of life? a. Monocytes c. Neutrophils b. Platelets d. Lymphocytes ANS: A Only monocyte counts are high in the first year of life and then decrease to adult levels. PTS: 1 REF: Page 1057 4. In a full-term infant, the normal erythrocyte life span is days, whereas the adult erythrocyte life span is _ days. a. 30 to 50; 80 c. 90 to 110; 140 b. 60 to 80; 120 d. 120 to 130; 150 ANS: B In full-term infants, normal erythrocyte life span is 60 to 80 days; in premature infants it may be as short as 20 to 30 days; and in children and adolescents, it is 120 days, the same as that in adults. PTS: 1 REF: Page 1057 5. What is the most common cause of insufficient erythropoiesis in children? a. Folic acid deficiency c. Hemoglobin abnormality b. Iron deficiency d. Erythrocyte abnormality ANS: B Similar to the anemias of adulthood, ineffective erythropoiesis or premature destruction of erythrocytes causes the anemias of childhood. The most common cause of insufficient erythropoiesis is iron deficiency. The other options may be causes but are not common ones. PTS: 1 REF: Page 1058 6. How does hemolytic disease of the newborn (HDN) cause acquired congenital hemolytic anemia? a. HDN develops when hypoxia or dehydration causes the erythrocytes to change shapes, which are then recognNizUeRdSaIsNfGoTrBei.gCnOMand removed from circulation. b. HDN is an alloimmune disease in which the mother’s immune system produces antibodies against fetal erythrocytes, which are recognized as foreign and removed from circulation. c. HDN develops when the polycythemia present in fetal life continues after birth, causing the excessive number of erythrocytes to be removed from circulation. d. HDN is an autoimmune disease in which the fetus’s immune system produces antibodies against fetal erythrocytes, which are recognized as foreign and removed from circulation. ANS: B HDN is an alloimmune disease in which maternal blood and fetal blood are antigenically incompatible, causing the mother’s immune system to produce antibodies against fetal erythrocytes. Fetal erythrocytes that have been attacked by (or bound to) maternal antibodies are recognized as foreign or defective by the fetal mononuclear phagocyte system and are removed from the circulation by phagocytosis, usually in the fetal spleen. This selection is the only option that accurately explains how HDN causes acquired congenital hemolytic anemia. PTS: 1 REF: Page 1058 7. Erythroblastosis fetalis is defined as an: a. Allergic disease in which maternal blood and fetal blood are antigenically incompatible b. Alloimmune disease in which maternal blood and fetal blood are antigenically incompatible c. Autoimmune disease in immature nucleated cells that are released into the bloodstream d. Autosomal dominant hereditary disease ANS: B Erythroblastosis fetalis, also known as hemolytic disease of the newborn (HDN), is an alloimmune disease in which maternal blood and fetal blood are antigenically incompatible, causing the mother’s immune system to produce antibodies against fetal erythrocytes. This selection is the only option that accurately defines erythroblastosis fetalis. PTS: 1 REF: Page 1058 8. An infant’s hemoglobin must fall below g/dl before signs of pallor, tachycardia, and systolic murmurs occur. a. 11 c. 7 b. 9 d. 5 ANS: D When the hemoglobin falls below 5 g/dl, pallor, tachycardia, and systolic murmurs may occur. PTS: 1 REF: Page 1059 9. Which vitamin improves the absorption of oral iron taken to treat iron deficiency anemia in children? NURSINGTB.COM a. A c. C b. B d. E ANS: C Of the available options, only additional vitamin C may be used to promote iron absorption. PTS: 1 REF: Page 1059 10. Hemolytic disease of the newborn (HDN) can occur if the mother: a. Is Rh-positive and the fetus is Rh-negative b. Is Rh-negative and the fetus is Rh-positive c. Has type A blood and the fetus has type O d. Has type AB blood and the fetus has type B ANS: B HDN can occur only if antigens on fetal erythrocytes differ from antigens on maternal erythrocytes. Maternal-fetal incompatibility exists only if the mother and fetus differ in ABO blood type or if the fetus is Rh-positive and the mother is Rh-negative. This erythrocyte incompatibility does not exist in any of the other options. PTS: 1 REF: Page 1059 11. When diagnosed with hemolytic disease of the newborn (HDN), why does the newborn develop hyperbilirubinemia after birth but not in utero? a. Excretion of unconjugated bilirubin through the placenta into the mother’s circulation is no longer possible. b. Hemoglobin does not break down into bilirubin in the intrauterine environment. c. The liver of the fetus is too immature to conjugate bilirubin from a lipid-soluble to water-soluble form. d. The destruction of erythrocytes producing bilirubin is greater after birth. ANS: A Hyperbilirubinemia occurs in the neonate after birth because excretion of lipid-soluble unconjugated bilirubin through the placenta is no longer possible. This selection is the only option that accurately explains why HDN causes hyperbilirubinemia after birth but not in utero. PTS: 1 REF: Page 1060 12. Fetuses who do not survive anemia in utero are usually stillborn with gross edema of the entire body. Which term is used to identify this condition? a. Spherocytosis c. Erythroblastosis fetalis b. Icterus gravis neonatorum d. Hydrops fetalis ANS: D Fetuses that do not survive anemia in utero are usually stillborn with gross edema in the entire body, a condition called hydrops fetalis. This selection is the only option that accurately identifies the condition with these specific symptoms. PTS: 1 REF: Page 10N6U0RSINGTB.COM 13. What is the name of the disorder in which levels of bilirubin remain excessively high in the newborn and are deposited in the brain? a. Kernicterus c. Jaundice b. Icterus neonatorum d. Icterus gravis neonatorum ANS: A Without replacement transfusions, during which the child receives Rh-negative erythrocytes, the bilirubin is deposited in the brain, a condition called kernicterus. This selection is the only option that accurately identifies the condition with the provided symptoms. PTS: 1 REF: Page 1060 14. What treatment prevents the development of kernicterus in an infant born with hemolytic disease of the newborn (HDN)? a. Administration of intravenous fluids to dilute the blood b. Replacement transfusion of new Rh-positive blood that is not contaminated with anti-Rh antibodies c. Performance of a splenectomy to prevent the destruction of abnormal erythrocytes d. Replacement transfusion of Rh-negative erythrocytes ANS: B If antigenic incompatibility of the mother’s erythrocytes is not discovered in time to administer Rh immunoglobulin and the child is born with HDN, then the treatment consists of exchange transfusions in which the neonate’s blood is replaced with new Rh-positive blood that is not contaminated with anti-Rh antibodies. This treatment is administered during the first 24 hours of extrauterine life to prevent kernicterus. This selection is the only option that accurately prevents kernicterus. PTS: 1 REF: Page 1061 15. Glucose 6-phosphate dehydrogenase (G6PD) deficiency is what type of inherited disorder? a. X-linked dominant c. Autosomal dominant b. X-linked recessive d. Autosomal recessive ANS: B G6PD deficiency is an inherited, X-linked recessive disorder. This selection is the only option that accurately identifies the mode of inheritance for the stated disorder. PTS: 1 REF: Page 1062 16. Sickle cell disease is classified as a(an): a. Inherited X-linked recessive disorder b. Inherited autosomal recessive disorder c. Disorder initiated by hypoxemia and acidosis d. Disorder that is diagnosed equally in men and women ANS: C Sickling is an occasional, intermittent phenomenon that can be triggered or sustained by one or more of the following stresNsoUrRsS: IdNeGcrTeBa.sCeOdMoxygen tension (PO2) of the blood (hypoxemia), increased hydrogen ion concentration in the blood (decreased pH), increased plasma osmolality, decreased plasma volume, and low temperature (see Figure 30-7). This selection is the only option that accurately identifies the classification of the stated disorders. PTS: 1 REF: Page 1062 | Page 1065 17. Hemoglobin S (HbS) is formed in sickle cell disease as a result of which process? a. Deficiency in glucose 6-phosphate dehydrogenase (G6PD) that changes hemoglobin A (HbA) to HbS. b. Genetic mutation in which two amino acids (histidine and leucine) are missing. c. Genetic mutation in which one amino acid (valine) is replaced by another (glutamic acid). d. Autoimmune response in which one amino acid (proline) is detected as an antigen by abnormal immunoglobulin G (IgG). ANS: C HbS is formed by a genetic mutation in which one amino acid (valine) replaces another (glutamic acid) (see Figure 30-5, A). This selection is the only option that accurately identifies how HbS is formed in sickle cell disease. PTS: 1 REF: Page 1063 18. Sickle cell disease (SCD) is what type of inherited disorder? a. Autosomal dominant c. X-linked dominant b. Autosomal recessive d. X-linked recessive ANS: B SCD is an inherited autosomal recessive disorder that is expressed as sickle cell anemia, sickle cell–thalassemia disease, or sickle cell–hemoglobin C (HbC) disease, depending on mode of inheritance. This selection is the only option that accurately identifies the mode of inheritance for the stated disorder. PTS: 1 REF: Page 1063 19. What is the reason most children diagnosed with sickle cell anemia are not candidates for either bone marrow or stem cell transplants? a. Well-matched stem cell donors are difficult to find. b. The child is usually too weak to survive the procedure. c. The child’s immune system will not appropriately respond to the antirejection medications. d. Although effective for adults, neither procedure has been proven effective for children. ANS: A Bone marrow or stem cell transplants can cure sickle cell anemia. However, the only reason these procedures are not currently an option for most children is because well-matched stem cell donors are often difficult to find. PTS: 1 REF: Page 1063 20. Which manifestations of vasoocclNuUsiRvSeIcNrGisTisB.aCrOe Massociated with sickle cell disease (SCD) in infants? a. Atelectasis and pneumonia b. Edema of the hands and feet c. Stasis ulcers of the hands, ankles, and feet d. Splenomegaly and hepatomegaly ANS: B Symmetric, painful swelling of the hands and feet (hand-foot syndrome) caused by infarction in the small vessels of the extremities is often the initial manifestation of SCD in infants. This selection is the only option that accurately identifies the manifestations of a vasoocclusive crisis associated with SCD in infancy. PTS: 1 REF: Page 1066 21. What is the chance with each pregnancy that a child born to two parents with the sickle trait will have sickle cell disease (SCD)? a. 20% c. 33% b. 25% d. 50% ANS: B A 25% chance exists with each pregnancy that a child born to two parents with sickle cell trait will have SCD. Genetic counseling enables people with SCD or with the sickle cell trait to make informed decisions about transmitting this genetic disorder to their offspring. PTS: 1 REF: Pages 1068-1069 22. Which type of anemia occurs as a result of thalassemia? a. Microcytic, hypochromic c. Macrocytic, hyperchromic b. Microcytic, normochromic d. Macrocytic, normochromic ANS: A The only option that identifies the anemic manifestation of thalassemia is microcytic-hypochromic hemolytic anemia. PTS: 1 REF: Page 1069 23. What is the fundamental defect that results in beta-thalassemia major? a. The spleen prematurely destroys the precipitate-carrying cells. b. A severe uncoupling of - and -chain synthesis occurs. c. All four beta-forming genes are defective. d. Hemoglobin H (HbH) develops when three genes are defective. ANS: B The fundamental defect in beta-thalassemia is the uncoupling of - and -chain synthesis. -Chain production is moderately depressed in the heterozygous form—beta-thalassemia minor, and severely depressed in the homozygous form—beta-thalassemia major (also called Cooley anemia). The defects described in the other options are not related to beta-thalassemia major. PTS: 1 REF: Page 1069 24. The alpha- and beta-thalassemias NarUeRcSoINnsGidTeBr.eCdOMwhat types of inherited disorder? a. Autosomal recessive c. X-linked recessive b. Autosomal dominant d. X-linked dominant ANS: A The alpha- and beta-thalassemias are inherited autosomal recessive disorders. This selection is the only option that correctly identifies how these disorders are inherited. PTS: 1 REF: Page 1069 25. Hemophilia B is caused by a deficiency of which clotting factor? a. V c. IX b. VIII d. X ANS: C Only factor IX deficiency causes hemophilia B (Christmas disease). PTS: 1 REF: Page 1071 26. Hemophilia A is considered to be what type of inherited disorder? a. Autosomal dominant c. X-linked recessive b. Autosomal recessive d. X-linked dominant ANS: C Hemophilia A (classic hemophilia) is inherited as an X-linked recessive disorder. This selection is the only option that correctly identifies how this disorder is inherited. PTS: 1 REF: Page 1071 27. Which disease is an autosomal dominant inherited hemorrhagic disease? a. Hemophilia A c. Christmas disease b. von Willebrand disease d. Hemophilia B ANS: B The bleeding disorder, von Willebrand disease, results only from an inherited autosomal dominant trait. PTS: 1 REF: Page 1071 28. Idiopathic thrombocytopenic purpura (ITP) is an autoimmune process involving antibodies attacking which type of cells? a. Neutrophils c. Platelets b. Eosinophils d. Basophils ANS: C ITP is a disorder of platelet consumption. This selection is the only option that correctly identifies which type of cell is attacked by ITP. PTS: 1 REF: Page 1073 29. Which disorder results in decreased erythrocytes and platelets with changes in leukocytes and has clinical manifestations of pallor, fatigue, petechiae, purpura, bleeding, and fever? a. Idiopathic thrombocytopenic purpura (ITP) b. Acute lymphocytic leukemia (NAULRLS)INGTB.COM c. Non-Hodgkin lymphoma (NHL) d. Iron deficiency anemia (IDA) ANS: B Pallor, fatigue, petechiae, purpura, bleeding, and fever are generally present with the most common symptoms reflecting the consequence of bone marrow failure, which results in decreased red blood cells and platelets and changes in white blood cells. This selection is the only option that correctly identifies the disease with the symptoms described. PTS: 1 REF: Page 1076 30. When does fetal erythrocyte production shift from the liver to the bone marrow? a. Fourth month of gestation c. Eighth month of gestation b. Fifth month of gestation d. At birth ANS: B By the fifth month of gestation, hematopoiesis begins to occur in the bone marrow and increases rapidly until hematopoietic (red) marrow fills the entire bone marrow space. By the time of delivery, the bone marrow is the only significant site of hematopoiesis. PTS: 1 REF: Pages 1055-1056 31. Which disease is caused by clotting factor VIII deficiency and is an autosomal dominant trait? a. Hemophilia A c. Hemophilia C b. Hemophilia B d. von Willebrand disease ANS: D Of the options available, only von Willebrand disease is caused by factor VIII deficiency and is the result of an inherited autosomal dominant trait. PTS: 1 REF: Page 1071 32. Which type of hemophilia affects only men? a. Hemophilia A c. Hemophilia C b. Hemophilia B d. von Willebrand disease ANS: A Of the options available, only hemophilia A (classic hemophilia) is caused by factor VIII deficiency and is inherited as an X-linked recessive disorder that affects men and is transmitted by women. PTS: 1 REF: Page 1071 33. Which hemophilia occurs equally in both men and women? a. Hemophilia A c. Hemophilia C b. Hemophilia B d. von Willebrand disease ANS: C Of the options available, only hemophilia C (factor XI deficiency) occurs as an autosomal recessive disease while occurring equally in men and women. PTS: 1 REF: Page 10N7U1RSINGTB.COM 34. During childhood, when is dietary iron deficiency commonly diagnosed? a. Between 2 months and 1 year c. Between 12 months and 3 years b. Between 6 months and 2 years d. Between 18 months and 4 years ANS: B Iron deficiency anemia is the most common blood disorder of infancy and childhood, with the highest incidence occurring between 6 months and 2 years of age. PTS: 1 REF: Page 1059 35. What is the significance of hyperdiploidy when diagnosing and treating leukemia? a. Hyperdiploidy indicates a good prognosis. b. Hyperdiploidy indicates a poor prognosis. c. Hyperdiploidy indicates poor response to a specific treatment. d. Hyperdiploidy indicates the achievement of remission. ANS: A Hyperdiploidy (increased number of chromosome copies) is associated with a good prognosis. The other options are incorrect regarding the significance of hyperdiploidy. PTS: 1 REF: Page 1075 MULTIPLE RESPONSE 36. What are the common triggers for sickle cell crisis? (Select all that apply.) a. Fever b. Infection c. Dehydration d. Alkalosis e. Exposure to the cold ANS: A, B, C, E Crises can be prevented by avoiding fever, infection, acidosis, dehydration, constricting clothes, and exposure to cold. PTS: 1 REF: Page 1068 MATCHING Match each sickle cell crisis with its description. Terms may be used more than once. A. Vasoocclusive crisis B. Aplastic crisis C. Sequestration crisis D. Hyperhemolytic crisis 37. Large amounts of blood become acutely pooled in the liver and spleen. 38. Blood flow is impaired by tangled masses of rigid, sickled cells. 39. Crisis occurs in association with certain drugs or infection. 40. Compensatory erythropoiesis is coNmUpRrSoINmGisTeBd.,CtOhMus limiting the number of erythrocytes that are replaced. 37. ANS: C PTS: 1 REF: Page 1066 MSC: In sequestration crisis, large amounts of blood become acutely pooled in the liver and spleen. 38. ANS: A PTS: 1 REF: Page 1066 MSC: Vasoocclusive crisis (thrombotic crisis) begins with sickling in the microcirculation. As blood flow is obstructed by tangled masses of rigid, sickled cells, vasospasm occurs and a log jam effect brings all blood flow through the vessel to a halt. 39. ANS: D PTS: 1 REF: Page 1066 40. ANS: B PTS: 1 REF: Page 1066 MSC: Aplastic crisis, a transient cessation in red blood cell production that results in acute anemia, occurs as a result of a viral infection. The virus causes a temporary shutdown of red blood cell production in the bone marrow, or reticulocytosis. Chapter 32: Structure and Function of the Cardiovascular and Lymphatic Systems MULTIPLE CHOICE 1. Which statement does not accurately describe the pericardium? a. The pericardium is a double-walled membranous sac that encloses the heart. b. It is made up of connective tissue and a surface layer of squamous cells. c. The pericardium protects the heart against infection and inflammation from the lungs and pleural space. d. It contains pain and mechanoreceptors that can elicit reflex changes in blood pressure and heart rate. ANS: B The pericardium is made up of a surface layer of mesothelium over a thin layer of connective tissue. The remaining options accurately describe the pericardium. PTS: 1 REF: Page 1085 2. Which cardiac chamber has the thinnest wall and why? a. The right and left atria; they are low-pressure chambers that serve as storage units and conduits for blood. b. The right and left atria; they are not directly involved in the preload, contractility, or afterload of the heart. c. The left ventricle; the mean pressure of blood coming into this ventricle is from the lung, which has a low pressure. d. The right ventricle; it pumps bNloUoRdSIiNntGoTtBh.eCOpuMlmonary capillaries, which have a lower pressure compared with the systemic circulation. ANS: A The two atria have the thinnest walls because they are low-pressure chambers that serve as storage units and conduits for blood that is emptied into the ventricles. This selection is the only option that correctly identifies which heart chambers have the thinnest walls and why that helps cardiac function. PTS: 1 REF: Page 1086 3. Which chamber of the heart endures the highest pressures? a. Right atrium c. Left ventricle b. Left atrium d. Right ventricle ANS: C Pressure is greatest in the systemic circulation, driven by the left ventricle. PTS: 1 REF: Page 1086 4. What is the process that ensures mitral and tricuspid valve closure after the ventricles are filled with blood? a. Chordae tendineae relax, which allows the valves to close. b. Increased pressure in the ventricles pushes the valves to close. c. Trabeculae carneae contract, which pulls the valves closed. d. Reduced pressure in the atria creates a negative pressure that pulls the valves closed. ANS: B During ventricular relaxation, the two atrioventricular valves open and blood flows from the higher pressure atria to the relaxed ventricles. With increasing ventricular pressure, these valves close and prevent backflow into the atria as the ventricles contract. This selection is the only option that correctly identifies the process that ensures closing of the mitral and tricuspid valves. PTS: 1 REF: Page 1088 5. Regarding the heart’s valves, what is a function of the papillary muscles? a. The papillary muscles close the semilunar valve. b. These muscles prevent backward expulsion of the atrioventricular valve. c. They close the atrioventricular valve. d. The papillary muscles open the semilunar valve. ANS: B The papillary muscles are extensions of the myocardium that pull the cusps together and downward at the onset of ventricular contraction, thus preventing their backward expulsion into the atria. This selection is the only option that correctly describes the function of the papillary muscles. PTS: 1 REF: Pages 1087-1088 6. During the cardiac cycle, why do the aortic and pulmonic valves close after the ventricles relax? NURSINGTB.COM a. Papillary muscles relax, which allows the valves to close. b. Chordae tendineae contract, which pulls the valves closed. c. Reduced pressure in the ventricles creates a negative pressure, which pulls the valves closed. d. Blood fills the cusps of the valves and causes the edges to merge, closing the valves. ANS: D When the ventricles relax, blood fills the cusps and causes their free edges to meet in the middle of the vessel, closing the valve and preventing any backflow. This selection is the only option that accurately explains why the aortic and pulmonic valves close after the ventricles contract. PTS: 1 REF: Page 1088 7. Oxygenated blood flows through which vessel? a. Superior vena cava c. Pulmonary artery b. Pulmonary veins d. Coronary veins ANS: B Only the four pulmonary veins, two from the right lung and two from the left lung, carry oxygenated blood from the lungs to the left side of the heart. PTS: 1 REF: Page 1088 8. The significance of the atrial kick is that it affects the contraction of the: a. Right atria, which is necessary to open the tricuspid valve. b. Right atria, which is necessary to increase the blood volume from the vena cava. c. Left atria, which increases the blood volume into the ventricle. d. Left atria, that is necessary to open the mitral valve. ANS: C Left atrial contraction, the atrial kick, provides a significant increase of blood to the left ventricle. PTS: 1 REF: Page 1088 9. Occlusion of the left anterior descending artery during a myocardial infarction would interrupt blood supply to which structures? a. Left and right ventricles and much of the interventricular septum b. Left atrium and the lateral wall of the left ventricle c. Upper right ventricle, right marginal branch, and right ventricle to the apex d. Posterior interventricular sulcus and the smaller branches of both ventricles ANS: A The left anterior descending artery (LAD), also called the anterior interventricular artery, delivers blood to portions of the left and right ventricles and much of the interventricular septum. This selection is the only option affected by the occlusion described. PTS: 1 REF: Page 1090 10. Occlusion of the circumflex arteryNdUuRrSinINgGaTmB.yCoOcMardial infarction would interrupt blood supply to which area? a. Left and right ventricles and much of the interventricular septum b. Posterior interventricular sulcus and the smaller branches of both ventricles c. Upper right ventricle, right marginal branch, and right ventricle to the apex d. Left atrium and the lateral wall of the left ventricle ANS: D The circumflex artery supplies blood to the left atrium and the lateral wall of the left ventricle. The circumflex artery often branches to the posterior surfaces of the left atrium and left ventricle. This selection is the only option affected by the occlusion described. PTS: 1 REF: Pages 1090-1091 11. The coronary ostia are located in the: a. Left ventricle c. Coronary sinus b. Aortic valve d. Aorta ANS: D Coronary arteries receive blood through openings in the aorta, called the coronary ostia. PTS: 1 REF: Page 1090 12. The coronary sinus empties into which cardiac structure? a. Right atrium c. Superior vena cava b. Left atrium d. Aorta ANS: A The cardiac veins empty only into the right atrium through another ostium, the opening of a large vein called the coronary sinus. PTS: 1 REF: Page 1090 13. What is the ratio of coronary capillaries to cardiac muscle cells? a. 1:1 (one capillary per one muscle cell) b. 1:2 (one capillary per two muscle cells) c. 1:4 (one capillary per four muscle cells) d. 1:10 (one capillary per ten muscle cells) ANS: A The heart has an extensive capillary network, with approximately 3300 capillaries per square millimeter (ca/mm2) or approximately one capillary per one muscle cell (muscle fiber). PTS: 1 REF: Page 1092 14. During the cardiac cycle, which structure directly delivers action potential to the ventricular myocardium? a. Sinoatrial (SA) node c. Purkinje fibers b. Atrioventricular (AV) node d. Bundle branches ANS: C Each cardiac action potential travels from the SA node to the AV node to the bundle of His NURSINGTB.COM(AV bundle), through the bundle lly to the Purkinje fibers and the ventricular myocardium, where the impulse is stopped. The refractory period of cells that have just been polarized prevents the impulse from reversing its path. The refractory period ensures that diastole (relaxation) will occur, thereby completing the cardiac cycle. This selection is the only option that accurately describes the structure that delivers the action potential directly to the myocardium. PTS: 1 REF: Page 1124 15. What causes depolarization of a cardiac muscle cell to occur? a. Decrease in the permeability of the cell membrane to potassium b. Rapid movement of sodium into the cell c. Decrease in the movement of sodium out of the cell d. Rapid movement of calcium out of the cell ANS: B Phase 0 consists of depolarization, which lasts 1 to 2 milliseconds (ms) and represents rapid sodium entry into the cell. This selection is the only option that accurately describes the cause of cardiac muscle cell depolarization. PTS: 1 REF: Page 1094 16. Which event occurs during phase 1 of the normal myocardial cell depolarization and repolarization? a. Repolarization when potassium moves out of the cells b. Repolarization when sodium rapidly enters into the cells c. Early repolarization when sodium slowly enters the cells d. Early repolarization when calcium slowly enters the cells ANS: D Phase 1 is early repolarization and the only time during which calcium slowly enters the cell. PTS: 1 REF: Page 1094 17. Which phase of the normal myocardial cell depolarization and repolarization correlates with diastole? a. Phase 1 c. Phase 3 b. Phase 2 d. Phase 4 ANS: D Potassium is moved out of the cell during phase 3, with a return to resting membrane potential only in phase 4. The time between action potentials corresponds to diastole. PTS: 1 REF: Page 1094 18. In the normal electrocardiogram, what does the PR interval represent? a. Atrial depolarization b. Ventricular depolarization c. Atrial activation to onset of ventricular activity d. Electrical systole of the ventricles NURSINGTB.COM ANS: C The PR interval is a measure of time from the onset of atrial activation to the onset of ventricular activation; it normally ranges from 0.12 to 0.20 second. The PR interval represents the time necessary to travel from the sinus node through the atrium, the atrioventricular (AV) node, and the His–Purkinje system to activate ventricular myocardial cells. This selection is the only option that accurately describes the PR interval. PTS: 1 REF: Page 1095 19. The cardiac electrical impulse normally begins spontaneously in the sinoatrial (SA) node because it: a. Has a superior location in the right atrium. b. Is the only area of the heart capable of spontaneous depolarization. c. Has rich sympathetic innervation via the vagus nerve. d. Depolarizes more rapidly than other automatic cells of the heart. ANS: D The electrical impulse normally begins in the SA node because its cells depolarize more rapidly than other automatic cells. This selection is the only option that accurately explains why cardiac electrical impulses normally begin spontaneously in the SA node. PTS: 1 REF: Page 1095 20. What period follows depolarization of the myocardium and represents a period during which no new cardiac potential can be propagated? a. Refractory c. Threshold b. Hyperpolarization d. Sinoatrial (SA) ANS: A During the refractory period, no new cardiac action potential can be initiated by a stimulus. This selection is the only option that accurately identifies the period described in the question. PTS: 1 REF: Page 1095 21. Which complex (wave) represents the sum of all ventricular muscle cell depolarizations? a. PRS c. QT interval b. QRS d. P ANS: B Only the QRS complex represents the sum of all ventricular muscle cell depolarizations. PTS: 1 REF: Page 1095 22. What can shorten the conduction time of action potential through the atrioventricular (AV) node? a. Parasympathetic nervous system c. Vagal stimulation b. Catecholamines d. Sinoatrial node (SA) ANS: B Catecholamines speed the heart rate, shorten the conduction time through the AV node, NURSINGTB.COMand increase the rhythmicity of fibers. This selection is the only option that can perform that function. PTS: 1 REF: Page 1096 23. If the sinoatrial (SA) node fails, then at what rate (depolarizations per minute) can the atrioventricular (AV) node depolarize? a. 60 to 70 c. 30 to 40 b. 40 to 60 d. 10 to 20 ANS: B If the SA node is damaged, then the AV node will become the heart’s pacemaker at a rate of approximately 40 to 60 spontaneous depolarizations per minute. PTS: 1 REF: Page 1095 24. What is the effect of epinephrine on 3 receptors on the heart? a. Decreases coronary blood flow. b. Supplements the effects of both 1 and 2 receptors. c. Increases the strength of myocardial contraction. d. Prevents overstimulation of the heart by the sympathetic nervous system. ANS: D 3 receptors are found in the myocardium and coronary vessels. In the heart, stimulation of these receptors opposes the effects of 1- and 2-receptor stimulation and negative inotropic effect. Thus 3 receptors may provide a safety mechanism that decreases myocardial contractility to prevent overstimulation of the heart by the sympathetic nervous system. This selection is the only option that accurately describes the effect of epinephrine on 2 receptors on the heart. PTS: 1 REF: Page 1097 25. Where in the heart are the receptors for neurotransmitters located? a. Semilunar and atrioventricular (AV) valves b. Endocardium and sinoatrial (SA) node c. Myocardium and coronary vessels d. Epicardium and AV node ANS: C Sympathetic neural stimulation of the myocardium and coronary vessels depends on the presence of adrenergic receptors, which specifically bind with neurotransmitters of the sympathetic nervous system. The 1 receptors are found mostly in the heart, specifically the conduction system (AV and SA nodes, Purkinje fibers) and the atrial and ventricular myocardium, whereas the 2 receptors are found in the heart and also on vascular smooth muscle. 3 receptors are also found in the myocardium and coronary vessels. This selection is the only option that accurately identifies the location of the receptors for neurotransmitters. PTS: 1 REF: Page 1097 NURSINGTB.COM 26. What enables electrical impulses to travel in a continuous cell-to-cell fashion in myocardial cells? a. Sarcolemma sclerotic plaques c. Trabeculae carneae b. Intercalated disks d. Bachmann bundles ANS: B Only intercalated disks, thickened portions of the sarcolemma, enable electrical impulses to spread quickly in a continuous cell-to-cell (syncytial) fashion. PTS: 1 REF: Page 1097 27. Within a physiologic range, what does an increase in left ventricular end-diastolic volume (preload) result in? a. Increase in force of contraction c. Increase in afterload b. Decrease in refractory time d. Decrease in repolarization ANS: A This concept is expressed in the Frank-Starling law; the cardiac muscle, like other muscles, increases its strength of contraction when it is stretched. This selection is the only option that accurately describes the result of an increase in preload. PTS: 1 REF: Pages 1101-1102 28. As stated in the Frank-Starling law, a direct relationship exists between the of the blood in the heart at the end of diastole and the of contraction during the next systole. a. Pressure; force c. Viscosity; force b. Volume; strength d. Viscosity; strength ANS: B As stated in the Frank-Starling law, the volume of blood in the heart at the end of diastole (the length of its muscle fibers) is directly related to the force (strength) of contraction during the next systole. This selection is the only option that accurately describes the relationship associated with the Frank-Starling law. PTS: 1 REF: Pages 1101-1102 29. Pressure in the left ventricle must exceed pressure in which structure before the left ventricle can eject blood? a. Superior vena cava c. Inferior vena cava b. Aorta d. Pulmonary veins ANS: B Pressure in the ventricle must exceed aortic pressure before blood can be pumped out during systole. The aorta is the only structure in which pressure must be less than the amount of blood in the left ventricle for ejection to occur. PTS: 1 REF: Page 1103 30. Continuous increases in left ventricular filing pressures result in which disorder? a. Mitral regurgitation NURSINGTcB..COPMulmonary edema b. Mitral stenosis d. Jugular vein distention ANS: C Pressure changes are important because increased left ventricular filling pressures back up into the pulmonary circulation, where they force plasma out through vessel walls, causing fluid to accumulate in lung tissues (pulmonary edema). This selection is the only option that accurately identifies the disorder described in the question. PTS: 1 REF: Page 1103 31. When the volume of blood in the ventricle at the end of diastole increases, the force of the myocardial contraction during the next systole will also increase, which is an example of which law or theory about the heart? a. Laplace’s law c. Cross-bridge theory b. Poiseuille law d. Frank-Starling law ANS: D This concept is expressed only in the Frank-Starling law; the cardiac muscle, like other muscles, increases its strength of contraction when it is stretched. PTS: 1 REF: Page 1101 32. The resting heart rate in a healthy person is primarily under the control of which nervous system? a. Sympathetic c. Somatic b. Parasympathetic d. Spinal ANS: B The resting heart rate in healthy individuals is primarily under the control of parasympathetic stimulation. This selection is the only option that accurately identifies the nervous system responsible for the healthy resting heart. PTS: 1 REF: Page 1106 33. The Bainbridge reflex is thought to be initiated by sensory neurons in which cardiac location? a. Atria c. Sinoatrial (SA) node b. Aorta d. Ventricles ANS: A The Bainbridge reflex causes changes in the heart rate after intravenous infusions of blood or other fluid. The changes in heart rate are thought to be caused by a reflex mediated by volume receptors found only in the atria that are innervated by the vagus nerve. PTS: 1 REF: Page 1106 34. After the baroreceptor reflex is stimulated, the resulting impulse is transmitted from the carotid artery by which sequence of events? a. Vagus nerve to the medulla to increase parasympathetic activity and to decrease sympathetic activity b. Glossopharyngeal cranial nerve through the vagus nerve to the medulla to increase sympathetic activity and to deNcrUeRasSeINpGaTraBs.yCmOMpathetic activity c. Glossopharyngeal cranial nerve through the vagus nerve to the medulla to increase parasympathetic activity and to decrease sympathetic activity d. Glossopharyngeal cranial nerve through the vagus nerve to the hypothalamus to increase parasympathetic activity and to decrease sympathetic activity ANS: C Neural impulses are transmitted over the glossopharyngeal nerve (ninth cranial nerve) from the carotid artery and through the vagus nerve from the aorta to the cardiovascular control centers in the medulla. These centers initiate an increase in parasympathetic activity and a decrease in sympathetic activity, causing blood vessels to dilate and the heart rate to decrease. This selection is the only option that accurately describes the correct sequence of events asked for in the question. PTS: 1 REF: Page 1106 35. Reflex control of total cardiac output and total peripheral resistance is controlled by what mechanism? a. Parasympathetic stimulation of the heart, arterioles, and veins b. Sympathetic stimulation of the heart, arterioles, and veins c. Autonomic control of the heart only d. Somatic control of the heart, arterioles, and veins ANS: B Reflex control of total cardiac output and peripheral resistance includes (1) sympathetic stimulation of the heart, arterioles, and veins; and (2) parasympathetic stimulation of the heart only. Neither autonomic nor somatic controls are involved in this process. PTS: 1 REF: Pages 1114-1115 36. What is the most important negative inotropic agent? a. Norepinephrine c. Acetylcholine b. Epinephrine d. Dopamine ANS: C Chemicals affecting contractility are called inotropic agents. The most important negative inotropic agent is acetylcholine released from the vagus nerve. The most important positive inotropic agents produced by the body are norepinephrine released from the sympathetic nerves that supply the heart and epinephrine released by the adrenal cortex. Other positive inotropes include thyroid hormone and dopamine. Many medications have positive or negative inotropic properties that can have profound effects on cardiac function. This selection is the only option that accurately identifies the regulation that is involved in the described process. PTS: 1 REF: Page 1103 37. The right lymphatic duct drains into which structure? a. Right subclavian artery c. Right subclavian vein b. Right atrium d. Superior vena cava ANS: C The right lymphatic duct drains lyNmUpRhSoINnGlyTBin.CtoOtMhe right subclavian vein. PTS: 1 REF: Page 1118 38. Where is the major cardiovascular center in the central nervous system? a. Frontal lobe c. Brainstem b. Thalamus d. Hypothalamus ANS: C The major cardiovascular control center is in the brainstem in the medulla with secondary areas in the hypothalamus, the cerebral cortex, the thalamus, and the complex networks of exciting or inhibiting interneurons (connecting neurons) throughout the brain. This selection is the only option that accurately identifies the cardiovascular control center. PTS: 1 REF: Page 1104 39. What is an expected change in the cardiovascular system that occurs with aging? a. Arterial stiffening b. Decreased left ventricular wall tension c. Decreased aortic wall thickness d. Arteriosclerosis ANS: A Arterial stiffening occurs with aging even in the absence of clinical hypertension. Aging is not responsible for the other conditions. PTS: 1 REF: Page 1123 40. What is the major determinant of the resistance that blood encounters as it flows through the systemic circulation? a. Volume of blood in the systemic circulation b. Muscle layer of the metarterioles c. Muscle layer of the arterioles d. Force of ventricular contraction ANS: C Of the options available, only the thick, smooth muscle layer of the arterioles is a major determinant of the resistance blood encounters as it flows through the systemic circulation. PTS: 1 REF: Page 1108 41. Which function of the cardiovascular system is often affected by ischemia? a. Cardiac output (CO) c. Heart rate (HR) b. Stroke volume (SV) d. Cardiac index (CI) ANS: C Common causes of an abnormal heart rate include ischemia, electrolyte imbalance, and drug toxicity. The other options are related to vascular resistance changes. PTS: 1 REF: Page 1120 | Table 31-4 42. What physical sign is the result of turbulent blood flow through a vessel? a. Increased blood pressure durinNgUpReSrIiNoGdsTBo.fCsOtrMess b. Bounding pulse felt on palpation c. Cyanosis observed on excretion d. Murmur heard on auscultation ANS: D Where flow is obstructed, the vessel turns or blood flows over rough surfaces. The flow becomes turbulent with whorls or eddy currents that produce noise, causing a murmur to be heard on auscultation, such as occurs during blood pressure measurement with a sphygomanometer. This selection is the only option that accurately identifies the physical sign of turbulent vascular blood flow. PTS: 1 REF: Page 1113 43. What is the major effect of a calcium channel blocker such as verapamil on cardiac contractions? a. Increases the rate of cardiac contractions. b. Decreases the strength of cardiac contractions. c. Stabilizes the rhythm of cardiac contractions. d. Stabilizes the vasodilation during cardiac contractions. ANS: B The L-type, or long-lasting, channels are the predominant type of calcium channels and are the channels blocked by calcium channel–blocking drugs (verapamil, nifedipine, diltiazem). The major effect of these medications is to decrease the strength of cardiac contraction. This selection is the only option that accurately identifies the effect of a calcium channel blocker on the cardiac contractions. PTS: 1 REF: Page 1099 44. An early diastole peak caused by filling of the atrium from peripheral veins is identified by which intracardiac pressure? a. A wave c. C wave b. V wave d. X descent ANS: B The V wave is an early diastolic peak caused by the filling of the atrium from the peripheral veins. This event is not identified by any of the other options. PTS: 1 REF: Page 1089 45. Which intracardiac pressure is generated by the atrial contraction? a. A wave c. Y descent b. C wave d. X descent ANS: A Atrial pressure curves are made up of only the A wave, which is generated by atrial contraction. PTS: 1 REF: Page 10N8U9RSINGTB.COM 46. Which intracardiac pressure is produced because of the descent of the tricuspid valve ring and by the ejection of blood from both ventricles? a. V wave c. Y descent b. C wave d. X descent ANS: D The X descent follows an A wave and is produced because of the descent of the tricuspid valve ring and by the ejection of blood from both ventricles. PTS: 1 REF: Page 1089 MULTIPLE RESPONSE 47. Which statements are true concerning the method in which substances pass between capillaries and the interstitial fluid? (Select all that apply.) a. Substances pass through junctions between endothelial cells. b. Substances pass through pores or oval windows (fenestrations). c. Substances pass between vesicles by active transport across the endothelial cell membrane. d. Substances pass across the endothelial cell membrane by osmosis. e. Substances pass through endothelial cell membranes by diffusion. ANS: A, B, C, E Substances pass between the capillary lumen and the interstitial fluid in several ways: (1) through junctions between endothelial cells, (2) through fenestrations in endothelial cells, (3) in vesicles moved by active transport across the endothelial cell membrane, or (4) by diffusion through the endothelial cell membrane. PTS: 1 REF: Page 1108 MATCHING Match the description with the corresponding terms. A. Relationship among blood flow, pressure, and resistance B. Increased heart rate from increased volume C. Relationship of wall tension, intraventricular pressure, internal radius, and wall thickness D. Cycles of attachment, movement, and dissociation of thin filaments during the attachments of actin to myosin E. Length-tension relationship of cardiac muscle 48. Poiseuille law 49. Cross-bridge theory 50. Frank-Starling law 51. Laplace’s law 52. Bainbridge reflex 48. ANS: A PTS: 1 REF: Pages 1111-1112 MSC: Poiseuille law for resistance to fluid flow through a tube takes into account the length of the tube, the viscosity of the fluid, and thNeUraRdSiIuNsGoTf Bth.Ce OtuMbe's lumen. 49. ANS: D PTS: 1 REF: Page 1099 MSC: With the attachment of actin to myosin at the cross-bridge, the myosin head molecule undergoes a position change, exerting traction on the rest of the myosin bridge, causing the thin filaments to slide past the thick filaments. During contraction, each cross-bridge undergoes cycles of attachment, movement, and dissociation from the thin filaments. 50. ANS: E PTS: 1 REF: Page 1101 MSC: The Frank-Starling law states that the cardiac muscle, like other muscles, increases its strength of contraction when it is stretched. 51. ANS: C PTS: 1 REF: Pages 1102-1103 MSC: In Laplace's law, wall tension is directly related to the product of intraventricular pressure and internal radius and inversely to the wall thickness. 52. ANS: B PTS: 1 REF: Page 1106 MSC: The Bainbridge reflex causes changes in the heart rate after intravenous infusions of blood or other fluid. Chapter 33: Alterations of Cardiovascular Function MULTIPLE CHOICE 1. What is the initiating event that leads to the development of atherosclerosis? a. Release of the inflammatory cytokines b. Macrophages adhere to vessel walls. c. Injury to the endothelial cells that line the artery walls d. Release of the platelet-deprived growth factor ANS: C Atherosclerosis begins with an injury to the endothelial cells that line the arterial walls. Possible causes of endothelial injury include the common risk factors for atherosclerosis, such as smoking, hypertension, diabetes, increased levels of low-density lipoprotein (LDL), decreased levels of high-density lipoprotein (HDL), and autoimmunity. The remaining options occur only after the endothelial cells are injured. PTS: 1 REF: Page 1145 2. What is the effect of oxidized low-density lipoproteins (LDLs) in atherosclerosis? a. LDLs cause smooth muscle proliferation. b. LDLs cause regression of atherosclerotic plaques. c. LDLs increase levels of inflammatory cytokines. d. LDLs direct macrophages to the site in the endothelium. ANS: A Oxidized LDLs are toxic to endotNheUlRiaSlIcNeGllTsB, .cCaOuMse smooth muscle proliferation, and activate further immune and inflammatory responses. This selection is the only option that accurately identifies the effects of LDLs. PTS: 1 REF: Page 1145 3. Which inflammatory cytokines are released when endothelial cells are injured? a. Granulocyte-macrophage colony-stimulating factor (GM-CSF) b. Interferon-beta (IFN-), interleukin 6 (IL-6), and granulocyte colony-stimulating factor (G-CSF) c. Tumor necrosis factor–alpha (TNF-), interferon-gamma (IFN-), and interleukin 1 (IL-1) d. Interferon-alpha (IFN-), interleukin-12 (IL-12), and macrophage colony-stimulating factor (M-CSF) ANS: C Numerous inflammatory cytokines are released, including TNF-, IFN-, IL-1, toxic oxygen radicals, and heat shock proteins. This selection is the only option that accurately identifies which inflammatory cytokines are associated with endothelial cell injury. PTS: 1 REF: Page 1145 4. When endothelia cells are injured, what alteration contributes to atherosclerosis? a. The release of toxic oxygen radicals that oxidize low-density lipoproteins (LDLs). b. Cells are unable to make the normal amount of vasodilating cytokines. c. Cells produce an increased amount of antithrombotic cytokines. d. Cells develop a hypersensitivity to homocysteine and lipids. ANS: B Injured endothelial cells become inflamed and cannot make normal amounts of antithrombotic and vasodilating cytokines. This selection is the only option that accurately identifies the factor that contributes to atherosclerosis. PTS: 1 REF: Page 1145 5. Which factor is responsible for the hypertrophy of the myocardium associated with hypertension? a. Increased norepinephrine c. Angiotensin II b. Adducin d. Insulin resistance ANS: C Of the available options, only angiotensin II is responsible for the hypertrophy of the myocardium and much of the renal damage associated with hypertension. PTS: 1 REF: Pages 1132-1138 6. What pathologic change occurs to the kidney’s glomeruli as a result of hypertension? a. Compression of the renal tubules b. Ischemia of the tubule c. Increased pressure from within the tubule d. Obstruction of the renal tubule NURSINGTB.COM ANS: B In the kidney, vasoconstriction and resultant decreased renal perfusion cause tubular ischemia and preglomerular arteriopathy. This selection is the only option that accurately identifies the pathologic change to the kidney that occurs as a result of hypertension. PTS: 1 REF: Pages 1134-1136 7. What effect does atherosclerosis have on the development of an aneurysm? a. Atherosclerosis causes ischemia of the intima. b. It increases nitric oxide. c. Atherosclerosis erodes the vessel wall. d. It obstructs the vessel. ANS: C Atherosclerosis is a common cause of aneurysms because plaque formation erodes the vessel wall. This selection is the only option that accurately identifies the effect that atherosclerosis has on aneurysm development. PTS: 1 REF: Pages 1141-1142 8. Regarding the endothelium, what is the difference between healthy vessel walls and those that promote clot formation? a. Inflammation and roughening of the endothelium of the artery are present. b. Hypertrophy and vasoconstriction of the endothelium of the artery are present. c. Excessive clot formation and lipid accumulation in the endothelium of the artery are present. d. Evidence of age-related changes that weaken the endothelium of the artery are present. ANS: A Invasion of the tunica intima by an infectious agent also roughens the normally smooth lining of the artery, causing platelets to adhere readily. This selection is the only option that accurately describes the mechanism that supports abnormal clot formation. PTS: 1 REF: Pages 1142-1143 9. What is the usual source of pulmonary emboli? a. Deep venous thrombosis c. Valvular disease b. Endocarditis d. Left heart failure ANS: A Pulmonary emboli originate in the venous circulation (mostly from the deep veins of the legs) or in the right heart. This selection is the only option that accurately identifies the usual source of pulmonary emboli. PTS: 1 REF: Page 1143 10. Which factor can trigger an immune response in the bloodstream that may result in an embolus? a. Amniotic fluid c. Bacteria b. Fat d. Air NURSINGTB.COM ANS: A Of the options available, only amniotic fluid displaces blood, thereby reducing oxygen, nutrients, and waste exchange; however, it also introduces antigens, cells, and protein aggregates that trigger inflammation, coagulation, and the immune response in the bloodstream. PTS: 1 REF: Pages 1143-1144 11. Which statement best describes thromboangiitis obliterans (Buerger disease)? a. Inflammatory disorder of small- and medium-size arteries in the feet and sometimes in the hands b. Vasospastic disorder of the small arteries and arterioles of the fingers and, less commonly, of the toes c. Autoimmune disorder of the large arteries and veins of the upper and lower extremities d. Neoplastic disorder of the lining of the arteries and veins of the upper extremities ANS: A Buerger disease is an inflammatory disease of the peripheral arteries. Inflammation, thrombus formation, and vasospasm can eventually occlude and obliterate portions of small- and medium-size arteries. The digital, tibial, and plantar arteries of the feet and the digital, palmar, and ulnar arteries of the hands are typically affected. This selection is the only option that accurately describes Buerger disease. 12. Which statement best describes Raynaud disease? a. Inflammatory disorder of small- and medium-size arteries in the feet and sometimes in the hands b. Neoplastic disorder of the lining of the arteries and veins of the upper extremities c. Vasospastic disorder of the small arteries and arterioles of the fingers and, less commonly, of the toes d. Autoimmune disorder of the large arteries and veins of the upper and lower extremities ANS: C Attacks of vasospasm in the small arteries and arterioles of the fingers and, less commonly, of the toes characterize Raynaud phenomenon and Raynaud disease and is the only option that accurately describes this disease. PTS: 1 REF: Page 1144 13. What change in a vein supports the development of varicose veins? a. Increase in osmotic pressure c. Damage to the venous endothelium b. Damage to the valves in veins d. Increase in hydrostatic pressure ANS: B If a valve is damaged, permitting backflow, then a section of the vein is subjected to the pressure exerted by a larger volume of blood under the influence of gravity. The vein swells as it becomes engorged, and the surrounding tissue becomes edematous because increased hydrostatic pressure pushes plasma through the stretched vessel wall. This selection is the only option that acNcUurRaSteINlyGdTeBs.CcrOibMes the development of varicose veins. PTS: 1 REF: Pages 1129-1130 14. Superior vena cava syndrome is a result of a progressive increase of which process? a. Inflammation c. Distention b. Occlusion d. Sclerosis ANS: B Superior vena cava syndrome (SVCS) is a progressive occlusion of the superior vena cava (SVC) that leads to venous distention in the upper extremities and head. The remaining options are not associated with this disorder. PTS: 1 REF: Page 1131 15. What term is used to identify when a cell is temporarily deprived of blood supply? a. Infarction c. Necrosis b. Ischemia d. Inflammation ANS: B Coronary artery disease (CAD) can diminish the myocardial blood supply until deprivation impairs myocardial metabolism enough to cause ischemia, a local state in which the cells are temporarily deprived of blood supply. This term is the only option that is used to identify a temporarily deprived blood supply. PTS: 1 REF: Page 1148 16. The risk of developing coronary artery disease is increased up to threefold by which factor? a. Diabetes mellitus c. Obesity b. Hypertension d. High alcohol consumption ANS: B Hypertension is the only factor responsible for a twofold-to-threefold increased risk of atherosclerotic cardiovascular disease. PTS: 1 REF: Page 1151 17. Which risk factor is associated with coronary artery disease (CAD) because of its relationship with the alteration of hepatic lipoprotein? a. Diabetes mellitus c. Obesity b. Hypertension d. High alcohol consumption ANS: A Of the available options, only diabetes mellitus is associated with CAD because of the resulting alteration of hepatic lipoprotein synthesis; it increases triglyceride levels and is involved in low-density lipoprotein oxidation. PTS: 1 REF: Pages 1148-1151 18. Nicotine increases atherosclerosis by the release of which neurotransmitter? a. Histamine c. Angiotensin II b. Nitric oxide ANS: D NURSINGTdB..COEMpinephrine Nicotine stimulates the release of catecholamines (e.g., epinephrine, norepinephrine), which increases the heart rate and causes peripheral vascular constriction. As a result, blood pressure increases, as do both cardiac workload and oxygen demand. None of the other options are associated with this mechanism. PTS: 1 REF: Page 1151 19. Which substance is manufactured by the liver and primarily contains cholesterol and protein? a. Very low–density lipoproteins (VLDLs) b. Low-density lipoproteins (LDLs) c. High-density lipoproteins (HDLs) d. Triglycerides ANS: B A series of chemical reactions in the liver results in the production of several lipoproteins that vary in density and function. These include VLDLs, primarily triglycerides and protein; LDLs, mostly cholesterol and protein; and HDLs, mainly phospholipids and protein. LDLs are the only lipoproteins that are manufactured by the liver and primarily contain cholesterol and protein. 20. Which elevated value may be protective of the development of atherosclerosis? a. Very low–density lipoproteins (VLDLs) b. Low-density lipoproteins (LDLs) c. High-density lipoproteins (HDLs d. Triglycerides ANS: C Low levels of HDL cholesterol are also a strong indicator of coronary risk, whereas high levels of HDLs may be more protective for the development of atherosclerosis than low levels of LDLs. Neither VLDLs nor elevated triglycerides are associated with a protective mechanism. PTS: 1 REF: Pages 1149-1151 21. Which laboratory test is an indirect measure of atherosclerotic plaque? a. Homocysteine b. Low-density lipoprotein (LDL) c. Erythrocyte sedimentation rate (ESR) d. C-reactive protein (CRP) ANS: D Highly sensitive CRP (hs-CRP) is an acute phase reactant or protein mostly synthesized in the liver and, of the available options, is an indirect measure of atherosclerotic plaque-related inflammation. PTS: 1 REF: Page 1152 NURSINGTB.COM 22. Cardiac cells can withstand ischemic conditions and still return to a viable state for how many minutes? a. 10 c. 20 b. 15 d. 25 ANS: C Cardiac cells remain viable for approximately 20 minutes under ischemic conditions. If blood flow is restored, then aerobic metabolism resumes, contractility is restored, and cellular repair begins. If the coronary artery occlusion persists beyond 20 minutes, then myocardial infarction (MI) occurs. PTS: 1 REF: Page 1153 23. Which form of angina occurs most often during sleep as a result of vasospasms of one or more coronary arteries? a. Unstable c. Silent b. Stable d. Prinzmetal ANS: D Of the options available, only Prinzmetal angina (also called variant angina) is chest pain attributable to transient ischemia of the myocardium that occurs unpredictably and almost exclusively at rest. PTS: 1 REF: Page 1154 24. When is the scar tissue that is formed after a myocardial infarction (MI) most vulnerable to injury? a. Between 5 and 9 days c. Between 15 and 20 days b. Between 10 and 14 days d. Between 20 and 30 days ANS: B During the recovery period (10 to 14 days after infarction), individuals feel more capable of increasing activities and thus may stress the newly formed scar tissue. After 6 weeks, the necrotic area is completely replaced by scar tissue, which is strong but unable to contract and relax like healthy myocardial tissue. PTS: 1 REF: Page 1160 25. An individual who is demonstrating elevated levels of troponin, creatine kinase–isoenzyme MB (CK-MB), and lactic dehydrogenase (LDH) is exhibiting indicators associated with which condition? a. Myocardial ischemia c. Myocardial infarction (MI) b. Hypertension d. Coronary artery disease (CAD) ANS: C Cardiac troponins (troponin I and troponin T) are the most specific indicators of MI. Other biomarkers released by myocardial cells include CK-MB and LDH, but they are not associated with the other options. PTS: 1 REF: Pages 1160-1161 26. What is the expected electrocardioNgUrRamSIN(EGCTBG.C) OpaMttern when a thrombus in a coronary artery permanently lodges in the vessel and the infarction extends through the myocardium from the endocardium to the epicardium? a. Prolonged QT interval b. ST elevation myocardial infarction (STEMI) c. ST depression myocardial infarction (STDMI) d. Non-ST elevation myocardial infarction (non-STEMI) ANS: B Individuals with this pattern on an ECG usually have significant elevations in the ST segments and are categorized as having STEMI. The other options are not associated with the described pathologic condition. PTS: 1 REF: Pages 1157-1158 27. How does angiotensin II increase the workload of the heart after a myocardial infarction (MI)? a. By increasing the peripheral vasoconstriction b. By causing dysrhythmias as a result of hyperkalemia c. By reducing the contractility of the myocardium d. By stimulating the sympathetic nervous system ANS: A Angiotensin II is released during myocardial ischemia and contributes to the pathogenesis of a myocardial infarction (MI) in several ways. First, it results in the systemic effects of peripheral vasoconstriction and fluid retention. These homeostatic responses are counterproductive in that they increase myocardial work and thus exacerbate the effects of the loss of myocyte contractility. Angiotensin II is also locally released, where it is a growth factor for vascular smooth muscle cells, myocytes, and cardiac fibroblasts; promotes catecholamine release; and causes coronary artery spasm. This selection is the only option that accurately describes how angiotensin II increases workload after a MI. PTS: 1 REF: Page 1159 28. The pulsus paradoxus that occurs as a result of pericardial effusion is caused by a dysfunction in which mechanism? a. Diastolic filling pressures of the right ventricle and reduction of blood volume in both ventricles b. Blood ejected from the right atrium and reduction of blood volume in the right ventricle c. Blood ejected from the left atrium and reduction of blood volume in the left ventricle d. Diastolic filling pressures of the left ventricle and reduction of blood volume in all four heart chambers. ANS: D Pulsus paradoxus means that the arterial blood pressure during expiration exceeds arterial pressure during inspiration by more than 10 mm Hg. This clinical finding reflects impairment of diastolic filling of the left ventricle plus a reduction of blood volume within all four cardiac chambers. This seNleUctRiSoInNiGsTthBe.CoOnMly option that accurately describes the mechanism. PTS: 1 REF: Page 1164 29. A patient reports sudden onset of severe chest pain that radiates to the back and worsens with respiratory movement and when lying down. These clinical manifestations describe: a. Myocardial infarction (MI) c. Restrictive pericarditis b. Pericardial effusion d. Acute pericarditis ANS: D Most individuals with acute pericarditis describe several days of fever, myalgias, and malaise, followed by the sudden onset of severe chest pain that worsens with respiratory movements and with lying down. Although the pain may radiate to the back, it is generally felt in the anterior chest and may be initially confused with the pain of an acute MI. Individuals with acute pericarditis also may report dysphagia, restlessness, irritability, anxiety, and weakness. This selection is the only option with these symptoms. PTS: 1 REF: Page 1163 30. Ventricular dilation and grossly impaired systolic function, leading to dilated heart failure, characterize which form of cardiomyopathy? a. Congestive c. Septal b. Hypertrophic d. Dystrophic ANS: A Only dilated cardiomyopathy (congestive cardiomyopathy) is characterized by ventricular dilation and grossly impaired systolic function, leading to dilated heart failure. PTS: 1 REF: Page 1165 31. A disproportionate thickening of the interventricular septum is the hallmark of which form of cardiomyopathy? a. Dystrophic c. Restrictive b. Hypertrophic d. Dilated ANS: B Only hypertrophic cardiomyopathy is characterized by a thickening of the septal wall, which may cause outflow obstruction to the left ventricle outflow tract. PTS: 1 REF: Page 1166 32. Amyloidosis, hemochromatosis, or glycogen storage disease usually causes which form of cardiomyopathy? a. Infiltrative c. Septal b. Restrictive d. Hypertrophic ANS: B Restrictive cardiomyopathy may occur idiopathically or as a cardiac manifestation of systemic diseases, such as scleroderma, amyloidosis, sarcoidosis, lymphoma, and hemochromatosis, or a number of inherited storage diseases. This characterization is not true of the other forms of cardiomyopathy. NURSINGTB.COM PTS: 1 REF: Page 1167 33. Which condition is a cause of acquired aortic regurgitation? a. Congenital malformation c. Rheumatic fever b. Cardiac failure d. Coronary artery disease (CAD) ANS: C Rheumatic heart disease, bacterial endocarditis, syphilis, hypertension, connective tissue disorders (e.g., Marfan syndrome, ankylosing spondylitis), appetite suppressing medications, trauma, or atherosclerosis can cause acquired aortic regurgitation. This selection is the only available option that is known to cause acquired aortic regurgitation. PTS: 1 REF: Page 1169 34. Which predominantly female valvular disorder is thought to have an autosomal dominant inheritance pattern, as well as being associated with connective tissue disease? a. Mitral valve prolapse c. Tricuspid valve prolapse b. Tricuspid stenosis d. Aortic insufficiency ANS: A Mitral valve prolapse tends to be most prevalent in young women. Studies suggest an autosomal dominant and X-linked inheritance pattern. Because mitral valve prolapse often is associated with other inherited connective tissue disorders (e.g., Marfan syndrome, Ehlers-Danlos syndrome, osteogenesis imperfecta), it is thought to result from a genetic or environmental disruption of valvular development during the fifth or sixth week of gestation. This provided history is not associated with any of the other options. PTS: 1 REF: Page 1170 35. Which disorder causes a transitory truncal rash that is nonpruritic and pink with erythematous macules that may fade in the center, making them appear as a ringworm? a. Fat emboli b. Rheumatic fever c. Bacterial endocarditis d. Myocarditis of acquired immunodeficiency syndrome ANS: B Erythema marginatum is a distinctive truncal rash that often accompanies acute rheumatic fever. It consists of nonpruritic, pink erythematous macules that never occur on the face or hands. This presentation is not associated with any of the other options. PTS: 1 REF: Page 1172 36. What is the most common cause of infective endocarditis? a. Virus c. Bacterium b. Fungus d. Rickettsiae ANS: C NURSINGTB.COM Infective endocarditis is a general term used to describe infection and inflammation of the endocardium—especially the cardiac valves. Bacteria are the most common cause of infective endocarditis, especially streptococci, staphylococci, or enterococci. PTS: 1 REF: Page 1173 37. What is the most common cardiac disorder associated with acquired immunodeficiency syndrome (AIDS) a. Cardiomyopathy c. Left heart failure b. Myocarditis d. Heart block ANS: C Pericardial effusion and left heart failure are the most common complications of human immunodeficiency virus (HIV) infection. Other conditions include cardiomyopathy, myocarditis, tuberculous pericarditis, infective and nonbacterial endocarditis, heart block, pulmonary hypertension, and nonantiretroviral drug-related cardiotoxicity. PTS: 1 REF: Page 1175 38. A patient is diagnosed with pulmonary disease and elevated pulmonary vascular resistance. Which form of heart failure may result from pulmonary disease and elevated pulmonary vascular resistance? a. Right heart failure c. Low-output failure b. Left heart failure d. High-output failure ANS: A Right heart failure is defined as the inability of the right ventricle to provide adequate blood flow into the pulmonary circulation at a normal central venous pressure. This condition is often a result of pulmonary disease and the resulting elevated pulmonary vascular resistance. PTS: 1 REF: Page 1181 39. What cardiac pathologic condition contributes to ventricular remodeling? a. Left ventricular hypertrophy c. Myocardial ischemia b. Right ventricular failure d. Contractile dysfunction ANS: C Of the options available, myocardial ischemia contributes to inflammatory, immune, and neurohumoral changes that mediate a process called ventricular remodeling. PTS: 1 REF: Page 1175 40. In systolic heart failure, what effect does the renin-angiotensin-aldosterone system (RAAS) have on stroke volume? a. Increases preload and decreases afterload. b. Increases preload and increases afterload. c. Decreases preload and increases afterload. d. Decreases preload and decreases afterload. ANS: B Activation of the RAAS not only causes an increase in preload and afterload, but it also NURSINGTB.COMcauses direct toxicity to the myoc tion is the only option that accurately identifies the effect that the RAAS has on stroke volume in this situation. PTS: 1 REF: Page 1175 | Page 1177 41. What is the cause of the dyspnea resulting from a thoracic aneurysm? a. Pressure on surrounding organs c. Formation of atherosclerotic lesions b. Poor oxygenation d. Impaired blood flow ANS: A Clinical manifestations depend on the location of the aneurysm. Pressure of a thoracic aneurysm on surrounding organs cause symptoms of dysphagia (difficulty in swallowing) and dyspnea (breathlessness). This selection is the only option that accurately describes the cause of dyspnea resulting from a thoracic aneurysm. PTS: 1 REF: Page 1142 42. Which statement is true concerning the cells’ ability to synthesize cholesterol? a. Cell production of cholesterol is affected by the aging process. b. Cells produce cholesterol only when dietary fat intake is low. c. Most body cells are capable of producing cholesterol. d. Most cholesterol produced by the cells is converted to the low-density form. ANS: C Although cholesterol can easily be obtained from dietary fat intake, most body cells can also manufacture cholesterol. This selection is the only option that accurately describes the cellular role in cholesterol synthesis. PTS: 1 REF: Page 1149 43. What is the trigger for angina pectoris? a. Atherosclerotic lesions c. Myocardial necrosis b. Hyperlipidemia d. Myocardial ischemia ANS: D Angina pectoris is chest pain caused by myocardial ischemia. None of the other options are considered triggers for angina pectoris. PTS: 1 REF: Page 1154 44. Individuals being effectively managed for type 2 diabetes mellitus often experience a healthy decline in blood pressure as a result of what intervention? a. Managed carbohydrate intake b. Appropriate exercise c. Insulin-sensitivity medication therapy d. Introduction of minimal doses of insulin ANS: C Many people with type 2 diabetes mellitus, who are treated with drugs that increase insulin sensitivity, experience a decline in their blood pressure without taking antihypertensive drugs. Although the other medications may be included in the management plan, the other options are not associated with a dNeUcrReSaIsNeGiTnBh.CypOeMrtension. PTS: 1 REF: Page 1136 MULTIPLE RESPONSE 45. Which statements are true regarding fatty streaks? (Select all that apply.) a. Fatty streaks progressively damage vessel walls. b. Fatty streaks are capable of producing toxic oxygen radials. c. When present, inflammatory changes occur to the vessel walls. d. Oxidized low-density lipoproteins (LDLs) are involved in their formation. e. Fatty streaks are formed by killer T cells filled with oxidized LDLs. ANS: A, B, C, D The oxidized LDLs penetrate the intima of the arterial wall and are engulfed by macrophages. Macrophages filled with oxidized LDLs are called foam. Once these lipid-laden foam cells accumulate in significant amounts, they form a lesion called a fatty streak. Once formed, fatty streaks produce more toxic oxygen radicals and cause immunologic and inflammatory changes, resulting in progressive damage to the vessel wall. PTS: 1 REF: Page 1145 | Page 1147 46. What factors contribute to the development of orthostatic hypotension? (Select all that apply.) a. Altered body chemistry b. Drug action of certain antihypertensive agents c. Prolonged immobility d. Effects of aging on postural reflexes e. Any condition that produces volume overload ANS: A, B, C, D Orthostatic hypotension may be acute or chronic. Acute orthostatic hypotension (temporary type) may result from (1) altered body chemistry, (2) drug action (e.g., antihypertensives, antidepressants), (3) prolonged immobility caused by illness, (4) starvation, (5) physical exhaustion, (6) any condition that produces volume depletion (e.g., massive diuresis, potassium or sodium depletion), and (7) venous pooling (e.g., pregnancy, extensive varicosities of the lower extremities). Older adults are susceptible to this type of orthostatic hypotension, in which postural reflexes are slowed as part of the aging process. PTS: 1 REF: Page 1140 47. Which assessment findings are clinical manifestations of aortic stenosis? (Select all that apply.) a. Jugular vein distention b. Bounding pulses c. Hypotension d. Angina e. Syncope ANS: D, E NURSINGTB.COM The classic manifestations of aortic stenosis are angina, syncope, and heart failure. None of the other options are associated with aortic stenosis. PTS: 1 REF: Pages 1168-1169 48. Which risk factors are associated with infective endocarditis? (Select all that apply.) a. Rheumatic fever b. Intravenous drug use c. Long-term indwelling catheterization d. Aortic regurgitation e. Heart valve disease ANS: B, C, E Risk factors for infective endocarditis include acquired valvular heart disease, intravenous drug abuse, long-term indwelling catheterization (e.g., for pressure monitoring, hyperalimentation, or hemodialysis), and recent cardiac surgery. Neither rheumatic fever nor aortic regurgitation is considered a risk factor for infective endocarditis. PTS: 1 REF: Page 1173 | Box 32-3 MATCHING Match the descriptions with the corresponding terms. A. Impairs flow from left atrium to left ventricle B. Impairs flow from the left ventricle C. Backflow into left atrium D. Backflow into right atrium E. Backflow into left ventricle 49. Aortic stenosis 50. Aortic regurgitation 51. Mitral stenosis 52. Tricuspid regurgitation 53. Mitral regurgitation 49. ANS: B PTS: 1 REF: Page 1168 MSC: Outflow obstruction increases pressure within the left ventricle as it tries to eject blood through the narrowed opening. Left ventricular hypertrophy develops to compensate for the increased workload. 50. ANS: E PTS: 1 REF: Pages 1169-1170 MSC: During systole, blood is ejected from the left ventricle into the aorta. If the aortic semilunar valve fails to close completely, then some of the ejected blood flows back into the left ventricle during diastole. 51. ANS: A PTS: 1 REF: Page 1169 MSC: Mitral stenosis impairs the flow of blood from the left atrium to the left ventricle. 52. ANS: D PTS: 1 REF: Page 1170 MSC: Tricuspid regurgitation is more common than tricuspid stenosis and is usually associated with cardiac failure and dilation of the right ventricle, secondary to pulmonary hypertension. 53. ANS: C PTS: 1 REF: Page 1170 MSC: Mitral regurgitation permits the backflow of blood from the left ventricle into the left atrium during ventricular systole, givNinUgRrSisIeNGtoTaBl.oCuOdMpansystolic (throughout systole) murmur heard best at the apex that radiates into the back and axillae. Chapter 34: Alterations of Cardiovascular Function in Children MULTIPLE CHOICE 1. Most cardiovascular developments occur between which weeks of gestation? a. Fourth and seventh weeks c. Twelfth and fourteenth weeks b. Eighth and tenth weeks d. Fifteenth and seventeenth weeks ANS: A Cardiogenesis begins at approximately 3 weeks’ gestation; however, most cardiovascular development occurs between 4 and 7 weeks’ gestation. PTS: 1 REF: Page 1194 2. The function of the foramen ovale in a fetus allows what to occur? a. Right-to-left blood shunting c. Blood flow from the umbilical cord b. Left-to-right blood shunting d. Blood flow to the lungs ANS: A The nonfused septum secundum and ostium secundum result in the formation of a flapped orifice known as the foramen ovale, which allows the right-to-left shunting necessary for fetal circulation. The foramen ovale is not involved in the blood flow described by the other options. PTS: 1 REF: Pages 1195-1196 3. At birth, which statement is true?NURSINGTB.COM a. Systemic resistance and pulmonary resistance fall. b. Gas exchange shifts from the placenta to the lung. c. Systemic resistance falls and pulmonary resistance rises. d. Systemic resistance and pulmonary resistance rise. ANS: B From the available options, the only change that takes place in the circulation at birth is the shift of gas exchange from the placenta to the lungs. PTS: 1 REF: Page 1197 4. When does systemic vascular resistance in infants begin to increase? a. One month before birth b. During the beginning stage of labor c. One hour after birth d. Once the placenta is removed from circulation ANS: D The low-resistance placenta is removed from circulation, which causes an immediate increase in systemic vascular resistance to approximately twice of that before birth. PTS: 1 REF: Page 1197 5. Which event triggers congenital heart defects that cause acyanotic congestive heart failure? a. Right-to-left shunts c. Obstructive lesions b. Left-to-right shunts d. Mixed lesions ANS: B Congenital heart defects that cause acyanotic congestive heart failure usually involve left-to-right shunts (see Table 33-4). Acyanotic congestive heart failure does not involve any of the other options. PTS: 1 REF: Pages 1201-1202 | Table 33-4 6. Older children with an unrepaired cardiac septal defect experience cyanosis because of which factor? a. Right-to-left shunts c. Obstructive lesions b. Left-to-right shunts d. Mixed lesions ANS: A Older children who have an unrepaired septal defect with a left-to-right shunt may become cyanotic because of pulmonary vascular changes secondary to increased pulmonary blood flow. None of the other options accurately describe the process that results in cyanosis. PTS: 1 REF: Page 1202 7. Which congenital heart defects occur in trisomy 13, trisomy 18, and Down syndrome? a. Coarctation of the aorta (COA) and pulmonary stenosis (PS) b. Tetralogy of Fallot and persistent truncus arteriosus c. Atrial septal defect (ASD) andNdUeRxStrINocGaTrBd.iCaOM d. Ventricular septal defect (VSD) and patent ductus arteriosus (PDA) ANS: D Congenital heart defects that are related to dysfunction of trisomy 13, trisomy 18, and Down syndrome include VSD and PDA (see Table 33-2). The other defects are not associated with dysfunction of trisomy 13, trisomy 18, and Down syndrome. PTS: 1 REF: Page 1200 | Table 33-2 8. An infant has a continuous machine-type murmur best heard at the left upper sternal border throughout systole and diastole, as well as a bounding pulse and a thrill on palpation. These clinical findings are consistent with which congenital heart defect? a. Atrial septal defect (ASD) c. Patent ductus arteriosus (PDA) b. Ventricular septal defect (VSD) d. Atrioventricular canal (AVC) defect ANS: C If pulmonary vascular resistance has fallen, then infants with PDA will characteristically have a continuous machine-type murmur best heard at the left upper sternal border throughout systole and diastole. If the PDA is significant, then the infant also will have bounding pulses, an active precordium, a thrill on palpation, and signs and symptoms of pulmonary overcirculation. The presentations of the other congenital heart defects are not consistent with the described the symptoms. PTS: 1 REF: Pages 1203-1204 9. An infant has a crescendo-decrescendo systolic ejection murmur located between the second and third intercostal spaces along the left sternal border. A wide fixed splitting of the second heart sound is also found. These clinical findings are consistent with which congenital heart defect? a. Atrial septal defect (ASD) c. Patent ductus arteriosus (PDA) b. Ventricular septal defect (VSD) d. Atrioventricular canal (AVC) defect ANS: A Because most children with ASD are asymptomatic, diagnosis is usually made during a routine physical examination by the auscultation of a crescendo-decrescendo systolic ejection murmur that reflects increased blood flow through the pulmonary valve. The location of the murmur is between the second and third intercostal spaces along the left sternal border. A wide fixed splitting of the second heart sound is also characteristic of ASD, reflecting volume overload to the right ventricle and causing prolonged ejection time and a delay of pulmonic valve closure. The presentations of other congenital heart defects are not consistent with the described symptoms. PTS: 1 REF: Pages 1204-1205 10. An infant has a loud, harsh, holosystolic murmur and systolic thrill that can be detected at the left lower sternal border that radiates to the neck. These clinical findings are consistent with which congenital heart defect? a. Atrial septal defect (ASD) c. Patent ductus arteriosus (PDA) b. Ventricular septal defect (VSD) d. Atrioventricular canal (AVC) defect ANS: B On physical examination, a loud, NhaUrRshS,INhGolToBs.yCsOtoMlic murmur and systolic thrill can be detected at the left lower sternal border. The intensity of the murmur reflects the pressure gradient across the VSD. An apical diastolic rumble may be present with a moderate-to-large defect, reflecting increased flow across the mitral valve. The presentations of the other congenital heart defects are not consistent with the described symptoms. PTS: 1 REF: Page 1205 11. Where can coarctation of the aorta (COA) be located? a. Exclusively on the aortic arch b. Proximal to the brachiocephalic artery c. Between the origin of the aortic arch and the bifurcation of the aorta in the lower abdomen d. Between the origin of the aortic arch and the origin of the first intercostal artery ANS: C COA can occur anywhere between the origin of the aortic arch and the bifurcation of the aorta in the lower abdomen. The other options do not accurately describe the location of a COA. PTS: 1 REF: Page 1210 12. Classic manifestations of a systolic ejection murmur heard at the left interscapular area, cool mottled skin on the lower extremities but hypertension noted in the upper extremities, and decreased or absent femoral pulse are indicative of an older child with which congenital defect? a. Tetralogy of Fallot c. Ventricular septum defect (SD) b. Aortic stenosis d. Coarctation of the aorta (OA) ANS: D Clinical manifestations of coarctation of the aorta include hypertension noted in the upper extremities with decreased or absent pulses in the lower extremities. Children may also have cool mottled skin and occasionally experience leg cramps during exercise. A systolic ejection murmur, heard best at the left interscapular area, is also considered a classic clinical manifestation of this disorder. The other options are not initially associated with these symptoms. PTS: 1 REF: Pages 1210-1212 13. What is the initial manifestation of aortic coarctation observed in a neonate? a. Congestive heart failure (CHF) c. Pulmonary hypertension b. Cor pulmonale d. Cerebral hypertension ANS: A Initially, the newborn usually exhibits symptoms of CHF. The other options are not initially associated with aortic coarctation. PTS: 1 REF: Page 1212 14. Which compensatory mechanism NisUsRpSoInNtGanTeBo.CuOslMy used by children diagnosed with tetralogy of Fallot to relieve hypoxic spells? a. Lying on their left side c. Squatting b. Performing the Valsalva maneuver d. Hyperventilating ANS: C Squatting is a spontaneous compensatory mechanism used by older children to alleviate hypoxic spells. Squatting and its variants increase systemic resistance while decreasing venous return to the heart from the inferior vena cava. The other options would not result in these changes. PTS: 1 REF: Page 1209 15. An infant diagnosed with a small patent ductus arteriosus (PDA) would likely exhibit which symptom? a. Intermittent murmur c. Need for surgical repair b. Lack of symptoms d. Triad of congenital defects ANS: B Infants with a small PDA usually remain asymptomatic; the other options are incorrect. PTS: 1 REF: Pages 1203-1204 16. What is the most common cause of chronic sustained hypertension observed only in a newborn? a. Renal parenchymal disease c. Renal artery stenosis b. Primary hypertension d. Congenital renal malformation ANS: D Congenital renal malformation is a cause of chronic sustained hypertension in a newborn. Although renal artery stenosis is observed in newborns, it is also observed in older children. Renal parenchymal disease and primary hypertension are commonly observed in older children diagnosed with chronic sustained hypertension. PTS: 1 REF: Page 1220 | Table 33-8 17. Which condition is consistent with the cardiac defect of transposition of the great vessels? a. The aorta arises from the right ventricle. b. The pulmonary trunk arises from the right ventricle. c. The right ventricle pumps blood to the lungs. d. An intermittent murmur is present. ANS: A Transposition of the great arteries refers to a condition in which the aorta arises from the right ventricle and the pulmonary artery arises from the left ventricle. A transposition of the great vessels is not associated with any of the other options. PTS: 1 REF: Pages 1214-1215 18. Which scenario describes total anomalous pulmonary venous return? a. The foramen ovale closes after birth. b. Pulmonary venous return is to the right atrium. c. Pulmonary venous return is toNtUheRSleINftGaTtrBiu.CmO.M d. The left atrium receives oxygenated blood. ANS: B Total anomalous pulmonary venous return occurs when the pulmonary veins abnormally connect to the right side of the heart either directly or through one or more systemic veins that drain into the right atrium. None of the other options accurately describe the presentation of a total anomalous pulmonary venous return. PTS: 1 REF: Page 1216 19. Which heart defect produces a systolic ejection murmur at the right upper sternal border that transmits to the neck and left lower sternal border? a. Coarctation of the aorta c. Aortic stenosis b. Pulmonic stenosis d. Hypoplastic left heart syndrome ANS: C Blood flow through the stenotic area of the aorta produces a systolic ejection murmur at the right upper sternal border that transmits to the neck and left lower sternal border. None of the other options produce the described assessment findings. PTS: 1 REF: Page 1212 20. Which heart defect produces a systolic ejection click at the upper left sternal border with a thrill palpated at the upper left sternal border? a. Coarctation of the aorta (COA) c. Aortic stenosis b. Pulmonary stenosis (PS) d. Hypoplastic left heart syndrome ANS: B PS results in a systolic ejection murmur at the left upper sternal border, reflecting an obstruction to flow through the narrowed pulmonary valve. A variable systolic ejection click is present in some children, as well as valvular stenosis at the upper left sternal border. PS also produces a thrill that may be palpated at the upper left sternal border. None of the other options produce the described assessment findings. PTS: 1 REF: Page 1213 21. Which heart defect results in a single vessel arising from both ventricles, providing blood to both the pulmonary and systemic circulations? a. Coarctation of the aorta b. Tetralogy of Fallot c. Total anomalous pulmonary connection d. Truncus arteriosus ANS: D Truncus arteriosus is the failure of the large embryonic artery, the truncus arteriosus, to divide into the pulmonary artery and the aorta, which results in a single vessel arising from both ventricles, providing blood flow to the pulmonary and systemic circulations. None of the other options produce the described structural malformation. PTS: 1 REF: Page 1217 22. What is the suggested mean bloodNpUrResSsINurGeTfBo.rCaOnM8- to 9-year-old child? a. 104/55 mm Hg c. 112/62 mm Hg b. 106/58 mm Hg d. 121/70 mm Hg ANS: B The suggested mean blood pressure for an 8- to 9- year-old child is 106/58 mm Hg. For a child of 6 to 7 years old, 104/55 mm Hg is appropriate; for a 12- to 13-year-old child, 112/62 mm Hg is appropriate, and for a 16- to 18-year-old young man, 121/70 mm Hg is appropriate. PTS: 1 REF: Page 1220 | Table 33-6 MULTIPLE RESPONSE 23. What congenital heart defects are associated with intrauterine exposure to rubella? (Select all that apply.) a. Pulmonary stenosis (PS) b. Cardiomegaly c. Patent ductus arteriosus (PDA) d. Coarctation of aorta (COA) e. Ventricular septal defect (VSD) ANS: A, C, D PS, PDA, and COA are congenital heart defects associated with intrauterine exposure to rubella. Cardiomegaly and VSD are associated with maternal diabetes. PTS: 1 REF: Page 1199 | Table 33-1 24. Which symptoms meet the diagnostic criteria for Kawasaki disease in a child? (Select all that apply.) a. Fever for 5 days or longer b. “Strawberry tongue” c. Peripheral edema d. Inguinal lymphadenopathy e. Bilateral conjunctival infection ANS: A, B, C, E The child must exhibit five of the following six criteria: (1) fever for 5 days or longer, (2) bilateral conjunctival infection without exudation, (3) changes in oral mucus such as strawberry tongue, (4) a polymorphous rash, (5) cervical lymphadenopathy, and (6) changes in the extremities such as peripheral edema. PTS: 1 REF: Page 1218 | Box 33-3 25. Which statements related to the ambulatory blood pressure monitoring (ABPM) system with children are true? (Select all that apply.) a. ABPM monitors blood pressure for a 24-hour period. b. ABPM assists in identifying children with white coat hypertension. c. ABPM is effective in identifying children at risk for target organ damage d. ABPM assists in identifying children who demonstrate masked hypertension. e. ABPM is effective in determinNiUngRSbIlNoGodTBp.rCeOssMure load or hypertension for at least 48 hours. ANS: A, B, C, D ABPM records blood pressure over a 24-hour period to help identify those children with white coat hypertension and masked hypertension. ABPM is useful in documenting the blood pressure load, which is the total amount of time the blood pressure is elevated above normal limits during a 24-hour period. By measuring blood pressure load, the ABPM may be able to identify those children who are at greatest risk for target organ damage. PTS: 1 REF: Page 1221 | What's New box MATCHING Match the phrases with the corresponding terms. A. Causes atrial separation B. Gap between the septum primum and the septum secundum C. Conal portion of the ventricular septum D. Abnormal communication between the atria E. Allows right-to-left shunting 26. Atrial septal defect 27. Foramen ovale 28. Septum secundum 29. Ostium primum 30. Bulbus cordis 26. ANS: D PTS: 1 REF: Page 1196 MSC: An atrial septal defect is an abnormal communication between the atria. 27. ANS: E PTS: 1 REF: Pages 1195-1196 MSC: The nonfused septum secundum and ostium secundum result in the formation of a flapped orifice known as the foramen ovale, which allows the right-to-left shunting necessary for fetal circulation. 28. ANS: A PTS: 1 REF: Pages 1195-1196 MSC: The septum secundum is also a fenestrated, membranelike structure located anteriorly that grows toward the endocardial cushions. During fetal development, this structure does not completely fuse with the endocardial cushions, which results in atrial separation. 29. ANS: B PTS: 1 REF: Page 1195 MSC: The septum primum forms along the posterior wall of the common atrium and grows downward toward the septum secundum. The gap between the two structures, known as the ostium primum, normally closes by extensions from the endocardial cushions. 30. ANS: C PTS: 1 REF: Page 1196 MSC: The conal portion of the ventricular septum that separates the aorta from the pulmonary artery forms from the bulbus cordis. NURSINGTB.COM Chapter 35: Structure and Function of the Pulmonary System MULTIPLE CHOICE 1. What pulmonary defense mechanism propels a mucous blanket that entraps particles moving toward the oropharynx? a. Nasal turbinates c. Cilia b. Alveolar macrophages d. Irritant receptors on the nares ANS: C The submucosal glands of the bronchial lining produce mucus, contributing to the mucous blanket that covers the bronchial epithelium. The ciliated epithelial cells rhythmically beat this mucous blanket toward the trachea and pharynx, where it can be swallowed or expectorated by coughing. This selection is the only option that accurately identifies the pulmonary defense mechanism described. PTS: 1 REF: Page 1229 2. Which term is used to identify the movement of gas and air into and out of the lungs? a. Perfusion c. Respiration b. Ventilation d. Diffusion ANS: B Of the options available, ventilation is the only term used to identify the mechanical movement of gas or air into and out of the lungs. PTS: 1 REF: Page 12N3U2RSINGTB.COM 3. When an individual aspirates food particles, where would the nurse expect to hear decreased or absent breath sounds? a. Left lung c. Trachea b. Right lung d. Carina ANS: B The right mainstem bronchus extends from the trachea more vertically than the left main bronchus; therefore aspirated fluids or foreign particles tend to enter the right lung rather than the left or any of the other locations listed. PTS: 1 REF: Page 1228 4. Aspiration is most likely to occur in the right mainstem bronchus because it: a. Extends vertically from the trachea. b. Is narrower than the left mainstem bronchus. c. Comes into contact with food and drink first. d. Is located at the site where the bronchi bifurcate. ANS: A The right mainstem bronchus extends from the trachea more vertically than the left mainstem bronchus; therefore aspirated fluids or foreign particles tend to enter the right lung rather than the left. The size of both mainstems is equal. The trachea comes into contact with food and drink first, and the carina is the site where the bronchi bifurcate. PTS: 1 REF: Page 1228 5. Air passage among alveoli is collateral and evenly distributed because of the function of which structures? a. Type I alveolar cells c. Acinus pores b. Pores of Kohn d. Alveolar pores ANS: B Tiny passages called pores of Kohn permit some air to pass through the septa from alveolus to alveolus, promoting collateral ventilation and even distribution of air among the alveoli. This selection is the only option that accurately describes the function that allows air passage among alveoli. PTS: 1 REF: Page 1229 6. Where in the lung does gas exchange occur? a. Trachea c. Alveolocapillary membrane b. Segmental bronchi d. Main bronchus ANS: C Gas exchange occurs only across the alveolocapillary membrane. PTS: 1 REF: Page 1230 7. Surfactant produced by type II alveolar cells facilitates alveolar distention and ventilation by which mechanism? a. Decreasing thoracic compliancNeURSINGTB.COM b. Attracting water to the alveolar surface c. Decreasing surface tension in the alveoli d. Increasing surface tension in the alveoli ANS: C Surfactant, a lipoprotein produced by type II alveolar cells, has a detergent-like effect that separates the liquid molecules, thereby decreasing alveolar surface tension. This selection is the only option that accurately describes the mechanism that allows surfactant to facilitate alveolar distention and ventilation. PTS: 1 REF: Pages 1235-1236 8. Which part of the brainstem provides basic automatic rhythm of respiration by sending efferent impulses to the diaphragm and intercostal muscles? a. Dorsal respiratory group (DRG) c. Pneumotaxic center b. Ventral respiratory group d. Apneustic center ANS: A The basic automatic rhythm of respiration is set by the DRG, a cluster of inspiratory nerve cells located in the medulla that sends efferent impulses to the diaphragm and inspiratory intercostal muscles. This selection is the only option that accurately identifies the appropriate brainstem location. PTS: 1 REF: Page 1233 9. Which structures secrete surfactant? a. Type I alveolar cells c. Alveolar macrophages b. Type II alveolar cells d. Stretch receptors ANS: B Two major types of epithelial cells appear in the alveolus. Type I alveolar cells provide structure, and type II alveolar cells secrete surfactant, a lipoprotein that coats the inner surface of the alveolus and facilitates its expansion during inspiration, lowers alveolar surface tension at end-expiration, and thereby prevents lung collapse. Neither alveolar macrophages nor stretch receptors secrete surfactant. PTS: 1 REF: Page 1229 10. Which structure is not associated with any lymphatic vessels? a. Trachea c. Acinus b. Bronchi d. Terminal bronchioles ANS: C No lymphatic structures are located in the acinus. The other options are associated with lymphatic vessels. PTS: 1 REF: Page 1230 11. Which describes the pressure in the pleural space? a. Atmospheric c. Above atmospheric b. Below atmospheric d. Variable ANS: B NURSINGTB.COM Pressure in the pleural space is usually negative or subatmospheric (4 to 10 mm Hg). This selection is the only option that accurately describes pleural space pressure. PTS: 1 REF: Page 1231 12. The adequacy of a person’s alveolar ventilation is assessed best by monitoring which mechanism? a. Ventilatory rate c. Respiratory effort b. Ventilatory pattern d. Arterial blood gas ANS: D Observation of the ventilatory rate, pattern, or effort cannot determine the adequacy of alveolar ventilation. If a health care professional needs to determine the adequacy of ventilation, then an arterial blood gas analysis must be performed to measure partial pressure of arterial carbon dioxide (PaCO2). PTS: 1 REF: Page 1232 13. Which normal physiologic change occurs in the aging pulmonary system? a. Decreased flow resistance c. Stiffening of the chest wall b. Fewer alveoli d. Improved elastic recoil ANS: C Normal alterations include (1) loss of elastic recoil, (2) stiffening of the chest wall, (3) alterations in gas exchange, and (4) increases in flow resistance (see Figure 34-18). The number of alveoli is not affected by age. PTS: 1 REF: Page 1244 14. How is most of the oxygen in the blood transported? a. Dissolved in plasma c. In the form of carbon dioxide (CO2) b. Bound to hemoglobin d. Bound to protein ANS: B Oxygen is transported in the blood in two forms. A small amount dissolves in plasma, and the remainder binds to hemoglobin molecules. The other options are not involved in this process. PTS: 1 REF: Page 1240 15. Stretch receptors and peripheral chemoreceptors send afferent impulses regarding ventilation to which location in the brain? a. Pneumotaxic center in the pons b. Apneustic center in the pons c. Dorsal respiratory group (DRG) in the medulla oblongata d. Ventral respiratory group (VRG) in the medulla oblongata ANS: C The respiratory center is made up of several groups of neurons located bilaterally in the brainstem: the DRG, the VRG, the pneumotaxic center, and the apneustic center. Of the options available, only the DRG gNroUuRpSIiNnGthTeB.mCOedMulla oblongata receives afferent impulses in the situation described. PTS: 1 REF: Page 1234 16. Which substances cause airway epithelium to constrict? a. Epinephrine and acetylcholine c. Bradykinin and thromboxane A b. Histamine and prostaglandin d. Leukotrienes and prostacyclin ANS: B Constriction occurs if the irritant receptors in the airway epithelium are stimulated by irritants in inspired air, by endogenous substances (e.g., histamine, serotonin, prostaglandins), by many drugs, and by humoral substances. Of the options available, only histamine and prostaglandin cause constriction. PTS: 1 REF: Page 1234 17. If a patient develops acidosis, the nurse would expect the oxyhemoglobin dissociation curve to react in which manner? a. Shift to the right, causing more oxygen (O2) to be released to the cells b. Shift to the left, allowing less O2 to be released to the cells c. Show no change, allowing the O2 concentration to remain stable d. Show dramatic fluctuation, allowing the O2 concentration to increase ANS: A A shift to the right depicts hemoglobin’s decreased affinity for O2 or an increase in the ease with which oxyhemoglobin dissociates and O2 moves into the cells. The oxyhemoglobin dissociation curve is shifted to the right by acidosis (low pH) and hypercapnia (increased partial pressure of arterial carbon dioxide [PaCO2]). This selection is the only option that accurately identifies what will happen to the oxyhemoglobin dissociation curve if acidosis occurs. PTS: 1 REF: Pages 1241-1243 18. How is most carbon dioxide (CO2) in the blood transported? a. Attached to oxygen c. Combined with albumin b. In the form of bicarbonate d. Dissolved in the plasma ANS: B Approximately 60% of the CO2 in venous blood and 90% of the CO2 in arterial blood are carried in the form of bicarbonate. PTS: 1 REF: Page 1243 19. The sternocleidomastoid and scalene muscles are referred to as which group? a. Diaphragmatic muscles c. Intercostal muscles b. Muscles of expiration d. Muscles of inspiration ANS: A The accessory muscles of inspiration are the sternocleidomastoid and scalene muscles. These muscles are not associated with the other options. PTS: 1 REF: Page 12N3U5RSINGTB.COM 20. An increase in surface tension caused by decreased surfactant production results in which alteration? a. Decrease in alveolar macrophage production b. Increase in lung compliance c. Decrease in alveoli collapse d. Increase in alveoli fluid collection ANS: D The decrease in surface tension caused by surfactant is also responsible for keeping the alveoli free of fluid. In the absence of surfactant, the surface tension tends to attract fluid into the alveoli. If surfactant production is disrupted or surfactant is not produced in adequate quantities, then the alveolar surface tension increases, causing alveolar collapse, decreased lung expansion, increased work of breathing, and severe gas-exchange abnormalities. The decrease in surface tension caused by surfactant is also responsible for keeping the alveoli free of fluid. The remaining options are not associated with decreased surfactant production. PTS: 1 REF: Pages 1235-1236 21. Decreased lung compliance means that the lungs are demonstrating which characteristic? a. Difficult deflation c. Stiffness b. Easy inflation d. Inability to diffuse oxygen ANS: C A decrease in compliance indicates that the lungs or chest wall is abnormally stiff or difficult to inflate. This selection is the only option that accurately identifies the meaning of decreased compliance. PTS: 1 REF: Page 1236 22. The lung is innervated by the parasympathetic nervous system via which nerve? a. Vagus c. Brachial b. Phrenic d. Pectoral ANS: A Fibers of the parasympathetic division of the autonomic nervous system (ANS) travel only in the vagus nerve to the lung. PTS: 1 REF: Page 1234 23. What event is characteristic of the function in Zone 1 of the lung? a. Blood flow through the pulmonary capillary bed increases in regular increments. b. Alveolar pressure is greater than venous pressure but not greater than arterial pressure. c. The capillary bed collapses, and normal blood flow ceases. d. Blood flows through Zone 1, but it is impeded to a certain extent by alveolar pressure. ANS: C Alveolar pressure exceeds pulmonary arterial and venous pressures in Zone 1. The capillary bed collapses, and normaNlUbRloSoINdGfTloBw.COceMases. Zone II is the portion where alveolar pressure is greater than venous pressure but not greater than arterial pressure. Blood flows through zone II, but it is impeded to a certain extent by alveolar pressure. Zone II is normally above the level of the left atrium. In zone III, arterial and venous pressures are greater than alveolar pressure and blood flow is not affected by alveolar pressure. Zone III is in the base of the lung. Blood flow through the pulmonary capillary bed increases in regular increments from the apex to the base. PTS: 1 REF: Pages 1239-1240 24. Hypoventilation that results in the retention of carbon dioxide will stimulate which receptors in an attempt to maintain a normal homeostatic state? a. Irritant receptors c. Peripheral chemoreceptors b. Central chemoreceptors d. Stretch receptors ANS: B Central chemoreceptors indirectly monitor arterial blood by sensing changes in the pH of cerebrospinal fluid (CSF). The central chemoreceptors are sensitive to very small changes in the pH of CSF (equivalent to a 1 to 2 mm Hg change in partial pressure of carbon dioxide [PCO2]) and are able to maintain a normal partial pressure of arterial carbon dioxide (PaCO2) under many different conditions, including strenuous exercise. This selection is the only option that accurately identifies the receptors that are associated with the retention of carbon dioxide. 25. What is the most important cause of pulmonary artery constriction? a. Low alveolar partial pressure of arterial oxygen (PaO2) b. Hyperventilation c. Respiratory alkalosis d. Epinephrine ANS: A The most important cause of pulmonary artery constriction is a low alveolar PaO2. PTS: 1 REF: Page 1230 26. Where does the tracheal bifurcation occur? a. Larynx c. Carina b. Bronchi d. Nasopharynx ANS: C The trachea, which is supported by U-shaped cartilage, connects the larynx to the bronchi, the conducting airways of the lungs. The trachea divides into the two main airways, or bronchi, at the carina (see Figure 34-1). The division occurs only at the carina. PTS: 1 REF: Page 1228 27. How low must the partial pressure of arterial oxygen (PaO2) drop before the peripheral chemoreceptors influence ventilation? a. Below 100 mm Hg c. Below 70 mm Hg b. Below 80 mm Hg d. Below 60 mm Hg ANS: C NURSINGTB.COM The PaO2 must drop well below normal (to approximately 60 mm Hg) before the peripheral chemoreceptors have much influence on ventilation. PTS: 1 REF: Page 1234 28. Which receptors are located in the smooth muscles of airways? a. Central chemoreceptors c. Peripheral chemoreceptors b. Stretch receptors d. J-receptors ANS: B Of the options available, only the stretch receptors are located in the smooth muscles of airways. PTS: 1 REF: Page 1234 29. Which receptors are located near the respiratory center? a. Peripheral chemoreceptors c. Central chemoreceptors b. Stretch receptors d. J-receptors ANS: C Of the options available, only the central chemoreceptors are located near the respiratory center. 30. Which receptors are located in the aortic bodies, aortic arch, and carotid bodies? a. Central chemoreceptors c. J-receptors b. Stretch receptors d. Peripheral chemoreceptors ANS: D Of the options available, only the peripheral chemoreceptors are located in the aortic bodies, aortic arch, and carotid bodies at the bifurcation of the carotids, near the baroreceptors. PTS: 1 REF: Page 1234 31. What is the purpose of the spirometry measurement? a. To evaluate the cause of hypoxia b. To measure the volume and flow rate during forced expiration c. To measures the gas diffusion rate at the alveolocapillary membrane d. To determine pH and oxygen and carbon dioxide concentrations ANS: B Spirometry measures volume and flow rate during forced expiration. The alveolar-arterial oxygen gradient is used to evaluate the cause of hypoxia. Diffusing capacity is a measure of the gas diffusion rate at the alveolocapillary membrane. Arterial blood gas analysis can be used to determine pH and oxygen and carbon dioxide concentrations. PTS: 1 REF: Page 1243 MULTIPLE RESPONSE NURSINGTB.COM 32. Which structures belong to the upper conduction airway? (Select all that apply.) a. Oropharynx b. Larynx c. Nasopharynx d. Trachea e. Bronchi ANS: A, C The conducting airways are the portion of the pulmonary system that provides a passage for the movement of air into and out of the gas-exchange portions of the lung. The nasopharynx, oropharynx, and related structures are often called the upper airway. The remaining options are not considered to be included in the upper conduction airway. PTS: 1 REF: Pages 1225-1226 33. Regarding the respiratory process referred to as remodeling, which statements are true? (Select all that apply.) a. Remodeling involves the vascular walls. b. Scarring and thickening occurs during this respiratory process. c. Remodeling results in a permanent change. d. Pulmonary artery hypotension results. e. Remodeling increases blood flow resistance. ANS: A, B, C, E Remodeling is a process by which the vascular wall becomes scarred and thickened, thus resulting in permanent decreases in luminal diameter, increased resistance to blood flow, and permanent pulmonary artery hypertension. PTS: 1 REF: Page 1232 | What's New box 34. What are the effects of aging on the pulmonary system? a. Decreased chest wall compliance b. Decreased lung recoil c. Reduced ventilatory reserve d. Decreased partial pressure of arterial oxygen (PaO2) e. Reduced respiratory rate ANS: A, B, C, D Aging affects the mechanical aspects of ventilation by decreasing chest wall compliance and elastic recoil of the lungs. Changes in these elastic properties reduce ventilatory reserve. Aging causes the PaO2 to decrease but does not affect the partial pressure of arterial carbon dioxide (PaCO2) or respiratory rate. PTS: 1 REF: Pages 1244-1245 MATCHING Match the receptor with its function. A. Irritant receptors B. Stretch receptors C. J-receptors NURSINGTB.COM D. Peripheral chemoreceptors E. Central chemoreceptors 35. Initiates rapid, shallow breathing 36. Monitors pH, partial pressure of carbon dioxide (PaCO2), and partial pressure of oxygen (PaO2) in arterial blood 37. Initiates cough reflex 38. Senses pH of cerebrospinal fluid 39. Hering-Breuer expiratory reflex 35. ANS: C PTS: 1 REF: Page 1234 MSC: J-receptors are sensitive to increased pulmonary capillary pressure, which stimulates them to initiate rapid, shallow breathing; hypotension; and bradycardia. 36. ANS: D PTS: 1 REF: Page 1234 MSC: Although the peripheral chemoreceptors are sensitive to changes in PaCO2 and pH, they are primarily sensitive to oxygen levels in arterial blood (PaO2) and are responsible for all of the increase in ventilation that occurs in response to arterial hypoxemia. 37. ANS: A PTS: 1 REF: Page 1234 MSC: Irritant receptors are sensitive to noxious aerosols (vapors), gases, and particulate matter (e.g., inhaled dusts), which cause them to initiate the cough reflex. 38. ANS: E PTS: 1 REF: Page 1234 MSC: Central chemoreceptors monitor arterial blood indirectly by sensing changes in the pH of cerebrospinal fluid (CSF). 39. ANS: B PTS: 1 REF: Page 1234 MSC: Stretch receptors decrease ventilatory rate and volume when stimulated, an occurrence sometimes referred to as the Hering-Breuer expiratory reflex. NURSINGTB.COM Chapter 36: Alterations of Pulmonary Function MULTIPLE CHOICE 1. Besides dyspnea, what is the most common characteristic associated with pulmonary disease? a. Chest pain c. Cough b. Digit clubbing d. Hemoptysis ANS: C Pulmonary disease is associated with many signs and symptoms, and their specific characteristics often help in identifying the underlying disorder. The most common characteristics are dyspnea and cough. Others include abnormal sputum, hemoptysis, altered breathing patterns, hypoventilation and hyperventilation, cyanosis, clubbing of the digits, and chest pain. PTS: 1 REF: Page 1248 2. Sitting up in a forward-leaning position generally relieves which breathing disorder? a. Hyperpnea c. Apnea b. Orthopnea d. Dyspnea on exertion ANS: B Of the options available, only orthopnea is generally relieved by sitting up in a forward-leaning posture or supporting the upper body on several pillows. PTS: 1 REF: Page 12N4U9RSINGTB.COM 3. Kussmaul respirations as a respiratory pattern may be associated with which characteristic(s)? a. Alternating periods of deep and shallow breathing b. Pulmonary fibrosis c. Chronic obstructive pulmonary disease d. Slightly increased ventilatory rate, large tidal volumes, and no expiratory pause ANS: D Kussmaul respirations are characterized by a slightly increased ventilatory rate, very large tidal volume, and no expiratory pause. Kussmaul respirations are not associated with any of the other options. PTS: 1 REF: Page 1249 4. Respirations that are characterized by alternating periods of deep and shallow breathing are a result of which respiratory mechanism? a. Decreased blood flow to the medulla oblongata b. Increased partial pressure of arterial carbon dioxide (PaCO2), decreased pH, and decreased partial pressure of arterial oxygen (PaO2) c. Stimulation of stretch or J-receptors d. Fatigue of the intercostal muscles and diaphragm ANS: A Alternating periods of deep and shallow breathing are characteristic of Cheyne-Stokes respirations and are the result of any condition that slows the blood flow to the brainstem, which in turn slows impulses that send information to the respiratory centers of the brainstem. None of the remaining options are responsible for the described breathing pattern. PTS: 1 REF: Page 1250 5. With a total hemoglobin of 9 g/dl, how many grams per deciliter of hemoglobin must become desaturated for cyanosis to occur? a. 3 c. 7 b. 5 d. 9 ANS: B Cyanosis generally develops when 5 g/dl of hemoglobin is desaturated, regardless of hemoglobin concentration. PTS: 1 REF: Page 1250 6. Which statement is true regarding ventilation? a. Hypoventilation causes hypocapnia. b. Hyperventilation causes hypercapnia. c. Hyperventilation causes hypocapnia. d. Hyperventilation results in an increased partial pressure of arterial carbon dioxide (PaCO2). ANS: C Hyperventilation is alveolar ventilNaUtiRonSItNhGaTt Bex.CcOeeMds metabolic demands. The lungs remove carbon dioxide at a faster rate than produced by cellular metabolism, resulting in decreased PaCO2 or hypocapnia. None of the remaining options are accurate statements. PTS: 1 REF: Page 1250 7. What term is used to describe the selective bulbous enlargement of the distal segment of a digit that is commonly associated with diseases that interfere with oxygenation of the blood? a. Edema c. Angling b. Clubbing d. Osteoarthropathy ANS: B Clubbing is the selective bulbous enlargement of the end (distal segment) of a digit (finger or toe) (see Figure 35-1) and is commonly associated with diseases that interfere with oxygenation, such as bronchiectasis, cystic fibrosis, pulmonary fibrosis, lung abscess, and congenital heart disease. None of the remaining options are terms used to identify the condition described. PTS: 1 REF: Page 1250 8. Pulmonary edema and pulmonary fibrosis cause hypoxemia by which mechanism? a. Creating alveolar dead space b. Decreasing the oxygen in inspired gas c. Creating a right-to-left shunt d. Impairing alveolocapillary membrane diffusion ANS: D Diffusion of oxygen through the alveolocapillary membrane is impaired if the alveolocapillary membrane is thickened or if the surface area available for diffusion is decreased. Abnormal thickness, as occurs with edema (tissue swelling) and fibrosis (formation of fibrous lesions), increases the time required for diffusion across the alveolocapillary membrane. None of the remaining options accurately describes the mechanism that triggers hypoxemia as a result of pulmonary edema or pulmonary fibrosis. PTS: 1 REF: Pages 1251-1252 9. High altitudes may produce hypoxemia through which mechanism? a. Shunting c. Decreased inspired oxygen b. Hypoventilation d. Diffusion abnormalities ANS: C The presence of adequate oxygen content of the inspired air is the first factor. Oxygen content is lessened at high altitudes. At high altitudes none of the remaining options would be the cause of hypoxemia. PTS: 1 REF: Page 1251 10. Which condition is capable of producing alveolar dead space? a. Pulmonary edema c. Atelectasis b. Pulmonary emboli d. Pneumonia ANS: B NURSINGTB.COM A pulmonary embolus that impairs blood flow to a segment of the lung results in an area where alveoli are ventilated but not perfused, which causes alveolar dead space. Alveolar dead space is not the result of any of the remaining options. PTS: 1 REF: Page 1252 11. What is the most common cause of pulmonary edema? a. Right-sided heart failure c. Mitral valve prolapse b. Left-sided heart failure d. Aortic stenosis ANS: B The most common cause of pulmonary edema is heart disease. When the left ventricle fails, filling pressures on the left side of the heart increase and cause a concomitant increase in pulmonary capillary hydrostatic pressure. The remaining options are not common triggers for pulmonary edema. PTS: 1 REF: Page 1260 12. Pulmonary edema usually begins to develop at a pulmonary capillary wedge pressure or left atrial pressure of how many millimeters of mercury (mm Hg)? a. 10 c. 30 b. 20 d. 40 ANS: B Pulmonary edema usually begins to develop at a pulmonary capillary wedge pressure or left atrial pressure of 20 mm Hg. PTS: 1 REF: Page 1260 13. The collapse of lung tissue caused by the lack of collateral ventilation through the pores of Kohn is referred to as what type of atelectasis? a. Compression c. Absorption b. Perfusion d. Hypoventilation ANS: C Absorption atelectasis is a result of the gradual absorption of air from obstructed or hypoventilated alveoli or from inhalation of concentrated oxygen or anesthetic agents. The other forms of atelectasis are not a result of the described mechanism. PTS: 1 REF: Pages 1256-1257 | Figure 35-5 14. In what form of bronchiectasis do both constrictions and dilations deform the bronchi? a. Varicose c. Cylindric b. Symmetric d. Saccular ANS: A Varicose bronchiectasis exists when both constrictions and dilations deform the bronchi. None of the other options involve both constriction and dilation, resulting in bronchi deformity. PTS: 1 REF: Pages 1256-1258 NURSINGTB.COM 15. Which pleural abnormality involves a site of pleural rupture that acts as a one-way valve, permitting air to enter on inspiration but preventing its escape by closing during expiration? a. Spontaneous pneumothorax c. Open pneumothorax b. Tension pneumothorax d. Secondary pneumothorax ANS: B In tension pneumothorax, the site of pleural rupture acts as a one-way valve, permitting air to enter on inspiration but preventing its escape by closing up during expiration. As more and more air enters the pleural space, air pressure in the pneumothorax begins to exceed barometric pressure. None of the other options result from the pathologic condition described. PTS: 1 REF: Page 1254 16. In which type of pleural effusion does the fluid become watery and diffuse out of the capillaries as a result of increased blood pressure or decreased capillary oncotic pressure? a. Exudative c. Transudative b. Purulent d. Large ANS: C In transudative pleural effusion, the fluid, or transudate, is watery and diffuses out of the capillaries as a result of disorders that increase intravascular hydrostatic pressure or decrease capillary oncotic pressure. The described mechanism is not associated with the other forms of pleural effusion. PTS: 1 REF: Page 1254 17. Which condition involves an abnormally enlarged gas-exchange system and the destruction of the lung’s alveolar walls? a. Transudative effusion c. Exudative effusion b. Emphysema d. Abscess ANS: B Emphysema is abnormal permanent enlargement of gas-exchange airways (acini) accompanied by the destruction of alveolar walls without obvious fibrosis. The described mechanism is not associated with the other options. PTS: 1 REF: Page 1268 18. Which term is used to identify a circumscribed area of suppuration and destruction of lung parenchyma? a. Consolidation c. Empyema b. Cavitation d. Abscess ANS: D An abscess is a circumscribed area of suppuration and destruction of lung parenchyma. The described pathologic abnormality is not associated with the other options. NURSINGTB.COM PTS: 1 REF: Page 1274 19. Which condition is not a cause of chest wall restriction? a. Pneumothorax c. Gross obesity b. Severe kyphoscoliosis d. Neuromuscular disease ANS: A Unlike the other options that result in chest wall restriction, a pneumothorax is the presence of air or gas in the pleural space caused by a rupture in the visceral pleura (which surrounds the lungs) or the parietal pleura and chest wall. PTS: 1 REF: Page 1254 20. What causes pneumoconiosis? a. Pneumococci bacteria c. Exposure to asbestos b. Inhalation of inorganic dust particles d. Inhalation of cigarette smoke ANS: B Pneumoconiosis represents any change in the lung caused by the inhalation of inorganic dust particles, which usually occurs in the workplace. Pneumoconiosis is not a result of any of the other options. PTS: 1 REF: Page 1259 21. Which condition is a fulminant form of respiratory failure characterized by acute lung inflammation and diffuse alveolocapillary injury? a. Acute respiratory distress syndrome (ARDS) b. Pneumonia c. Pulmonary emboli d. Acute pulmonary edema ANS: A ARDS is a fulminant form of respiratory failure characterized by acute lung inflammation and diffuse alveolocapillary injury. The described pathologic characteristics are not associated with the other options. PTS: 1 REF: Page 1261 22. Which structure(s) in acute respiratory distress syndrome (ARDS) release inflammatory mediators such as proteolytic enzymes, oxygen-free radicals, prostaglandins, leukotrienes, and platelet-activating factor? a. Complement cascade c. Macrophages b. Mast cells d. Neutrophils ANS: D The role of neutrophils is central to the development of ARDS. Activated neutrophils release a battery of inflammatory mediators, among them proteolytic enzymes, oxygen-free radicals (superoxide radicals, hydrogen peroxide, hydroxyl radicals), arachidonic acid metabolites (prostaglandins, thromboxanes, leukotrienes), and platelet-activating factor. These mediators cause extensive damage to the alveolocapillary membrane and greatly increase capillary membrane permeability. The described responses are not associated with the other oNpUtiRonSIsN. GTB.COM PTS: 1 REF: Page 1261 23. Pulmonary edema in acute respiratory distress syndrome (ARDS) is the result of an increase in: a. Levels of serum sodium and water c. Capillary hydrostatic pressure b. Capillary permeability d. Oncotic pressure ANS: B Increased capillary permeability, a hallmark of ARDS, allows fluids, proteins, and blood cells to leak from the capillary bed into the pulmonary interstitium and alveoli. The resulting pulmonary edema and hemorrhage severely reduce lung compliance and impair alveolar ventilation. The other options would not trigger ARDS-associated pulmonary edema. PTS: 1 REF: Page 1261 24. In acute respiratory distress syndrome (ARDS), alveoli and respiratory bronchioles fill with fluid as a result of which mechanism? a. Compression on the pores of Kohn, thus preventing collateral ventilation b. Increased capillary permeability, which causes alveoli and respiratory bronchioles to fill with fluid c. Inactivation of surfactant and the impairment of type II alveolar cells d. Increased capillary hydrostatic pressure that forces fluid into the alveoli and respiratory bronchioles ANS: C Lung inflammation and injury damage the alveolar epithelium and the vascular endothelium. Surfactant is inactivated, and its production by type II alveolar cells is impaired as alveoli and respiratory bronchioles fill with fluid or collapse. The other options would not trigger the described response. PTS: 1 REF: Page 1261 25. Which type of pulmonary disease requires more force to expire a volume of air? a. Restrictive c. Acute b. Obstructive d. Communicable ANS: B Obstructive pulmonary disease is characterized by airway obstruction that is worse with expiration. Either more force (i.e., the use of accessory muscles of expiration) or more time is required to expire a given volume of air. The other options are not associated with a need for an increase of force to expire air. PTS: 1 REF: Page 1263 26. Which immunoglobulin (Ig) may contribute to the pathophysiologic characteristics of asthma? a. IgA c. IgG b. IgE d. IgM NURSINGTB.COM ANS: B Asthma is a familial disorder, and more than 100 genes have been identified that may play a role in the susceptibility of and the pathogenetic mechanisms that cause asthma, including those that influence the production of interleukin (IL)–4, IL-5, and IL-13; IgE; eosinophils; mast cells; adrenergic receptors; and leukotrienes. The pathophysiologic characteristics of asthma are not associated with the other immunoglobulins. PTS: 1 REF: Pages 1263-1264 27. Which statement about the late asthmatic response is true? a. Norepinephrine causes bronchial smooth muscle contraction and mucus secretion. b. The release of toxic neuropeptides contributes to increased bronchial hyperresponsiveness. c. The release of epinephrine causes bronchial smooth muscle contraction and increases capillary permeability. d. Immunoglobulin G initiates the complement cascade and causes smooth muscle contraction and increased capillary permeability. ANS: B The late asthmatic response begins 4 to 8 hours after the early response when the release of toxic neuropeptides contributes to increased bronchial hyperresponsiveness. This selection is the only option associated with the late asthmatic response. PTS: 1 REF: Page 1264 28. Clinical manifestations of inspiratory and expiratory wheezing, dyspnea, nonproductive cough, and tachypnea are indicative of which condition? a. Chronic bronchitis c. Pneumonia b. Emphysema d. Asthma ANS: D At the beginning of an attack, the individual experiences chest constriction, expiratory wheezing, dyspnea, nonproductive coughing, prolonged expiration, tachycardia, and tachypnea. Severe attacks involve the use of accessory muscles of respiration, and wheezing is heard during both inspiration and expiration. The presentations of none of the other options are consistent with the described symptoms. PTS: 1 REF: Page 1264 | Page 1266 29. The most successful treatment for chronic asthma begins with which action? a. Avoidance of the causative agent b. Administration of broad-spectrum antibiotics c. Administration of drugs that reduce bronchospasm d. Administration of drugs that decrease airway inflammation ANS: A Chronic management of asthma begins with the avoidance of allergens and other triggers. The effectiveness of the other options is reliant on the avoidance of triggers. PTS: 1 REF: Page 1266 30. Which factor contributes to the prNodUuRcStIiNonGToBf .mCOuMcus associated with chronic bronchitis? a. Airway injury c. Increased Goblet cell size b. Pulmonary infection d. Bronchospasms ANS: C Continual bronchial inflammation causes bronchial edema and increases the size and number of mucous glands and goblet cells in the airway epithelium. Thick, tenacious mucus is produced and cannot be cleared because of impaired ciliary function (see Figure 35-13). The lung’s defense mechanisms are therefore compromised, increasing a susceptibility to pulmonary infection, which contributes to airway injury. Frequent infectious exacerbations are complicated by bronchospasm with dyspnea and productive cough. PTS: 1 REF: Page 1267 31. Clinical manifestations of decreased exercise tolerance, wheezing, shortness of breath, and productive cough are indicative of which respiratory disorder? a. Chronic bronchitis c. Pneumonia b. Emphysema d. Asthma ANS: A The symptoms that lead individuals with chronic bronchitis to seek medical care include decreased exercise tolerance, wheezing, and shortness of breath. Individuals usually have a productive cough (“smoker’s cough”). The described symptoms are not associated with any of the other options. PTS: 1 REF: Page 1267 32. Clinical manifestations that include unexplained weight loss, dyspnea on exertion, use of accessory muscles, and tachypnea with prolonged expiration are indicative of which respiratory disorder? a. Chronic bronchitis c. Pneumonia b. Emphysema d. Asthma ANS: B Individuals with emphysema usually have dyspnea on exertion that later progresses to significant dyspnea, even at rest (see Table 35-3). Little coughing and very little sputum are produced. The individual is often thin, has tachypnea with prolonged expiration, and must use accessory muscles for ventilation. The anteroposterior diameter of the chest is increased (barrel chest), and the chest has a hyperresonant sound with percussion. The described symptoms are not associated with any of the other options. PTS: 1 REF: Page 1270 33. Which of the following is the most common route of lower respiratory tract infection? a. Aspiration of oropharyngeal secretions b. Inhalation of microorganisms c. Microorganisms spread to the lung via blood d. Poor mucous membrane protection ANS: A Aspiration of oropharyngeal secretions is the most common route of lower respiratory tract infection; thus the nasopharynx anNdUoRrSoIpNhGaTryBn.CxOcMonstitute the first line of defense for most infectious agents. The other options are not common routes of lower respiratory tract infections. PTS: 1 REF: Page 1271 34. What is the initial step in the management of emphysema? a. Inhaled anticholinergic agents c. Cessation of smoking b. Beta agonists d. Surgical reduction of lung volume ANS: C Chronic management of emphysema begins with smoking cessation. Pharmacologic management includes inhaled anticholinergic agents, and beta agonists should be prescribed. Pulmonary rehabilitation, improved nutrition, and breathing techniques all can improve symptoms. Oxygen therapy is indicated in chronic hypoxemia but must be administered with care. In selected patients, lung volume reduction surgery or transplantation can be considered. PTS: 1 REF: Pages 1270-1271 35. In tuberculosis, the body walls off the bacilli in a tubercle by stimulating which action? a. Macrophages that release tumor necrosis factor–alpha (TNF- b. Phagocytosis by neutrophils and eosinophils c. Formation of immunoglobulin G to initiate the complement cascade d. Apoptotic infected macrophages that activate cytotoxic T cells ANS: D In defense, macrophages and lymphocytes release interferon, which inhibits the replication of the microorganism and stimulates more macrophages to attack the bacterium. Apoptotic infected macrophages can also activate cytotoxic T cells (cluster of differentiation [CD] 8). Tuberculosis does not trigger the mechanisms described by the other options. PTS: 1 REF: Pages 1273-1274 36. The progression of chronic bronchitis is best halted by which intervention? a. Regular use of bronchodilators b. Smoking cessation c. Postural chest drainage techniques d. Identification of early signs of infection ANS: B By the time an individual seeks medical care for symptoms, considerable airway damage is present. If the individual stops smoking, then disease progression can be halted. If smoking is stopped before symptoms occur, then the risk of chronic bronchitis decreases considerably and eventually reaches that of nonsmokers. The other interventions, although appropriate, are not directed toward halting the progression of the disease process. PTS: 1 REF: Page 1267 37. Clinical manifestations of inspiratory crackles, increased tactile fremitus, egophony, and whispered pectoriloquy are indicative of which respiratory condition? a. Chronic bronchitis NURSINGTcB..COPMneumonia b. Emphysema d. Asthma ANS: C Physical examination may reveal signs of pulmonary consolidation, such as inspiratory crackles, increased tactile fremitus, egophony, and whispered pectoriloquy, which support a diagnosis of pneumonia. The presentations of the other options are not consistent with the described symptoms. PTS: 1 REF: Page 1273 38. Pulmonary artery hypertension (PAH) results from which alteration? a. Narrowed pulmonary capillaries c. Destruction of alveoli b. Narrowed bronchi and bronchioles d. Ischemia of the myocardium ANS: A PAH is characterized by endothelial dysfunction with an overproduction of vasoconstrictors (e.g., thromboxane, endothelin) and decreased production of vasodilators (e.g., nitric oxide, prostacyclin), resulting in narrowed pulmonary capillaries. None of the remaining options result in pulmonary hypertension. PTS: 1 REF: Page 1277 39. Squamous cell carcinoma of the lung is best described as a tumor that causes which alterations? a. Abscesses and ectopic hormone production b. Airway obstruction and atelectasis c. Pleural effusion and shortness of breath d. Chest wall pain and early metastasis ANS: B Typically, the tumors are centrally located near the hila and project into bronchi. Because of this central location, nonproductive cough or hemoptysis is common. Pneumonia and atelectasis are often associated with squamous cell carcinoma. Chest pain is a late symptom associated with large tumors. These tumors can remain fairly well localized and tend not to metastasize until late in the course of the disease. Squamous cell carcinomas are not associated with any of the other options. PTS: 1 REF: Pages 1280-1281 | Table 35-4 40. What medical term is used to identify the accumulation of air in the pleural space? a. Flail chest c. Pleural effusion b. Pneumothorax d. Exudate effusion ANS: B Pneumothorax is the presence of air or gas in the pleural space caused by a rupture in the visceral pleura (which surrounds the lungs) or the parietal pleura and chest wall. The condition is not identified by any of the other options. PTS: 1 REF: Page 1254 41. What medical term is used to identify the presence of pus in the pleural space? a. Plural effusion NURSINGTcB..COEMmpyema b. Asthma d. Pneumonia ANS: C Empyema is the presence of pus in the pleural space. This condition is not identified by any of the other options. PTS: 1 REF: Page 1255 42. Fluid in the pleural space characterizes which condition? a. Pleural effusion c. Bronchiectasis b. Atelectasis d. Ischemia ANS: A Pleural effusion is the presence of fluid in the pleural space. This condition is not identified by any of the other options. PTS: 1 REF: Page 1254 43. Which statement is true regarding hypoxemia? a. Hypoxemia results in the increased oxygenation of arterial blood. b. Respiratory alterations cause hypoxemia. c. Hypoxemia results in the decreased oxygenation of tissue cells. d. Various system changes cause hypoxemia. ANS: B Hypoxemia, or reduced oxygenation of arterial blood (PaO2), is caused by respiratory alterations, whereas hypoxia, or reduced oxygenation of cells in tissues, may be caused by alterations of other systems as well. PTS: 1 REF: Page 1251 44. Which medication classification is generally included in the treatment of silicosis? a. Corticosteroids c. Bronchodilators b. Antibiotics d. Expectorants ANS: A No specific treatment exists for silicosis, although corticosteroids may produce some improvement in the early, more acute stages. The other options are not generally prescribed. PTS: 1 REF: Page 1259 45. What medical term is used for a condition that results from pulmonary hypertension, creating chronic pressure overload in the right ventricle? a. Hypoxemia c. Bronchiectasis b. Hypoxia d. Cor pulmonale ANS: D Cor pulmonale develops as pulmonary hypertension and creates chronic pressure overload in the right ventricle similar to that created in the left ventricle by systemic hypertension. None of the other options identify the condition. PTS: 1 REF: Page 12N7U8RSINGTB.COM MULTIPLE RESPONSE 46. What are the causes of dyspnea? (Select all that apply.) a. Decreased pH, increased partial pressure of arterial carbon dioxide (PaCO2) and decreased partial pressure of arterial oxygen (PaO2) b. Decreased blood flow to the medulla oblongata c. Stimulation of stretch or J-receptors d. Presence of anxiety e. Presence of pain ANS: A, C, D Dyspnea can be triggered by decreased pH, increased PaCO2, and decreased PaO2. Stimulation of either stretch or J-receptors is also known as a cause of dyspnea. Dyspnea may be the result of pulmonary disease or many other conditions, such as pain, heart disease, trauma, and anxiety. No data are available to support the role of decreased blood flow to the medulla oblongata as being a cause of dyspnea. PTS: 1 REF: Pages 1248-1249 47. Which inflammatory mediators are produced in asthma? (Select all that apply.) a. Histamine b. Bradykinin c. Leukotrienes d. Prostaglandins e. Neutrophil proteases ANS: A, B, C, D A large number of inflammatory mediators, such as histamine, prostaglandins, and leukotrienes, are produced by asthma. Neutrophil proteases are not produced in relationship to asthma. PTS: 1 REF: Page 1265 | Figure 35-12 48. Which clinical manifestation is associated with pulmonary hypertension? (Select all that apply.) a. Systemic blood pressure greater than 130/90 mm Hg b. Rhonchi bilaterally c. Dyspnea on exertion d. Peripheral edema e. Jugular venous distention ANS: C, D, E Symptoms of fatigue, chest discomfort, tachypnea, and dyspnea on exertion, palpitations, and cough are common. Examination may reveal peripheral edema, jugular venous distention, a precordial heave, and accentuation of the pulmonary compartment of the second heart sound. Neither rhonchi nor a systemic blood pressure of 130/90 mm Hg are associated with pulmonary hypertension. PTS: 1 REF: Pages 1277-1278 NURSINGTB.COM 49. Which statements are true regarding exudative effusion? (Select all that apply.) a. Exudative effusion contains high concentrations of white blood cells. b. Exudative effusion produces a very thick exudate. c. Exudative effusion may occur in response to an inflammatory process. d. The presence of a malignant cancer can trigger exudative effusion. e. Exudative effusion is a result of increased capillary permeability. ANS: A, C, D, E Exudative effusion is less watery and contains high concentrations of white blood cells and plasma proteins. Exudative effusion occurs in response to inflammation, infection, or malignancy and involves inflammatory processes that increase capillary permeability. PTS: 1 REF: Page 1254 50. Which characteristics are symptomatic of a flail chest? (Select all that apply.) a. Involves the fracture of several consecutive ribs. b. Involves multiple fractures to individual ribs. c. Can involve the fracture of the sternum. d. Is generally a result of the inflammatory process. e. Is more common among the older adult population. ANS: A, B, C A flail chest results from the fracture of several consecutive ribs in more than one place or the fracture of the sternum and several consecutive ribs. Age and inflammation are not generally considered factors in this disorder. PTS: 1 REF: Page 1253 51. Which statements regarding Mycobacterium tuberculosis are true regarding the bacilli’s ability to go into dormancy? (Select all that apply.) a. Neutrophils and macrophages all play a role in its dormancy. b. Mycobacterium tuberculosis is capable of dormancy but for only a short period. c. The immune system is the controlling factor regarding its length of dormancy. d. The bacilli are sealed off in tubercles to allow for dormancy. e. An attack by lymphocytes brings the bacilli out of their dormant state. ANS: A, C, D Neutrophils, lymphocytes, and macrophages seal off the colonies of bacilli, forming a granulomatous lesion called a tubercle. Once the bacilli are isolated in tubercles and immunity develops, tuberculosis may remain dormant for life. If the immune system is impaired, however, or if live bacilli escape into the bronchi, active disease occurs and may spread through the blood and lymphatic system to other organs. This microorganism can remain dormant for extended periods. Reverting from dormancy is not related to a lymphocyte attack. PTS: 1 REF: Pages 1273-1274 MATCHING NURSINGTB.COM Match the descriptions with the corresponding terms. A. Passage of fluid and solid particles into the lung B. Inflammatory obstruction of small airways C. Excessive amount of connective tissue in the lung D. Lung tissue collapse E. Abnormal dilation of the bronchi 52. Pulmonary fibrosis 53. Atelectasis 54. Bronchiectasis 55. Aspiration 56. Bronchiolitis 52. ANS: C PTS: 1 REF: Page 1258 MSC: Pulmonary fibrosis is an excessive amount of fibrous or connective tissue in the lung. 53. ANS: MSC: D PTS: 1 REF: Page 1256 Atelectasis is the collapse of lung tissue. 54. ANS: MSC: E PTS: 1 REF: Page 1256 Bronchiectasis is persistent abnormal dilation of the bronchi. 55. ANS: MSC: A PTS: 1 REF: Page 1255 Aspiration is the passage of fluid and solid particles into the lung. 56. ANS: MSC: B PTS: 1 REF: Page 1258 Bronchiolitis is inflammation of the small airways or bronchioles. Chapter 37: Alterations of Pulmonary Function in Children MULTIPLE CHOICE 1. How does chest wall compliance in an infant differ from that of an adult? a. An adult’s chest wall compliance is lower than an infant’s. b. An adult’s chest wall compliance is higher than an infant’s. c. An adult’s chest wall compliance is the same as an infant’s. d. An adult’s chest wall compliance is dissimilar to that of an infant’s. ANS: A Chest wall compliance is higher in infants than it is in adults, particularly in premature infants. PTS: 1 REF: Page 1292 2. Why is nasal congestion a serious threat to young infants? a. Infants are obligatory nose breathers. b. Their noses are small in diameter. c. Infants become dehydrated when mouth breathing. d. Their epiglottis is proportionally greater than the epiglottis of an adult’s. ANS: A Infants up to 2 to 3 months of age are obligatory nose breathers and are unable to breathe in through their mouths. Nasal congestion is therefore a serious threat to a young infant. This selection is the only option that accurately describes why nasal congestion is a serious threat to young infants. NURSINGTB.COM PTS: 1 REF: Page 1290 3. The risk for respiratory distress syndrome (RDS) decreases for premature infants when they are born between how many weeks of gestation? a. 16 and 20 c. 24 and 30 b. 20 and 24 d. 30 and 36 ANS: D Surfactant is secreted into fetal airways between 30 and 36 weeks. The other options are not true regarding the timeframe when the risk for RDS decreases. PTS: 1 REF: Page 1292 4. Which type of croup is most common? a. Bacterial c. Fungal b. Viral d. Autoimmune ANS: B In 85% of children with croup, a virus is the cause, most commonly parainfluenza. However, other viruses such as influenza A or respiratory syncytial virus (RSV) also can cause croup. PTS: 1 REF: Pages 1295-1296 5. What is the chief predisposing factor for respiratory distress syndrome (RDS) of the newborn? a. Low birth weight b. Alcohol consumption during pregnancy c. Premature birth d. Smoking during pregnancy ANS: C RDS of the newborn, also known as hyaline membrane disease (HMD), is a major cause of morbidity and mortality in premature newborns. None of the other options are considered the chief predisposing factors for RDS. PTS: 1 REF: Page 1301 6. What is the primary cause of respiratory distress syndrome (RDS) of the newborn? a. Immature immune system c. Surfactant deficiency b. Small alveoli d. Anemia ANS: C RDS is primarily caused by surfactant deficiency and secondarily by a deficiency in alveolar surface area for gas exchange. None of the other options are related to the cause of RDS. PTS: 1 REF: Page 1301 7. What is the primary problem resulting from respiratory distress syndrome (RDS) of the newborn? NURSINGTB.COM a. Consolidation c. Atelectasis b. Pulmonary edema d. Bronchiolar plugging ANS: C The primary problem is atelectasis, which causes significant hypoxemia and is difficult for the neonate to overcome because a significant negative inspiratory pressure is required to open the alveoli with each breath. None of the other options are considered a primary problem associated with RDS. PTS: 1 REF: Page 1301 8. Which option shows the correct sequence of events after atelectasis develops in respiratory distress syndrome of the newborn? a. Increased pulmonary vascular resistance, atelectasis, hypoperfusion b. Hypoxic vasoconstriction, right-to-left shunt hypoperfusion c. Respiratory acidosis, hypoxemia, hypercapnia d. Right-to-left shunt, hypoxic vasoconstriction, hypoperfusion ANS: B Atelectasis results in a decrease in tidal volume, causing alveolar hypoventilation and hypercapnia. Hypoxia and hypercapnia cause pulmonary vasoconstriction, which increases intrapulmonary resistance and shunting. This results in hypoperfusion of the lung and a decrease in effective pulmonary blood flow. This selection is the only option that identifies the correct sequence of events. PTS: 1 REF: Page 1301 9. Which statement about the advances in the treatment of respiratory distress syndrome (RDS) of the newborn is incorrect? a. Administering glucocorticoids to women in preterm labor accelerates the maturation of the fetus’s lungs. b. Administering oxygen to mothers during preterm labor increases their arterial oxygen before the birth of the fetus. c. Treatment includes the instillation of exogenous surfactant down an endotracheal tube of infants weighing less than 1000 g. d. Using continuous positive airway pressure (CPAP) supports the infant’s respiratory function. ANS: B Administering oxygen to the mother is not a valid treatment of RDS. The other statements provide correct information regarding RDS. PTS: 1 REF: Pages 1301-1303 10. Bronchiolitis tends to occur during the first years of life and is most often caused by what type of infection? a. Respiratory syncytial virus (RSV) c. Adenoviruses b. Influenzavirus d. Rhinovirus ANS: A The most common associated pathogen is RSV, but bronchiolitis may also be associated with adenovirus, rhinovirus, influeNnUzRaS, IpNaGraTiBn.fCluOeMnza virus (PIV), and Mycoplasma pneumoniae. PTS: 1 REF: Page 1305 11. Which immunoglobulin (Ig) is present in childhood asthma? a. IgM c. IgE b. IgG d. IgA ANS: C Included in the long list of asthma-associated genes are those that code for increased levels of immune and inflammatory mediators (e.g., interleukin [IL]–4, IgE, leukotrienes), nitric oxide, and transmembrane proteins in the endoplasmic reticulum. None of the other options are associated with childhood asthma. PTS: 1 REF: Pages 1308-1309 12. Which T-lymphocyte phenotype is the key determinant of childhood asthma? a. Cluster of differentiation (CD) 4 T-helper Th1 lymphocytes b. CD4 T-helper Th2 lymphocytes c. CD8 cytotoxic T lymphocytes d. Memory T lymphocytes ANS: B Asthma develops because the Th2 response (in which CD4 T-helper cells produce specific cytokines, such as interleukin [IL]–4, IL-5, and IL-13) promotes an atopic and allergic response in the airways. This selection is the only option that accurately identifies the appropriate T-lymphocyte phenotype. PTS: 1 REF: Page 1309 13. Which cytokines activated in childhood asthma produce an allergic response? a. Interleukin (IL)–1, IL-2, and interferon-alpha (IFN-) b. IL-8, IL-12, and tumor necrosis factor-alpha (TNF-) c. IL-4, IL-10, and colony-stimulating factor (CSF) d. IL-4, IL-5, and IL-13 ANS: D Related to asthma, IL-4 and IL-13 are particularly important for B-cell switching to favor immunoglobulin E (IgE) production, and IL-5 is crucial for local differentiation and enhanced survival of eosinophils within the airways. This selection is the only option that accurately describes how cytokines produce a childhood asthmatic response. PTS: 1 REF: Page 1309 14. Which statement accurately describes childhood asthma? a. An obstructive airway disease characterized by reversible airflow obstruction, bronchial hyperreactivity, and inflammation b. A pulmonary disease characterized by severe hypoxemia, decreased pulmonary compliance, and diffuse densities on chest x-ray imaging c. A pulmonary disorder involvinNgURanSIaNbGnToBrm.CaOlMexpression of a protein, producing viscous mucus that lines the airways, pancreas, sweat ducts, and vas deferens d. An obstructive airway disease characterized by atelectasis and increased pulmonary resistance as a result of a surfactant deficiency ANS: A Asthma is an obstructive airway disease characterized by reversible airflow obstruction, bronchial hyperreactivity, and inflammation. This selection is the only option that accurately describes childhood asthma. PTS: 1 REF: Page 1308 15. Which criterion is used to confirm a diagnosis of asthma in an 8-year-old child? a. Parental history of asthma b. Serum testing that confirms increased immunoglobulin E (IgE) and eosinophil levels c. Reduced expiratory flow rates confirmed by spirometry testing d. Improvement on a trial of asthma medication ANS: C Confirmation of the diagnosis of asthma relies on pulmonary function testing using spirometry, which can be accomplished only after the child is 5 to 6 years of age. Reduced expiratory flow rates that are reversible in response to an inhaled bronchodilator would be characteristic abnormalities. For younger children, an empiric trial of asthma medications is commonly initiated. The remaining options are major historical and physical factors that contribute but do not confirm the diagnosis of asthma in children. PTS: 1 REF: Pages 1309-1310 16. Which statement best describes acute respiratory distress syndrome (ARDS)? a. An obstructive airway disease characterized by reversible airflow obstruction, bronchial hyperreactivity, and inflammation b. A pulmonary disease characterized by severe hypoxemia, decreased pulmonary compliance, and the presence of bilateral infiltrates on chest x-ray imaging c. A respiratory disorder involving an abnormal expression of a protein producing viscous mucus that lines the airways, pancreas, sweat ducts, and vas deferens d. A pulmonary disorder characterized by atelectasis and increased pulmonary resistance as a result of a surfactant deficiency ANS: B ARDS is a condition that can result from either a direct or indirect pulmonary insult. It is defined as respiratory failure of acute onset characterized by severe hypoxemia that is refractory to treatment with supplemental oxygen, bilateral infiltrates on chest x-ray imaging, and no evidence of heart failure, as well as decreased pulmonary compliance. This selection is the only option that accurately describes ARDS. PTS: 1 REF: Page 13N1U0RSINGTB.COM 17. When considering the signs and symptoms of acute respiratory distress syndrome (ARDS), the absence of which condition is considered characteristic? a. Progressive respiratory distress c. Decreased pulmonary compliance b. Bilateral infiltrates d. Heart failure ANS: D ARDS is characterized by progressive respiratory distress, severe hypoxemia refractory to treatment with supplemental oxygen, decreased pulmonary compliance, bilateral infiltrates on chest x-ray imaging, and no evidence of heart failure. PTS: 1 REF: Page 1310 18. Examination of the throat in a child demonstrating signs and symptoms of acute epiglottitis may contribute to which life-threatening complication? a. Retropharyngeal abscess c. Rupturing of the tonsils b. Laryngospasms d. Gagging induced aspiration ANS: B Examination of the throat may trigger laryngospasm and cause respiratory collapse. Death may occur in a few hours. This selection is the only option that accurately identifies the life-threatening complication that can result from an examination of the throat of a child who demonstrates the signs and symptoms of acute epiglottitis. PTS: 1 REF: Page 1297 19. Which statement best describes cystic fibrosis? a. Obstructive airway disease characterized by reversible airflow obstruction, bronchial hyperreactivity, and inflammation b. Respiratory disease characterized by severe hypoxemia, decreased pulmonary compliance, and diffuse densities on chest x-ray imaging c. Pulmonary disorder involving an abnormal expression of a protein-producing viscous mucus that obstructs the airways, pancreas, sweat ducts, and vas deferens d. Pulmonary disorder characterized by atelectasis and increased pulmonary resistance as a result of a surfactant deficiency ANS: C Cystic fibrosis is best described as a pulmonary disorder involving an abnormal expression of a protein-producing viscous mucus that obstructs the airways, pancreas, sweat ducts, and vas deferens. This selection is the only option that accurately describes cystic fibrosis. PTS: 1 REF: Pages 1310-1311 20. Cystic fibrosis is caused by which process? a. Autosomal recessive inheritance c. Infection b. Autosomal dominant inheritance d. Malignancy ANS: A Cystic fibrosis is an autosomal recessive inherited disorder that is associated with defective epithelial ion transport. None of the other options cause cystic fibrosis. PTS: 1 REF: Page 13N1U0RSINGTB.COM 21. What are the abnormalities in cytokines found in children with cystic fibrosis (CF)? a. Deficit of interleukin (IL)–1 and an excess of IL-4, IL-12, and interferon-alpha (IFN- b. Deficit of IL-6 and an excess of IL-2, IL-8, and granulocyte colony-stimulating factor (G-CSF) c. Deficit of IL-10 and an excess of IL-1, IL-8, and TNF- d. Deficit of IL-3 and an excess of IL-14, IL-24, and colony-stimulating factor (CSF) ANS: C Abnormal cytokine profiles have been documented in CF airway fluids, including deficient IL-10 and excessive IL-1, IL-8, and TNF-, all changes conducive to promoting inflammation. This selection is the only option that accurately identifies the abnormalities in cytokines observed in children with CF. PTS: 1 REF: Pages 1311-1312 22. Between which months of age does sudden infant death syndrome (SIDS) most often occur? a. 0 and 1 c. 5 and 6 b. 2 and 4 d. 6 and 7 ANS: B The incidence of SIDS is low during the first month of life but sharply increases in the second month of life, peaking at 2 to 4 months and is unusual after 6 months of age. PTS: 1 REF: Page 1313 23. Where in the respiratory tract do the majority of foreign objects aspirated by children finally lodge? a. Trachea c. Bronchus b. Left lung d. Bronchioles ANS: C Approximately 75% of aspirated foreign bodies lodge in a bronchus. The other options are not locations where children aspirate the majority of foreign objects. PTS: 1 REF: Page 1298 24. What is the most common predisposing factor to obstructive sleep apnea in children? a. Chronic respiratory infections c. Obligatory mouth breathing b. Adenotonsillar hypertrophy d. Paradoxic breathing ANS: B In otherwise healthy children, the most common predisposing factor is adenotonsillar hypertrophy, which causes physical impingement on the nasopharyngeal airway. The other options are not associated with obstructive sleep apnea in children. PTS: 1 REF: Page 1300 MULTIPLE RESPONSE NURSINGTB.COM 25. Which statement is true regarding alveoli? (Select all that apply.) a. The number of functioning alveoli is determined by birth. b. The alveoli begin to increase in size starting at 8 years of age. c. The complexity of the alveoli increases into adulthood. d. These structures produce surfactant. e. Capillaries are the origin of alveoli. ANS: B, C, E Capillaries grow into the distal respiratory units that keep subdividing (alveolarization) to maximize the surface area for gas exchange. The number of alveoli continues to increase during the first 5 to 8 years of life, after which the alveoli increase in size and complexity. Surfactant is a lipid-protein mix that is produced by type II alveolar cells. PTS: 1 REF: Page 1291 26. Children diagnosed with chronic asthma are likely to exhibit which symptoms? (Select all that apply.) a. Nasal flaring b. Musical expiratory wheezing c. Clubbing of fingers and toes d. Substernal retractions e. Diaphoresis ANS: A, B, D, E On physical examination, expiratory wheezing that is often described as high pitched and musical is exhibited, along with prolongation of the expiratory phase of the respiratory cycle. Hyperinflation is sometimes visible. The respiratory rate is elevated, as is the heart rate. Nasal flaring and accessory muscle use are evident, with retractions in the substernal, subcostal, intercostal, suprasternal, or sternocleidomastoid areas. Infants may appear to be “head bobbing” because of sternocleidomastoid muscle use. Pulsus paradoxus may also be present. The child may appear anxious or diaphoretic, which are important signs of respiratory compromise. Clubbing of fingers and toes is not typically associated with asthma. PTS: 1 REF: Page 1309 27. Which symptom is not a clinical manifestation of croup? a. Rhinorrhea b. Sore throat c. Low-grade fever d. Barking cough e. Coarse rhonchi ANS: E Typically, a prodrome of rhinorrhea, sore throat, and low-grade fever is exhibited for a few days with croup. The child then develops the characteristic harsh (seal-like) barking cough, hoarse voice, and inspiratory stridor. Rhonchi are associated with lower respiratory diseases. PTS: 1 REF: Page 1295 NURSINGTB.COM 28. What are the clinical manifestations of bacterial pneumonia in children? (Select all that apply.) a. Fever with chills b. Productive cough c. Dyspnea d. Respiratory alkalosis e. Malaise ANS: A, B, C, E The clinical presentation of bacterial pneumonia, particularly pneumococcal, may include a preceding viral illness, followed by fever with chills and rigors, shortness of breath, and an increasingly productive cough. Auscultation usually reveals such abnormalities as crackles or decreased breath sounds. Other less specific findings may include malaise, emesis, abdominal pain, and chest pain. Respiratory alkalosis is not usually associated with bacterial pneumonia in children. PTS: 1 REF: Page 1306 MATCHING Match the sound of stridor with the location of the problem. A. Sonorous snoring B. Muffled voice C. High-pitched inspiratory sound, voice change, hoarse D. Expiratory stridor or monophonic wheeze E. Inspiratory wheezes 29. Tracheal problems 30. Laryngeal problems 31. Upper trachea obstruction 32. Nasopharyngeal obstruction, such as adenotonsillar hypertrophy. 33. Supralaryngeal obstructions 29. ANS: D PTS: 1 REF: Page 1293 MSC: Expiratory stridors or monophonic wheezes suggest tracheal problems. 30. ANS: C PTS: 1 REF: Page 1294 MSC: Abnormalities of voice or cry (weak or hoarse) suggest problems at the larynx. 31. ANS: E PTS: 1 REF: Page 1298 MSC: Foreign bodies lodged in the upper trachea typically produce inspiratory stridor. 32. ANS: A PTS: 1 REF: Page 1293 MSC: Sonorous snoring is associated with nasopharyngeal obstruction, such as adenotonsillar hypertrophy. 33. ANS: B PTS: 1 REF: Page 1294 MSC: Muffling of the voice, especially in an acute condition, suggests supralaryngeal obstruction, such as epiglottitis or retropharyngeal abscess. NURSINGTB.COM Chapter 38: Structure and Function of the Renal and Urologic Systems MULTIPLE CHOICE 1. The area of the kidneys that contains the glomeruli and portions of the tubules is called the: a. Medulla c. Pyramids b. Cortex d. Columns ANS: B The cortex contains all the glomeruli and portions of the tubules of the kidneys. Although the other options are also located in the kidney, they do not contain the glomeruli. PTS: 1 REF: Page 1320 2. What is the functional unit of the kidney called? a. Glomerulus c. Collecting duct b. Nephron d. Pyramid ANS: B The nephron is the functional unit of the kidney. Although the other options are also located in the kidney, they are not its functional units. PTS: 1 REF: Page 1320 3. Which cells have phagocytic properties similar to monocytes and contract like smooth muscles cells, thereby influencingNtUheRSgIlNoGmTeBru.ClaOrMfiltration rate? a. Principle cells c. Mesangial cells b. Podocin cells d. Intercalated cells ANS: C Mesangial cells and the mesangial matrix, secreted by mesangial cells, lie between and support the glomerular capillaries. Different mesangial cells contract like smooth muscle cells to regulate glomerular capillary blood flow. They also have phagocytic properties similar to monocytes. The other options are not capable of these functions. PTS: 1 REF: Page 1321 4. The only surface inside the nephron where cells are covered with microvilli to increase the reabsorptive surface area is called the: a. Proximal convoluted tubules c. Ascending loop of Henle b. Distal tubules d. Descending loop of Henle ANS: A The only surface inside the nephron where the cells are covered with microvilli (a brush border) is called the proximal convoluted tubules. This proximal convoluted tubular lumen consists of one layer of cuboidal cells with a surface layer of microvilli that increases the reabsorptive surface area. PTS: 1 REF: Page 1322 5. What part of the kidney controls renal blood flow, glomerular filtration, and renin secretion? a. Macula densa c. Juxtaglomerular apparatus (JGA) b. Visceral epithelium d. Filtration slits ANS: C Control of renal blood flow, glomerular filtration, and renin secretion occur at the JGA. Together, the juxtaglomerular cells and macula densa cells form the JGA. The control of renal blood flow, glomerular filtration, and renin secretion is not directed by any of the other options. PTS: 1 REF: Page 1322 6. Kidney stones in the upper part of the ureter would produce pain referred to which anatomical area? a. Vulva or penis c. Thighs b. Umbilicus d. Lower abdomen ANS: B Kidney stones in the upper part of the ureter would produce pain in the umbilicus. Sensory innervation for the upper part of the ureter arises from the tenth thoracic nerve roots with referred pain to the umbilicus. The other options would not experience such referred pain. PTS: 1 REF: Page 1325 7. Innervation of the bladder and internal urethral sphincter is supplied by which nerves? a. Peripheral nerves c. Sympathetic nervous system b. Parasympathetic fibers ANS: B NURSINGTdB..COTMenth thoracic nerve roots The innervation of the bladder and internal urethral sphincter is supplied by parasympathetic fibers of the autonomic nervous system. The process is not dependent on any of the other options. PTS: 1 REF: Page 1326 8. How much urine accumulates in the bladder before the mechanoreceptors sense bladder fullness? a. 75 to 100 ml c. 250 to 300 ml b. 100 to 150 ml d. 350 to 400 ml ANS: C When the bladder accumulates 250 to 300 ml of urine, it contracts and the internal urethral sphincter relaxes through activation of the spinal reflex arc (known as the micturition reflex). PTS: 1 REF: Page 1326 9. What is the trigone? a. A smooth muscle that comprises the orifice of the ureter b. The inner mucosal lining of the kidneys c. A smooth triangular area between the openings of the two ureters and the urethra d. One of the three divisions of the loop of Henle ANS: C The trigone is a smooth triangular area lying between the openings of the two ureters and the urethra. The other options do not accurately identify the trigone. PTS: 1 REF: Page 1325 10. The glomerular filtration rate is directly related to which factor? a. Perfusion pressure in the glomerular capillaries b. Diffusion rate in the renal cortex c. Diffusion rate in the renal medulla d. Glomerular active transport ANS: A The filtration of the plasma per unit of time is known as the glomerular filtration rate (GFR), which is directly related to only the perfusion pressure in the glomerular capillaries. PTS: 1 REF: Page 1326 11. On average, what percent of cardiac output do the kidneys receive? a. 10% to 20% c. 20% to 25% b. 15% to 20% d. 30% to 35% ANS: C The kidneys are highly vascular organs and usually receive 1000 to 1200 ml of blood per minute, or approximately 20% to 25% of the cardiac output. NURSINGTB.COM PTS: 1 REF: Page 1326 12. What effects do exercise and body position have on renal blood flow? a. Exercise and body position activate renal parasympathetic neurons and cause mild vasoconstriction. b. They activate renal sympathetic neurons and cause mild vasoconstriction. c. Both activate renal parasympathetic neurons and cause mild vasodilation. d. They activate renal sympathetic neurons and cause mild vasodilation. ANS: B Exercise and change of body position activate renal sympathetic neurons and cause mild vasoconstriction. The other options do not have these effects on renal blood flow. PTS: 1 REF: Page 1327 13. Blood vessels of the kidneys are innervated by the: a. Vagus nerve c. Somatic nervous system b. Sympathetic nervous system d. Parasympathetic nervous system ANS: B The blood vessels of the kidney are innervated by the sympathetic noradrenergic fibers that cause arteriolar vasoconstriction and reduce renal blood flow. The other options are not involved in this process. 14. When renin is released, it is capable of which action? a. Inactivation of autoregulation b. Direct activation of angiotensin II c. Direct release of antidiuretic hormone (ADH) d. Formation of angiotensin I ANS: D When renin is released, it cleaves an -globulin (angiotensinogen produced by liver hepatocytes) in the plasma to form angiotensin I. PTS: 1 REF: Page 1327 15. What effect do natriuretic peptides have during heart failure when the heart dilates? a. Stimulates antidiuretic hormones. c. Stimulates renin and aldosterone. b. Inhibits antidiuretic hormones. d. Inhibits renin and aldosterone. ANS: D Natriuretic peptides inhibit renin and aldosterone during heart failure when the heart dilates. These make up a group of peptide hormones, including atrial natriuretic peptide (ANP), secreted from myocardial cells in the atria and brain natriuretic peptide (BNP) secreted from myocardial cells in the cardiac ventricles. When the heart dilates during volume expansion or heart failure, ANP and BNP inhibit sodium and water absorption by kidney tubules, inhibit the secretion of renin and aldosterone, vasodilate the afferent arterioles, and constrict the efferent arterioles. The result is increased urine formation, leading to decreased blood volume and blood pressure. NURSINGTB.COM PTS: 1 REF: Page 1327 16. What is the direct action of atrial natriuretic hormone? a. Sodium retention c. Water retention b. Sodium excretion d. Water excretion ANS: B Atrial natriuretic peptide (ANP) and brain natriuretic peptide (BNP) inhibit the secretion of renin, inhibit angiotensin-induced secretion of aldosterone, vasodilate the afferent and constrict the efferent glomerular arterioles, and inhibit sodium and water absorption by kidney tubules. The other actions are not a result of the atrial natriuretic hormone. PTS: 1 REF: Page 1327 17. What term is used to identify the movement of fluids and solutes from the tubular lumen to the peritubular capillary plasma? a. Tubular secretion c. Tubular reabsorption b. Ultrafiltration d. Tubular excretion ANS: C Tubular reabsorption is the movement of fluids and solutes from the tubular lumen to the peritubular capillary plasma. This selection is the only option that correctly identifies the process. PTS: 1 REF: Page 1328 18. How high does the plasma glucose have to be before the threshold for glucose is achieved? a. 126 mg/dl c. 180 mg/dl b. 150 mg/dl d. 200 mg/dl ANS: C When the plasma glucose reaches 180 mg/dl, as occurs in the individual with uncontrolled diabetes mellitus, the threshold for glucose is achieved. PTS: 1 REF: Page 1330 19. Which hormone is required for water to be reabsorbed in the distal tubule and collecting duct? a. Antidiuretic hormone c. Cortisol b. Aldosterone d. Adrenocorticotropin hormone ANS: A Antidiuretic hormone is required for water to be reabsorbed in the distal tubule and collecting duct. The later, straight segment of the distal tubule and the collecting duct are permeable to water as controlled by antidiuretic hormone. The other options are not involved in this process. PTS: 1 REF: Page 1331 20. Which glycoprotein protects against urolithiasis and is a ligand for lymphokines? a. Uromodulin c. Urodilatin b. Nephrin ANS: A NURSINGTdB..COCMystatin Tamm-Horsfall glycoprotein, also known as uromodulin, is the most abundant urinary protein, protects against bacterial adhesion and urolithiasis, and is a ligand for lymphokines. This statement is not true of the other options. PTS: 1 REF: Page 1331 21. What is the end-product of protein metabolism that is excreted in urine? a. Glucose c. Bile b. Ketones d. Urea ANS: D Of the options available, only urea is an end-product of protein metabolism and is the major constituent of urine along with water. PTS: 1 REF: Page 1332 22. What is the action of urodilatin? a. Urodilatin causes vasoconstriction of afferent arterioles. b. It causes vasodilation of the efferent arterioles. c. Urodilatin inhibits antidiuretic hormone secretion. d. It inhibits salt and water reabsorption. ANS: D Urodilatin (a natriuretic peptide) inhibits sodium and water reabsorption from the medullary part of collecting duct, thereby producing diuresis. It is not involved in the actions described by the other options. PTS: 1 REF: Pages 1327-1328 | Table 37-1 23. The concentration of the final urine is determined by antidiuretic hormone (ADH), which is secreted by which gland? a. Posterior pituitary c. Parathyroid b. Thyroid d. Anterior pituitary ANS: A ADH, which is secreted from the posterior pituitary gland, controls the concentration of the final urine. ADH is not secreted by any of the other options. PTS: 1 REF: Pages 1332-1333 24. Which statement is true regarding urodilatin? a. Urodilatin inhibits sodium chloride and water reabsorption in the medullary part of the collecting duct. b. It inhibits antidiuretic hormone (ADH) to prevent water reabsorption in the medullary part of the collecting duct. c. Urodilatin is stimulated by a rise in blood pressure and an increase in extracellular volume. d. It is stimulated by a fall in blood pressure and a decrease in extracellular volume. ANS: C When the circulating volume and NinUcRreSaINseGdTbBl.oCoOdMpressure are increased, the distal tubule and collecting duct produces urodilatin (a natriuretic peptide). Urodilatin inhibits sodium and water reabsorption from the medullary part of collecting duct, thereby producing diuresis. PTS: 1 REF: Page 1328 | Table 37-1 25. What substance stimulates renal hydroxylation in the process of producing vitamin D? a. Erythropoietin c. Calcitonin b. Thyroid hormone d. Parathyroid hormone ANS: D Parathyroid hormone stimulates renal hydroxylation in the process of producing vitamin D. The first step occurs in the liver with hydroxylation at the 25th carbon (calcifediol); the second step in hydroxylation occurs at the first carbon position in the kidneys. The other options are not involved in this process. PTS: 1 REF: Page 1334 26. Which hormone is synthesized and secreted by the kidneys? a. Antidiuretic hormone c. Erythropoietin b. Aldosterone d. Angiotensinogen ANS: C Erythropoietin is produced by the fetal liver and in the adult kidney and is essential for normal erythropoiesis. This statement is not true of the other options. PTS: 1 REF: Page 1334 27. What provides the best estimate of the functioning of renal tissue? a. Glomerular filtration rate b. Hourly urine output c. Serum blood urea nitrogen and creatinine d. The specific gravity of the solute concentration of the urine ANS: A The glomerular filtration rate provides the best estimate of the level of functioning of renal tissue. The other options are not used to assess renal tissue function. PTS: 1 REF: Page 1334 28. Which renal change is found in older adults? a. Sharp decline in glomerular filtration rate b. Sharp decline in renal blood flow c. Decrease in the number of nephrons d. Decrease in urine output ANS: C With aging, the number of nephrons decreases. The other options are not necessarily related to aging. PTS: 1 REF: Page 13N3U6RSINGTB.COM 29. Compared with a younger individual, how is the specific gravity of urine in older adults affected? a. Specific gravity of urine in older adults is increased. b. Specific gravity of urine in older adults is considered high normal. c. Specific gravity of urine in older adults is considered low normal. d. Specific gravity of urine in older adults is decreased. ANS: C The specific gravity of the urine in older individuals tends to be on the low side of normal. PTS: 1 REF: Page 1336 30. What process allows the kidney to respond to an increase in workload? a. Glomerular filtration b. Secretion of 1,25-dihydroxyvitamin D3 c. Increased heart rate d. Compensatory hypertrophy ANS: D Compensatory hypertrophy allows the kidney to respond to an increase in workload throughout life. The remaining options are not relevant to accommodating an increased workload. PTS: 1 REF: Page 1336 31. Which process makes it possible for ureters to be transplanted successfully? a. Compensatory hypertrophy c. Peristalsis b. Erythropoietin secretion d. Collateral circulation ANS: C Peristalsis is the process which makes it possible for ureters to be transplanted successfully. This process is maintained even when the ureter is denervated. The remaining options are not relevant to transplant success. PTS: 1 REF: Page 1325 MULTIPLE RESPONSE 32. Which structures are parts of the nephron? (Select all that apply.) a. Loop of Henle b. Renal corpuscle c. Proximal convoluted tubule d. Calyx e. Collecting duct ANS: A, B, C, E The nephron is a tubular structure with subunits that include the renal corpuscle, proximal convoluted tubule, loop of Henle, distal convoluted tubule, and collecting duct, all of which contribute to the formation of final urine. The calyx is not a structure contained in the nephrons. NURSINGTB.COM PTS: 1 REF: Page 1320 33. Which forces create passive transport of water in the proximal tubule? (Select all that apply.) a. Peritubular capillary hydrostatic pressure b. Peritubular capillary oncotic pressure c. Interstitial hydrostatic pressure d. Interstitial osmotic pressure e. Peritubular capillary osmotic pressure ANS: B, E The osmotic force generated by active sodium transport promotes the passive diffusion of water out of the tubular lumen and into the peritubular capillaries. The elevated oncotic pressure of the blood in the peritubular capillaries further enhances the passive transport of water. The remaining options are not forces that create passive transport of water in the proximal tubule. PTS: 1 REF: Page 1330 34. Which hormones are produced by the kidney? (Select all that apply.) a. Renin b. Erythropoietin c. 1,25-dihydroxyvitamin D3 d. Calcitonin e. Aldosterone ANS: A, B, C The kidney also has an endocrine function, secreting the hormones renin, erythropoietin, and 1,25-dihydroxyvitamin D3 for the regulation of blood pressure, erythrocyte production, and calcium metabolism, respectively. Neither calcitonin nor aldosterone is produced by the kidney. PTS: 1 REF: Page 1319 35. Which statements are true regarding renal circulation? (Select all that apply.) a. The interlobar arteries travel down into the renal columns. b. The arcuate arteries branch to form the interlobar arteries. c. The arcuate arteries arch over the base of the pyramids. d. The interlobar arteries run parallel to the surface of the kidneys. e. The interlobar arteries run between the pyramids. ANS: A, C, E The interlobar arteries are further subdivisions that travel down the renal columns and between the pyramids. At the cortical medullary junction, interlobar arteries branch into the arcuate arteries that arch over the base of the pyramids and run parallel to the surface of the kidney. PTS: 1 REF: Page 1325 MATCHING NURSINGTB.COM Match the descriptions with the corresponding terms. A. Good estimate of glomerular filtration rate B. Form in concentrated acidic or alkaline urine C. Cylindric with distinct borders D. Hematuria E. Pyuria 36. Crystals 37. Casts 38. Leukocytes 39. Creatinine clearance 40. Erythrocytes 36. ANS: B PTS: 1 REF: Page 1336 MSC: Crystals tend to form in a concentrated acidic or alkaline urine. 37. ANS: MSC: C PTS: 1 REF: Page 1336 Casts are cylindrical with distinct borders. 38. ANS: MSC: E PTS: 1 REF: Page 1336 White blood cells in the urine (a condition termed pyuria) are indicative of urinary tract infection, particularly when bacteria are present. 39. ANS: A PTS: 1 REF: Page 1334 MSC: Creatinine clearance provides a good measure of glomerular filtration rate because only one blood sample is required in addition to a 24-hour volume of urine. 40. ANS: D PTS: 1 REF: Pages 1335-1336 MSC: Normal urine contains few or no red blood cells. If a large number of red blood cells are present, then a condition known as hematuria results, and the sediment may be red. NURSINGTB.COM Chapter 39: Alterations of Renal and Urinary Tract Function MULTIPLE CHOICE 1. How does progressive nephrons injury affect angiotensin II activity? a. Angiotensin II activity is decreased. b. It is elevated. c. Angiotensin II activity is totally suppressed. d. It is not affected. ANS: B Angiotensin II activity is elevated with progressive nephron injury. This selection is the only accurate identification of the effect of progressive nephron injury on angiotensin II activity. PTS: 1 REF: Page 1364 | Page 1366 2. Which mineral accounts for the most common type of renal stone? a. Magnesium-ammonium-phosphate c. Calcium oxalate b. Uric acid d. Magnesium phosphate ANS: C Calcium stones (calcium phosphate or calcium oxalate) account for 70% to 80% of all stones requiring treatment. PTS: 1 REF: Page 1343 NURSINGTB.COM 3. Regarding the formation of renal calculi, what function does pyrophosphate, potassium citrate, and magnesium perform? a. They inhibit crystal growth. b. Pyrophosphate, potassium citrate, and magnesium stimulate the supersaturation of salt. c. They facilitate the precipitation of salts from a liquid to a solid state. d. Pyrophosphate, potassium citrate, and magnesium enhance crystallization of salt crystals to form stones. ANS: A Stone or crystal growth inhibiting substances, including potassium citrate, pyrophosphate, and magnesium, are capable of crystal growth inhibition. They are not capable of the functions stated by the other options. PTS: 1 REF: Page 1343 4. Hypercalciuria is primarily attributable to which alteration? a. Defective renal calcium reabsorption b. Intestinal hyperabsorption of dietary calcium c. Bone demineralization caused by prolonged immobilization d. Hyperparathyroidism ANS: B Hypercalciuria is usually attributable to intestinal hyperabsorption of dietary calcium and less commonly to a defect in renal calcium reabsorption. Hyperparathyroidism and bone demineralization associated with prolonged immobilization are also known to cause hypercalciuria but too a much lesser degree. PTS: 1 REF: Page 1343 5. Detrusor hyperreflexia develops from neurologic disorders that originate where? a. Spinal cord between C2 and S1 c. Above the pontine micturition center b. Spinal cord between S2 and S4 d. Below the cauda equina ANS: C Neurologic disorders that develop above the pontine micturition center result in detrusor hyperreflexia, also known as an uninhibited or reflex bladder. This selection is the only option responsible for detrusor hyperreflexia. PTS: 1 REF: Page 1345 6. Considering the innervation of the circular muscles of the bladder neck, which classification of drug is used to treat bladder neck obstruction? a. -Adrenergic blocking medications c. Parasympathomimetic medications b. -Adrenergic blocking medications d. Anticholinesterase medications ANS: B Because the bladder neck consists of circular smooth muscle with adrenergic innervation, detrusor sphincter dyssynergia may be managed by -adrenergic blocking (antimuscarinic) medications. This selection is the only option capable of this specific function. NURSINGTB.COM PTS: 1 REF: Page 1346 7. Renal cell carcinoma, classified as clear cell tumors, arises from epithelial cells in which structure? a. Proximal tubule c. Nephron b. Distal tubule d. Glomerulus ANS: A Renal cell carcinoma, classified as clear cell tumors according to cell type and extent of metastasis, arises from the proximal tubular epithelium. These tumors are not associated with the other options. PTS: 1 REF: Page 1347 8. Bladder cancer is associated with the gene mutation of which gene? a. c-erbB2 b. Human epidermal growth factor receptor 2 (HER2) c. TP53 d. myc ANS: C Oncogenes of the ras gene family and tumor-suppressor genes including TP53 mutations and the inactivation of the retinoblastoma gene (pRb) are implicated in bladder cancer. This process is not associated with the other options. PTS: 1 REF: Page 1348 9. What is the most common cause of uncomplicated urinary tract infections? a. Staphylococcus c. Proteus b. Klebsiella d. Escherichia coli ANS: D The most common infecting microorganisms are uropathic strains of E. coli (80% to 85%). PTS: 1 REF: Page 1350 10. Which differentiating sign is required to make the diagnosis of pyelonephritis from that of cystitis? a. Difficulty starting the stream of urine b. Spasmodic pain that radiates to the groin c. Increased glomerular filtration rate d. Urinalysis confirmation of white blood cell casts ANS: D Clinical assessment, alone, is difficult to differentiate the symptoms of cystitis from those of pyelonephritis. Urine culture, urinalysis, and clinical signs and symptoms establish the specific diagnosis. White blood cell casts indicate pyelonephritis, but they are not always present in the urine. This selection is the only option that is considered a required sign of pyelonephritis. PTS: 1 REF: Pages 1351-1352 NURSINGTB.COM 11. Considering host defense mechanisms, which element in the urine is bacteriostatic? a. High pH (alkaline urine) c. High glucose b. High urea d. High calcium ANS: B Dilute urine washes out bacteria, and urine with higher urea concentrations (high osmolarity) is more bacteriostatic. PTS: 1 REF: Page 1350 12. Which clinical manifestations of a urinary tract infection may be demonstrated in an 85-year-old individual? a. Confusion and poorly localized abdominal discomfort b. Dysuria, frequency, and suprapubic pain c. Hematuria and flank pain d. Pyuria, urgency, and frequency ANS: A Older adults with cystitis may demonstrate confusion or vague abdominal discomfort or otherwise be asymptomatic. PTS: 1 REF: Page 1351 13. Pyelonephritis is usually caused by which type of organism? a. Bacteria c. Viruses b. Fungi d. Parasites ANS: A Pyelonephritis is usually caused by the bacteria Escherichia coli, Proteus, or Pseudomonas. PTS: 1 REF: Pages 1351-1352 14. Which abnormal laboratory value is found in glomerular disorders? a. Elevated creatinine concentration c. Elevated immunoglobulin A (IgA) b. Low blood urea nitrogen (BUN) d. Low serum complement ANS: A Elevated creatinine concentration is an abnormal laboratory value found in glomerular disorders. Reduced glomerular filtration rate during glomerular disease is evidenced by elevated plasma urea, creatinine concentration, or reduced renal creatinine clearance. Glomerular disorders are not associated with the other options. PTS: 1 REF: Page 1357 15. Which glomerular lesion is characterized by thickening of the glomerular capillary wall with immune deposition of immunoglobulin G (IgG) and C3? a. Proliferative c. Mesangial b. Membranous d. Crescentic ANS: B The thickening of the glomerular capillary wall characterizes only membranous lesions. NURSINGTB.COM PTS: 1 REF: Page 1355 | Table 38-6 16. Goodpasture syndrome is an example of which of the following? a. Antiglomerular basement membrane disease b. Acute glomerulonephritis c. Chronic glomerulonephritis d. Immunoglobulin A (IgA) nephropathy ANS: A Antiglomerular basement membrane disease (Goodpasture syndrome) is associated with immunoglobulin G (IgG) antibody formation against pulmonary capillary and glomerular basement membranes. Goodpasture syndrome is not an example of any of the other options. PTS: 1 REF: Page 1355 | Table 38-5 17. A patient exhibits symptoms including hematuria with red blood cell casts and proteinuria exceeding 3 to 5 g/day, with albumin as the major protein. These data suggest the presence of which disorder? a. Cystitis c. Glomerulonephritis b. Chronic pyelonephritis d. Nephrotic syndrome ANS: C The data suggest the patient has the disorder known as glomerulonephritis. Two major changes distinctive of more severe glomerulonephritis are (1) hematuria with red blood cell casts and (2) proteinuria exceeding 3 to 5 g/day with albumin as the major protein. These symptoms do not support the diagnosis of the other options. PTS: 1 REF: Page 1357 18. Hypothyroidism, edema, hyperlipidemia, and lipiduria characterize which kidney disorder? a. Nephrotic syndrome c. Chronic glomerulonephritis b. Acute glomerulonephritis d. Pyelonephritis ANS: A Symptoms of nephrotic syndrome include edema, hyperlipidemia, lipiduria, vitamin D deficiency, and hypothyroidism. These symptoms do not support the other options. PTS: 1 REF: Page 1359 19. Which antibiotics are considered “major culprits” in causing nephrotoxic acute tubular necrosis (ATN)? a. Penicillin and ampicillin c. Gentamicin and tobramycin b. Vancomycin and bacitracin d. Cefazolin and cefepime ANS: C Although numerous antibiotics can produce nephrotoxic ATN, the aminoglycosides (gentamicin, tobramycin) are the major culprits. PTS: 1 REF: Page 13N6U1RSINGTB.COM 20. Which urine characteristics are indicative of acute tubular necrosis (ATN) caused by intrinsic (intrarenal) failure? a. Urine sodium >30 mEq/L b. Urine osmolality >500 mOsm c. Fractional excretion of sodium (FENa) <1% d. Urine sediment has no cells, some hyaline casts ANS: A Urine sodium >30 mEq/L is the only option indicative of ATN. PTS: 1 REF: Page 1363 | Table 38-11 21. How are glucose and insulin used to treat hyperkalemia associated with acute renal failure? a. Glucose has an osmotic effect, which attracts water and sodium, resulting in more dilute blood and a lower potassium concentration. b. When insulin transports glucose into the cell, it also carries potassium with it. c. Potassium attaches to receptors on the cell membrane of glucose and is carried into the cell. d. Increasing insulin causes ketoacidosis, which causes potassium to move into the cell in exchange for hydrogen. ANS: B This selection is the only option that accurately describes glucose metabolism, causing potassium to move to the intracellular fluid; insulin infusions therefore can be effective in shifting potassium from the extracellular to intracellular space, along with the transport of glucose. PTS: 1 REF: Page 1363 22. Creatinine is primarily excreted by glomerular filtration after being constantly released from what type of tissue? a. Nervous system c. Muscle b. Kidneys d. Liver ANS: C Creatinine is constantly released from only muscle tissue to be excreted by glomerular filtration. PTS: 1 REF: Page 1366 23. Which statement is false concerning the skeletal alterations caused by chronic renal failure when the glomerular filtration rate (GFR) declines to 25% of normal? a. Parathyroid hormone is no longer effective in maintaining serum phosphate levels. b. The parathyroid gland is no longer able to secrete sufficient parathyroid hormone. c. The synthesis of 1,25-vitamin D3, which reduces intestinal absorption of calcium, is impaired. d. The synthesis of 1,25-vitamin D3, which impairs the effectiveness of calcium and phosphate resorption from bone by parathyroid hormone, is impaired. ANS: B NURSINGTB.COM Bone and skeletal changes develop with alterations in calcium and phosphate metabolism (see Table 38-16). These changes begin when the GFR decreases to 25% or less. The combined effect of hyperparathyroidism and vitamin D deficiency can result in renal osteodystrophies (e.g., osteomalacia, osteitis fibrosa with increased risk for fractures). Other consequences of secondary hyperparathyroidism include soft-tissue and vascular calcification, cardiovascular disease, and, less commonly, calcific uremic arteriolopathy. The other options are true. PTS: 1 REF: Page 1368 | Table 38-16 24. Anemia of chronic renal failure can be successfully treated with which element? a. Intrinsic factor c. Vitamin D b. Vitamin B12 d. Erythropoietin ANS: D Anemia of chronic renal failure can be successfully treated with erythropoietin. Reduced erythropoietin secretion and reduced red cell production are evident in anemia resulting from chronic renal failure. The other options are not relevant to this condition. PTS: 1 REF: Page 1365 | Page 1369 | Table 38-13 25. When the right kidney is obstructed, how will the glomeruli and tubules in the left kidney compensate? a. Increase in number c. Develop collateral circulation b. Increase in size d. Increase speed of production ANS: B These processes cause the contralateral (unobstructed) kidney to increase the size of individual glomeruli and tubules. The changes described by the other options are not correct. PTS: 1 REF: Page 1342 26. What medical term is used to identify a functional urinary tract obstruction caused by an interruption of the nerve supply to the bladder? a. Neurogenic bladder c. Necrotic bladder b. Obstructed bladder d. Retrograde bladder ANS: A Neurogenic bladder is a general term for bladder dysfunction caused by neurologic disorders. The types of dysfunction are related to the sites in the nervous system that control sensory and motor bladder function (see Figure 38-3). None of the other options correctly identify the described condition. PTS: 1 REF: Pages 1344-1345 MULTIPLE RESPONSE 27. In glomerulonephritis, what damages the epithelial cells resulting in proteinuria? (Select all that apply.) a. Ischemia b. Lysosomal enzymes c. Compression from edema d. Activated complement NURSINGTB.COM e. Altered membrane permeability ANS: D, E Activated complement, inflammatory cytokines, oxidants, proteases, and growth factors attack epithelial cells, alter membrane permeability, and cause proteinuria. None of the other options are responsible for this process. PTS: 1 REF: Pages 1353-1357 28. Which statements are true concerning struvite stones? (Select all that apply.) a. They are more common in women than in men. b. Struvite stones are associated with chronic laxative use in women. c. They grow large and branch into a staghorn configuration in renal pelvis and calyces. d. Struvite stones are closely associated with urinary tract infections caused by urease-producing bacteria, such as Pseudomonas. e. They are more common in men than in women. ANS: A, C, D Women are at greater risk for developing struvite stones, but the risk is greater because women have an increased incidence of urinary tract infections not because of chronic laxative use. Such stones grow large and branch into a staghorn configuration and are associated with Pseudomonas. PTS: 1 REF: Pages 1343-1344 29. Which statements are true regarding renal colic? (Select all that apply.) a. Renal colic results in mild-to-moderate pain. b. Pain originates in the flank area. c. Renal colic indicates the presence of renal stones. d. Pain radiated to the groin. e. Renal colic indicates obstruction of the renal pelvis or proximal ureter. ANS: B, C, D, E Renal colic is described as moderate-to-severe pain often originating in the flank and radiating to the groin. It usually indicates obstruction by renal stones of the renal pelvis or proximal ureter. PTS: 1 REF: Page 1344 30. Which conditions related to the bladder would result from the effects of lesions of the sacral segments below S1? (Select all that apply.) a. Frequency b. Urge incontinence c. Bladder distension d. Urgency e. Urinary retention ANS: C, E NURSINGTB.COM Lesions that involve the sacral micturition center (below S1; may also be termed cauda equina syndrome) or peripheral nerve lesions result in detrusor areflexia (acontractile detrusor), a lower motor neuron disorder. The result is an acontractile detrusor or atonic bladder with retention of urine and distention. The other options are associated with neurologic lesions that occur between C2 and S1 PTS: 1 REF: Page 1345 31. What are considered risk factors for developing bladder and kidney cancers? (Select all that apply.) a. Cigarette smoking b. Hypertension c. Exposure to aniline dyes d. Below normal body weight e. Male gender ANS: A, B, C Risk factors for renal cancer include cigarette smoking, obesity, and hypertension. The risk of primary bladder cancer is greater among people who smoke or those who are exposed to metabolites of aniline dyes or other aromatic amines or chemicals and with heavy consumption of phenacetin. Gender is not a recognized risk factor. PTS: 1 REF: Pages 1347-1349 32. Which renal disorders are considered causes of intrarenal renal failure? (Select all that apply.) a. Acute glomerulonephritis b. Allograft rejection c. Tumors d. Acute tubular necrosis (ATN) e. Prostatic hypertrophy ANS: A, B, C, D Intrarenal (intrinsic) acute kidney injury (AKI) may result from ischemic ATN, nephrotoxic ATN, acute glomerulonephritis, vascular disease, allograft rejection, or interstitial disease (drug allergy, infection, tumor growth). Prostatic hypertrophy is not associated with intrarenal renal failure. PTS: 1 REF: Pages 1360-1361 33. Prerenal injury from poor perfusion can result from which condition? (Select all that apply.) a. Bilateral ureteral obstruction b. Renal vasoconstriction c. Renal artery thrombosis d. Hemorrhage e. Hypotension ANS: B, C, D, E NURSINGTB.COM Poor perfusion can result from renal artery thrombosis, hypotension related to hypovolemia (dehydration, diarrhea, fluid shifts) or hemorrhage, renal vasoconstriction and alterations in renal regional blood flow, microthrombi, or kidney edema that restricts arterial blood flow. Bilateral ureteral obstruction is not associated with prerenal injuries. PTS: 1 REF: Page 1360 MATCHING Match the predisposing factors causing pyelonephritis to the pathology mechanism. A. Kidney stones B. Vesicoureteral reflux C. Pregnancy D. Neurogenic bladder E. Female sexual trauma 34. Movement of organisms from the urethra into the bladder with infection and retrograde spread to the kidney 35. Dilation and relaxation of the ureter with hydroureter and hydronephrosis 36. Impairment to the bladder, interfering with normal bladder contraction and causing residual urine and ascending infection 37. Obstruction and stasis of urine, contributing to bacteremia and hydronephrosis; irritation of epithelial lining with entrapment of bacteria 38. Chronic reflux of urine up the ureter and into the kidney during micturition, contributing to bacterial infection 34. ANS: E PTS: 1 REF: Page 1352 | Table 38-4 MSC: Female sexual trauma can result from the movement of organisms from the urethra into the bladder with infection and retrograde spread to the kidney. 35. ANS: C PTS: 1 REF: Page 1352 | Table 38-4 MSC: Pregnancy causes the dilation and relaxation of the ureter with hydroureter and hydronephrosis. 36. ANS: D PTS: 1 REF: Page 1352 | Table 38-4 MSC: Neurogenic bladder, caused by neurologic impairment, results in the interference of normal bladder and urethral sphincter contraction and causes residual urine and ascending infection. 37. ANS: A PTS: 1 REF: Page 1352 | Table 38-4 MSC: Kidney stones result in the obstruction and stasis of urine, contributing to bacteriuria and hydronephrosis and causing irritation of epithelial lining with entrapment of bacteria. 38. ANS: B PTS: 1 REF: Page 1352 | Table 38-4 MSC: Vesicoureteral reflux results in the chronic reflux of urine up the ureter and into the kidney during micturition, contributing to bacterial infection. NURSINGTB.COM Chapter 40: Alterations of Renal and Urinary Tract Function in Children MULTIPLE CHOICE 1. The functional kidney is associated with which embryonic organ? a. Metanephros c. Pronephros b. Mesonephros d. Endonephros ANS: A The functional kidney is associated with the metanephros. The kidney develops from three sets of structures: the pronephros (nonfunctional by the end of the embryonic period), mesonephros (nonfunctional), and metanephros (the functional kidney). Endonephros is not relevant to this process. PTS: 1 REF: Pages 1376-1377 | Figure 39-1 2. When does urine formation and excretion begin? a. At birth c. By 6 months’ gestation b. By 3 months’ gestation d. By 8 months’ gestation ANS: B Urine formation and excretion begin by the third month of gestation. PTS: 1 REF: Page 1377 3. Compared with an adult, an infant has a greater content of extracellular fluid, as well as a greater rate of fluid exchange. WhNaUt RefSfIeNcGt TdBoe.CsOthMis have on the fluid balance of a child compared with that of an adult? a. Edema development is less of a problem. b. Overhydration is not difficult to manage. c. Daily fluid requirements are greater. d. The control of dehydration is more difficult. ANS: D An infant not only has a greater content of extracellular fluid, but infants also have a greater rate of fluid exchange. The adult takes in and excretes approximately 2000 ml of water daily, representing 5% of the total body fluid and 14% of the extracellular fluid. In contrast, the infant’s daily exchange of 600 to 700 ml of water represents 290% of the total body fluid or nearly 50% of the extracellular volume, making control of dehydration and overhydration more difficult. This fact makes the other options incorrect. PTS: 1 REF: Page 1378 4. What term is used to identify the condition that exists when the urethral meatus is located on the undersurface of the penis? a. Hypospadias c. Hyperspadias b. Epispadias d. Chordee ANS: A Hypospadias is a congenital condition in which the urethral meatus is located on the ventral side or undersurface of the penis. This is the only term used for this condition. PTS: 1 REF: Page 1378 5. What initiates inflammation in acute poststreptococcal glomerulonephritis? a. Lysosomal enzymes b. Endotoxins from Streptococcus c. Immune complexes d. Immunoglobulin E (IgE)–mediated response ANS: C The immune complexes initiate inflammation and glomerular injury in acute poststreptococcal glomerulonephritis. Antigen-antibody complexes are deposited in the glomerulus, or the antigen may be trapped within the glomerulus and immune complexes formed in situ. The other options are not involved in initiating inflammation in this situation. PTS: 1 REF: Page 1381 6. Acute glomerulonephritis (AGN) may be accompanied by a positive throat or skin culture for which bacteria? a. Staphylococcus aureus c. Pseudomonas aeruginosa b. Streptococcus d. Haemophilus ANS: B AGN may be accompanied by a positive throat or skin culture for Streptococcus. AGN is not associated with any of the other options. PTS: 1 REF: Page 13N8U2RSINGTB.COM 7. What is the cause of smoky, brown-colored urine resulting from acute poststreptococcal glomerulonephritis? a. Presence of red blood cells c. Slough from the collecting tubules b. Presence of urobilinogen d. Protein in the urine ANS: A The urine is usually smoky brown or cola-colored because of the presence of red blood cells. The other options are not correct statements regarding this condition. PTS: 1 REF: Pages 1381-1382 8. In immunoglobulin G (IgG) nephropathies such as glomerulonephritis, IgG is deposited in which location? a. Juxtamedullary nephrons b. Glomerulus basement membranes c. Mesangium of the glomerular capillaries d. Parietal epithelium ANS: B Glomerulonephritis develops with the deposition of antigen-antibody complexes (IgG, immunoglobulin A [IgA], and C3 complement) in the glomerulus, or the antigen may be trapped within the glomerulus and immune complexes formed in situ. Immunofluorescence microscopy shows lumpy deposits of IgG and C3 complement on the glomerular basement membrane (see Figure 39-5). When considering IgG nephropathies, the only location of the IgG immunoglobulins is the correct option. PTS: 1 REF: Page 1381 9. What is the pathophysiologic process responsible for the autoimmune disorder of hemolytic-uremic syndrome (HUS)? a. Immunoglobulin A (IgA) coats erythrocytes that are destroyed by the spleen, and remnants are excreted through the kidneys. b. Verotoxin from Escherichia coli is absorbed from the intestines and damages erythrocytes and endothelial cells. c. Endotoxins from E. coli block the erythropoietin produced by the kidneys, which reduces the number of erythrocytes produced by the bone marrow. d. Failure of the nephron to filter urea increases the blood urea nitrogen, which binds to erythrocytes that are subsequently destroyed by the spleen. ANS: B In HUS, verotoxin from E. coli is absorbed from the intestines, the glomerular arterioles become swollen, and these narrowed vessels damage erythrocytes as they pass through. HUS is responsible for causing a cascade of effects, including lysis of glomerular capillary endothelial cells. The other options are not relevant. PTS: 1 REF: Page 13N8U3RSINGTB.COM 10. What is the first indication of nephrotic syndrome in children? a. Periorbital edema c. Frothy urine b. Scrotal or labial edema d. Ascites ANS: A Onset of nephritic syndrome is insidious, with periorbital edema as the first sign of the disorder. None of the other options represent the first indication of nephritic syndrome in children. PTS: 1 REF: Page 1384 11. Bacteria gain access to the female urinary tract by which means? a. Systemic blood that is filtered through the kidney b. Bacteria traveling from the lymph adjacent to the bladder and kidneys c. Bacteria ascending the urethra into the bladder d. Colonization of the bladder when urine is static ANS: C Urinary tract infections (UTIs) in girls occur as a result of perineal bacteria, especially Escherichia coli, ascending the urethra. None of the other options represent the means by which bacteria gain access to the female urinary tract. PTS: 1 REF: Page 1386 12. What causes vesicoureteral reflux to occur in children? a. Children do not ask for help in urinating in a timely manner, and urine is forced up into the ureters. b. The submucosal segment of a child’s ureter is short, making the antireflux mechanism inefficient. c. The trigone lying between the opening to the ureters and the urethra is underdeveloped in children. d. As the bladder fills in infants and children, it pulls the smooth lining of the transitional epithelium away from the ureters, making the reflux valves ineffective. ANS: B Although reflux is considered abnormal at any age, the shortness of the submucosal segment of the ureter during infancy and childhood renders the antireflux mechanism relatively inefficient and delicate. The other options are not considered reasons for this reflux. PTS: 1 REF: Page 1387 13. What is the mechanism for developing Wilms tumor? a. The development of a Wilms tumor involves tumor-suppressor genes located on chromosome 11. b. Development involves an autosomal dominant inherited disorder involving the Y chromosome. c. Wilms tumor is an autoimmune disorder. d. The development of a Wilms tumor is a congenital anomaly. ANS: A Wilms tumor–suppressor genes WNTU1RaSnIdNGWTTB.2COarMe located on chromosome 11. The other options are not considered relevant to the development of a Wilms tumor. PTS: 1 REF: Pages 1385-1386 14. Which anomaly is often associated with Wilms tumor? a. Renal anaplasia c. Anemia b. Aniridia d. Hypothyroidism ANS: B Aniridia (lack of an iris in the eye) is an anomaly often associated with Wilms tumor. Approximately 10% of children who have Wilms tumor also have a loss of other important genes and therefore have a number of congenital anomalies. The other anomalies listed are not associated with a Wilms tumor. PTS: 1 REF: Pages 1385-1386 15. Which statement is false about the causes of enuresis? a. A maturational lag may cause enuresis. b. Enuresis may be related to increased light sleep. c. Obstructive sleep apnea may be a symptom of enuresis. d. Excessive nocturnal levels of vasopressin may cause enuresis. ANS: D Children who do not have the normal nocturnal elevation of vasopressin produce a higher volume of urine with a lower osmolality. The other options are accurate statements regarding enuresis. PTS: 1 REF: Pages 1388-1389 16. When does an individual have their full complement of renal nephrons? a. At birth c. At puberty b. At 6 months of age d. Between the ages 18 and 21 years ANS: A All the nephrons are present at birth, and their number does not increase as the kidney grows and matures. PTS: 1 REF: Pages 1376-1377 MULTIPLE RESPONSE 17. Which statements are true regarding the narrow chemical safety margin that infants demonstrate? (Select all that apply.) a. The immaturity of tubules may diminish the response to antidiuretic hormone (ADH). b. An immature tubular transport capacity impairs the excretion of potassium. c. An immature tubular transport capacity impairs the reabsorption of bicarbonate. d. The immaturity of tubules may diminish the response to aldosterone. e. The safety margin begins to expand after 2 years of age. ANS: A, B, C, E NURSINGTB.COM Because of a high hydrogen ion concentration, a limited ability to regulate the internal environment, and a lowered osmotic pressure, the infant’s renal system has a narrow chemical safety margin. The immaturity and smaller surface area of the tubules also may diminish the water reabsorption response to ADH. An immature tubular transport capacity means that the ability to excrete a potassium load, reabsorb bicarbonate, or buffer hydrogen with ammonia does not become efficient until approximately 2 years of age. PTS: 1 REF: Page 1378 18. The excretion of urea is low in infants because of which feature? (Select all that apply.) a. Medullary nephrons with comparatively short loops at this stage of development b. Immature tubular transport capacity, impairing the excretion of urea c. High anabolic state d. Dilute urine as a result of the immaturity of an infant’s glomeruli e. Available protein used for physical growth ANS: C, E Urea excretion is low in infants primarily because infants are in a high anabolic state and use their protein for growth. The other options are not involved in their low urea excretion. PTS: 1 REF: Page 1378 19. Which factors are involved in the development of hypospadias? (Select all that apply.) a. Disruption in male hormones, including testosterone b. Mutation of gene HP-1 (chromosome 16) c. 5-reductase mutations d. Hormones administered for in vitro fertilization e. Advanced maternal age ANS: A, C, D, E The cause of hypospadias is multifactorial and related to disruptions in male hormones, including testosterone biosynthesis defects, 5-reductase mutations, hormones administered for in vitro fertilization, advanced maternal age, and other environmental factors. A mutation of gene HP-1 (chromosome 16) is not relevant. PTS: 1 REF: Page 1378 20. Which cells of the inflammatory process are found in acute poststreptococcal glomerulonephritis? (Select all that apply.) a. Immunoglobulin G (IgG) b. Immunoglobulin A (IgA) c. Complement C3 d. Immunoglobulin E (IgE) e. Immune complexes ANS: A, B, C, E Antigen-antibody complexes of IgG, IgA, and C3 complement are deposited in the glomerulus, or the antigen may be trapped within the glomerulus and immune complexes formed in situ. IgE is not relevant to this situation. PTS: 1 REF: Page 13N8U1RSINGTB.COM 21. Which symptoms are considered part of the nephrotic syndrome in children? (Select all that apply.) a. Proteinuria b. Pyuria c. Hyperlipidemia d. Edema e. Hypoalbuminemia ANS: A, C, D, E Nephrotic syndrome is a term used to describe a symptom complex characterized by proteinuria, hypoalbuminemia, hyperlipidemia, and edema. Pyuria is not observed in this condition. PTS: 1 REF: Page 1383 22. Which statements are true regarding hemolytic uremic syndrome (HUS)? (Select all that apply.) a. Microangiopathic hemolytic anemia characterizes HUS. b. Thrombocytopenia is a comorbid condition. c. Wilms tumors are often the trigger for HUS. d. HUS is the most common community-acquired cause of acute renal failure in young children. e. HUS is chronic in nature. ANS: A, B, D HUS is an acute disorder characterized by microangiopathic hemolytic anemia and thrombocytopenia and is the most common community-acquired cause of acute renal failure in young children. Wilms tumors are not necessarily associated with this disorder. PTS: 1 REF: Pages 1382-1383 23. What problems are commonly associated with the diagnosis of horseshoe kidneys? (Select all that apply.) a. Hydronephrosis b. Nephritis c. Infection d. Kidney stones e. Pyuria ANS: A, C, D Approximately one-third of individuals with horseshoe kidneys are asymptomatic, and the most common problems are hydronephrosis, infection, and stone formation. Nephritis and pyuria are not associated with this diagnosis. PTS: 1 REF: Page 1378 24. Which structural anomalies are associated with urinary tract malformations? (Select all that apply.) a. Low-set ears b. Prune-belly syndrome c. Broad-bridged nose d. Imperforate anus e. Malformed lips ANS: A, B, D NURSINGTB.COM Structural anomalies that are associated with urinary tract malformations include low-set, malformed ears; absent abdominal muscles (prune-belly syndrome); and imperforate anus or genital deviation. Anomalies of the either the nose or the lips are not generally associated with urinary tract malformations. PTS: 1 REF: Page 1378 MATCHING Match the kidney disorder with the corresponding descriptions. A. Hemolytic-uremic syndrome B. Henoch-Schönlein purpura nephritis C. Renal dysplasia D. Ureteropelvic junction obstruction E. Polycystic kidney disease 25. Is usually the result of abnormal differentiation of the renal tissues. 26. Is an autosomal dominant inherited disorder. 27. Fibrin-rich thrombi can be found throughout the microcirculation. 28. Is the most common cause of hydronephrosis in neonates. 29. Immunoglobulin A (IgA) nephropathy causes inflammation to glomerular blood vessels. 25. ANS: C PTS: 1 REF: Page 1380 MSC: Renal dysplasia usually results from abnormal differentiation of the renal tissues. 26. ANS: E PTS: 1 REF: Page 1381 MSC: Polycystic kidney disease is an autosomal dominant inherited disorder that occurs in approximately 1 in 1000 live births. 27. ANS: A PTS: 1 REF: Pages 1382-1383 MSC: Hemolytic-uremic syndrome (HUS) results in fibrin-rich thrombi found throughout the microcirculation. 28. ANS: D PTS: 1 REF: Page 1380 MSC: Ureteropelvic junction obstruction is the most common cause of hydronephrosis in neonates. 29. ANS: B PTS: 1 REF: Page 1382 MSC: Henoch-Schönlein purpura nephritis, also known as anaphylactoid purpura, is an IgA nephropathy that affects the glomerular blood vessels, causing inflammation and damage to the vessel wall. NURSINGTB.COM Chapter 41: Structure and Function of the Digestive System MULTIPLE CHOICE 1. In the mouth and stomach, salivary -amylase initiates the digestion of which nutrients? a. Proteins c. Fats b. Carbohydrates d. Fiber ANS: B Salivary -amylase is an enzyme that initiates only carbohydrate digestion in the mouth and stomach. PTS: 1 REF: Page 1395 2. Which elements in saliva protect against tooth decay? a. Salivary -amylase and ptyalin b. Secretin and IgA c. Endogenous enamelin and salivary -amylase d. Exogenous fluoride and a pH of 7.4 ANS: D The bicarbonate concentration of saliva sustains a pH of approximately 7.4, which neutralizes bacterial acids and prevents tooth decay. Exogenous fluoride (e.g., fluoride in drinking water) is absorbed and then secreted in the saliva, providing additional protection against tooth decay. Decay prevention is not associated with any of the other options. PTS: 1 REF: Page 13N9U5RSINGTB.COM 3. Saliva contains which immunoglobulin (Ig)? a. IgA c. IgG b. IgE d. IgM ANS: A Saliva contains only IgA, which helps prevent infection. PTS: 1 REF: Page 1395 4. What effect is a result of inhibiting the parasympathetic nervous system with a drug such as atropine? a. Salivation becomes thinner. c. The pH of saliva changes. b. Salivation decreases. d. Digestive enzymes are inhibited. ANS: B The sympathetic and parasympathetic divisions of the autonomic nervous system control salivation. Because cholinergic parasympathetic fibers stimulate the salivary glands, atropine (an anticholinergic agent) inhibits salivation and makes the mouth dry. None of the other options is a result of drugs such as atropine. PTS: 1 REF: Page 1395 5. Food enters the stomach via which sphincter? a. Cardiac c. Gastric b. Upper esophageal d. Fundal ANS: A Each end of the esophagus is opened and closed by a sphincter. The upper esophageal sphincter (cricopharyngeal muscle) prevents entry of air into the esophagus during respiration. The lower esophageal sphincter (cardiac sphincter) prevents regurgitation from the stomach. The lower esophageal sphincter is located near the esophageal hiatus—the opening in the diaphragm where the esophagus ends at the stomach. The cardiac sphincter is the only option that fulfills the function described in the question. PTS: 1 REF: Page 1395 6. Which gastric cells secrete hydrochloric acid and intrinsic factor? a. Parietal c. G b. Chief d. H ANS: A Of the available options, only the parietal cells (oxyntic cells) secrete hydrochloric acid and intrinsic factor. PTS: 1 REF: Page 1398 7. Which cells in the stomach secrete histamine? a. Oxyntic c. D b. Chief d. Enterochromaffin-like ANS: D NURSINGTB.COM Of the available options, only enterochromaffin-like cells secrete histamine. PTS: 1 REF: Page 1398 8. The presence of chyme in the duodenum stimulates which hormones? a. Motilin and histamine b. Secretin and cholecystokinin c. Enteroglucagon and gastric inhibitory peptide d. Somatostatin and acetylcholine ANS: B The movement of chyme from the stomach into the duodenum initiates the intestinal phase of secretion. This phase represents a slowdown of the gastric secretory response; however, the presence of digested protein and amino acids in the duodenum continue to stimulate some gastric secretion. The release of secretin and cholecystokinin stimulate pancreatic secretions and inhibit gastric secretions. This selection is the only option that correctly identifies the hormones that are stimulated by the presence of chyme in the duodenum. PTS: 1 REF: Page 1400 9. Which gastric hormone inhibits acid and pepsinogen secretion, as well as decrease the release of gastrin? a. Bombesin c. Somatostatin b. Histamine d. Acetylcholine ANS: C Of the options available, only somatostatin inhibits the secretion of acid and pepsinogen and decreases gastrin release. PTS: 1 REF: Page 1397 | Table 40-1 10. Which enzyme breaks down protein-forming polypeptides in the stomach? a. Acetylcholine c. Gastrin b. Pepsin d. Secretin ANS: B Of the options available, only pepsin, a proteolytic enzyme, breaks down protein-forming polypeptides in the stomach. PTS: 1 REF: Page 1399 11. Exposure to which substance protects the mucosal barrier of the stomach? a. Prostaglandins c. Helicobacter pylori b. Aspirin d. Regurgitated bile ANS: A Prostaglandins and enterogastrones, such as gastric inhibitory peptide, somatostatin, and secretin, inhibit acid secretion. This selection is the only option that accurately identifies a substance that protects the mucosal barrier of the stomach. PTS: 1 REF: Page 1399 12. The ileum and jejunum are suspenNdUeRdSbIyNGfoTlBd.sCoOfMthe peritoneum that contain an extensive vascular and nervous network. What are these folds called? a. Ligament of Treitz c. Auerbach folds b. Mesentery d. Lamina propria ANS: B The ileum and jejunum are suspended in loose folds from the posterior abdominal wall by a peritoneal membrane called the mesentery. The mesentery facilitates intestinal motility and supports blood vessels, nerves, and lymphatics. None of the other options accurately fills this description. PTS: 1 REF: Page 1401 13. Where in the small intestines are lymphocytes, plasma cells, and macrophages produced? a. Brush border c. Lamina propria b. Microvilli d. Crypts of Lieberkühn ANS: C The lamina propria, which is a connective tissue layer of the mucous membrane, lies beneath the epithelial cells of the villi and contains lymphocytes, plasma cells, which produce immunoglobulins, and macrophages. None of the other options performs the described function. PTS: 1 REF: Page 1401 14. Why are water and electrolytes transported in both directions through tight junctions and intercellular spaces rather than across cell membranes? a. The intercellular hydrostatic pressure is inadequate to push the water and electrolytes across the cell membranes. b. A balance of cations and ions among the electrolytes on each side of the cell membranes cannot be maintained. c. The epithelial cell membranes are formed of lipids that are hydrophobic and therefore repel water. d. Receptors on those cell membranes are occupied with a diffusion of amino acids and monosaccharides. ANS: C The epithelial cell membranes of the small intestine are formed of lipids and are hydrophobic or tend to repel water. Therefore water and electrolytes are transported in both directions—toward the capillary blood or toward the intestinal lumen—through the tight junctions and intercellular spaces rather than across cell membranes. This selection is the only option that accurately describes why water and electrolytes are transported in both directions. PTS: 1 REF: Pages 1401-1402 15. Which statement best describes the gastrointestinal tract? a. The gastrointestinal tract is a muscular tube that transports food from the mouth to the stomach. b. The gastrointestinal tract is a hollow tube that extends from the mouth to the anus. c. The gastrointestinal tract is a baglike structure that propels partially digested food (chyme). NURSINGTB.COM d. The structure is 5 m long and consists of three segments. ANS: B The gastrointestinal tract is a hollow tube that extends from the mouth to the anus. The esophagus is a muscular tube that transports food from the mouth to the stomach. The stomach is a baglike structure that secretes digestive juices, mixes and stores food, and propels partially digested food (chyme) into the duodenum. The small intestine is 5 m long and has three segments. PTS: 1 REF: Page 1395 16. Glucose transport enhances the absorption of which electrolyte? a. Sodium c. Potassium b. Phosphate d. Chloride ANS: A Sodium passes through the tight junctions and is actively transported across cell membranes. Sodium and glucose share a common active transport carrier (sodium-glucose ligand transporter 1 [SGLT1]). This statement is not true of any other option. PTS: 1 REF: Page 1402 17. What process is capable of increasing both intrathoracic and intraabdominal pressure, thereby facilitating defecation? a. Relaxation of the internal anal sphincter b. Intestinal peristalsis c. Valsalva maneuver d. Ileogastric reflex ANS: C Squatting or sitting facilitate defecation; these positions straighten the angle between the rectum and anal canal and increase the efficiency of straining (increasing intraabdominal pressure). Initiating the Valsalva maneuver increases intraabdominal pressure. This maneuver consists of inhaling and forcing the diaphragm and chest muscles against the closed glottis and increases both intrathoracic and intraabdominal pressure, which is transmitted to the rectum. None of the other options facilitate defecation as described. PTS: 1 REF: Page 1408 18. Which pancreatic enzyme is responsible for the breakdown of carbohydrates? a. Trypsin c. Lipase b. Amylase d. Chymotrypsin ANS: B Salivary and pancreatic amylases break down starches to oligosaccharides by splitting -1,4-glucosidic linkages of long-chain molecules. None of the other options is relevant to this process. PTS: 1 REF: Page 1402 19. The formation of water-soluble molecules to facilitate the absorption of the by-products of lipid hydrolysis is accomplished bNyU: RSINGTB.COM a. Micelles c. Chylomicrons b. Phospholipase d. Colipase ANS: A The products of lipid hydrolysis must be made water soluble if they are to be efficiently absorbed from the intestinal lumen. The formation of water-soluble molecules known as micelles (see Figure 40-14) accomplishes this process. This selection is the only option that accurately identifies the molecule needed to facilitate lipid hydrolysis. PTS: 1 REF: Page 1404 20. What is the primary source of physiologic iron? a. Transferrin from plasma c. Bile from bilirubin b. Pepsin form pepsinogen d. Heme from animal protein ANS: D The primary source of iron is heme from animal protein. The other options are not considered primary sources of physiologic iron. PTS: 1 REF: Page 1405 21. Which statement is incorrect regarding the characteristics of vitamin B12? a. Vitamin B12 is absorbed in the terminal ileum. b. Vitamin B12 is absorbed in its free (unbound) form but only in small amounts. c. Vitamin B12 is necessary for platelet maturation. d. Vitamin B12 binds to intrinsic factor. ANS: C Vitamin B12 is not necessary for platelet maturation. Most vitamin B12 (cobalamin) is bound to intrinsic factor (making it resistant to digestion) and is absorbed in the terminal ileum, although a small amount of the vitamin is absorbed in its free (unbound) form. PTS: 1 REF: Page 1406 22. Which water-soluble vitamin is absorbed by passive diffusion? a. Vitamin B6 c. Vitamin K b. Vitamin B1 d. Folic acid ANS: A Of the options available, only Vitamin B6 is the water-soluble vitamin absorbed by passive diffusion. PTS: 1 REF: Page 1406 | Table 40-2 23. Which vitamin facilitates the absorption of iron by the epithelial cells of the duodenum and jejunum? a. B6 c. E b. C d. B12 ANS: B Of the options available, only vitamin C reduces ferric iron to ferrous iron, which is the form more easily absorbed by the epithelial cells of the duodenum and jejunum. NURSINGTB.COM PTS: 1 REF: Page 1405 24. What is the role of the normal intestinal bacterial flora? a. Metabolizing bile salts, estrogens, and lipids b. Breaking down proteins into amino acids c. Facilitating the motility of the colon d. Metabolizing aldosterone and insulin ANS: A The intestinal bacteria play a role in the metabolism of bile salts, contributing to the intestinal reabsorption of bile and the elimination of toxic bile metabolites. These bacteria also play a role in the metabolism of estrogens, androgens, and lipids, as well as in the conversion of unabsorbed carbohydrates to absorbable organic acids, the synthesis of vitamin K2, and the metabolism of various nitrogenous substances and drugs. This selection is the only option that accurately describes the role of normal intestinal bacterial flora. PTS: 1 REF: Page 1409 25. Kupffer cells are best described as: a. Natural killer cells that produce interferon-gamma (IFN-) b. Contractile and therefore capable of regulating the sinusoid blood flow c. Bactericidal and therefore central to innate immunity d. Able to metabolize estrogen, progesterone, and androgens ANS: C The sinusoids are lined with phagocytic cells, known as Kupffer cells, which are part of the mononuclear phagocyte system and are the largest population of tissue macrophages in the body. They are bactericidal and central to innate immunity. None of the other options accurately describe Kupffer cells. PTS: 1 REF: Pages 1409-1410 26. Bilirubin is a by-product of the destruction of which aged cells? a. Platelets c. Leukocytes b. Protein d. Erythrocytes ANS: D Bilirubin is a by-product of only the destruction of aged red blood cells or erythrocytes. PTS: 1 REF: Page 1411 27. The process of conjugation of bilirubin in the liver is best described as which transformation? a. Unconjugated (fat-soluble) bilirubin into urobilinogen b. Unconjugated (fat-soluble) bilirubin into conjugated (water-soluble) bilirubin c. Conjugated (water-soluble) bilirubin into unconjugated (fat-soluble) bilirubin d. Conjugated (water-soluble) bilirubin into urobilinogen ANS: B In the liver, unconjugated bilirubin moves from plasma in the sinusoids into the NURSINGTB.COMhepatocytes. Within hepatocytes acid to form conjugated bilirubin, which is water-soluble. This selection is the only option that accurately describes the transformation associated with the conjugation of bilirubin. PTS: 1 REF: Pages 1411-1412 28. Which statement is incorrect regarding gallbladder function? a. Within 30 minutes of eating, the gallbladder forces bile into the stomach. b. Cholinergic branches of the vagus nerve mediate gallbladder contraction. c. Cholecystokinin provides hormonal regulation of gallbladder contraction. d. The sphincter of Oddi controls the flow of bile from the gallbladder. ANS: A Within 30 minutes after eating, the gallbladder begins to contract and the sphincter of Oddi relaxes, forcing bile into the duodenum through the major duodenal papilla. The other options are correct statements regarding the functioning of the gallbladder. PTS: 1 REF: Page 1413 29. Which structure synthesizes clotting factors and the vitamin K necessary for hemostasis? a. Colon c. Gallbladder b. Spleen d. Liver ANS: D The liver has hemostatic functions. It synthesizes prothrombin, fibrinogen, and factors I, II, VII, IX, and X, all of which are necessary for effective clotting. Vitamin K, a fat-soluble vitamin, is essential for the synthesis of other clotting factors. Because bile salts are needed for the absorption of fats, vitamin K absorption depends on adequate bile production in the liver. Impairment of vitamin K absorption diminishes the production of clotting factors and increases the risk of bleeding. The other structures mentioned are not associated with synthesizing of clotting factors and vitamin K. PTS: 1 REF: Page 1412 30. How many days does it take for the entire epithelial population of the small intestines to be replaced? a. 30 to 45 c. 7 to 15 b. 15 to 25 d. 4 to 7 ANS: D The entire epithelial population is replaced approximately every 4 to 7 days. PTS: 1 REF: Page 1401 31. Which statement is true regarding the state of the intestinal tract at birth? a. The intestinal tract is colonized by Escherichia coli. b. The intestinal tract is sterile. c. Clostridium welchii is present in the intestinal tract but in very small numbers. d. Streptococcus colonization in the intestinal tract has begun. ANS: B The intestinal tract is sterile at birtNhUbRuStINbeGcToBm.CeOs Mcolonized with Escherichia coli, Clostridium welchii, and Streptococcus within a few hours. PTS: 1 REF: Page 1409 MULTIPLE RESPONSE 32. Gastric emptying is delayed by the presence of which substances? (Select all that apply.) a. Solids b. Carbohydrates c. Nonisotonic solutions d. Bacteria e. Fats ANS: A, C, E Of the available options, only solids, fats, and nonisotonic solutions delay gastric emptying. PTS: 1 REF: Page 1397 33. Which statements are true regarding the ileogastric reflex? (Select all that apply.) a. The purpose of ileogastric reflex is to inhibit gastric motility. b. Ileum distension triggers the ileogastric reflex. c. The ileogastric reflex causes the relaxation of the ileocecal sphincter. d. Increased gastric secretion triggers the ileogastric reflex. e. The ileogastric reflex stimulates an increase in ileal motility. ANS: A, B The ileogastric reflex inhibits gastric motility when the ileum becomes distended. The gastroileal reflex, which is activated by an increase in gastric motility and secretion, stimulates an increase in ileal motility and the relaxation of the ileocecal sphincter. PTS: 1 REF: Page 1407 34. Which hormones are involved in the relaxation of the stomach’s fundus during swallowing? (Select all that apply.) a. Progesterone b. Glucagon c. Motilin d. Gastrin e. Cholecystokinin ANS: D, E Swallowing causes the fundus to relax (receptive relaxation) to receive a bolus of food from the esophagus. Relaxation is coordinated by efferent, nonadrenergic, noncholinergic vagal fibers and, of the available options, is facilitated only by gastrin and cholecystokinin, two polypeptide hormones secreted by the gastrointestinal mucosa. PTS: 1 REF: Page 1397 35. Which hormones are involved in regulating gastric motility by making the threshold potential of muscle fibers more neNgUatRivSIeN?G(TSeBl.eCcOtMall that apply.) a. Estrogen b. Secretin c. Somatostatin d. Gastrin e. Motilin ANS: B, D, E Gastrin and motilin (small intestine hormones) and the vagus nerve increase contraction by making the threshold potential of muscle fibers less negative. Sympathetic activity and secretin (another small intestine hormone) are inhibitory and make threshold potential more negative. The rate of peristalsis is mediated by pacemaker cells that initiate a wave of depolarization (basic electrical rhythm), which moves from the upper part of the stomach to the pylorus. Neither estrogen nor somatostatin inhibits gastric motility as described in the question. PTS: 1 REF: Page 1397 36. What requirements are necessary for calcium to be absorbed through the ileum at concentrations of less than 5 mmol/L? (Select all that apply.) a. Receptor site on the ileum b. Vitamin D3 c. Vitamin K d. Carboxypeptidase e. A carrier protein ANS: B, E At concentrations less than 5 mmol/L, calcium is transported actively across cell membranes, bound to a carrier protein. The carrier formation requires the presence of the active form of vitamin D3 (1,25-dihydroxyvitamin D). These selections are the only options relevant to the transport of calcium. PTS: 1 REF: Page 1405 37. Which water-soluble vitamins are dependent on sodium for absorption? (Select all that apply.) a. Pantothenic acid b. Vitamin B1 c. Vitamin C d. Vitamin B12 e. Folic acid ANS: C, E Of the options available, only vitamin C and folic acid are water-soluble vitamins that are dependent on sodium for absorption. PTS: 1 REF: Page 1406 | Table 40-2 38. Which statement is correct regarding the functions of the pancreas? (Select all that apply.) a. Cholecystokinin stimulates the release of pancreatic enzymes. b. Bilirubin and S cells inhibit the secretion of pancreatic enzymes. c. Pancreatic polypeptide is releaNsUedRSaIfNteGrTeBa.tCinOgM. d. Acetylcholine (ACh) is liberated from the pancreatic branches of the vagus nerve. e. ACh stimulates the release of pancreatic enzymes. ANS: A, C, D Enzymatic secretion follows, stimulated by cholecystokinin, which activates ACh from the vagus nerve and the release of ACh from pancreatic stellate cells. Cholecystokinin is released in the duodenum in response to the essential amino and fatty acids already present in chyme. Once in the small intestine, activated pancreatic enzymes inhibit the release of more cholecystokinin and ACh. This feedback mechanism inhibits the secretion of more pancreatic enzymes. Pancreatic polypeptide is released after eating and inhibits postprandial pancreatic exocrine secretion. (Table 40-1 summarizes the hormonal stimulation of pancreatic secretions.) PTS: 1 REF: Page 1415 MATCHING Match the hormone from the small intestine with its action. Hormones may be used more than once. A. Peptide YY B. Secretin C. Cholecystokinin D. Enteroglucagon E. Pancreatic polypeptide 39. Stimulates the gallbladder to eject bile and the pancreas to secrete alkaline fluid. 40. Stimulates the pancreas to secrete alkaline pancreatic juices. 41. Delays gastric and small bowel emptying. 42. Decreases pancreatic and enzyme secretions. 43. Enhances insulin release, lipolysis, and ketogenesis. 39. ANS: C PTS: 1 REF: Page 1397 | Table 40-1 MSC: fluid. Cholecystokinin stimulates the gallbladder to eject bile and the pancreas to secrete alkaline 40. ANS: MSC: B PTS: 1 REF: Page 1397 | Table 40-1 Secretin stimulates the pancreas to secrete alkaline pancreatic juice. 41. ANS: MSC: A PTS: 1 REF: Page 1397 | Table 40-1 Peptide YY delays gastric and small bowel emptying. 42. ANS: MSC: E PTS: 1 REF: Page 1397 | Table 40-1 Pancreatic polypeptide decreases pancreatic and enzyme secretions. 43. ANS: MSC: D PTS: 1 REF: Page 1397 | Table 40-1 Enteroglucagon enhances insulin release, lipolysis, and ketogenesis. NURSINGTB.COM Chapter 42: Alterations of Digestive Function MULTIPLE CHOICE 1. Where in the brain is the vomiting center located? a. Hypothalamus c. Pons b. Medulla oblongata d. Midbrain ANS: B The vomiting center of the brain lies in the medulla oblongata. The other locations listed are not related to vomiting. PTS: 1 REF: Page 1424 2. Antiemetic agents, such as domperidone and haloperidol, are antagonists for which receptors? a. 5-Hydroxytryptamine (5-HT) serotonin b. Histamine-2 c. Acetylcholine d. Dopamine ANS: D Metoclopramide, domperidone, and haloperidol are dopamine antagonists, making them effective antiemetic agents. This selection is the only option that identifies a receptor that is involved in the process of vomiting. PTS: 1 REF: Page 14N2U4RSINGTB.COM 3. What type of vomiting is caused by the direct stimulation of the vomiting center by neurologic lesions involving the brainstem? a. Retch c. Duodenal b. Periodic d. Projectile ANS: D Of the available options, only projectile vomiting is caused by the direct stimulation of the vomiting center by neurologic lesions, such as increased intracranial pressure, tumors, or aneurysms involving the brainstem. PTS: 1 REF: Page 1424 4. Considering the normal frequency of bowel evacuation, how infrequently can evacuation occur and still be considered within normal range? a. Once a day c. Once a week b. Once every 2 days d. Once every 2 weeks ANS: C Normal bowel habits range from two or three evacuations per day to one per week. PTS: 1 REF: Page 1428 5. How many stools per day are considered the upper limits of normal? a. Two c. Five b. Three d. Seven ANS: B More than three stools per day is considered abnormal. PTS: 1 REF: Page 1428 6. The adult intestine processes approximately how many liters of luminal content per day? a. 3 c. 9 b. 6 d. 12 ANS: C The adult intestine processes approximately 9 L of luminal content per day. Of this amount, 2 L is ingested and the remaining 7 L consists of intestinal secretions. PTS: 1 REF: Page 1425 7. A person who has cholera would be expected to have which type of diarrhea? a. Osmotic c. Small volume b. Secretory d. Motility ANS: B Primary causes of secretory diarrhea are bacterial enterotoxins, particularly those released by cholera or strains of Escherichia coli, and neoplasms, such as gastrinoma or thyroid carcinoma. None of the other options are associated with secretory diarrhea. PTS: 1 REF: Page 1425 NURSINGTB.COM 8. What type of diarrhea is a result of lactase deficiency? a. Motility c. Secretory b. Osmotic d. Small-volume ANS: B Malabsorption related to lactase deficiency, pancreatic enzyme or bile salt deficiency, small intestine bacterial overgrowth, and celiac disease cause osmotic diarrhea. None of the other options are associated with lactase deficiencies. PTS: 1 REF: Page 1425 9. Which statement is false concerning how abdominal pain is produced? a. Chemical mediators, such as histamine, bradykinin, and serotonin, produce abdominal pain. b. Edema and vascular congestion produce abdominal pain by stretching. c. Ischemia, caused by distention of bowel obstruction or mesenteric vessel thrombosis, produces abdominal pain. d. Low concentrations of anaerobes, such as Streptococci, Lactobacilli, Staphylococci, Enterobacteria, and Bacteroides, produce abdominal pain. ANS: D Low concentrations of anaerobes are not typically a cause of abdominal pain. PTS: 1 REF: Page 1426 10. How can abdominal pain that is visceral in nature best be described? a. Abdominal pain that is visceral in nature is diffused, vague, poorly localized, and dull. b. It travels from a specific organ to the spinal cord. c. The pain lateralizes from only one side of the nervous system. d. Abdominal pain is associated with the peristalsis of the gastrointestinal tract. ANS: A Pain is usually felt near the midline in the epigastrium (upper midabdomen), midabdomen, or lower abdomen. The pain is poorly localized, is dull rather than sharp, and is difficult to describe. None of the other options accurately describe this type of pain. PTS: 1 REF: Page 1426 11. What is the cause of gastroesophageal reflux disease? a. Excessive production of hydrochloric acid b. Zone of low pressure of the lower esophageal sphincter c. Presence of Helicobacter pylori in the esophagus d. Reverse muscular peristalsis of the esophagus ANS: B Normally, the resting tone of the lower esophageal sphincter maintains a zone of high pressure that prevents gastroesophageal reflux. In individuals who develop reflux esophagitis, this pressure tends to be lower than normal from either transient relaxation or a weakness of the sphincter. This selection is the only option that accurately describes the cause of gastroesophageal reflux disease. NURSINGTB.COM PTS: 1 REF: Page 1429 12. What term is used to identify frank bleeding of the rectum? a. Melena c. Occult bleeding b. Hematochezia d. Hematemesis ANS: B Hematochezia is the only available option that is associated with frank bright red or burgundy blood from the rectum. PTS: 1 REF: Page 1428 13. What is the cause of functional dysphagia? a. Intrinsic mechanical obstruction c. Tumor b. Extrinsic mechanical obstruction d. Neural or muscular disorders ANS: D Neural or muscular disorders that interfere with voluntary swallowing or peristalsis cause functional dysphagia. This selection is the only option that accurately identifies a cause of functional dysphagia. PTS: 1 REF: Page 1428 14. What is the cause of reflux esophagitis? a. Immune response to gastroesophageal reflux b. Delayed gastric emptying c. Congenital anomaly d. Secretory response to gastroesophageal reflux ANS: B Delayed gastric emptying contributes to reflux esophagitis by (1) lengthening the period during which reflux is possible and (2) increasing the acid content of chyme. None of the other options are accurate descriptions of the cause of reflux esophagitis. PTS: 1 REF: Page 1429 15. By what mechanism does intussusception cause an intestinal obstruction? a. Telescoping of part of the intestine into another section of intestine, usually causing strangulation of the blood supply b. Twisting the intestine on its mesenteric pedicle, causing occlusion of the blood supply c. Loss of peristaltic motor activity in the intestine, causing an adynamic ileus d. Forming fibrin and scar tissue that attach to the intestinal omentum, causing obstruction ANS: A Intussusception is the telescoping of part of the intestine into another section of intestine, usually causing strangulation of the blood supply. This selection is the only option that accurately describes how intussusception causes an intestinal obstruction. PTS: 1 REF: Page 1431 | Table 41-2 NURSINGTB.COM 16. What is the most immediate result of a small intestinal obstruction? a. Vomiting c. Electrolyte imbalances b. Dehydration d. Distention ANS: D Distention begins almost immediately, as gases and fluids accumulate proximal to the obstruction. Within 24 hours, up to 8 L of fluid and electrolytes enters the lumen in the form of saliva, gastric juice, bile, pancreatic juice, and intestinal secretions. Copious vomiting or sequestration of fluids in the intestinal lumen prevents their reabsorption and produces severe fluid and electrolyte disturbances. PTS: 1 REF: Pages 1431-1432 17. An intestinal obstruction at the pylorus or high in the small intestine causes metabolic alkalosis by causing which outcome? a. Gain of bicarbonate from pancreatic secretions that cannot be absorbed b. Excessive loss of hydrogen ions normally absorbed from gastric juices c. Excessive loss of potassium, promoting atony of the intestinal wall d. Loss of bile acid secretions that cannot be absorbed ANS: B If the obstruction is at the pylorus or high in the small intestine, then metabolic alkalosis initially develops as a result of excessive loss of hydrogen ions that normally would be reabsorbed from the gastric juices. This selection is the only option that accurately describes the cause of metabolic alkalosis in this situation. PTS: 1 REF: Page 1432 18. What are the cardinal symptoms of small intestinal obstruction? a. Constant, dull pain in the lower abdomen relieved by defecation b. Acute, intermittent pain 30 minutes to 2 hours after eating c. Colicky pain caused by distention, followed by vomiting d. Excruciating pain in the hypogastric area caused by ischemia ANS: C Of the options available, only colicky pain caused by distention followed by vomiting are considered the cardinal symptoms of a small intestinal obstruction. PTS: 1 REF: Pages 1432-1433 19. What is a cause of chronic antral gastritis? a. Helicobacter pylori bacteria b. Development of autoantibodies to gastric H+/K+ ATPase c. Pernicious anemia d. Reflux of bile and alkaline pancreatic secretions ANS: A Chronic antral gastritis generally involves only the antrum and is more common than fundal gastritis. It is caused by H.NpUylRoSrIiNbGaTcBte.CriOa Mor the chronic use of alcohol, tobacco, and nonsteroidal antiinflammatory drugs. None of the other options are associated with the cause of chronic antral gastritis. PTS: 1 REF: Page 1435 20. What is the primary cause of peptic ulcers? a. Hypersecretion of gastric acid c. Helicobacter pylori b. Hyposecretion of pepsin d. Escherichia coli ANS: C Infection with H. pylori is a primary cause of peptic ulcers. PTS: 1 REF: Page 1435 21. A peptic ulcer may occur in all of the following areas except the: a. Stomach c. Jejunum b. Duodenum d. Esophagus ANS: C A peptic ulcer is a break, or ulceration, in the protective mucosal lining of the lower esophagus, stomach, or duodenum. This type of ulcer is not associated with the jejunum. 22. Which statement is false regarding the contributing factors of duodenal ulcers? a. Bleeding from duodenal ulcers causes hematemesis or melena. b. Gastric emptying is slowed, causing greater exposure of the mucosa to acid. c. The characteristic pain begins 30 minutes to 2 hours after eating when the stomach is empty. d. Duodenal ulcers occur with greater frequency than other types of peptic ulcers. ANS: B Duodenal ulcers can be associated with altered mucosal defenses, rapid gastric emptying, elevated serum gastrin levels, or acid production stimulated by smoking. The other options provide correct information regarding duodenal ulcers. PTS: 1 REF: Page 1435 23. After a partial gastrectomy or pyloroplasty, clinical manifestations that include increased pulse, hypotension, weakness, pallor, sweating, and dizziness are the results of which mechanism? a. Anaphylactic reaction in which chemical mediators, such as histamine, prostaglandins, and leukotrienes, relax vascular smooth muscles, causing shock b. Postoperative hemorrhage during which a large volume of blood is lost, causing hypotension with compensatory tachycardia c. Concentrated bolus that moves from the stomach into the small intestine, causing hyperglycemia and resulting in polyuria and eventually hypovolemic shock d. Rapid gastric emptying and the creation of a high osmotic gradient in the small intestine, causing a sudden shift of fluid from the blood vessels to the intestinal lumen ANS: D NURSINGTB.COM Dumping syndrome occurs with varying severity in 5% to 10% of individuals who have undergone partial gastrectomy or pyloroplasty. Rapid gastric emptying and the creation of a high osmotic gradient in the small intestine cause a sudden shift of fluid from the vascular compartment to the intestinal lumen. Plasma volume decreases, causing vasomotor responses, such as increased pulse rate, hypotension, weakness, pallor, sweating, and dizziness. Rapid distention of the intestine produces a feeling of epigastric fullness, cramping, nausea, vomiting, and diarrhea. This selection is the only option that accurately identifies the mechanism responsible for the described situation. PTS: 1 REF: Page 1440 24. Which statement is consistent with dumping syndrome? a. Dumping syndrome usually responds well to dietary management. b. It occurs 1 to 2 hours after eating. c. Constipation is often a result of the dumping syndrome. d. It can result in alkaline reflux gastritis. ANS: A Most individuals with the dumping syndrome respond well to dietary management. None of the other options is associated with the dumping syndrome. PTS: 1 REF: Page 1440 25. What stimulates the desire to eat? a. Agouti-related protein (AgRP) b. Alpha-melanocyte–stimulating hormone (-MSH) c. Cocaine- and amphetamine-regulated transcript (CART) d. Peptide YY (PYY) ANS: A Specific neurons produce neuropeptide Y (NPY) and AgRP, which stimulates eating and decreases metabolism (anabolic). PTS: 1 REF: Page 1448 26. Which structure regulates eating behavior and energy metabolism? a. Anterior pituitary c. Posterior pituitary b. Hypothalamus d. Parietal lobe ANS: B The arcuate nucleus (ARC) in the hypothalamus has two sets of neurons with opposing effects that interact to regulate and balance food intake and energy metabolism. This selection is the only option that regulates eating behavior and energy metabolism. PTS: 1 REF: Pages 1447-1448 27. Which symptom is characteristic of bulimia nervosa? a. Recurrent episodes of binge eating with fears of not being able to stop eating. b. Fear of becoming obese, despite progressive weight loss. c. Perception that the body is fat when it is actually underweight. d. Absence of three consecutive NmUeRnSstIrNuGaTl Bpe.CriOoMds. ANS: A Diagnosis of bulimia is based on, among other findings, recurrent episodes of binge eating during which the individual fears not being able to stop. The remaining options are characteristic of anorexia nervosa. PTS: 1 REF: Page 1450 | Box 41-5 28. The most common clinical manifestation of portal hypertension is what type of bleeding? a. Rectal c. Esophageal b. Duodenal d. Intestinal ANS: C The vomiting of blood from bleeding esophageal varices is the most common clinical manifestation of portal hypertension. PTS: 1 REF: Page 1453 29. What is the most common manifestation of portal hypertension–induced splenomegaly? a. Leukopenia c. Erythrocytopenia b. Thrombocytopenia d. Pancytopenia ANS: B Thrombocytopenia (decreased platelet count) is the most common manifestation of congestive splenomegaly and can contribute to a tendency of increased bleeding. PTS: 1 REF: Page 1452 30. Which statement is false concerning the accumulation of fluid in the peritoneal cavity? a. Impaired excretion of sodium by the kidneys promotes water retention. b. Decreased oncotic pressure and increased hepatic sinusoidal hydrostatic pressure cause the movement of fluid into the peritoneal cavity. c. Decreased blood flow to the kidneys activates aldosterone, which retains sodium. d. Circulating nitric oxide causes vasoconstriction, which forces fluid from the capillaries into the peritoneal cavity. ANS: D The arterial vasodilation theory proposes that circulating nitric oxide or the release of endotoxin from translocation of intestinal bacteria triggers arterial vasodilation of the splanchnic organs early in the course of cirrhosis and stimulates renal sodium retention through the renin-angiotensin-aldosterone system, increased sympathetic tone, and changes in the intrarenal blood flow. The other options provide accurate information regarding the accumulation of fluid in the peritoneal cavity. PTS: 1 REF: Page 1453 31. Which statement is false regarding the sources of increased ammonia that contribute to hepatic encephalopathy? a. End products of intestinal protein digestion are sources of increased ammonia. b. Digested blood leaking from ruptured varices is a source of increased ammonia. c. Accumulation of short-chain fNatUtyRSaIcNidGsTtBh.aCtOiMs attached to ammonia is a source of increased ammonia. d. Ammonia-forming bacteria in the colon are sources of increased ammonia. ANS: C The accumulation of short-chain fatty acids, serotonin, tryptophan, and false neurotransmitters probably contributes to neural derangement and is not associated with ammonia levels. The other options provide accurate information regarding how the sources of ammonia contribute to hepatic encephalopathy. PTS: 1 REF: Pages 1454-1455 32. Hepatic fat accumulation is observed in which form of cirrhosis? a. Biliary c. Postnecrotic b. Metabolic d. Alcoholic ANS: D Alcoholic cirrhosis is a complex process that begins with fatty infiltration (hepatic steatosis). Fat deposition (deposition of triglycerides) within the liver hepatocytes is primarily caused by increased lipogenesis and decreased fatty acid oxidation by hepatocytes. This selection is the only option that accurately identifies the correct form of cirrhosis. PTS: 1 REF: Pages 1460-1461 33. Which statement is false concerning the pathophysiologic process of alcoholic cirrhosis? a. Inflammation and damage leading to cirrhosis begin in the bile canaliculi. b. Alcohol is transformed to acetaldehyde, which promotes liver fibrosis. c. Mitochondrial function is impaired, decreasing oxidation of fatty acids. d. Acetaldehyde inhibits export of proteins from the liver. ANS: A Biliary cirrhosis differs from alcoholic cirrhosis in that the damage and inflammation leading to cirrhosis begin in bile canaliculi and bile ducts, rather than in the hepatocytes. The other options provide true information regarding the pathophysiologic process of alcoholic cirrhosis. PTS: 1 REF: Pages 1460-1462 34. Which statement is false regarding the pathophysiologic process of acute pancreatitis? a. Bile duct or pancreatic duct obstruction blocks the outflow of pancreatic digestive enzymes. b. Acute pancreatitis can also result from direct cellular injury from drugs or viral infection. c. Acute pancreatitis is an autoimmune disease in which immunoglobulin G (IgG) coats the pancreatic acinar cells; consequently, the pancreatic enzymes destroy the cells. d. Acute pancreatitis is usually mild and spontaneously resolves. ANS: C The backup of pancreatic secretions and the activation and release of enzymes (activated trypsin activates chymotrypsin, lipase, and elastase) within the pancreatic acinar cells cause acute pancreatitis, an obstruNcUtivReSIdNiGseTaBs.eC.OTMhe activated enzymes cause autodigestion (e.g., proteolysis, lipolysis) of the pancreatic cells and tissues, resulting in inflammation. Acute pancreatitis is usually a mild disease and spontaneously resolves; however, approximately 20% of those with the disease develop a severe acute pancreatitis that requires hospitalization. Pancreatitis develops because of a blockage to the outflow of pancreatic digestive enzymes caused by bile duct or pancreatic duct obstruction (e.g., gallstones). Acute pancreatitis can also result from direct cellular injury from drugs or viral infection. PTS: 1 REF: Page 1464 35. The mutation of which gene is an early event associated with the pathogenetic origin of esophageal cancer? a. K-ras mutation c. myc b. TP53 d. HER2 ANS: B Mutation of the TP53 gene is an early event associated with esophageal cancer. This selection is the only mutation from among the provided options. PTS: 1 REF: Page 1466 36. Obesity is defined as a body mass index (BMI) greater than what measurement? a. 22 c. 28 b. 25 d. 30 ANS: D Obesity is an energy imbalance, with caloric intake exceeding energy expenditure, and is defined as a BMI greater than 30. PTS: 1 REF: Page 1446 MULTIPLE RESPONSE 37. Which statements are true regarding parietal pain? (Select all that apply.) a. Parietal pain arises from the parietal peritoneum. b. It is generally more localized than visceral pain. c. Parietal pain is usually less intense than visceral pain. d. Nerve fibers that travel to the spinal cord are involved in parietal pain. e. Parietal pain corresponds to dermatomes T6 and L1. ANS: A, B, D, E Parietal pain arises from the parietal peritoneum and is more localized and intense than visceral pain. Nerve fibers from the parietal peritoneum travel with peripheral nerves to the spinal cord, and the sensation of pain corresponds to skin dermatomes T6 and L1. PTS: 1 REF: Page 1426 38. Which statements are true regarding chronic gastritis? (Select all that apply.) a. Chronic gastritis tends to occur in older adults. b. It causes thinning and degeneration of the stomach wall. c. Chronic gastritis results in chrNonUiRcSiInNfGlaTmBm.CaOtMion and mucosal atrophy. d. Mucosal atrophy is a common outcome of chronic gastritis. e. Epithelial metaplasia is often observed with chronic gastritis. ANS: A, C, D, E Chronic gastritis tends to occur in older adults and causes chronic inflammation, mucosal atrophy, and epithelial metaplasia. Neither thinning nor degeneration of the stomach wall is associated with chronic gastritis. PTS: 1 REF: Page 1434 39. Which hormones are natural appetite suppressants? (Select all that apply.) a. Insulin b. Cortisol c. Galanin d. Calcitonin e. Serotonin ANS: A, D, E Insulin, calcitonin, and serotonin are natural appetite suppressants, whereas cortisol and galanin are natural appetite stimulants. PTS: 1 REF: Page 1448 | Box 41-4 40. Which are the early (prodromal) clinical manifestations of hepatitis? (Select all that apply.) a. Fatigue b. Vomiting c. Itching d. Splenomegaly e. Hyperalgia ANS: A, B, E The prodromal (preicteric) phase of hepatitis begins approximately 2 weeks after exposure and ends with the appearance of jaundice. Fatigue, anorexia, malaise, nausea, vomiting, headache, hyperalgia, cough, and low-grade fever are prodromal symptoms that precede the onset of jaundice. Itching and splenomegaly are not associated with the prodromal phase of hepatitis. PTS: 1 REF: Page 1459 41. Which clinical manifestations are consistent with cancer of the cecum and ascending colon? (Select all that apply.) a. Mahogany-colored blood mixed with stool b. Anemia c. Pain d. Constipation e. Palpable mass in the lower right quadrant ANS: A, B, C, E Clinical manifestations consistent with cancer of the cecum and ascending colon include pain, a palpable mass in the lowerNrUigRhStINquGaTdBr.aCnOt,Manemia, and dark red or mahogany-colored blood mixed with the stool. Constipation is not associated with this diagnosis. PTS: 1 REF: Page 1471 MATCHING Match the descriptions with the corresponding terms. A. Ulcerative colitis B. Crohn disease 42. Alterations in immunoglobulin G (IgA) production have been found in individuals with this disorder. 43. Inflammation develops in crypts of Lieberkühn in the large intestine. 42. ANS: B PTS: 1 REF: Page 1442 MSC: In Crohn disease, elevations in IgG are associated with the severity of the disease. 43. ANS: A PTS: 1 REF: Page 1441 MSC: Inflammation begins at the base of the crypts of Lieberkühn in the large intestine, primarily the left colon, with infiltration and release of inflammatory cytokines from neutrophils, lymphocytes, plasma cells, macrophages, eosinophils, and mast cells. Chapter 43: Alterations of Digestive Function in Children MULTIPLE CHOICE 1. Incomplete fusion of the nasomedial and intermaxillary process during the fourth week of embryonic development causes which condition in an infant? a. Cleft palate c. Cleft lip b. Sinus dysfunction d. Esophageal malformation ANS: C Of the available options, only a cleft lip is caused by the incomplete fusion of the nasomedial and intermaxillary process during the fourth week of embryonic development. PTS: 1 REF: Page 1486 2. Increased gastrin secretion by the mother in the last trimester of pregnancy may cause which condition in the infant? a. Pyloric stenosis c. Esophageal atresia b. Meconium ileus d. Galactosemia ANS: A The cause is unknown but increased gastrin secretion by the mother in the last trimester of pregnancy increases the likelihood of pyloric stenosis in the infant. The overproduction of gastric secretions in the infant may be caused by stress-related factors in the mother. This statement is not true of the other options. PTS: 1 REF: Pages 1N4U88R-S1I4N8G9TB.COM 3. At 2 or 3 weeks of age, an infant who has been well fed and has gained weight begins to vomit for no apparent reason. The vomiting gradually becomes more forceful. These symptoms may be indicative of which disorder? a. Esophageal atresia c. Pyloric stenosis b. Congenital aganglionic megacolon d. Galactosemia ANS: C Of the options available, only the clinical manifestations of pyloric stenosis—an infant who has been well fed and has gained weight begins to vomit without an apparent reason—can appear between 2 and 3 weeks after birth. The vomiting gradually becomes more forceful. PTS: 1 REF: Page 1489 4. Which term is used to identify a condition in which the developing colon remains in the upper right quadrant instead of moving to its normal location? a. Intestinal malrotation c. Duodenal obstruction b. Ileocecal displacement d. Pyloric stenosis ANS: A Intestinal malrotation is the only term used to identify a condition in which rotation does not occur and the colon remains in the upper right quadrant, where an abnormal membrane may press on and obstruct the duodenum. PTS: 1 REF: Page 1489 5. Which term is used to identify an intestinal obstruction caused by meconium formed in utero that is abnormally sticky and adheres firmly to the mucosa of the small intestine? a. Meconium cecum c. Meconium obstruction b. Meconium ileus d. Meconium vivax ANS: B Meconium ileus is the only term used to identify an intestinal obstruction caused by meconium formed in utero that is abnormally sticky and adheres firmly to the mucosa of the small intestine, resisting passage beyond the terminal ileum. The cause is usually a lack of digestive enzymes during fetal life. PTS: 1 REF: Page 1490 6. With which medical diagnosis is meconium ileus often associated? a. Muscular dystrophy c. Cystic fibrosis b. Cerebral palsy d. Congenital aganglionic megacolon ANS: C The detection of albumin in meconium has been used as a screening test for cystic fibrosis. This condition is not associated with any of the other options. PTS: 1 REF: Page 1490 7. Congenital aganglionic megacolon (Hirschsprung disease) involves inadequate motility of the colon caused by neural malforNmUaRtiSoInNGofTBw.hCiOcMh nervous system? a. Central c. Sympathetic b. Parasympathetic d. Somatic ANS: B A malformation related only to the parasympathetic nervous system causes congenital aganglionic megacolon. PTS: 1 REF: Page 1491 8. Which term is used to describe an intestinal obstruction caused by the invagination of the ileum into the cecum and part of the ascending colon by collapsing through the ileocecal valve? a. Congenital aganglionic megacolon c. Intussusception b. Malrotation d. Volvulus ANS: C Intussusception is the telescoping or invagination of one portion of the intestine into another section of intestine. Usually, the ileum invaginates the cecum and part of the ascending colon by collapsing through the ileocecal valve. The other terms are not used to describe this event. PTS: 1 REF: Page 1492 9. An infant suddenly develops abdominal pain, becomes irritable (colicky), and draws up the knees. Vomiting occurs soon afterward. The mother reports that the infant passed a normal stool, followed by one that looked like currant jelly. Based on these data, which disorder does the nurse suspect? a. Congenital aganglionic megacolon c. Malrotation b. Intussusception d. Volvulus ANS: B Based on these data, the nurse should suspect intussusception. A single normal stool may be passed, evacuating the colon distal to the apex of the intussusception. After passing a normal stool, 60% of infants will pass “currant jelly” stools, which appear dark and gelatinous because of their blood and mucus content. Intussusception is the only option that describes the symptoms listed. PTS: 1 REF: Page 1493 10. Cystic fibrosis is characterized by which symptom? a. Excessive mucus production c. Low sodium content in perspiration b. Elevated blood glucose levels d. Abnormally thin exocrine secretions ANS: A Excessive mucus production characterizes cystic fibrosis. However, the pathophysiologic triad that is the hallmark of cystic fibrosis includes (1) pancreatic enzyme deficiency, which causes maldigestion; (2) overproduction of mucus in the respiratory tract and an inability to clear secretions, which cause progressive chronic obstructive pulmonary disease; and (3) abnormally elevated sodium and chloride concentrations in sweat. Exocrine secretions tend to be abnormally thick and precipitate in the glandular ducts, obstructing flow. An elevated blooNdURglSuIcNoGsTeBl.eCvOeMl is not associated with this disorder. PTS: 1 REF: Page 1494 11. Which medication compensates for the deficiency that occurs as a result of cystic fibrosis? a. Salt tablets c. Antihypertensives b. Pancreatic enzymes d. Antibiotics ANS: B Pancreatic replacement enzymes are administered before or with meals, and high-calorie, high-protein diets with frequent snacks and vitamin supplements are used to treat the deficiency. These statements are not true of the other options. PTS: 1 REF: Page 1494 12. What causes a person with cystic fibrosis to experience an exocrine pancreatic insufficiency? a. Pancreatic ducts are obstructed with mucus. b. Impaired blood supply to the pancreas causes ischemia. c. A genetically impaired pancreas is unable to produce digestive enzymes. d. The pancreas has a volvulus at the ampulla of Vater. ANS: A Approximately 85% of individuals with cystic fibrosis experience pancreatic insufficiency. Obstruction of the pancreatic ducts with thick mucus blocks the flow of pancreatic enzymes and causes degenerative and fibrotic changes in the pancreas. This selection is the only option that accurately describes why an exocrine pancreatic insufficiency is observed in individuals diagnosed with cystic fibrosis. PTS: 1 REF: Page 1494 13. What is the cause of faulty digestion of fats in those diagnosed with cystic fibrosis? a. Bile ducts obstructed with mucus, prohibiting the release of bile b. Failure to metabolize fat-soluble vitamins c. Deficiency of pancreatic enzymes d. Fat malabsorption that now occurs in the jejunum ANS: C Severe problems with maldigestion of proteins, carbohydrates, and fats occur because of the insufficient secretion of pancreatic enzymes. This selection is the only option that accurately describes why fatty stools are observed in individuals diagnosed with cystic fibrosis. PTS: 1 REF: Page 1494 14. Which disorder is characterized by damage to the mucosa of the duodenum and jejunum and impaired secretion of secretin, cholecystokinin, and pancreatic enzymes? a. Wilson disease c. Gluten-sensitive enteropathy b. Cystic fibrosis d. Galactosemia ANS: C NURSINGTB.COM Gluten-sensitive enteropathy is characterized by damage to the mucosa of the duodenum and jejunum and has secondary effects that exacerbate malabsorption. The secretion of intestinal hormones, such as secretin and cholecystokinin, may be diminished. Because these chemical messengers are scarce, secretion of pancreatic enzymes and expulsion of bile from the gallbladder decrease. These statements are not true of the other options. PTS: 1 REF: Page 1495 15. What factor associated with gluten-sensitive enteropathy (celiac sprue) causes an infant to bruise and bleed easily? a. Vitamin K deficiency from fat malabsorption b. Bone marrow function depression c. Iron, folate, and B12 deficiency anemias d. Prescribed daily warfarin (Coumadin) ANS: A Deficiencies of fat-soluble vitamins (such as vitamin K) are common in children with gluten-sensitive enteropathy. Vitamin K malabsorption leads to hypoprothrombinemia, causing the child to bruise and bleed easily. This selection is the only option that accurately describes the mechanism that causes bruising and bleeding in children diagnosed with celiac sprue. 16. What distinguishes kwashiorkor from marasmus? a. All nutrients, proteins, fats, and carbohydrates are reduced in kwashiorkor. b. Physical growth of children is stunted in kwashiorkor but not in marasmus. c. Muscle wasting, diarrhea, low hemoglobin, and infection characterize kwashiorkor. d. Subcutaneous fat, hepatomegaly, and fatty liver are present in kwashiorkor. ANS: D The presence of subcutaneous fat, hepatomegaly, and fatty liver distinguishes kwashiorkor from marasmus. These manifestations are missing in marasmus because caloric intake is not sufficient to support fat synthesis and storage. None of the other options accurately describes the differences among these conditions. PTS: 1 REF: Page 1498 17. Why is prolonged diarrhea more severe in children than it is in adults? a. Less water is absorbed from the colon in children. b. Fluid reserves are smaller in children. c. Children have a higher fluid volume intake. d. Children have diarrhea more often than adults. ANS: B Infants have low fluid reserves and relatively rapid peristalsis and metabolism. Therefore the danger of dehydration is great. This selection is the only option that correctly identifies the reason prolonged diarrhea is more severe in children. PTS: 1 REF: Page 1500 NURSINGTB.COM 18. In an infant who is 5 weeks old, an increase in bilirubin production and persistent jaundice support which diagnosis? a. Pathologic hyperbilirubinemia c. Hepatitis A b. Physiologic jaundice d. Infantile cirrhosis ANS: A Physiologic jaundice develops during the second or third day after birth and usually subsides in 1 to 2 weeks in full-term infants and in 2 to 4 weeks in premature infants. After this development, increased bilirubin values and persistent jaundice indicate pathologic hyperbilirubinemia. This selection is the only option that accurately identifies the diagnosis associated with these symptoms and timeline. PTS: 1 REF: Pages 1500-1501 19. Which type of diarrhea results from lactose intolerance? a. Secretory c. Osmotic b. Motility d. Small volume ANS: C The malabsorption of lactose results in osmotic diarrhea, in which fluids move by osmosis from the vascular compartment into the intestinal lumen. Of the available options, this selection is the only type of diarrhea that results from lactose intolerance. PTS: 1 REF: Page 1500 20. Physiologic jaundice in a newborn is caused by: a. Reabsorption of bilirubin in the small intestine b. Impaired hepatic uptake and excretion of bilirubin c. Increased bilirubin production d. Mild conjugated (indirect-reacting) hyperbilirubinemia ANS: D Of the available options, physiologic jaundice in a newborn is caused by mild unconjugated (indirect-reacting) hyperbilirubinemia. PTS: 1 REF: Page 1500 21. In children, the risk factors for hepatitis B virus (HBV) are primarily associated with: a. Living in urban communities b. Mothers who are hepatitis C carriers c. Transfusion therapy for hemophilia d. Those of Hispanic ethnic background ANS: C Risk factors for HBV infection include infants of mothers who are carriers of chronic hepatitis B surface antigen (HBsAg), hemophiliacs who receive frequent blood transfusions, children who abuse parenteral drugs, and children who live in residences for those who are mentally delayed. HBV is endemic in China and other parts of Asia where most infections occur in infants and children as a result of maternal-neonatal transmission. PTS: 1 REF: Page 1502 NURSINGTB.COM 22. Cirrhosis causes intrahepatic portal hypertension in children as a result of which mechanism? a. Fibrosis that increases the resistance to blood flow within the portal system b. Increased pressure from the twisting of the common bile ducts c. Development of collateral circulation within the portal system d. Shunting of fluid to the spleen or abdomen ANS: A The two basic causes of portal hypertension in children are (1) increased resistance to blood flow within the portal system and (2) increased volume of portal blood flow. This selection is the only option that correctly identifies the cause of intrahepatic portal hypertension in children. PTS: 1 REF: Page 1503 23. What is the most common clinical sign of portal hypertension in children? a. Right heart failure c. Splenomegaly b. Pulmonary edema d. Diarrhea ANS: C Splenomegaly is the most common sign of portal hypertension in children. PTS: 1 REF: Page 1503 MULTIPLE RESPONSE 24. What factors can contribute to the development of a cleft lip and a cleft palate? (Select all that apply.) a. Maternal deficiency of B vitamins b. Exposure to heavy metals during the second trimester of pregnancy c. Maternal use of tobacco d. Maternal diabetes mellitus e. Genetic mutation of the transforming growth factor ANS: A, C, D In most cases, cleft lip and cleft palate are caused by multiple gene and environmental interactions, including maternal deficiency of B vitamins (B6, folic acid, and B12), maternal tobacco and alcohol use, maternal diabetes mellitus, and genetic variations of several biomolecules including transforming growth factor, interferon regulatory factor-6, fibroblast growth factor, and other growth factors. Exposure to heavy metals during the second trimester of pregnancy is not a known trigger for such development. PTS: 1 REF: Page 1486 25. Which situations have been associated with possible causes of the failure to thrive (FTT) in infants? (Select all that apply.) a. Gastroesophageal reflux b. Pyloric stenosis c. Intestinal parasites d. Adoption at birth e. Psychosocial isolation ANS: A, B, C, E NURSINGTB.COM FTT is a disorder having organic (e.g., gastrointestinal and endocrine disorders) and nonorganic (e.g., psychosocial) deprivation causes. Organic FTT has a pathophysiologic cause, such as gastroesophageal reflux, pyloric stenosis, gastroenteritis, malabsorption syndromes, infection by intestinal parasites, congenital anomalies, very low birth weight, or chronic diseases of major body systems. Nonorganic FTT occurs in the absence of any gastrointestinal, endocrine, or other chronic diseases. It is usually associated with psychosocial deprivation, although behavior problems may contribute to its occurrence in the absence of maternal pathologic findings. Adoption, in of itself, is not a cause. PTS: 1 REF: Page 1498 26. Which statements regarding Wilson disease in children are true? (Select all that apply.) a. Wilson disease is a rare autosomal recessive defect. b. Wilson disease affects copper metabolism. c. A lack of necessary copper is a result of Wilson disease. d. The liver is often affected in Wilson disease. e. Corneal damage can be a result of Wilson disease. ANS: A, C, D, E Wilson disease (hepatolenticular degeneration) is an autosomal recessive defect of copper metabolism that causes toxic amounts of copper to accumulate in the liver, brain, kidneys, and corneas. PTS: 1 REF: Page 1504 27. What are the classic symptoms associated with hepatitis A in children? (Select all that apply.) a. Nausea b. Vomiting c. Diarrhea d. Jaundice e. Muscle pain ANS: A, B, C Clinical manifestations associated with hepatitis A may include nausea, vomiting, and diarrhea. Neither jaundice nor muscle pain is associated with hepatitis A in children. PTS: 1 REF: Page 1502 28. Cystic fibrosis is directly responsible for complications to which structures? (Select all that apply.) a. Muscles b. Kidneys c. Lymph nodes d. Cervix e. Liver ANS: D, E Of the options available, only cervical inflammation and portal hypertension (liver) are complications directly related to cNyUstRicSIfNibGrToBsi.sC.OM PTS: 1 REF: Page 1495 | Table 42-1 29. Which foods should be eliminated from the diet for children who have gluten-sensitive enteropathy (celiac sprue)? (Select all that apply.) a. Citrus fruits b. Starchy vegetables c. Cereal grains d. Red meat e. Dairy ANS: C, E Treatment for celiac sprue consists of the immediate and permanent institution of a diet free of cereal grains (e.g., wheat, rye, barley, oats, malt). Lactose intolerance is presumed because of damage to the villi; therefore lactose (milk sugar) is excluded from the diet. The remaining options are not contraindicated on a celiac sprue–related diet. PTS: 1 REF: Page 1497 If needing more Test Banks, just let me know: [email protected] MATCHING Match the terms with the corresponding characteristics. A. Gluten sensitivity B. Periduodenal band C. Congenital aganglionic megacolon D. Protein energy malnutrition E. Lack of digestive enzymes during fetal life 30. Malrotation 31. Hirschsprung disease 32. Marasmus and kwashiorkor 33. Celiac sprue 34. Meconium ileus 30. ANS: B PTS: 1 REF: Page 1489 MSC: Intestinal malrotation is a condition in which rotation does not occur and the colon remains in the upper right quadrant, where an abnormal membrane may press and obstruct the duodenum. The obstructing band over the duodenum, called a periduodenal band, is one of the most significant findings in malrotation. 31. ANS: C PTS: 1 REF: Page 1491 MSC: Congenital aganglionic megacolon (Hirschsprung disease) is a functional obstruction of the colon caused by the absence of the enteric ganglia along a variable length of the colon with inadequate motility. 32. ANS: D PTS: 1 REF: Page 1497 MSC: Kwashiorkor and marasmus are the two most common types of malnutrition in children. These disorders are collectively known as protein energy malnutrition (PEM). 33. ANS: A PTS: 1 NURSINRGETFB:.CPOaMge 1494 MSC: Gluten-sensitive enteropathy, formerly called celiac sprue or celiac disease, is an autoimmune disease that damages the small intestinal villous epithelium when ingestion of gluten (gliadin), the protein component of cereal grains, occurs. 34. ANS: E PTS: 1 REF: Page 1490 MSC: Meconium ileus is intestinal obstruction caused by meconium formed in utero that is abnormally sticky and adheres firmly to the mucosa of the small intestine, resisting passage beyond the terminal ileum. The cause is usually a lack of digestive enzymes during fetal life. Chapter 44: Structure and Function of the Musculoskeletal System MULTIPLE CHOICE 1. Which bone cells produce osteocalcin when stimulated by 1,25-dihydroxyvitamin D and synthesize osteoid? a. Osteoclasts c. Fibrocytes b. Osteocytes d. Osteoblasts ANS: D Osteoblasts are cells derived from mesenchymal stem cells and produce several substances, including osteocalcin, transforming growth factor–beta (TGF-) (a growth inhibitor for many cells), macrophage colony–stimulating factor, receptor activator of nuclear factor kappa-B ligand, osteoprotegerin (OPG), and bone matrix when stimulated by 1,25-dihydroxyvitamin D. This statement is not true of the other options. PTS: 1 REF: Page 1514 2. What happens to the original bone during the second phase of bone remodeling? a. The original bone is replaced. c. The original bone is resorbed. b. It hardens. d. It is synthesized. ANS: C The distribution of these apoptotic osteocytes provides osteoclasts with information about where to begin resorbing damaged bone. In the second phase of remodeling (resorption), the osteoclasts form a cutting cone that gradually resorbs bone, leaving behind an elongated cavity termed a resorptiNoUnRcSaIvNiGtyT.BT.ChiOsMselection is the only option that accurately describes what happens when bone is resorbed. PTS: 1 REF: Pages 1519-1520 3. Which cells function to maintain bone matrix? a. Osteoclasts c. Osteoblasts b. Osteocytes d. Osteophytes ANS: B Osteocytes help synthesize and replace needed elements of the matrix by signaling osteoclasts and osteoblasts to resorb and form new bone. This selection is the only option that performs that function. PTS: 1 REF: Pages 1514-1515 4. Which bone cells are large and multinucleated and contain lysosomes filled with hydrolytic enzymes? a. Osteoblasts c. Osteocytes b. Osteoclasts d. Fibrocytes ANS: B Osteoclasts are the major resorptive cells of bone. They are large multinucleated cells with a short life span and contain lysosomes (digestive vacuoles) filled with hydrolytic enzymes. This selection is the only option that is described in this manner. PTS: 1 REF: Page 1515 5. Which bone cell secretes hydrochloric acid to help dissolve bone minerals and collagenase, thus aiding in the digestion of collagen? a. Osteocytes c. Osteoclasts b. Osteoblasts d. Osteophytes ANS: C Only osteoclasts bind to the bone surface of cell attachment proteins called integrins. They bring about resorption of bone by secreting hydrochloric acid and cathepsin K (a protease enzyme) that help dissolve bone minerals and collagenase, which aids in digesting collagen, along with the action of cytokines. PTS: 1 REF: Page 1515 6. Which glucoprotein is believed to inhibit calcium phosphate precipitation and play a part in bone resorption by recruiting osteoclasts? a. Osteocalcin c. Laminin b. Osteonectin d. Osteopontin ANS: A The roles of osteocalcin may be to inhibit calcium phosphate precipitation and play a part in bone resorption by recruiting osteoclasts. This statement is not true of any other option. PTS: 1 REF: Page 1516 7. Which glucoprotein is thought to NtrUanRsSpIoNrGt TeBss.CenOtMial elements such as hormones, ions, and other metabolites to and from the bone cells? a. Osteocalcin c. Laminin b. Osteonectin d. Bone albumin ANS: D Researchers believe bone albumin transports essential elements such as hormones, ions, and other metabolites to and from the bone cells and maintain the osmotic pressure of bone fluid (fluid surrounding mineral crystals and osteoblasts). This statement is not true of any other option. PTS: 1 REF: Page 1516 8. How is the work function of a muscle usually calculated? a. Muscle type c. Foot pounds b. Calculating force  distance d. Kilograms ANS: B The ultimate function of muscle is to accomplish work. Although expressed in such measures as foot-pounds or kilogram-meters, work refers to the amount of energy liberated or the amount of force exerted over a distance (work  force  distance). Muscle type is not relevant. PTS: 1 REF: Page 1533 9. What is the basic structural unit in compact bone? a. Small channels called canaliculi c. Tiny spaces within the lacunae b. Osteocytes within the lacunae d. Haversian system ANS: D The basic structural unit in compact bone is the haversian system (see Figure 43-4). This selection is the only option that accurately identifies the basic structure of compact bone. PTS: 1 REF: Page 1517 10. Which part of an injured joint becomes insensitive to pain and regenerates slowly and minimally? a. Synovium c. Bursa b. Articular cartilage d. Tendon ANS: B Articular cartilage has no blood vessels, lymph vessels, or nerves. Therefore it is insensitive to pain and regenerates slowly and minimally after injury. These statements are not true of the other options. PTS: 1 REF: Page 1525 11. The outer layer of the periosteum contains blood vessels and nerves that penetrate the inner structures of the bone by way of which structure? a. Volkmann canals c. Sharpey canals b. Canaliculi d. Trabeculae ANS: A NURSINGTB.COM All bones are covered with a double-layered connective tissue called the periosteum. The outer layer of the periosteum contains blood vessels and nerves, some of which penetrate to the inner structures of the bone only through channels called Volkmann canals (see Figure 43-4). PTS: 1 REF: Page 1517 12. What is the function of Sharpey fibers? a. To anchor the outer layer of the periosteum to the inner layer b. To contain blood vessels and nerves on the outer layer of the periosteum c. To help attach tendons and ligaments to the periosteum d. To attach muscles to the periosteum ANS: C Collagenous fibers (Sharpey fibers) that penetrate the bone anchor the inner layer of the periosteum to the bone. Sharpey fibers help hold or attach tendons and ligaments, not muscle, but to the periosteum of bones. PTS: 1 REF: Page 1517 13. After puberty, the epiphyseal plate calcifies, and the epiphysis merges with the . a. Epiphyseal line c. Metaphysis b. Epiphyseal plate d. Articular cartilage ANS: C After puberty, the epiphyseal plate calcifies and the epiphysis merges only with the metaphysis. PTS: 1 REF: Page 1518 14. The stage of healing in the bone that involves procallus formation entails which process? a. Formation of a hematoma that allows the development of a fibrin network b. Production of granulation tissue by fibroblasts, capillary buds, and osteoblasts c. Development of a primitive bone matrix termed woven bone d. Remodeling of the periosteal and endosteal bone surfaces ANS: B Fibroblasts, capillary buds, and osteoblasts move into the wound to produce granulation tissue called a procallus. None of the other options are associated with this process. PTS: 1 REF: Page 1520 15. Which term describes a freely movable joint? a. Synarthrosis c. Biarthrosis b. Amphiarthrosis d. Diarthrosis ANS: D Based on movement, a joint is classified as a synarthrosis (immovable joint), an amphiarthrosis (slightly movable joint), or a diarthrosis (freely movable joint). Biarthrosis is not a term used to describe a freely movable joint. PTS: 1 REF: Page 15N2U0RSINGTB.COM 16. Which type of joint holds teeth in the maxilla or mandible? a. Amphiarthrosis c. Synarthrosis b. Diarthrosis d. Gomphosis ANS: D A gomphosis is a special type of fibrous joint in which a conical projection fits into a complementary socket and is held in place by a ligament. Gomphosis is the only term used to identify the joint that holds teeth in the maxilla or mandible. PTS: 1 REF: Page 1520 17. The hyaline cartilage joints between the ribs and sternum are examples of which type of joint? a. Synchondrosis c. Gomphosis b. Symphysis d. Syndesmosis ANS: A A synchondrosis is the only type of joint in which hyaline cartilage, rather than fibrocartilage, connects the two bones. The joints between the ribs and the sternum are synchondroses and are not examples of any of the other types. PTS: 1 REF: Page 1520 18. The joint that contains a synovial membrane that lines the inner joint capsule is an example of which type of joint? a. Amphiarthrosis c. Synarthrosis b. Diarthrosis d. Biarthrosis ANS: B The joint that contains a synovial membrane that lines the inner joint capsule is a diarthrosis. A synovial joint consists of the following parts: (1) fibrous joint capsule (articular capsule), (2) synovial membrane that lines the inner surface of the joint capsule, (3) joint cavity (synovial cavity or space formed by the capsule), (4) synovial fluid, which fills the joint cavity and lubricates the joint surface, and (5) articular cartilage, which covers and pads the articulating bony surfaces. PTS: 1 REF: Page 1522 19. What is the function of the synovial membrane’s type A cells within the intima? a. To release mast cells, initiating the inflammatory process after joint injury b. To ingest and remove bacteria and debris by phagocytosis in the joint cavity c. To secrete hyaluronate, a binding agent that gives synovial fluid its viscous quality d. To store fat cells and glycogen, providing adenosine triphosphate for joint activity ANS: B The intima contains two types of synovial cells: A and B. Type A synovial cells ingest and remove bacteria and particles of debris by phagocytosis in the joint cavity. None of the remaining options accurately describes the function of the synovial membrane’s type A cells. PTS: 1 REF: Page 15N2U2RSINGTB.COM 20. Which statement does not present accurate information concerning synovial fluid? a. Synovial fluid contains protein polysaccharides to repair the synovial membrane after injury. b. It lubricates the joint surfaces. c. Synovial fluid nourishes the pad of the articular cartilage. d. It contains leukocytes to phagocytize joint debris and microorganisms. ANS: A Synovial fluid lubricates the joint surfaces, nourishes the pad of the articular cartilage that covers the ends of the bones, and contains free-floating synovial cells and various leukocytes that phagocytose joint debris and microorganisms. The other options provide accurate information about synovial fluid. PTS: 1 REF: Page 1522 21. What anchors articular cartilage to the underlying bone? a. Sharpey fibers c. Glycoproteins b. Collagen fibers d. Elastin fibers ANS: B Collagen fibers are important components of the cartilage matrix because they anchor the cartilage securely to underlying bone. This statement is not true of the other options. PTS: 1 REF: Pages 1524-1525 22. What controls the movement of synovial fluid through cartilage? a. Cartiloregulins c. Proteoglycans b. Hyaluronate d. Chondroitin ANS: C Only proteoglycans give articular cartilage its stiff quality and regulate the movement of synovial fluid through the cartilage. PTS: 1 REF: Page 1525 23. Which statement is false about muscles? a. Muscle comprises 50% of an adult’s body weight and 40% of a child’s body weight. b. Muscle is 75% water, 20% protein, and 5% organic and inorganic compounds. c. Muscle contains 32% of all protein stores for energy and metabolism. d. Muscles are encased in fascia. ANS: A Muscle constitutes 40% of an adult’s body weight and 50% of a child’s body weight. The other options are true statements regarding muscles. PTS: 1 REF: Pages 1526-1527 24. Which characteristic is true of type II (white fast-motor) muscle fibers? a. Slow contraction speed c. Profuse capillary supply b. Fast conduction velocities ANS: B NURSINGTdB..COOMxidative metabolism Type II fibers, also called white fast-motor fibers, are innervated by relatively large type II alpha motor neurons with fast conduction velocities. This selection is the only correct option provided. PTS: 1 REF: Page 1529 25. As the innervation ratio of a particular organ increases, what other property also increases? a. Control c. Coordination b. Movement d. Endurance ANS: D The greater the innervation ratio of a particular organ, the greater is its endurance. Low-innervation ratios promote control and coordination, whereas high ratios promote strength and endurance. An increase in the innervations ratio does not result in an increase in movement. PTS: 1 REF: Page 1528 26. Which statement describes a neuroregulin? a. Chemical mediator that initiates signals from the anterior horn cell of the spinal card to the axon of motor nerve branches of groups of muscle fibers b. Neurotransmitter that provides a means of reporting changes in length, tension, velocity, and tone in muscles c. Proteoglycan secreted by neurons, which increases acetylcholine receptors d. Mechanoreceptor that lies parallel to muscle fibers and responds to muscle stretching ANS: C Motor and sensory neurons secrete a proteoglycan called neuroregulin that increases acetylcholine receptors and helps in the formation of muscle spindle fibers. This selection is the only option that accurately describes a neuroregulin. PTS: 1 REF: Pages 1528-1529 27. Which four-step process correctly describes muscle contraction? a. Coupling, contraction, relaxation, excitation b. Contraction, relaxation, excitation, coupling c. Relaxation, excitation, coupling, contraction d. Excitation, coupling, contraction, relaxation ANS: D Muscle contraction is a four-step process: excitation, coupling, contraction, and relaxation. PTS: 1 REF: Page 1533 28. Which type of ion directly controls the contraction of muscles? a. Sodium c. Calcium b. Potassium d. Magnesium ANS: C NURSINGTB.COM Contraction begins as the calcium ions combine with troponin, a reaction that overcomes the inhibitory function of the troponin-tropomyosin system. This selection is the only option that has such a direct association with muscle contraction. PTS: 1 REF: Page 1533 29. In which type of contraction does the muscle maintain a constant tension as it moves? a. Isotonic c. Hypertonic b. Isometric d. Hypotonic ANS: A Only during an isotonic contraction does the muscle maintain a constant tension as it moves. PTS: 1 REF: Page 1535 30. Which term is used to identify a functional muscle contraction in which the muscle contracts but the limb does not move? a. Isotonic c. Eccentric b. Isometric d. Concentric ANS: B Only during an isometric contraction (static or holding contraction) does the muscle maintain a constant length as tension is increased. Isometric contraction occurs, for example, when the arm or leg is pushed against an immovable object. The muscle contracts, but the limb does not move. PTS: 1 REF: Page 1535 31. Which statement is false about aging and the musculoskeletal system? a. Haversian system erodes, the canals nearest the marrow cavity widened, and the endosteal cortex converts to spongy bone. b. The remodeling cycle increases because of a decreased ability of the basic multicellular units to resorb and deposit bone. c. Cartilaginous rigidity increases because of decreasing water content and decreasing concentrations of glycosaminoglycans. d. Muscle ribonucleic acid (RNA) synthesis declines, although the regenerative function of muscle tissue is reportedly normal in older adults. ANS: A Aging does not typically have an effect on the haversian system as described. The remaining options are accurate statements regarding the effects of aging. PTS: 1 REF: Page 1536 32. Which component is found in synovial fluid? a. Protein polysaccharides c. Leukocytes b. Water d. Chondrocytes ANS: C NURSINGTB.COM Synovial fluid lubricates the joint surfaces, nourishes the pad of the articular cartilage that covers the ends of the bones, and contains only free-floating synovial cells and various leukocytes that phagocytose joint debris and microorganisms. None of the remaining options are found in synovial fluid. PTS: 1 REF: Page 1522 33. What term is used to identify an interlacing bundle of dense, white fibrous tissue that is richly supplied with nerves, blood vessels, and lymphatic vessels? a. Procallus c. Hematoma b. Joint capsule d. Elastin fibers ANS: B The joint capsule is made up of parallel, interlacing bundles of dense, white fibrous tissue. It is richly supplied with nerves, blood vessels, and lymphatic vessels. None of the other options are associated with this structure. PTS: 1 REF: Page 1522 MULTIPLE RESPONSE 34. In adults, hematopoiesis takes place in which bone marrow cavities? (Select all that apply.) a. Skull b. Shoulders c. Sternum d. Long bones e. Pelvis ANS: A, B, C, E The marrow cavities within certain bones serve as sites of blood cell formation. In adults, blood cells exclusively originate in the marrow cavities of only the skull, vertebrae, ribs, sternum, shoulders, and pelvis. PTS: 1 REF: Page 1510 35. Which statements are true regarding osteocytes? (Select all that apply.) a. An osteocyte is a transformed osteoblast. b. An osteocyte obtains nutrients from capillaries in the canaliculi. c. The functioning of an osteocyte is well understood. d. Osteocytes signal osteoclasts and osteoblasts to form new bone. e. An osteocyte helps maintain levels of calcium and phosphorus in blood plasma. ANS: A, B, D, E An osteocyte is a transformed osteoblast. Osteocytes communicate with each other and help concentrate nutrients in the matrix. They obtain nutrients from capillaries in the canaliculi, which contain nutrient-rich fluids and also help synthesize and replace needed elements of the matrix by signaling osteoclasts and osteoblasts to resorb and form new bone. Through exchanges among these cells, hormone catalysts, minerals, and optimal levels of calcium, phosphorus, and other minerals are maintained in blood plasma. NURSINGTB.COM PTS: 1 REF: Pages 1514-1515 36. Which minerals are stored in bones? (Select all that apply.) a. Carbonate b. Phosphate c. Selenium d. Magnesium e. Calcium ANS: A, B, D, E Bones have a crucial role in mineral homeostasis and in storing and releasing minerals (e.g., calcium, phosphate, carbonate, magnesium) that are essential for the proper working of many delicate cellular mechanisms. Selenium is not stored in the bones. PTS: 1 REF: Page 1510 37. Which factors influence the rate of protein synthesis of skeletal muscles? (Select all that apply.) a. Insulin b. Cortisol c. Parathyroid hormone d. Growth hormone e. Amino acid substrates ANS: A, E The rate of protein synthesis is related to insulin levels amino acid substrates and to overall nutritional status. PTS: 1 REF: Page 1533 MATCHING Match the stages of wound healing of a bone with the description of each. Stages may be used more than once. A. Callus replacement B. Hematoma formation C. Remodeling D. Procallus formation E. Callus formation 38. Fibroblasts, capillary buds, and osteoblasts move into the wound to produce granulation tissue. 39. Periosteal and endosteal surfaces of the bone are formed to the size and shape of the bone before an injury. 40. Osteoblasts form membranous or woven bone. Enzymes increase the phosphate content and permit the phosphate to join with calcium to be deposited. 41. Fibrin and platelets form a meshwork within the initial framework for healing. 42. Osteoblasts continue to restore callus with lamellar bone or trabecular bone. wound to produce granulation MSC: Remodeling occurs as the periosteal and endosteal surfaces of the bone are remodeled to the size and shape of the bone before injury. 40. ANS: E PTS: 1 REF: Page 1520 MSC: Callus formation occurs as osteoblasts in the procallus form membranous or woven bone (callus). Enzymes increase the phosphate content and permit the phosphate to join with calcium to be deposited as mineral to harden the callus. 41. ANS: B PTS: 1 REF: Page 1520 MSC: Hematoma formation occurs if vessels have been damaged, causing hemorrhage. Fibrin and platelets within the hematoma form a meshwork that is the initial framework for healing with the help of hematopoietic growth factors such as platelet-derived growth factor and transforming growth factor-beta (TGF-b). 42. ANS: A PTS: 1 REF: Page 1520 MSC: Callus replacement occurs as osteoblasts continue to replace the callus with lamellar bone or trabecular bone. Chapter 45: Alterations of Musculoskeletal Function MULTIPLE CHOICE 1. What type of fracture occurs at a site of a preexisting bone abnormality and is a result of a force that would not normally cause a fracture? a. Idiopathic c. Pathologic b. Incomplete d. Greenstick ANS: C Only a pathologic fracture is a break at the site of a preexisting abnormality, usually by force that would not fracture a normal bone. PTS: 1 REF: Page 1541 2. Which type of fracture usually occurs in an individual who engages in a new activity that is strenuous and repetitive? a. Stress c. Insufficiency b. Greenstick d. Pathologic ANS: A Only a stress fracture occurs in normal or abnormal bone that is subjected to repeated stress, such as repetitive and strenuous activities that occur during athletics. PTS: 1 REF: Page 1541 3. Which term is used to identify theNtUemRSpIoNrGarTyBd.CisOpMlacement of two bones causing the bone surfaces to partially lose contact? a. Dislocation c. Malunion b. Subluxation d. Nonunion ANS: B Dislocation is the temporary displacement of a bone from its normal position in a joint. If the contact between the two surfaces is only partially lost, then the injury is referred to as a subluxation. This selection is the only option that identifies the temporary displacement of two bones, causing the bone surfaces to partially lose contact. PTS: 1 REF: Page 1544 4. Improper reduction or immobilization of a fractured femur can result in which outcome after cast removal? a. The muscles around the fracture site are weak. b. The fracture requires 6 to 8 weeks of physical therapy. c. The skin under the cast is dry and flaky. d. The bone is not straight. ANS: D Improper reduction or immobilization of a fractured bone may result in nonunion, delayed union, or malunion. Malunion is the healing of a bone in a nonanatomic position that could result in the bone not being straight. The other options are not outcomes of improper reduction or immobilization. PTS: 1 REF: Page 1543 5. Which structure attaches skeletal muscle to bone? a. Tendon c. Bursa b. Ligament d. Mesentery ANS: A A tendon is fibrous connective tissue that attaches skeletal muscle to bone. None of the other options are associated with this function. PTS: 1 REF: Page 1545 6. The tear in a ligament is referred to as a: a. Fracture c. Disunion b. Strain d. Sprain ANS: D Ligament tears are commonly known as sprains. None of the other options are associated with this damage. PTS: 1 REF: Page 1545 7. Which medical diagnosis is characterized by tissue degeneration or irritation of the extensor carpi radialis brevis tendon? a. Lateral epicondylitis c. Bursitis b. Medial tendinitis d. Lateral tendinitis ANS: A NURSINGTB.COM Only lateral epicondylopathy, commonly called tennis elbow, is the result of tissue degeneration or irritation of the extensor carpi radialis brevis tendon at its origin. PTS: 1 REF: Page 1546 8. The pain resulting from tendon and ligament injuries is usually described as: a. Dull and diffuse, persisting over the distribution of the tendon or ligament b. Sharp and localized, persisting over the distribution of the tendon or ligament c. Pins-and-needle sensations that occur distal to the injury with movement d. Intermittent and aching, occurring over the distribution of the tendon or ligament ANS: B The pain resulting from tendon and ligament injuries is usually described as being sharp and localized, persisting over the distribution of the tendon or ligament. This selection is the only option that accurately describes this type of pain. PTS: 1 REF: Page 1545 9. How is rhabdomyolysis characterized? a. Paralysis of skeletal muscles, resulting from an impaired nerve supply b. Smooth muscle degeneration, resulting from ischemia c. Lysis of skeletal muscle cells through the initiation of the complement cascade d. Release of myoglobin from damaged striated muscle cells ANS: D Rhabdomyolysis involves the release of myoglobin when muscle cells are damaged. This selection is the only accurate description of rhabdomyolysis. PTS: 1 REF: Pages 1547-1550 10. Which pathophysiologic alteration precedes crush syndrome after prolonged muscle compression? a. Muscle ischemia c. Volkmann contracture b. Myoglobinuria d. Neural injury ANS: B Myoglobinuria is an excess of myoglobin (an intracellular muscle protein) in the urine. Muscle cell damage releases the myoglobin. The most severe form is often called crush syndrome. Less severe and more localized forms of muscle damage are called compartment syndromes. This selection is the only option that accurately identifies the pathophysiologic alteration that precedes crush syndrome. PTS: 1 REF: Pages 1547-1549 11. By the time osteoporosis is visible on an x-ray examination, up to what percent of bone has been lost? a. 30% c. 50% b. 40% d. 60% ANS: A Generally, osteoporosis is radiographically detected as increased radiolucency of bone. By the time abnormalities are detecteNd UbRySxIN-rGayTBe.xCaOmMination, as much as 25% to 30% of bone tissue may have been lost. PTS: 1 REF: Page 1555 12. A bone density of 645 mg/cm2 would support which diagnosis? a. Osteoplasia c. Osteopenia b. Osteoporosis d. Osteomalacia ANS: B The World Health Organization (WHO) has defined osteoporosis on the basis of bone density. Normal bone is greater than 833 mg/cm2; osteopenia, or decreased bone mass, is 833 to 648 mg/cm2; osteoporosis is less than 648 mg/cm2. This selection is the only accepted option. PTS: 1 REF: Page 1550 13. Which type of osteoporosis would a person develop after having the left leg in a cast for 8 weeks to treat a compound displaced fracture of the tibia and fibula? a. Iatrogenic c. Idiopathic b. Regional d. Osteoblastic ANS: B Of the options available, only classic regional osteoporosis is associated with disuse or immobilization of a limb because of fractures, motor paralysis, or bone or joint inflammation. PTS: 1 REF: Page 1553 14. Considering the pathophysiologic process of osteoporosis, after being activated by receptor activator of nuclear factor B ligand (RANKL), receptor activator of nuclear factor B (RANK) activates which of the following? a. Osteoclast apoptosis c. Osteoprotegerin b. Osteoblast survival d. Osteoclast survival ANS: D RANKL activates the receptor RANK, which is expressed on osteoclasts and their precursors and suppresses apoptosis, which leads to activation and the prolongation of osteoclast survival. This statement is not true of any of the other options. PTS: 1 REF: Page 1553 15. Considering the pathophysiologic process of postmenopausal osteoporosis, which changes are believed to play a significant role in the development of age-related bone loss? a. Increased oxidative stress and increased intracellular reactive oxygen species b. Hypoparathyroidism c. Increased body weight d. Decreased formation and short life span of osteoclasts ANS: A Postmenopausal osteoporosis ocNcuUrRsSiInNmGTidBd.CleO-Maged and older women. It can occur because of estrogen deficiency, as well as estrogen-independent, age-related mechanisms (e.g., secondary causes such as hyperparathyroidism and decreased mechanical stimulation). Recent studies indicate that increased oxidative stress (OS) and increased intracellular reactive oxygen species (ROS) play significant roles in the development of age-related bone loss, as well as other age-related changes in the body. Hormonal deficiency also can increase with stress, excessive exercise, and low body weight. Increased formation and longevity of osteoclasts results in increased bone resorption and is associated with a cascade of proinflammatory cytokines. PTS: 1 REF: Page 1552 16. Considering the pathophysiologic process of osteoporosis, which hormone exerts antiapoptotic effects on osteoblasts but proapoptotic effects on osteoclasts? a. Parathyroid hormone c. Growth hormone b. Glucocorticoid d. Estrogen ANS: D Data reveal that sex steroids (e.g., estrogens) exert antiapoptotic effects on osteoblasts but exert proapoptotic effects on osteoclasts; in both scenarios, activating the extracellular signal-regulated kinases (ERKs) accomplish these effects. This process is not true of any of the other options. PTS: 1 REF: Page 1553 17. Considering the pathophysiologic process of osteoporosis, what are the effects of extracellular signal-regulated kinases (ERKs) and receptor activator of nuclear factor B ligand (RANKL) on osteoblasts and osteoclasts? a. ERKs increase the life span of osteoclasts, and RANKL decreases the life span of osteoblasts. b. ERKs and RANKL increase the life span of osteoclasts and decrease the life span of osteoblasts. c. ERKs and RANKL increase the life span of osteoblasts and decrease the life span of osteoclasts. d. ERKs increase the life span of osteoblasts, and RANKL decreases the life span of osteoclasts. ANS: B In addition to ERKs, RANKL is required for the antiapoptotic effect and thus longer life span of osteoclasts. This effect also shortens the life span of the bone-forming cells, or osteoblasts. This process is not true of any of the other options. PTS: 1 REF: Page 1553 18. What is the most common clinical manifestation of osteoporosis? a. Bone deformity c. Pathologic fracture b. Bone pain d. Muscle strain ANS: A The most common clinical manifestation of osteoporosis is bone deformity. PTS: 1 REF: Page 15N5U5RSINGTB.COM 19. Which disorder is characterized by the formation of abnormal new bone at an accelerated rate beginning with excessive resorption of spongy bone? a. Osteomalacia c. Osteoporosis b. Paget disease d. Osteosarcoma ANS: B Of the available options, only Paget disease (osteitis deformans) is a state of increased metabolic activity in bone characterized by abnormal and excessive bone remodeling, both resorption and formation. Chronic accelerated remodeling eventually enlarges and softens the affected bones. PTS: 1 REF: Pages 1557-1558 20. Which statement is false about factors that contribute to the difficulty in treating bone infections? a. Bone contains multiple microscopic channels that are impermeable to the cells and biochemicals of the body’s natural defenses. b. Microcirculation of bone is highly vulnerable to damage and destruction by bacterial toxins, leading to ischemic necrosis of bone. c. Bone cells have a limited capacity to replace bone destroyed by infections. d. Bacteria are walled off by macrophages and T lymphocytes; consequently, the antibiotics cannot penetrate the infected area. Bacteria are not walled off by macrophages and T lymphocytes, thus inhibiting the effects of antibiotics. The other options are true statements regarding factors that contribute to the difficulty in treating bone infections. PTS: 1 REF: Page 1560 21. Bone death as a result of osteomyelitis is due to what? a. Formation of immune complexes at the site of infection b. Localized ischemia c. Tumor necrosis factor–alpha (TNF-) and interleukin 1 (IL-1) d. Impaired nerve innervation at the site of infection ANS: B Vessel damage causes local thrombosis (blockage) of the small vessels, which leads to ischemic necrosis (death) of bone. This selection is the only option that is associated with bone death as a result of osteomyelitis. PTS: 1 REF: Page 1560 22. When considering osteomyelitis, sequestrum is identified as what? a. An area of devascularized and devitalized bone b. An enzyme that phagocytizes necrotic bone c. A subperiosteal abscess d. A layer of new bone surrounding the infected bone ANS: A Lifting of the periosteum disruptsNbUloRoSdINvGeTssBe.lCsOtMhat enter bone through the periosteum, which deprives the underlying bone of its blood supply. This deprivation leads to necrosis and death of the area of infected bone, producing sequestrum, an area of devitalized bone. None of the other available options accurately identify the term sequestrum. PTS: 1 REF: Page 1559 23. What pattern of bone destruction is described as not well-defined and not easily separated from normal bone? a. Moth-eaten c. Geographic b. Permeative d. Porous ANS: A Moth-eaten pattern is the only option that involves destruction that is not well-defined and not easily separated from normal bone. PTS: 1 REF: Pages 1562-1563 | Table 44-5 24. Which statement accurately describes a characteristic of osteosarcoma? a. Slow-growing tumor that begins in the bone marrow and infiltrates the trabeculae b. Solitary tumor that most often affects the metaphyseal region of the femur or tibia c. Aggressive tumor most often found in the bone marrow of long bones d. Tumor that infiltrates the trabeculae in spongy bone and implants in surrounding tissue by seeding ANS: C An osteosarcoma is a malignant bone-forming tumor. It is aggressive and most often found in bone marrow; it has a moth-eaten pattern of bone destruction. This selection is the only option that accurately describes a characteristic of osteosarcoma. PTS: 1 REF: Page 1563 25. Which statement is false concerning giant cell tumors? a. Giant cell tumors are an overexpression of genes including osteoprotegerin ligand (OPGL). b. The tumors are malignant, solitary, and irregularly shaped. c. Giant cell tumors are typically located in the epiphysis in the femur, tibia, radius, and humerus. d. They are slow-growing tumors that extend over the articular cartilage. ANS: B The giant cell tumor is generally a benign, solitary, circumscribed tumor that causes extensive bone resorption because of its osteoclastic origin. The other options are true statements concerning giant cell tumors. PTS: 1 REF: Pages 1564-1565 26. Which is a characteristic of inflammatory joint disease? a. Unilateral joint involvement b. Normal joint synovial fluid c. Absence of synovial membrane inflammation d. Systemic symptoms of inflammation NURSINGTB.COM ANS: D Inflammatory joint disease is characterized by systemic signs of inflammation (fever, leukocytosis, malaise, anorexia, hyperfibrinogenemia) and inflammatory damage or destruction in the synovial membrane or articular cartilage. This selection is the only option that accurately describes a characteristic of inflammatory joint disease. PTS: 1 REF: Page 1568 27. What is a primary defect in osteoarthritis? a. Stromelysin and acid metalloproteinase break down articular cartilage. b. Immunoglobulin G (IgG) destroys the synovial membrane. c. Synovial membranes become inflamed. d. Cartilage-coated osteophytes create bone spurs. ANS: A Of the options available, the primary defect in osteoarthritis is the loss of articular cartilage. PTS: 1 REF: Pages 1565-1566 28. In osteoarthritis, what is the effect of the disruption of the pumping action of proteoglycans? a. Pump malfunction stimulates the induction of nitric oxide synthase and nitric oxide, which degrades the cartilage. b. Cartilage is damaged by proteolytic enzymes because they cannot be pumped out of the joint. c. Cartilage becomes dry, brittle, and wears away because fluid cannot be pumped into the cartilage. d. Cartilage takes in too much fluid and is unable to withstand the stresses of weight bearing. ANS: D Changes in the conformation of proteoglycans disrupt the pumping action that regulates the movement of water and synovial fluid into and out of the cartilage. Without the regulatory action of the proteoglycan pump, cartilage imbibes too much fluid and becomes less able to withstand the stresses of weight bearing. This selection is the only option that accurately describes the disruption of the pumping action of proteoglycans when considering osteoarthritis. PTS: 1 REF: Page 1566 29. Which joint disease is characterized by joint stiffness on movement and joint pain of weight-bearing joints that is usually relieved by rest? a. Gouty arthritis c. Osteoarthritis b. Rheumatoid arthritis d. Suppurative arthritis ANS: C Pain and stiffness in one or more joints, usually weight-bearing or load-bearing joints, are the first symptoms of osteoarthritis. Use-related joint pain relieved by rest is a key feature. This selection is the only option that accurately identifies the disease associated with the described symptoms. NURSINGTB.COM PTS: 1 REF: Page 1567 30. Which medical diagnosis is described as a chronic inflammatory joint disease characterized by stiffening and fusion of the spine and sacroiliac joints? a. Ankylosing spondylitis c. Paget disease b. Rheumatoid arthritis d. Fibromyalgia ANS: A Of the options available, only ankylosing spondylitis (spondyloarthritis) is described as a chronic, inflammatory joint disease characterized by stiffening and fusion (ankylosis) of the spine and sacroiliac joints. PTS: 1 REF: Page 1572 31. What is the primary pathologic alteration resulting from ankylosing spondylitis (AS)? a. Inflammation of the sacroiliac joint b. Inflammation of the long bones c. Inflammation of fibrocartilaginous joints of the vertebrae d. Inflammation of the small hand and feet bones ANS: C AS involves inflammation of fibrocartilage in cartilaginous joints, primarily in the vertebrae. The other options do not accurately describe the primary pathologic alterations of AS. PTS: 1 REF: Pages 1573-1574 32. In ankylosing spondylitis, the CD8+ T cells are presented with which antigen? a. Synovium c. Tendons b. Cartilage d. Ligaments ANS: B Cartilage antigens are proposed as the targets for the immune response and the presentation of such antigens to CD8+ T cells. This statement is not true of any of the other options. PTS: 1 REF: Page 1573 33. People with gout are at high risk for which co-morbid condition? a. Renal calculi c. Anemia b. Joint trauma d. Hearing loss ANS: A Renal stones are 1000 times more prevalent in individuals with primary gout than they are in the general population. This statement is not true of any of the other options. PTS: 1 REF: Page 1578 34. What causes the crystallization within the synovial fluid of the joint affected by gouty arthritis? a. Reduced excretion of purines b. Overproduction of uric acid NURSINGTB.COM c. Increase in the glycosaminoglycan levels d. Overproduction of proteoglycans ANS: B When the uric acid reaches a certain concentration in fluids, it crystallizes, forming insoluble precipitates that are deposited in connective tissues throughout the body. Crystallization in synovial fluid causes acute, painful inflammation of the joint, a condition known as gouty arthritis. This selection is the only option that accurately identifies the cause of crystallization in synovial fluid associated with gouty arthritis. PTS: 1 REF: Pages 1574-1575 35. The pathophysiologic presentation of gout is closely linked to the metabolism of which chemical? a. Purine c. Vitamin E b. Pyrimidine d. Amino acid ANS: A The pathophysiologic presentation of gout is closely linked only to purine metabolism (or cellular metabolism of purines) and kidney function. PTS: 1 REF: Page 1575 36. Which clinical manifestations are associated with fibromyalgia? a. Hot, tender, and edematous muscle groups bilaterally b. Fasciculations of the upper and lower extremity muscles c. Exercise intolerance and painful muscle cramps d. Sensitivity at tender points and profound fatigue ANS: D Widespread joint and muscle pain, fatigue, and tender points are characteristics of fibromyalgia, a chronic musculoskeletal syndrome. Increased sensitivity to touch (i.e., tender points), the absence of systemic or localized inflammation, and fatigue and sleep disturbances are common. Fatigue is profound. The remaining options include symptoms not generally associated with fibromyalgia. PTS: 1 REF: Pages 1579-1580 37. At what age is peak bone mass and strength reached in women? a. 15 years c. 30 years b. 20 years d. 35 years ANS: C Bone formation continues at a pace faster than resorption until peak bone mass—or maximum bone density and strength—is reached at approximately 30 years of age, after which bone resorption slowly exceeds bone formation. PTS: 1 REF: Page 1550 38. What event is associated with the beginning of bone loss in women? a. Puberty c. Childbirth b. Sexual activity ANS: D NURSINGTdB..COMMenopause Bone loss in women is associated with menopause. Bone loss is most rapid in the first years after menopause but persists throughout the postmenopausal years. The other options are not relevant as triggers for bone loss. PTS: 1 REF: Page 1550 39. What term is used to identify the calcium crystals that are associated with chronic gout? a. Stones c. Tophi b. Spurs d. Nodes ANS: C With time, crystal deposition in subcutaneous tissues causes the formation of small white nodules, or tophi, that are visible through the skin. Crystal aggregates deposited in the kidneys can form urate renal stones and lead to renal failure. None of the other options are associated with the calcium crystals resulting from chronic gout. PTS: 1 REF: Page 1575 MULTIPLE RESPONSE 40. What are the primary sources of bacterial infections that lead to hematogenous bone infection? (Select all that apply.) a. Sinus b. Ear c. Dental d. Cutaneous e. Throat ANS: A, B, C, D Cutaneous, sinus, ear, and dental infections are all primary sources of bacteria in hematogenous bone infections. Throat infections are not generally associated with bone infections. PTS: 1 REF: Page 1559 41. Which structures are most often affected by Paget disease? (Select all that apply.) a. Vertebrae b. Skull c. Sternum d. Metacarpals e. Pelvis ANS: A, B, C, E Paget disease most often affects the axial skeleton, especially the vertebrae, skull, sacrum, sternum, and pelvis. The metacarpals are not associated with the axial skeleton or Paget disease. PTS: 1 REF: Page 1558 42. Which clinical manifestations areNcUhaRrSaIcNteGrTisBt.iCcOoMf rheumatoid arthritis? (Select all that apply.) a. Subcutaneous tissue crystals b. Anorexia c. Painful, stiffening of joints d. Edema of the wrists e. Fever ANS: B, C, E Rheumatoid arthritis begins with general systemic manifestations of inflammation, including fever, fatigue, weakness, anorexia, weight loss, and generalized aching and stiffness. Local manifestations also gradually appear over weeks or months. Typically, the joints become painful, tender, and stiff. Neither tissue crystals nor edema is associated with rheumatoid arthritis. PTS: 1 REF: Page 1571 MATCHING Match the phrases with the corresponding characteristics. A. Caused by sedatives and narcotics, particularly street heroin B. Caused by viruses, bacteria, and parasites C. Exercise intolerance with normal production of lactic acid D. Impairment of the breakdown of glycogen and production of lactic acid E. Autoimmune disease 43. McArdle disease 44. Myoadenylate deaminase deficiency 45. Rhabdomyolysis 46. Polymyositis 47. Myositis 43. ANS: D PTS: 1 REF: Page 1582 MSC: The individual with McArdle disease is not able to break down glycogen or produce lactic acid. 44. ANS: C PTS: 1 REF: Page 1583 MSC: Myoadenylate deaminase deficiency is an enzyme deficiency that produces changes in skeletal muscle and is associated with exercise intolerance. 45. ANS: A PTS: 1 REF: Page 1548 | Box 44-1 MSC: Sedatives and narcotics, particularly street heroin, clofibrate (a hypolipidemic agent), and the antifibrinolytic aminocaproic acid often cause rhabdomyolysis and myoglobinuria. 46. ANS: E PTS: 1 REF: Page 1584 MSC: Inflammation of connective tissue and muscle fibers that presumably causes the destruction of muscle fibers characterize polymyositis and dermatomyositis. The agent that causes the muscle inflammation has not been identified, but recent findings strongly suggest an autoimmune connection. 47. ANS: B PTS: 1 REF: Page 1583 MSC: Viral, bacterial, and parasitic infections of varying severity are known to produce inflammatory changes in skeletal muscle, a group of conditions collectively described by the term myositis. NURSINGTB.COM Chapter 46: Alterations of Musculoskeletal Function in Children MULTIPLE CHOICE 1. Until the skeleton matures and adult stature is reached, where does growth in the length of bone occur? a. Epiphyseal line c. Epiphyseal cartilage b. Physeal plate d. Metaphyseal plate ANS: B Until the skeleton matures and adult stature is reached, growth in the length of bone occurs only at the physeal plate through endochondral ossification. PTS: 1 REF: Page 1592 2. Which skeletal deformity is normal at birth but generally disappears by 21/2 years of age? a. Genu varum (bowleg) c. Equinovarus (clubfoot) b. Genu valgum (knock knee) d. Pes planus (flat feet) ANS: A Genu varum (bowleg) generally resolves itself by 21/2 years of age, whereas genu valgum (knock knee) maximizes by 5 to 6 years of age. This statement is not true of either equinovarus or pes planus. PTS: 1 REF: Page 1593 3. The total mass of muscle in the boNdUyRcSaInNGbeTBe.sCtiOmMated from which serum laboratory test value? a. Albumin c. Creatinine b. Blood urea nitrogen d. Creatine ANS: C Of the options available, the total mass of muscle in the body can be estimated from the amount of creatinine excreted in the urine, because the conversion of creatine to creatinine only takes place in muscle. PTS: 1 REF: Page 1593 4. What is the most common congenital skeletal defect of the upper extremity? a. Vestigial tabs c. Rickets b. Paget disease d. Syndactyly ANS: D The most common congenital skeletal defect of the upper extremity is syndactyly, or webbing of the fingers. PTS: 1 REF: Page 1593 5. What diagnosis is given when the infant’s hip maintains contact with the acetabulum but is not well seated within the hip joint? a. Dislocatable hip c. Dislocated hip b. Subluxated hip d. Subluxable hip ANS: B Subluxated hip is the only option used to identify the condition when the hip maintains contact with the acetabulum but is not well seated within the hip joint. PTS: 1 REF: Page 1594 6. Which sign or symptom is a very late indication of developmental dysplasia of the hip? a. Asymmetry of the gluteal or thigh folds b. Leg-length discrepancy c. Waddling gait d. Pain ANS: D Signs and symptoms of developmental dysplasia of the hip that should be noted include pain very late in the process. This statement is not true of the other options. PTS: 1 REF: Page 1595 7. Which serum laboratory test is elevated in all forms of osteogenesis imperfecta? a. Phosphorus c. Alkaline phosphatase b. Calcium d. Total protein ANS: C Of the available options, serum alkaline phosphatase is elevated in all forms of the disease. PTS: 1 REF: Page 1598 NURSINGTB.COM 8. The failure of bones to ossify, resulting in soft bones and skeletal deformity, characterizes which disorder? a. Osteogenesis imperfecta c. Osteochondrosis b. Rickets d. Legg-Calvé-Perthes disease ANS: B Of the available options, only rickets is a disorder in which growing bone fails to become mineralized (ossified) and results in soft bones and skeletal deformity. PTS: 1 REF: Page 1599 9. An insufficient dietary intake of which vitamin can lead to rickets in children? a. C c. B6 b. B12 d. D ANS: D Rickets results from either insufficient vitamin D, insensitivity to vitamin D, wasting of vitamin D by the kidney, or inability to absorb vitamin D and calcium in the gut. Vitamin D is the only vitamin associated with rickets. PTS: 1 REF: Pages 1599-1600 10. In scoliosis, curves in the thoracic spine greater than how many degrees result in decreased pulmonary function? a. 40 c. 60 b. 50 d. 80 ANS: D In scoliosis, curves in the thoracic spine greater than 80 degrees result in decreased pulmonary function. PTS: 1 REF: Page 1601 11. In osteomyelitis, bacteria gain access to the subperiosteal space in the metaphysis, which is considered the path of least resistance. What factor makes this route for bacteria the path of least resistance? a. Cortex of the bone in this area is porous or mazelike. b. Blood supply to the metaphysis is easily compromised. c. Macrophages and lymphocytes have limited access to the subperiosteal space. d. Bacteria usually spread down the medullary cavity of the bone. ANS: A The subperiosteal space in the metaphysis is the path of least resistance because the cortex of the bone in this area is porous or mazelike, and the inflammatory response blocks spread within the bone. This statement is not true of the other options. PTS: 1 REF: Page 1602 12. How do the clinical manifestations and onset of juvenile rheumatoid arthritis (JRA) differ from those of rheumatoid arthritis (RA) in adults? a. JRA begins insidiously with systemic signs of inflammation. b. JRA predominantly affects larNgUe RjoSiInNtGs.TB.COM c. JRA has more severe joint pain than adult RA. d. JRA has a rapid onset of generalized aches as the first symptom. ANS: B The onset of JRA is less gradual than it is in adult RA. JRA also differs from the adult form in that predominantly the large joints are affected in JRA. The remaining options are not accurate since the statements are true of RA as well. PTS: 1 REF: Page 1604 13. What is the cause of osteochondrosis? a. Imbalance between calcitonin and parathyroid hormone b. Nutritional deficiency of calcium and phosphorus c. Bacterial infection of the bone d. Vascular impairment and trauma to bone ANS: D Of the options available, only vascular impairment and trauma to bone, coupled with an underlying developmental or genetic predisposition, have been identified as probable causes of osteochondrosis. PTS: 1 REF: Page 1604 14. Which bones are affected in Legg-Calvé-Perthes disease? a. Heads of the femur c. Heads of the humerus b. Distal femurs d. Distal tibias ANS: A A recurrent interruption of the blood supply to only the femoral heads presumably produces Legg-Calvé-Perthes disease, which is a self-limited disease of the hip. PTS: 1 REF: Page 1604 15. Which statement is true regarding the pain experienced with Legg-Calvé-Perthes disease? a. Pain experienced with Legg-Calvé-Perthes disease in the elbows and upper and lower arms is aggravated by activity and relieved by rest. b. Pain experienced with Legg-Calvé-Perthes disease in the knees, inner thighs, and groin is described as a continuous ache and relieved by antiinflammatory drugs. c. Pain experienced with Legg-Calvé-Perthes disease in the knees, inner thighs, and groin is aggravated by activity and relieved by rest. d. Pain experienced with Legg-Calvé-Perthes disease in the elbows and upper and lower arms is described as a continuous ache and relieved by antiinflammatory drugs. ANS: C The child with Legg-Calvé-Perthes disease often complains of a limp or pain for several months. The pain is usually referred to the knee, inner thigh, and groin and aggravated by activity and relieved by rest. This selection is the only option that accurately describes the pain associated with Legg-Calvé-Perthes disease. PTS: 1 REF: Page 1605 NURSINGTB.COM 16. Which assessment finding characterizes Osgood-Schlatter disease? a. Lateral epicondylitis of the elbow b. Inflammation of the anterior cruciate ligament c. Bursitis of the subscapular bursa in the glenohumeral joint d. Tendinitis of the anterior patellar tendon ANS: D Tendinitis of the anterior patellar tendon, within which the patella (kneecap) is embedded, and associated osteochondrosis of the tubercle of the tibia are characteristics of Osgood-Schlatter disease. This selection is the only option that is associated with Osgood-Schlatter disease. PTS: 1 REF: Page 1605 17. At birth, the diagnosis of cerebral palsy (CP) may be made based on what factor? a. Brain trauma during birth c. Major brain malformation b. Prematurity d. Genetic defect ANS: A The diagnosis of CP is often made when gross motor milestones are not met by predicted ages. In some infants, diagnosis is made at birth because the child has an underlying diagnosis, such as a major brain malformation that is known to be associated with CP. None of the other options are known triggers for CP. PTS: 1 REF: Page 1606 18. Which muscular dystrophy syndrome develops from an X-linked recessive mode of inheritance? a. Duchenne c. Myotonic b. Facioscapulohumeral d. Limb girdle ANS: A A deletion of a segment of DNA or a single-gene defect on the short arm of the X chromosome is believed to be the cause of the X-linked inherited type of Duchenne muscular dystrophy. This statement is not true of the other options. PTS: 1 REF: Pages 1606-1607 | Table 45-4 19. Which protein, absent in muscle cells of Duchenne muscular dystrophy, mediates the anchoring of skeletal muscle fibers to the basement membrane? a. Syntrophin c. Dystrophin b. Laminin d. Troponin ANS: C Only dystrophin is present in normal muscle cells and absent in Duchenne muscular dystrophy. Dystrophin mediates anchorage of the actin cytoskeleton of skeletal muscle fibers to the basement membrane through a membrane glycoprotein complex. PTS: 1 REF: Page 1607 20. What pulmonary complication in children with Duchenne muscular dystrophy contributes to their death? NURSINGTB.COM a. Infection c. Impaired formation of alveoli b. Kyphoscoliosis d. Cancer ANS: A Although the life expectancy of boys with Duchenne continues to rise, death usually occurs from respiratory infection and a compromised respiratory system, with the majority living into their middle 20s. Some individuals who have chosen ventilatory support live a decade or more longer. This selection is the only option that is associated with the death and pulmonary complications experienced by children with this diagnosis. PTS: 1 REF: Pages 1607-1608 21. Generally, what is the first symptom of facioscapulohumeral muscular dystrophy? a. Inability to open the eyes completely c. Drooping shoulders b. Difficulty standing d. Facial pain ANS: C As the name implies, clinical manifestations begin with weakness and atrophy of facial and shoulder girdle (scapulohumeral) muscles. The illness progresses slowly. An inability to close the eyes completely may be noted in early childhood. The face is expressionless, and pouting of the lips makes whistling impossible. The first symptoms usually include drooping of the shoulders with difficulty in raising the arms above the head. The onset of weakness in the lower limbs is often delayed for 20 to 30 years, and pseudohypertrophy of muscles is rare. PTS: 1 REF: Page 1609 22. What is the most common malignant bone tumor diagnosed during childhood? a. Chondrosarcoma c. Ewing Sarcoma b. Fibrosarcoma d. Osteosarcoma ANS: D Of the options available, osteosarcoma is the most common bone tumor that occurs during childhood; it originates from bone-producing mesenchymal cells. PTS: 1 REF: Page 1610 23. Molecular analysis has demonstrated that osteosarcoma is associated with which gene? a. TP53 c. myc b. src d. TSC2 ANS: B Of the options available, only the oncogene src has been associated with osteosarcoma. PTS: 1 REF: Page 1610 24. Osteosarcoma usually metastasizes to the: a. Pancreas c. Lungs b. Liver d. Brain ANS: C Osteosarcoma disseminates through the bloodstream, usually to the lung. As many as 25% of children diagnosed with osteosaNrUcoRmSIaNGexThBi.bCiOt Mlung metastases at diagnosis. This statement is not true of any of the other options. PTS: 1 REF: Page 1610 25. A Ewing sarcoma arises from the: a. Bone marrow c. Metadiaphysis of long bones b. Bone-producing mesenchymal cells d. Embryonal osteocytes ANS: A Arising only from bone marrow, Ewing sarcoma can break through the cortex of the bone to form a soft-tissue mass. PTS: 1 REF: Page 1611 26. Rhabdomyosarcoma can develop in which type of muscle? a. Cardiac c. Involuntary b. Smooth d. Striated ANS: D Rhabdomyosarcoma can develop anywhere striated muscle is located. This statement is not true of the other options. PTS: 1 REF: Page 1612 27. Which organism is the primary cause of osteomyelitis in a newborn? a. Staphylococcus aureus c. Group B streptococcus b. Escherichia coli d. Bacillus anthracis ANS: A S. aureus is the primary cause of osteomyelitis in a newborn. Group B streptococcus and E. coli infections are responsible for some cases, especially those of multiple bone involvement and in high-risk infants. S. aureus is the responsible microorganism in 80% to 90% of osteomyelitis cases in older children. Bacillus anthracis (anthrax) is not associated with osteomyelitis. PTS: 1 REF: Page 1602 28. What is the cause of structural scoliosis? a. Poor posture c. Poor calcium absorption b. Vertebral rotation d. Intrauterine trauma ANS: B Structural scoliosis is the curvature of the spine associated with vertebral rotation. No research data suggest that any of the other options results in structural scoliosis. PTS: 1 REF: Page 1600 29. Which statements are true regarding bone growth before adulthood? (Select all that apply.) a. Growth in the length of bone occurs at the physeal plate. b. Growth occurs through endochondral ossification. c. Bone growth takes place under hormonal control. d. Cartilage cells at the epiphyseNalUsRidSeINoGfTtBhe.CpOhMyseal plate multiply and enlarge. e. Cartilage cells at the metaphyseal side of the plate are replaced by bone. ANS: A B, D, E Until adult stature is reached, growth in the length of bone occurs at the physeal plate through endochondral ossification. Cartilage cells at the epiphyseal side of the physeal plate multiply and enlarge. As rapidly as new cartilage cells form, cartilage cells at the metaphyseal side of the plate are destroyed and replaced by bone. Bone growth is not hormonally controlled. PTS: 1 REF: Page 1592 30. The anatomic makeup of which bone structures make them susceptible locations for osteomyelitis in children? (Select all that apply.) a. Hip joint b. Distal femur c. Proximal humerus d. Distal radius e. Lateral ankle ANS: A B, C, E Spread of infection to contiguous joints is related to the child’s age. Metaphyseal infection may spread to contiguous joints if the fibrous joint capsule includes the metaphysis and epiphysis. This special situation exists at the hip joint, distal femur, proximal humerus and radius, and lateral ankle. PTS: 1 REF: Page 1603 MATCHING Match the type of muscular dystrophy with its description. Types may be used more than once. A. Duchenne muscular dystrophy B. Facioscapulohumeral muscular dystrophy C. Myotonic dystrophy D. Becker muscular dystrophy 31. Onset occurs at approximately 3 years of age with rapid progression and frequently mental retardation. 32. The face is expressionless, and pouting of the lips makes whistling impossible. 33. Shares the X-linked inheritance pattern but produces milder symptoms. 34. Is an autosomal dominant condition with a slow rate of progression and frequently mental retardation. 31. ANS: A PTS: 1 REF: Pages 1606-1608 | Table 45-4 MSC: Duchenne muscular dystrophy is usually identified in children at approximately 3 years of age, when the parents first notice slow motor development with progressive weakness and muscle wasting. A moderate degree of mentaNlUreRtaSrIdNaGtiToBn.CcaOuMses these children to have a mean intelligence quotient (IQ) of approximately 85. 32. ANS: B PTS: 1 REF: Page 1609 MSC: Facioscapulohumeral muscular dystrophy exhibits a face that is expressionless, and pouting of the lips makes whistling impossible. 33. ANS: D PTS: 1 REF: Page 1608 MSC: Becker muscular dystrophy shares the X-linked inheritance pattern but produces milder clinical features. 34. ANS: C PTS: 1 REF: Page 1606 | Table 45-4 MSC: Myotonic dystrophy is a result of an autosomal dominant gene exhibiting a slow rate of progression and frequently mental retardation. Chapter 47: Structure, Function, and Disorders of the Integument MULTIPLE CHOICE 1. Which component of the epidermis initiates immune responses and provides a defense against environmental antigens? a. Langerhans cells c. Keratinocytes b. Merkel cells d. Melanocytes ANS: A Langerhans cells (a type of dendritic cell) and dermal dendritic cells initiate an immune response by presenting processed antigen to T cells, thus providing a defense against environmental antigens. This function is not true of the other options. PTS: 1 REF: Page 1617 2. Which cells of the dermis release histamine and play a role in the hypersensitivity reactions of the skin? a. Histiocytes c. Mast cells b. Fibroblasts d. Macrophages ANS: C Only mast cells release histamine and play a role in hypersensitivity reactions in the skin. PTS: 1 REF: Page 1618 3. Which cells of the dermis secrete NcoUnRnSeIcNtGivTeBt.iCsOsuMe matrix? a. Macrophages c. Fibroblasts b. Mast cells d. Histiocytes ANS: C Only fibroblasts secrete the connective tissue matrix and collagen. PTS: 1 REF: Page 1618 4. Which gland releases secretions that are important in thermoregulation and cooling of the body through evaporation? a. Sebaceous c. Eccrine b. Apocrine d. Exocrine ANS: C The eccrine sweat glands are distributed over the body, with the greatest numbers in the palms of the hands, soles of the feet, and forehead. These secretions are important in thermoregulation and cooling of the body through evaporation. This function is not true for the other options. PTS: 1 REF: Page 1618 5. Which receptors of the sympathetic nervous system regulate heat loss through the skin? a. Cholinergic c. -Adrenergic b. ß-Adrenergic d. Anticholinergic ANS: C Heat loss can be regulated by varying blood flow through the skin by opening and closing the arteriovenous anastomoses in conjunction with evaporative heat loss of sweat. The sympathetic nervous system regulates vasoconstriction and vasodilation through -adrenergic receptors only. PTS: 1 REF: Page 1619 6. What is the first change in the skin that indicates a pressure ulcer? a. Blanchable erythema of intact skin b. Nonblanchable erythema of intact skin c. Blister at the site of pressure d. Reddish-purple discoloration ANS: B Nonblanchable erythema of intact skin, usually over bony prominences, characterizes stage 1 pressure ulcers. This selection is the only option that accurately identifies the initial skin change indicative of a pressure ulcer. PTS: 1 REF: Page 1625 7. Which type of dressings should be applied to pressure ulcers? a. Flat and dry c. Bulky and dry b. Flat and moist d. Bulky and moist ANS: B Superficial pressure ulcers should be covered with flat, nonbulky dressings that cannot wrinkle or cause increased pressurNeUoRrSfIrNicGtTioBn.C. SOpMontaneous healing will occur more quickly if the ulcer is kept moist with an occlusive dressing. PTS: 1 REF: Page 1626 8. What term is used to identify skin lesions that are elevated, rounded, and firm with irregular clawlike margins that extend beyond the original site of injury? a. Psoriasis c. Acne b. Dermatitis d. Keloid ANS: D Of the options available, only keloids typically send out clawlike prolongations. PTS: 1 REF: Page 1626 9. Keloids are sharply elevated, irregularly shaped, progressively enlarging scars caused by excessive amounts of which substance in the corneum during connective tissue repair? a. Elastin c. Stroma b. Collagen d. Reticular fibers ANS: B Abnormal wound healing with excessive fibroblast activity and collagen formation during dermal connective tissue repair cause keloids. The other options are not associated with keloid formation. PTS: 1 REF: Pages 1626-1627 10. In allergic contact dermatitis, which cells possess the antigens and present them to T cells? a. Langerhans cells c. Keratinocytes b. Merkel cells d. Macrophages ANS: A In allergic contact dermatitis, only Langerhans cells process the antigen and carry it to T cells that then become sensitized to the antigen-releasing cytokines and chemokines, leading to leukocyte infiltration and inflammation. PTS: 1 REF: Pages 1627-1628 11. In latex allergies, which immunoglobulin (Ig) is associated with an immediate reaction? a. IgA c. IgG b. IgE d. IgM ANS: B Latex allergy can be either a type IV hypersensitivity to chemicals used in latex rubber processing or a type I immediate hypersensitivity, with IgE antibodies formed in response to latex rubber protein. This selection is the only correct option available. PTS: 1 REF: Page 1628 12. What medical term is used to identify an inflammatory disorder of the skin that is often considered synonymous with dermatitis and characterized by pruritus with lesions that have an indistinct border? a. Eczema NURSINGTcB..COAMtopic dermatitis b. Psoriasis d. Pityriasis rosea ANS: A The most common inflammatory disorder of the skin is eczema, or dermatitis. Eczema and dermatitis are general terms that describe a particular type of inflammatory response in the skin—these terms can be used interchangeably. Pruritus, lesions with indistinct borders, and epidermal changes generally characterize diseases that are considered eczematous. Of the available options, only eczema is associated with these characteristics. PTS: 1 REF: Page 1627 13. What clinical manifestations do allergic contact and stasis dermatitis have in common? a. Petechiae and hyperpigmentation c. Scaling and crusting of lesions b. Edema and vesicular lesions d. Erythema and pruritus ANS: D Only erythema and pruritus are exhibited with both conditions. PTS: 1 REF: Pages 1627-1628 14. Lesions on the elbows and knees that are well demarcated, thick, silvery, scaly, and erythematous characterize which type of psoriasis? a. Plaque c. Guttate b. Inverse d. Erythrodermic ANS: A The description provided is characteristic of only lesions observed in plaque psoriasis. These lesions are well-demarcated, thick, silvery, scaly, erythematous plaque surrounded by normal skin. PTS: 1 REF: Page 1629 15. Which skin disorder begins with a single lesion that is circular, demarcated, and salmon pink, measures approximately 3 to 4 cm in diameter, and is usually located on the trunk? a. Lichen planus c. Psoriasis b. Pityriasis rosea d. Acne rosacea ANS: B Of the options available, only pityriasis rosea begins as a single lesion known as a herald patch. This lesion is circular, demarcated, and salmon pink; it measures approximately 3 to 4 cm in diameter and is usually located on the trunk. PTS: 1 REF: Page 1630 16. Which immunoglobulin is found in skin biopsy with immunofluorescent observation of people with discoid lupus erythematosus? a. Immunoglobulin A (IgA) c. Immunoglobulin G (IgG) b. Immunoglobulin E (IgE) d. Immunoglobulin M (IgM) ANS: D Skin biopsy with immunofluorescent observation reveals lumpy deposits of immunoglobulins, especially IgM, in some individuals. This immunoglobulin is the only one associated with discoid lupus NeUryRthSIeNmGaTtoBs.CuOs.M PTS: 1 REF: Page 1632 17. Which term is used to identify a collection of infected hair follicles occurring most often on the back of the neck, the upper back, and the lateral thighs that forms a mass and evolves into an erythematous, painful, and edematous mass and drains through many openings? a. Erysipelas c. Carbuncles b. Furuncles d. Boils ANS: C The term, carbuncle, is used to identify a collection of infected hair follicles occurring most often on the back of the neck, the upper back, and the lateral thighs. The lesion begins in the subcutaneous tissue and lower dermis as a firm mass that evolves into an erythematous, painful, and swollen mass and drains through many openings. PTS: 1 REF: Page 1635 18. Chickenpox (varicella) may be followed years later by which disorder? a. Erysipelas c. Warts (Verrucae) b. Cytomegalovirus d. Herpes zoster ANS: D The same herpesvirus—varicella zoster virus (VZV)—causes both herpes zoster (shingles) and varicella (chickenpox). Varicella is the primary infection, followed years later by herpes zoster, particularly among those who are immunosuppressed. These statements are not true of any of the other options. PTS: 1 REF: Page 1636 19. What type of skin infection is tinea corporis (ringworm)? a. Nematode c. Viral b. Fungal d. Bacterial ANS: B Tinea corporis is a fungal infection of the skin. This selection is the only option that accurately identifies the nature of the infection. PTS: 1 REF: Page 1637 20. Cutaneous vasculitis develops from the deposit of . a. Immune complexes c. Complement proteins b. Immunoglobulin E (IgE) d. T lymphocytes ANS: A Cutaneous vasculitis involves the deposit of immune complexes, initiating an uncontrolled inflammatory response and causing the lesions that are often polymorphic. PTS: 1 REF: Page 1638 21. Which skin lesion is mediated by NthUeRiSmINmGuTnBo.gClOoMbulin E (IgE)–stimulated release of histamine, bradykinin, or kallikrein from mast cells? a. Dermatitis c. Urticaria b. Scleroderma d. Cutaneous vasculitis ANS: C Only urticaria is mediated by the IgE-stimulated release of histamine, bradykinin, or kallikrein from mast cells or basophils, or both, which causes the endothelial cells of skin blood vessels to contract. PTS: 1 REF: Page 1639 22. Scleroderma is associated with . a. X-linked recessive gene c. Virus b. X-linked dominant gene d. Autoantibodies ANS: D Scleroderma means sclerosis of the skin and is associated with immune dysregulation and several autoantibodies. This selection is the only option associated with scleroderma. PTS: 1 REF: Page 1639 23. What name is given to a benign proliferation of basal cells that produces smooth or warty elevated lesions on the chest, back, and face that are tan or waxy, yellow, flesh-colored, or dark brown to black? a. Basal keratosis c. Keratoacanthoma b. Seborrheic keratosis d. Actinic keratosis ANS: B Seborrheic keratosis is a benign proliferation of cutaneous basal cells that produces smooth or warty elevated lesions. The pathogenetic origin is unknown. Seborrheic keratosis usually occurs in older people as multiple lesions on the chest, back, and face. The color varies from tan to waxy, yellow, flesh-colored, or dark brown to black. No other option exhibits these characteristics. PTS: 1 REF: Page 1640 24. Lesions that usually have depressed centers with rolled borders and are frequently located on the face and neck characterize which malignancy? a. Squamous cell carcinoma c. Malignant melanoma b. Kaposi sarcoma d. Basal cell carcinoma ANS: D Of the available options, only basal cell carcinoma has depressed centers with rolled borders. Lesions are most often observed on people who live in regions with intense sunlight and on those areas of the skin most exposed—namely, the face and neck. PTS: 1 REF: Pages 1641-1642 25. The alteration of which gene is associated with basal cell carcinoma? a. myc c. src b. TP53 d. Ras NURSINGTB.COM ANS: B Basal cell carcinoma arises as a consequence of ultraviolet-associated mutation in only the TP53 tumor-suppressor gene leading to the loss of keratinocyte repair functions and apoptosis resistance of DNA-damaged cells. PTS: 1 REF: Page 1642 26. Bowen disease is a form of which type of cancer? a. Kaposi sarcoma c. Basal cell carcinoma b. Malignant melanoma d. Squamous cell carcinoma ANS: D Squamous cell carcinoma (SCC) is a tumor of the epidermis and the second most common human cancer. Two types are characterized: (1) in situ (Bowen disease) and (2) invasive. Bowen disease is not a form of any of the other options. PTS: 1 REF: Pages 1642-1643 27. A small (less than 1 cm) lesion with regular edges and bristlelike hairs with a color change that can range from skin tones to light brown characterize which form of nevus? a. Junctional c. Intradermal b. Epidermal d. Compound Only intradermal nevus is characterized by a small (less than 1 cm) lesion with regular edges and bristlelike hairs with a color change that can range from skin tones to light brown. PTS: 1 REF: Page 1643 | Table 46-9 28. Which cell is thought to be the progenitor cell of Kaposi sarcoma? a. Endothelial c. Melanocyte b. Keratinocyte d. Exothelial ANS: A Only the endothelial cell is thought to be the progenitor of Kaposi sarcoma, but the specific origin is elusive. PTS: 1 REF: Page 1645 29. Which intervention is included in the treatment of frostbite? a. Applying local, dry heat c. Immersing in warm water b. Rubbing or massaging the area d. Leaving the area uncovered ANS: C Immersion in a warm-water bath (40 to 42 C [104 to 107.6 F]) until frozen tissue is thawed is the best treatment for frostbite. Rubbing or massaging the area would damage the traumatized cells. The remaining options have no therapeutic value for frostbite. PTS: 1 REF: Page 1646 30. What is the purpose of administerNinUgRiSbIuNpGrToBfe.CnOtoMindividuals being treated for frostbite? a. Treating fever c. Reducing pain b. Preventing plate aggregation d. Inhibiting prostaglandins ANS: D Ibuprophen is used to inhibit prostaglandins. Although ibuprofen may be administered for the other reasons stated, those applications are not related to the treatment of frostbite. PTS: 1 REF: Page 1646 31. What is the medical term for a fungal infection of the nail plate? a. Paronychia c. Tinea corporis b. Onychomycosis d. Tinea capitis ANS: B Onychomycosis is the only term used to identify a fungal or dermatophyte infection of the nail plate that occurs in 2% to 18% of the population. PTS: 1 REF: Page 1647 32. What term is used to identify an inflamed hair root? a. Tinea c. Furuncle b. Verruca d. Erysipelas A furuncle, or “boil,” is used to identify an inflammation of the hair follicles (root) that may develop from a preceding folliculitis and spread through the follicular wall into the surrounding dermis. PTS: 1 REF: Page 1634 MULTIPLE RESPONSE 33. What effect does normal aging have on the wound healing process? (Select all that apply.) a. Decreased cell proliferation b. Decreased blood supply c. Depressed immune responses d. Depressed appetite e. Decreased cognitive function ANS: A, B, C Decreased cell proliferation, decreased blood supply, and depressed immune responses delay wound healing in aging skin. The other options are not related to wound healing progress. PTS: 1 REF: Page 1619 34. Which statements are true regarding a nevus? (Select all that apply.) a. A nevus is an aggregation of melanocytes b. Nevi begin to form at the ages of 3 to 5 years. c. Nevi are characterized by irregular borders. d. Nevi share a relationship withNmUeRlSaInNoGmTaB..COM e. Nevi always warrant being removed. ANS: A, B, D A nevus, or mole, is a benign pigmented or nonpigmented lesions that form from melanocytes beginning at ages 3 to 5 years. Over time the cells move down into the dermis and the nevi become nodular and symmetrical without irregular borders. Nevi may undergo transition to malignant melanomas. Nevi repeatedly traumatized, irritated by clothing, or large lesions can be excised. PTS: 1 REF: Page 1641 35. Which statements are true regarding Kaposi sarcoma? (Select all that apply.) a. South America has been the site of the endemic form of Kaposi sarcoma. b. Kaposi sarcoma is a vascular malignancy. c. It is associated with drug-induced immunosuppression. d. Kaposi sarcoma is associated with acquired immunodeficiency syndrome (AIDS). e. Presentation of the classic form of Kaposi sarcoma is observed on the lower legs of older men. ANS: B, C, D, E Kaposi sarcoma is a vascular malignancy with four different presentations: (1) it is associated with drug-induced immunosuppression; (2) its endemic form has been found in equatorial Africa, (3) the classic form appears on the lower legs of older men, and (4) it is associated with AIDS. PTS: 1 REF: Page 1645 36. Which medications may exacerbate existing psoriasis? (Select all that apply.) a. Antibiotics b. Calcium channel blockers c. Nonsteroidal antiinflammatory drugs (NSAIDs) d. Beta blockers e. Lithium ANS: C, D, E Antimalarial drugs, lithium, NSAIDs, and beta-blockers tend to exacerbate existing psoriasis. This statement is not true of the remaining options. PTS: 1 REF: Page 1629 37. Which inflammatory mediators are active in frostbite? (Select all that apply.) a. Leukotrienes b. Histamine c. Prostaglandins d. Bradykinin e. Thromboxanes ANS: B, C, D, E The inflammatory mediators of frostbite are similar to burns and include prostaglandins, thromboxanes, bradykinin, and histamine. Leukotrienes are not associated with frostbite. PTS: 1 REF: Page 16N4U6RSINGTB.COM 38. Women who develop hirsutism may be secreting hormones associated with which condition? (Select all that apply.) a. Adrenal hyperplasia b. Cushing disease c. Polycystic ovaries d. Addison disease e. Adrenal tumor ANS: A, C, E Women who develop hirsutism may be secreting hormones associated with ovarian or adrenal disease. Such women should be evaluated for polycystic ovaries, adrenal hyperplasia, or adrenal tumors. Neither Cushing nor Addison diseases are associated with hirsutism. PTS: 1 REF: Page 1647 MATCHING Match the descriptions of the skin lesions with the disorder where it is found. A. Elevated, firm circumscribed area less than 1 cm in diameter B. Elevated, firm, and rough lesion with a flat top surface greater than 1 cm in diameter C. Flat circumscribed area that is less than 1 cm in diameter D. Elevated irregular-shaped area of cutaneous edema; solid and transient with a variable diameter E. Elevated circumscribed, superficial lesion filled with serous fluid, less than 1 cm in diameter 39. Urticaria, allergic reaction 40. Varicella (chickenpox), herpes zoster (shingles) 41. Wart (verruca) or lichen planus 42. Psoriasis or seborrheic and actinic keratoses 43. Nevus (flat mole) 39. ANS: D PTS: 1 REF: Page 1621 | Table 46-3 MSC: Urticaria exhibits elevated, irregular-shaped areas of cutaneous edema. Its lesions are solid and transient with a variable diameter. 40. ANS: E PTS: 1 REF: Page 1621 | Table 46-3 MSC: Varicella (chickenpox) and herpes zoster (shingles) are elevated circumscribed, superficial lesions filled with serous fluid and measure less than 1 cm in diameter. 41. ANS: A PTS: 1 REF: Page 1620 | Table 46-3 MSC: Warts (verruca) or lichen planus are elevated, firm, circumscribed areas less than 1 cm in diameter. 42. ANS: B PTS: 1 REF: Page 1620 | Table 46-3 MSC: Psoriasis or seborrheic and actinic keratoses are elevated, firm, and rough lesions with flat-top surfaces that measure greater than 1 cm in diameter. 43. ANS: C PTS: 1 REF: Page 1620 | Table 46-3 MSC: A nevus is a flat, circumscribed area that is less than 1 cm in diameter. NURSINGTB.COM Chapter 48: Alterations of the Integument in Children MULTIPLE CHOICE 1. Which immunoglobulin (Ig) is elevated in atopic dermatitis? a. IgA c. IgE b. IgM d. IgG ANS: C In the acute phase of atopic dermatitis, inflammation is associated with the activation of T helper 1 (Th-1) cells with an overexpression of cytokines (interleukin [IL]–4, IL-5, and IL-13) and chemokine ligand 1 (CCL1) and 18 (CCL18) with increases in IgE, eosinophils, and macrophages. The remaining options will not be elevated as related to atopic dermatitis. PTS: 1 REF: Page 1654 2. Which leukocytes are elevated in atopic dermatitis? a. Eosinophils c. Basophils b. Neutrophils d. Monocytes ANS: A In the acute phase of atopic dermatitis, inflammation is associated with the activation of T helper 1 (Th-1) cells with an overexpression of cytokines (interleukin [IL]–4, IL-5, and IL13) and chemokine ligand 1 (CCL1) and 18 (CCL 18) with increases in IgE, eosinophils, and macrophages. Other leukocytes will not be elevated as related to atopic dermatitis. NURSINGTB.COM PTS: 1 REF: Page 1654 3. Which clinical manifestation is considered the hallmark of atopic dermatitis? a. Papular rash c. Vesicles that burst and form crusts b. High fever d. Itching ANS: D Of the options, only itching is considered a hallmark of atopic dermatitis, and rubbing and scratching to relieve the itch are responsible for many of the clinical changes of atopic dermatitis. PTS: 1 REF: Pages 1654-1655 4. Frequently, diaper dermatitis is secondarily infected with which organism? a. Escherichia coli c. Proteus spp. b. Candida albicans d. Staphylococcus aureus ANS: B Of the options available, frequently the infant with diaper dermatitis is secondarily infected with C. albicans. PTS: 1 REF: Page 1655 5. The disruption in cellular adhesion observed in bullous impetigo is caused by an exfoliative toxin related to which organism? a. Staphylococcus aureus c. Escherichia coli b. Streptococcus pyogenes d. Candida albicans ANS: A Bullous impetigo is a rare variant of impetigo caused by S. aureus. The staphylococci produce a bacterial toxin called exfoliative toxin (ET), which causes a disruption in desmosomal adhesion molecules with blister formation characteristic of bullous impetigo. This selection is the only option that identifies the correct organism. PTS: 1 REF: Page 1656 6. The staff member of a crowed day care center is a source for which bacterial infection? a. Atopic dermatitis b. Staphylococcal scalded-skin syndrome c. Impetigo d. Tinea capitis ANS: C Impetigo is a common bacterial skin infection in infants and children and is particularly infectious among people living in crowded conditions with poor sanitary facilities or in settings such as day care facilities. The other options are not as infectious as impetigo. PTS: 1 REF: Page 1656 7. Skin lesions that rupture, creating a thin, flat, honey-colored crust, are the hallmark clinical manifestation of which skin disordNeUrR? SINGTB.COM a. Rubella c. Atopic dermatitis b. Tinea capitis d. Bullous impetigo ANS: D Of the options available, only the vesicular impetigo lesions that begin as small vesicles with a honey-colored serum are the hallmark clinical manifestation of bullous impetigo. Yellow to white-brown crusts form as the vesicles rupture and extend radially (see Figure 47-4). PTS: 1 REF: Page 1656 8. Skin lesions caused by tinea corporis are best described as: a. Pink-to-red coalescing maculopapular rash on the scalp or trunk b. Vesicles that rupture, creating a thin, flat, honey-colored crust c. Circular (round or oval) lesions with erythema and scaling patches d. Red papules, vesicles, and pustules in clusters ANS: C Lesions caused by tinea corporis are often erythematous, round, or oval-scaling patches that peripherally spread with clearing in the center, creating the ring appearance, which is why this disease is commonly referred to as ringworm. None of the other options accurately describe the lesions associated with tinea corporis. PTS: 1 REF: Pages 1657-1658 9. What is a common contact source of tinea corporis? a. Mites c. Ponies b. Kittens d. Ticks ANS: B Contact with only kittens and puppies is considered a common source of tinea corporis. PTS: 1 REF: Page 1657 10. What is the cause of molluscum contagiosum? a. Bacteria c. Fungi b. Virus d. Rickettsia ANS: B The poxvirus is the only organism that induces epidermal cell proliferation and blocks immune responses that would control the virus as noted with molluscum contagiosum. PTS: 1 REF: Page 1658 11. Thrush is a superficial infection that commonly occurs in children and is caused by which organism? a. Staphylococcus c. Herpesvirus b. Streptococcus d. Candida albicans ANS: D C. albicans infection is a superficial fungal infection that commonly occurs in children. C. albicans is part of the normal skin flora in certain individuals and invades susceptible tissue sites if the predisposing facNtoUrsRaSrINe GnToBt e.CliOmMinated. This description is not accurate for any of the other options. PTS: 1 REF: Page 1658 12. How is thrush best defined? a. Presence of Candida in the mucous membranes of the mouths of infants b. Presence of bacteria in the nasal mucous membranes of infants c. Any viral infection of the mucous membranes of the mouths of infants d. Acute immune response to oral medication, located in the mucosal lining of the mouths of infants ANS: A Thrush is the term used to describe the presence of Candida in the mucous membranes of the mouth of infants and, less commonly, in the mouth of adults. This selection is the only option that accurately describes thrush. PTS: 1 REF: Page 1658 13. Which viral disease has an incubation period of 14 to 21 days and duration of 1 to 4 days? a. Rubeola c. Roseola b. Rubella d. Varicella ANS: B Only rubella has an incubation period that ranges from 14 to 21 days and a duration of 1 to 4 days. PTS: 1 REF: Page 1658 14. Rubella (German or 3-day measles) is a common communicable disease of children caused by what type of organism? a. Virus c. Fungus b. Bacterium d. Yeast ANS: A Rubella is a common communicable disease in children and young adults and is caused only by a ribonucleic acid (RNA) virus that enters the bloodstream through the respiratory route. PTS: 1 REF: Page 1658 15. Rubeola is a highly contagious acute disease in children caused by which type of infection? a. Bacterial c. Yeast b. Fungal d. Viral ANS: D Rubeola, a highly contagious acute disease in children, is a result of only a viral infection. PTS: 1 REF: Page 1659 16. Which clinical manifestation is prNesUeRnSt IiNnGrTuBbe.CoOlaMbut not in rubella? a. Conjunctivitis c. Presence of a cough b. Enlarged lymph nodes d. Runny nose ANS: A Prodromal symptoms of rubeola include high body temperature (up to 40.5 C [104.9 F]), malaise, enlarged lymph nodes, runny nose, conjunctivitis, and a “barking” cough. Prodromal symptoms of rubella are few but may include enlarged cervical and postauricular lymph nodes, low-grade fever, headache, sore throat, runny nose, and cough. PTS: 1 REF: Pages 1658-1660 17. What is the cause of chickenpox? a. Poxvirus c. Adenovirus b. Varicella-zoster virus d. Human papillomavirus ANS: B Only a varicella-zoster viral infection produces chickenpox (varicella) and herpes zoster (shingles). PTS: 1 REF: Page 1660 18. Which contagious disease has clinical manifestations of burrows several millimeters to 1 cm long, papules, and vesicular lesions with severe itching that worsens at night? a. Pediculosis c. Scabies b. Tinea capitis d. Rubeola ANS: C Only scabies has symptoms that appear 3 to 5 weeks after infestation. The burrows, papules, and vesicular lesions cause severe itching that worsens at night. PTS: 1 REF: Page 1661 19. Which contagious disease creates a primary skin lesion that is a pinpointed macule, papule, or wheal with hemorrhagic puncture site? a. Pediculosis c. Scabies b. Tinea capitis d. Rubeola ANS: A Of the options available, only the primary lesion of the body louse (pediculosis) exhibits a pinpoint red macule, papule, or wheal with a hemorrhagic puncture site. PTS: 1 REF: Page 1662 20. Which contagious disease is caused by the itch mite? a. Miliaria c. Pediculosis b. Tinea corporis d. Scabies ANS: D Scabies is the only contagious disease caused by the itch mite, Sarcoptes scabiei. PTS: 1 REF: Page 1661 21. Which vascular anomaly is associNatUeRdSwINitGhTaBc.CoOnMgenital malformation of dermal capillaries that does not fade with age? a. Cutaneous hemangioma c. Strawberry hemangioma b. Port-wine (nevus flammeus) stain d. Cavernous hemangioma ANS: B A port-wine stain is the only option that is present at birth or within a few days after birth and does not fade with age. PTS: 1 REF: Pages 1663-1664 22. Which skin disorder is characterized by a vesicular eruption after prolonged exposure to perspiration, with subsequent obstruction of the eccrine ducts? a. Scabies c. Pediculosis b. Tinea corporis d. Miliaria ANS: D Of the available options, only miliaria is characterized by a vesicular eruption after prolonged exposure to perspiration, with subsequent obstruction of the eccrine ducts. PTS: 1 REF: Page 1664 23. What is the treatment plan for a strawberry hemangioma? a. Cosmetic surgical removal c. Oral antibiotics b. Topical steroid therapy d. Support of its involution ANS: D Approximately 90% of strawberry hemangiomas involute by 5 to 6 years of age, usually without scarring; therefore self resolution is the usual treatment plan. PTS: 1 REF: Page 1663 If needing more Test Banks, just let me know: [email protected] MULTIPLE RESPONSE 24. Which statements are true of scalded skin syndrome? (Select all that apply.) a. Virulent group II staphylococci cause the scalded skin syndrome. b. A trauma burn triggers the syndrome. c. The syndrome results in a separation involving the epidermis. d. Toxins resulting from scalded skin syndrome are circulated through the blood. e. The lesions of scalded skin syndrome are sites of the infective organism. ANS: A, C, D Virulent group II staphylococci, which produce an exfoliative toxin that attacks desmoglein and adhesion molecules and that causes a separation of the skin just below the granular layer of the epidermis, cause the scalded-skin syndrome. The toxins are usually produced at body sites other than the skin and arrive at the epidermis through the circulatory system. Staphylococci are not typically found in the skin lesions themselves. PTS: 1 REF: Page 16N5U6RSINGTB.COM 25. Which acne lesions are classified as inflammatory? (Select all that apply.) a. Closed comedones b. Opened comedones c. Nodules d. Papules e. Pustules ANS: C, D, E Acne lesions may be divided into inflammatory lesions (pustules, papules, nodules) and noninflammatory lesions (closed and open comedones). PTS: 1 REF: Page 1653 26. Which statements are true regarding Koplik spots? (Select all that apply.) a. Koplik spots are associated with rubeola. b. They appear as white spots. c. The lesions are surrounded by a red ring. d. The lesions of Koplik spots are large in size. e. Koplik spots are primarily found on buccal mucosa. ANS: A, B, C, E Rubeola infection begins with characteristic pinpoint white spots surrounded by an erythematous ring that develops over the buccal mucosa and are known as Koplik spots. PTS: 1 REF: Pages 1659-1660 MATCHING Match the viral disease with the skin lesions produced. Viral diseases may be used more than once. A. Rubella (German measles) B. Rubeola (red measles) C. Roseola (exanthema subitum) D. Varicella (chickenpox) E. Herpes zoster (shingles) 27. Erythematous macular rash over the trunk and neck that lasts 24 hours 28. Pinpoint white spots surrounded by an erythematous ring over the buccal mucosa 29. Groups of vesicles on an inflammatory base following the course of a sensory nerve 30. Pink-to-red coalescing maculopapular rash on the scalp or trunk 31. Red papules, vesicles, and pustules in clusters on the trunk, scalp, or face 27. ANS: C PTS: 1 REF: Page 1660 MSC: After the fever, roseola exhibits an erythematous macular rash that lasts approximately 24 hours and primarily develops over the trunk and neck. 28. ANS: B PTS: 1 REF: Pages 1659-1660 MSC: Rubeola's characteristic pinpoint white spots are surrounded by an erythematous ring, develop over the buccal mucosa, and are known as Koplik spots. 29. ANS: E PTS: 1 REF: Page 1660 MSC: Herpes zoster occurs in partiaNllyURimSImNuGnTeBi.nCdOivMiduals who have had varicella. 30. ANS: A PTS: 1 REF: Page 1658 MSC: Rubella exhibits a faint, pink-to-red coalescing maculopapular rash that develops on the face and spreads to the trunk and extremities 1 to 4 days after the onset of initial symptoms (see Figure 47-8). 31. ANS: D PTS: 1 REF: Page 1660 MSC: The first sign of illness may be itching or the appearance of vesicles, usually on the trunk, scalp, or face. The rash later spreads to the extremities. Characteristically, lesions can be observed in various stages of maturation with macules, papules, and vesicles present in a particular area at the same time (see Figure 47-9). Chapter 49: Shock, Multiple Organ Dysfunction Syndrome, and Burns in Adults MULTIPLE CHOICE 1. What is the final outcome of impaired cellular metabolism? a. Cellular alterations in the heart and brain b. Buildup of cellular waste products c. Cellular alterations in the vasculature structures and kidneys d. Impairment of urine excretion ANS: B The common pathway in all types of shock is impairment of cellular metabolism as a result of decreased delivery of oxygen and nutrients, which are frequently coupled with an increased demand, the consumption of oxygen and nutrients, and a decreased removal of cellular waste products. Of the options available, this selection is the only accurate outcome. PTS: 1 REF: Page 1669 2. Which clinical manifestation of septic shock confirms an elevation in immune system response? a. Tachycardia c. Low respiratory rate b. Increased white blood cell count d. Hypothermia ANS: B Clinical manifestations common in septic shock are fever, high heart rate, high respiratory rate, or elevations in immune respNoUnsReSsI,NsGuTchB.aCsOiMncreased white blood cells and circulating blood glucose. PTS: 1 REF: Page 1671 3. The release of catecholamine by the adrenal glands compensate for which initial effects of hypovolemic shock? a. Interstitial fluid moves out of the vascular compartment. b. Systemic vascular resistance is decreased. c. Heart rate is increased. d. Water excretion is increased. ANS: C Compensatory mechanisms (see Figure 48-3) initially offset hypovolemia. Heart rate and systemic vascular resistance increase as a result of catecholamine release by the adrenal glands, which boosts cardiac output and tissue perfusion pressures. Compelled by a decrease in capillary hydrostatic pressures, interstitial fluid moves into the vascular compartment. The liver and spleen add to blood volume by disgorging stored red blood cells and plasma. In the kidneys, renin (through several intermediaries) stimulates aldosterone release and the retention of sodium and therefore water, whereas antidiuretic hormone (ADH), or vasopressin, from the posterior pituitary gland increases water retention. Data on the compensation of ADH, however, show that as shock worsens, ADH in plasma decreases. PTS: 1 REF: Page 1672 4. Hypovolemic shock begins to develop when intravascular volume has decreased by what percentage? a. 5 c. 15 b. 10 d. 20 ANS: C Hypovolemic shock begins to develop when intravascular volume has decreased by approximately 15%. PTS: 1 REF: Page 1672 5. What type of shock develops as a result of the overstimulation of the parasympathetic nervous system or the understimulation of the sympathetic nervous system? a. Septic c. Anaphylactic b. Cardiogenic d. Vasogenic ANS: D Only vasogenic shock refers to a widespread and massive vasodilation resulting from an imbalance between parasympathetic and sympathetic stimulation of vascular smooth muscle. PTS: 1 REF: Pages 1673-1674 6. What is the clinical hallmark of neurogenic shock as a result of the overstimulation of the parasympathetic nervous system? a. Vasoconstriction NURSINGTcB..COInMcreased metabolism b. Vasodilation d. Respiratory distress ANS: B Neurogenic shock refers to a widespread and massive vasodilation that results from an imbalance between parasympathetic and sympathetic stimulation of vascular smooth muscle. None of the other options are related to this condition. PTS: 1 REF: Page 1673 7. Which form of shock is often more severe than other forms because of its sudden, rapid systemic vasodilation? a. Septic c. Anaphylactic b. Hypovolemic d. Neurogenic ANS: C Anaphylactic shock is often more severe than other types of shock because the hypersensitivity reaction, which triggers vasodilation, has other pathophysiologic effects that rapidly involve the entire body. This action is not associated with the other options. PTS: 1 REF: Pages 1674-1675 8. What type of shock is related to a decrease in systemic vascular resistance? a. Septic c. Hypovolemic b. Cardiogenic d. Heart failure ANS: A Clinical manifestations of only septic shock are persistent low arterial pressure, low systemic vascular resistance from vasodilation, and an alteration in oxygen extraction by all cells. PTS: 1 REF: Page 1675 9. For which type of shock would antihistamines and corticosteroids be prescribed? a. Septic c. Hypovolemic b. Anaphylactic d. Cardiogenic ANS: B Only anaphylactic shock responds to the administration of epinephrine to decrease mast cell and basophil degranulation. Antihistamines and steroids are administered to stop the inflammatory reaction. PTS: 1 REF: Page 1675 10. Which condition is best defined as a clinical syndrome involving a systemic response to infection, which is manifested by two or more of the systemic inflammatory response syndrome criteria? a. Bacteremia c. Septicemia b. Sepsis d. Septic shock ANS: B Of the options available, only sepsis is best defined as a systemic response to infection that is manifested by two or more criteria of the systemic inflammatory response syndrome. NURSINGTB.COM PTS: 1 REF: Page 1676 | Table 48-1 11. In septic shock, which mediators are antiinflammatory? a. Interleukin (IL)–4 (IL-4), IL-10, and IL-13 b. Tumor necrosis factor–alpha (TNF-) and granulocyte cell-stimulating factor c. IL-1, IL-2, and IL-6 d. Prostaglandin, leukotrienes, and bradykinin ANS: A In septic shock, the only antiinflammatory mediators released include lipopolysaccharide-binding protein; IL-1 receptor antagonist; soluble cluster of differentiation 14 (CD-14); type 2 IL-1 receptor; leukotriene 4-receptor antagonist; IL-4, IL-10, and IL-13; and soluble TNF. PTS: 1 REF: Page 1675 12. What mechanism causes organ injury in primary multiple organ dysfunction syndrome (MODS)? a. Impaired immune response c. Impaired perfusion b. Impaired glucose use d. Impaired ventilation ANS: C In primary MODS, the organ injury is directly associated with a specific insult, most often ischemia or impaired perfusion from an episode of shock or trauma, thermal injury, soft-tissue necrosis, or invasive infection. None of the other options accurately identifies the cause of MODS. PTS: 1 REF: Page 1680 13. In secondary multiple organ dysfunction syndrome (MODS), what stimulates the normal endothelial cells to change to a proinflammatory state? a. Interleukin (IL)–4 (IL-4) and IL-13 b. IL-1, IL-6, and tumor necrosis factor (TNF) c. Interferon gamma (IFN-) and granulocyte cell-stimulating factor d. Prostaglandin, leukotrienes, histamine, and bradykinin ANS: B Normal endothelial cells have little interaction with leukocytes except when stimulated by TNF, IL-1, and IL-6. This selection is the only option that accurately describes what stimulates the normal endothelial cells to change to a proinflammatory state. PTS: 1 REF: Page 1680 14. What stimulates the respiratory burst and production of highly toxic free radicals in the multiple organ dysfunction syndrome (MODS)? a. Neutrophils adhering to the endothelium b. Activation of the complement cascade c. Release of prostaglandins, thromboxanes, and leukotrienes d. Activation of the fibrinolytic sNyUstReSmINGTB.COM ANS: A The accumulation of activated neutrophils in organs is thought to play a key role in the pathogenetic development of MODS. When neutrophils adhere to the endothelium, they undergo a respiratory burst (oxidative burst) and release oxygen radicals. The respiratory burst occurs as the activated neutrophil experiences a sudden increase in oxidative metabolism, producing large quantities of highly toxic oxygen free radicals. This selection is the only option that accurately identifies the stimulant of the respiratory burst that results in the production of toxic free radicals. PTS: 1 REF: Page 1682 15. In multiple organ dysfunction syndrome (MODS), the gut hypothesis attempts to explain which phenomena? a. Paralytic ileus b. Translocation of bacteria c. Maldistribution of blood flow d. Massive diarrhea accompanying septic shock ANS: B The loss of intestinal barrier function leads to the systemic spread of bacteria and/or endotoxin from the gut (systemic endotoxemia). This phenomenon is called translocation of bacteria. The gut hypothesis provides a possible explanation for the fact that an infectious focus is not always found in individuals with MODS. The gut hypothesis is not related to any other option. PTS: 1 REF: Page 1682 16. Blistering of the skin within minutes occurs in which type of burn injury? a. First degree c. Deep second degree b. Superficial second degree d. Third degree ANS: B The hallmark of superficial partial-thickness injury is the appearance of thin-walled, fluid-filled blisters that develop within only a few minutes after injury. Blistering that occurs within minutes of the burn injury is not a defining characteristic of the other options. PTS: 1 REF: Page 1686 17. Which form of shock occurs from an acute burn injury? a. Hypovolemic c. Cardiogenic b. Septic d. Vasogenic ANS: A Burn shock consists of a hypovolemic cardiovascular component and a cellular component. Hypovolemia associated with burn shock results from massive fluid losses from the circulating blood volumeN. UTRhSeINotGhTeBr .fCoOrmMs of shock are not directly related to an acute burn injury. PTS: 1 REF: Page 1689 18. Which fluid is most often used in fluid resuscitation after a major burn injury? a. Saline c. Lactated Ringer solution b. Albumin d. Dextrose in water ANS: C Lactated Ringer solution is used most often because it closely approximates extracellular fluid, the repository of fluid leaving the circulatory system during this phase of extensive edema formation (see Table 48-4). The other options are not most often used in fluid resuscitation after major burns. PTS: 1 REF: Page 1689 19. What is the most reliable criterion of adequate fluid resuscitation after a major burn injury? a. Blood pressure c. Respiratory rate b. Pulse rate d. Urine output ANS: D The most reliable criterion for adequate resuscitation of burn shock is urine output. None of the remaining options are considered reliable. PTS: 1 REF: Page 1690 20. The endpoint of burn shock is defined as the time when the individual is able to do which of the following? a. Maintain adequate blood pressure for 4 hours. b. Maintain adequate urine output for 2 hours. c. Manage pain without narcotics. d. Manage pain during dressing changes. ANS: B The endpoint of burn shock is defined as the state in which the individual is able to maintain adequate urine output for 2 hours with the intravenous fluid administration rate equal to the individual’s calculated maintenance rate (see Box 48-4). None of the remaining options are defined as the endpoint of burn shock. PTS: 1 REF: Page 1690 21. Which condition does a burn injury create for an extended period? a. Hypervolemia c. Hyponatremia b. Hypermetabolism d. Hypotension ANS: B Of the options available, a burn injury induces a hypermetabolic state that persists until wound closure. PTS: 1 REF: Page 1691 NURSINGTB.COM 22. What effect does a fatal burn injury have on interleukins (ILs)? a. Decreases levels of IL-2, which may decrease T helper 1 (Th1) lymphocytes. b. Decreases levels of IL-4, which causes a shift in production from Th1 to Th2 lymphocytes. c. Decreases levels of IL-6, which produces cytokines. d. Decreases levels of IL-12, which stimulates the production of immunoglobulins. ANS: A A fatal burn injury has often shown decreased levels of IL-2, which may result in decreased Th1 lymphocytes. This option is the only accurate description of the effect a fatal burn injury has on ILs. PTS: 1 REF: Page 1692 23. Daily evaporative water loss after a burn injury is approximately how many times the normal? a. 5 c. 15 b. 10 d. 20 ANS: D Moncrief and Mason attempted to determine the magnitude of such a loss and determined that daily evaporative water loss was in the range of 20 times normal in the early phase of injury, with gradual decreases as wound closure is achieved. PTS: 1 REF: Page 1693 24. What is the significance of a high level of interleukin 1 (IL-1) in a patient who has experienced severe burns? a. Prognosis is poor. c. Urinary function is improved. b. Antibiotic therapy is required. d. They are less at risk for death. ANS: D The level of IL-1 inversely correlates with burn survival; low levels may be associated with a higher mortality. This selection is the only option that accurately identifies the significance of a high level of IL-1. PTS: 1 REF: Page 1692 25. What is the purpose of monitoring procalcitonin (PCT) levels in a patient after a burn? a. To help evaluate the potential risk for respiratory complications b. To justify the initiation of antibiotic therapy c. To determine when discontinuing antibiotic use is feasible d. To help in the selection of appropriate antibiotic therapy agents ANS: C Seeking to decrease the use of antibiotics in the patient who is critically ill and thus prevent resistance to antibiotics is an important strategy in treating infection. Recent research suggests that monitoring serial PCT levels, a precursor hormone to calcitonin, may be used to shorten antibiotic use in the treatment of respiratory infections. PCT, normally not discernible on assay, when elevated may indicate specific proinflammatory response during a bacterial infection. PCT levels should not be used as an indicator to start antibiotics; however, if monitoredNsUeRquSIeNnGtiTalBly.CaOtMthe start of empiric antibiotics and then dropped to low levels, then discontinuation may be clinically indicated. PTS: 1 REF: Page 1679 | What's New box 26. How many milliliters of fluid replacement per hour does a 70-kg adult with a 50% total body surface area burn and a body surface area of 2 m require? a. 150 c. 350 b. 275 d. 500 ANS: B A 70-kg adult with a 50% total body surface area burn and a body surface area of 2 m requires the following: Basal = (1500 ml/day) (2 m2 body surface area) = 3000 ml/24 hr or 125 ml/hr Evaporative = (25 + 50% total body surface burn) (2m2 total body surface area) = (75) (2) = 150 ml/hr Total maintenance fluids = 125 ml + 150 ml = 275 ml/hr PTS: 1 REF: Page 1690 | Box 48-4 MULTIPLE RESPONSE 27. A patient will be referred to the burn unit when which criteria are met? (Select all that apply.) a. Patient is older than 5 years of age. b. The burn involves the face or a major joint. c. The source of the burn is electrical. d. Partial thickness burns are on more than 10% of the total body surface area (TBSA). e. Patient has a life-threatening trauma injury. ANS: B, C, D A burn unit may treat adults or children or both. Burn injuries that should be referred to a burn unit include the following: partial-thickness burns on more than 10% TBSA, burns that involve the face, hands, feet, genitalia, perineum, or major joints, and electrical burns, including lightning injury. If the trauma poses the greater immediate risk, then the patient’s condition may be initially stabilized in a trauma center before being transferred to a burn center. PTS: 1 REF: Page 1689 | Box 48-3 28. Which feedback loop will further impair oxygen in all types of shock? (Select all that apply.) a. Activation of the fibrinolytic cascade b. Increased circulating volume c. Hypermetabolic state d. Lysosomal enzyme release e. Activation of the clotting cascade ANS: D, E Both positive and negative compensatory mechanisms, such as anaerobic metabolism, lysosomal enzyme release, decreaNseUdRiSnItNraGvTaBs.cCuOlaMr volume, and activation of the clotting cascade, may further impair oxygen delivery and use. The remaining options are not related to impaired oxygen delivery. PTS: 1 REF: Page 1669 MATCHING Match the types of shock with the corresponding descriptions. Terms can be used more than once. A. Cardiogenic B. Hypovolemic C. Neurogenic D. Anaphylactic E. Septic 29. Follows a systemic inflammatory response. 30. Follows widespread hypersensitivity reaction. 31. Follows myocardial infarction. 32. Follows major burns. 33. Follows parasympathetic stimulation. 29. ANS: E PTS: 1 REF: Page 1675 MSC: Septic shock begins with systemic inflammatory response syndrome. It then evolves into sepsis, into severe sepsis, and finally into septic shock. 30. ANS: D PTS: 1 REF: Page 1674 MSC: Anaphylactic shock is the outcome of a widespread hypersensitivity reaction known as anaphylaxis. 31. ANS: A PTS: 1 REF: Page 1671 MSC: Cardiogenic shock results from the inability of the heart to pump adequate blood to tissues and end organs. This type of shock occurs from any cause, the most common being within hours of an acute myocardial infarction or severe episode of myocardial ischemia. 32. ANS: B PTS: 1 REF: Page 1689 MSC: Hypovolemia associated with burn shock results from massive fluid losses from the circulating blood volume. 33. ANS: C PTS: 1 REF: Pages 1673-1674 MSC: Any factor that stimulates parasympathetic activity or inhibits sympathetic activity of vascular smooth muscle can cause neurogenic shock. NURSINGTB.COM Chapter 50: Shock, Multiple Organ Dysfunction Syndrome, and Burns in Children MULTIPLE CHOICE 1. What is the most common type of shock in children? a. Hypovolemic c. Neurogenic b. Cardiogenic d. Septic ANS: A Hypovolemic shock, the most common type of shock in children, is associated with a reduction in the intravascular volume relative to the vascular space. PTS: 1 REF: Page 1703 2. Hypotension is likely to occur when an infant or child is greater than % dehydrated. a. 2 c. 7 b. 5 d. 10 ANS: D Hypotension typically develops when dehydration is greater than 10% in the infant or child or greater than 6% in the adolescent. PTS: 1 REF: Page 1703 3. A prolonged capillary refill time in children is consistent with the development of what type of shock? a. Hypovolemic NURSINGTcB..COCMompensated b. Septic d. Cardiogenic ANS: B The child with hypovolemic shock demonstrates signs of inadequate blood flow to some tissue beds and some evidence of organ system dysfunction. The infant or child may be irritable or lethargic. Respirations will be rapid and may be labored if shock is severe or associated with myocardial failure. The skin will be mottled, although pallor also may be observed. A prolonged capillary refill time (>2 seconds) is consistent with the development of septic shock. This statement is not necessarily true of the other types of shock. PTS: 1 REF: Page 1704 4. Bradycardia in young children experiencing shock symptoms often suggests which result? a. Onset of cardiac dysfunction b. Effective management of cardiac dysfunction c. Good management of anxiety d. Cardiovascular collapse ANS: D Bradycardia often indicates impending cardiovascular collapse or cardiac arrest and is the most common terminal cardiac rhythm observed in children. This selection is the only option that accurately describes the impact of bradycardia on a child. PTS: 1 REF: Page 1704 5. What is the cause of appropriately 50% of the deaths in children who have experienced a burn injury? a. Immunosuppression c. Inhalation injury b. Hypermetabolism d. Hypertrophic scarring ANS: C The leading cause of death in children after burn injury, as in adults, is an inhalation injury. Inhalation injuries cause approximately 50% of all deaths in children with burns. Although the other options are factors, they are not responsible for 50% of the deaths. PTS: 1 REF: Pages 1715-1716 6. In cardiogenic shock, what is the cause of hepatomegaly and periorbital edema? a. Mass vasodilation as a result of chemical mediators released from the myocardium b. Low cardiac output, causing a high central venous pressure c. Tissue damage to the myocardium, causing increased capillary permeability d. Low perfusion of the kidneys, stimulating the renin-angiotensin-aldosterone system to retain sodium and water ANS: B Evidence of an adequate or high central venous pressure, including hepatomegaly and periorbital edema, is observed in cardiogenic shock. This selection is the only option that accurately identifies the cause of these symptoms. PTS: 1 REF: Page 1704 NURSINGTB.COM 7. Approximately 80% of all hospital-acquired infections in children are a result of which type of organism? a. Bacteria c. Fungi b. Viruses d. Rickettsia ANS: A In adults and children, approximately 40% of all hospital-acquired infections are linked to gram-negative infections, 40% to gram-positive infections, and 20% to viruses, fungi, or rickettsial microorganisms. PTS: 1 REF: Page 1707 8. Which cytokines are antiinflammatory mediators? a. Interleukin (IL)–1, IL-6, and tumor necrosis factor–alpha (TNF-) b. IL-8, IL-12, and platelet-activating factor c. IL-24, arachidonic acid metabolites, and nitric oxide d. IL-4, IL-11, and colony-stimulating factor ANS: D Antiinflammatory mediators include only IL-4, IL-10, IL-11, and IL-13; transforming growth factor-beta; colony-stimulating factors; soluble TNF receptor; IL-1 receptor antagonist; and activated protein C. PTS: 1 REF: Page 1707 9. What type of injury is associated with cellular injury caused by the restoration of blood flow and physiologic concentrations of oxygen to cells that have been exposed to injurious but nonlethal hypoxic conditions? a. Hypoxic c. Reperfusion b. Hyperoxygenation d. Ischemic ANS: C Reperfusion (reoxygenation) injury is associated with cell damage caused by the restoration of blood flow and physiologic concentrations of oxygen to cells that have been exposed to injurious but nonlethal hypoxic conditions. PTS: 1 REF: Page 1710 10. What are the primary goals for the treatment of shock? a. Maximizing oxygen delivery and minimizing oxygen demand b. Maintaining hydration and adequate urinary output c. Supporting all facets to the cardiovascular system d. Maintaining all vital signs within normal functioning ranges ANS: A The primary goals of the treatment of shock are maximizing oxygen delivery and minimizing oxygen demand. Although the other options are desirable, they are each associated with the primary goals related to oxygen delivery and demand. PTS: 1 REF: Pages 1710-1711 11. To determine a child’s response toNfUluRiSdINthGeTrBap.CyOfMor shock, the nurse should monitor . a. Hematocrit and hemoglobin levels c. Blood pressure and pulse b. Urine output and specific gravity d. Arterial blood gases and heart rate ANS: B Monitoring of the volume of urine output and specific gravity is most useful in determining the child’s response to fluid therapy. PTS: 1 REF: Page 1711 12. For children who are experiencing shock, crystalloids and colloids are generally administered in boluses of how many milliliters per kilogram (ml/kg)? a. 5 c. 15 b. 10 d. 20 ANS: D In general, isotonic crystalloids (salt-containing solutions, such as normal saline or lactated Ringer solution) or colloids (protein-containing fluids, such as albumin or blood) are administered in boluses of 20 ml/kg. PTS: 1 REF: Page 1711 13. As the result of the inability to concentrate urine, children are at risk for dehydration before which age? a. 2 years c. 6 years b. 4 years d. 8 years ANS: A Children younger than 2 years lack the ability to concentrate urine because of the immaturity of the renal system and are therefore at increased risk for dehydration. PTS: 1 REF: Page 1719 14. What causes renal failure after electrical burns in children? a. Cytokines are released after the damaged tissue. b. Immature kidneys are unable to compensate for the electrical burn. c. Cardiac output is reduced. d. Myoglobin is released from damaged muscles. ANS: D The release of myoglobin may occlude the kidney tubules and result in renal failure. This selection is the only option that accurately describes the correlation between electrical burns and renal failure. PTS: 1 REF: Page 1719 15. Compared with the ebb phase, characteristics of the catabolic flow phase in metabolism after a burn injury in a child include which process? a. Reduced oxygen consumption c. Impaired circulation b. Elevation of catecholamines d. Cellular shock ANS: B NURSINGTB.COM After the resolution of the shock and the restoration of circulating volume, the metabolic response shifts to a catabolic (flow) phase. A state of hypermetabolism ensues, characterized by increased oxygen consumption and the elevation of catecholamines, glucocorticoids, and glucagon. This selection is the only option that accurately compares the characteristics of the ebb and flow phases. PTS: 1 REF: Page 1720 16. What advantage do impregnated silver dressings have for patients with burn injuries? a. Impregnated silver dressings contain natural-occurring collagens. b. They require only one dressing change every other day. c. Removal is less painful. d. Impregnated silver dressings are cost effective. ANS: C Impregnated silver dressings not only kill bacteria but most are processed with a special layer, making them less painful to remove. Some of these products can be left on the wound for up to 2 weeks, allowing for less frequent dressing changes. The other options are related to a new extracellular matrix (ECM) product that is now being researched for repair and remodeling of damaged tissues. PTS: 1 REF: Page 1722 | What's New box 17. What is the most serious outcome resulting from limited glycogen stores in children who have been seriously burned? a. Poor wound healing c. Decreased immunity b. Increased morbidity d. Loss of adipose tissue stores ANS: B Glycogen stores are limited in children, making it hard for them to meet the increased energy demands of the burn. This prolonged metabolic dysfunction may lead to the loss of lean body mass. The most serious affect is increased morbidity; that is, the risk of additional illnesses will impede recovery. The remaining options do not represent the most serious outcome of limited glycogen stores resulting from severe burns. PTS: 1 REF: Page 1720 18. What is the normal range of respirations for an infant (in breaths per minute)? a. 18 to 25 c. 27 to 37 b. 20 to 28 d. 30 to 53 ANS: D Although the other ranges may be normal for some age groups, the normal respiratory range for infants is 30 to 53 breaths per minute. PTS: 1 REF: Page 1702 | Table 49-1 19. While awake, what is the normal heart rate for toddlers (in beats per minute)? a. 100 to 205 c. 98 to 140 b. 100 to 180 d. 80 to 120 NURSINGTB.COM ANS: C Although the other ranges may be normal for some age groups, the normal heart rate range for toddlers during waking hours is 98 to 140 beats per minute. PTS: 1 REF: Page 1702 | Table 49-1 MULTIPLE RESPONSE 20. Which statements are true regarding multiple organ dysfunction syndrome (MODS)? (Select all that apply.) a. Diagnosis requires simultaneous failure of at least two organs. b. Primary MODS occurs immediately after the attributing cause. c. Secondary MODS occurs within 3 to 7 days of the initial insult. d. Chronic illness increases a child’s risk for MODS. e. Risk factors for MODS include severe or prolonged shock, sepsis, and trauma. ANS: A, D MODS is the simultaneous failure of at least two organs resulting from a single cause and may be either primary or secondary. Primary MODS is directly attributable to the insult and typically occurs 3 to 7 days after an insult. Secondary MODS typically occurs later and may be associated with the more sequential development of organ dysfunction. Risk factors for MODS include severe or prolonged shock, sepsis, trauma, cardiopulmonary arrest, congenital heart disease, and liver and bone marrow transplantation. Children with chronic diseases have an increased risk for MODS and increased mortality. PTS: 1 REF: Pages 1699-1700 21. The child with septic shock has significant hypovolemia that typically responds to which interventions? (Select all that apply.) a. Aggressive fluid administration b. Antibiotic therapy c. Titration of inotropes d. Vasopressors e. High caloric nasogastric feedings ANS: A, B, C, D The child with septic shock has significant hypovolemia that typically responds to aggressive fluid administration, antibiotic therapy, titration of inotropes, vasopressors, and vasodilators. Nasogastric feedings are not generally ordered for children with septic shock. PTS: 1 REF: Page 1709 22. Which behaviors in newborns would support the possibility of shock? (Select all that apply.) a. Decreased heart rate variabilitNyURSINGTB.COM b. Temperature instability c. Hyperalertness d. Increased muscle tone e. Hypoglycemia ANS: A, B, E Nonspecific signs of distress in newborns include jitteriness or lethargy with decreased muscle tone, bradycardia or decreased heart rate variability, temperature instability, and hypoglycemia. PTS: 1 REF: Page 1700 | Box 49-1 23. Which assessment finding would be recognized as a late sign of shock in a child? (Select all that apply.) a. Metabolic (lactic) acidosis b. Cool skin c. Bradycardia d. Prolonged capillary refill e. Hypotension ANS: C, E Although all of the options are observable, hypotension and bradycardia are recognized as late signs of shock in children. PTS: 1 REF: Page 1700 | Box 49-1 24. Which statements are true regarding how a child’s body compensates for cardiogenic shock? (Select all that apply.) a. Splanchnic arteries are constricted to divert blood from the skin, kidneys, and gut to the heart and brain. b. Peripheral blood vessels are constricted to raise blood pressure. c. Adrenergic responses produce tachycardia to increase cardiac output. d. The renin-angiotensin-aldosterone system is stimulated when renal function decreases. e. Compensation prevents the child from developing hepatic or mesenteric failure. ANS: A, B, C, D In the early stages of cardiogenic shock, adrenergic compensatory mechanisms produce tachycardia, peripheral vasoconstriction, and constriction of the splanchnic arteries to divert blood flow from the skin, gut, and kidneys to maintain flow to the heart and brain. These compensatory mechanisms may be sufficient to maintain the child’s systolic blood pressure and effective coronary artery and cerebral blood flow. However, tachycardia and systemic arterial constriction increase myocardial oxygen consumption. In addition, reduction in gut and kidney blood flow may produce hepatic, mesenteric, or renal ischemia or failure. Decreased renal perfusion stimulates the renin-angiotensin-aldosterone system, as described for hypovolemic shock. PTS: 1 REF: Page 1704 25. What considerations must the nurse take into account when assessing the severity of a burn injury? (Select all that applyN.)URSINGTB.COM a. Amount of fluid lost over a 24-hour period b. Circumference of the burn injury c. Depth of the burn injury d. Severity of the injury caused to other body systems e. Percentage of total body surface area involved ANS: C, E The severity of a burn injury is assessed on the basis of the percentage of the total body surface area involved. Because burn trauma represents a three-dimensional wound, the severity of injury is also assessed in relation to the depth of the injury. Although the other options are considered when planning treatment, the severity of the burn injury, itself, is not based on any of these. PTS: 1 REF: Pages 1716-1717 MATCHING Match the terms with the corresponding descriptions. A. Scald burn B. Contact burn C. Flame burn D. Electrical burn E. Chemical burn 26. Involves flammable liquids such as gasoline. 27. Is caused by hot grease. 28. Is the result of direct contact with high- and low-voltage current. 29. Is caused by a corrosive agent. 30. Involves cigarette burns and curling irons. 26. ANS: C PTS: 1 REF: Page 1716 MSC: Flame burns involving flammable liquids, especially gasoline, are most common in older children. 27. ANS: A PTS: 1 REF: Page 1715 MSC: Scald injuries (e.g., hot water, grease) are most common among young children. 28. ANS: D PTS: 1 REF: Page 1716 MSC: Electrical burns are the result of direct contact with high- or low-voltage current. 29. ANS: E PTS: 1 REF: Page 1716 MSC: Chemical burns occurring at home may be a result of swallowing corrosive agents. 30. ANS: B PTS: 1 REF: Page 1715 MSC: Contact burns may be intentionally inflicted by contact with cigarettes or other hot objects such as curling irons. NURSINGTB.COM [Show More]

Last updated: 1 year ago

Preview 1 out of 934 pages

Add to cart

Instant download

document-preview

Buy this document to get the full access instantly

Instant Download Access after purchase

Add to cart

Instant download

Reviews( 0 )

$15.00

Add to cart

Instant download

Can't find what you want? Try our AI powered Search

OR

REQUEST DOCUMENT
123
0

Document information


Connected school, study & course


About the document


Uploaded On

Feb 02, 2023

Number of pages

934

Written in

Seller


seller-icon
Nolan19

Member since 2 years

10 Documents Sold


Additional information

This document has been written for:

Uploaded

Feb 02, 2023

Downloads

 0

Views

 123

Document Keyword Tags

Recommended For You

Get more on EXAM »

$15.00
What is Browsegrades

In Browsegrades, a student can earn by offering help to other student. Students can help other students with materials by upploading their notes and earn money.

We are here to help

We're available through e-mail, Twitter, Facebook, and live chat.
 FAQ
 Questions? Leave a message!

Follow us on
 Twitter

Copyright © Browsegrades · High quality services·